0% found this document useful (0 votes)
433 views224 pages

Family Law Cases - Year 1 - Indian Law

The case involved a Hindu couple who got married in India but lived in multiple other countries like Sweden, Australia and India over the years. The key issue was whether the Hindu Marriage Act applied to Hindus who were not living in India and were not domiciled in India. The Supreme Court held that domicile is the connecting factor for applying the Hindu Marriage Act to Hindus living outside India. Since the wife was domiciled in India, the Hindu Marriage Act was applicable in her divorce proceedings even though neither party was living in India at the time. The court examined provisions of the Act and principles of domicile to arrive at its decision.

Uploaded by

aishvarya
Copyright
© © All Rights Reserved
We take content rights seriously. If you suspect this is your content, claim it here.
Available Formats
Download as PDF, TXT or read online on Scribd
0% found this document useful (0 votes)
433 views224 pages

Family Law Cases - Year 1 - Indian Law

The case involved a Hindu couple who got married in India but lived in multiple other countries like Sweden, Australia and India over the years. The key issue was whether the Hindu Marriage Act applied to Hindus who were not living in India and were not domiciled in India. The Supreme Court held that domicile is the connecting factor for applying the Hindu Marriage Act to Hindus living outside India. Since the wife was domiciled in India, the Hindu Marriage Act was applicable in her divorce proceedings even though neither party was living in India at the time. The court examined provisions of the Act and principles of domicile to arrive at its decision.

Uploaded by

aishvarya
Copyright
© © All Rights Reserved
We take content rights seriously. If you suspect this is your content, claim it here.
Available Formats
Download as PDF, TXT or read online on Scribd
You are on page 1/ 224

CASE BRIEFS

COURSE NAME: FAMILY LAW I


COURSE INSTRUCTOR: SWATI MALIK
CLASS: B.A.LLB 2014 SECTION A & D
FALL 2015
(Compilation based on Course Instructor’s notes & Student
submissions)

1|Page
TABLE OF CONTENTS
I. WEEK 1: Introduction to the Course (3rd August)
a. No Cases
II. WEEK 2 & 3: Hindu Marriage Act: Valid Marriages (10th August & 17th
August)
a. Domicile
i. Sondur Gopal v. Sondur Rajini AIR2013SC2678
b. Religion
i. Gullipilli Sowria Raj v. Bandaru Pavani AIR 2009 SC 1085
c. Bigamy/Ceremony
i. Ram Prasad v. State of UP AIR 1961 All 334
ii. Priya Bala Ghosh v. Suresh Chandra Ghosh AIR 1971 SC 1153
iii. Bhaurao Lokhande v. State of Maharashtra AIR 1965 SC 1564
iv. S. Nagalingam v. Sivagami AIR 2001 SC 3576
v. Balusami Reddiar v. Balakrishna Reddiar, AIR 1957 Mad 97
d. Religious Conversion
i. Lily Thomas v.Union of India AIR 2000 SC 1650
ii. Sarla Mudgal v. Union of India 1995 SCC (3) 635
e. Caste:
i. Smt. Kastoori Devi v. Chiranji Lal AIR 1960 All 446
f. Minority:
i. P. Venkataramana v. State AIR 1977 AP 43
ii. Ravi Kumar v. The State 124 (2005) DLT 1
g. Sapinda:
i. Navalkar v. Meena Arun Navalkar AIR 2006 Bom 342
III. WEEK 4 & 5: Muslim Marriage & Sources of Muslim Law (24th August & 31st
August)
a. Nature of a Muslim Marriage
i. Abdul Kadir v. Salima And Anr (1886) ILR 8 All 149 (Nature of a
Muslim Marriage)
b. Who is a Muslim?
i. Narantakath Avullah v. Parakkal Mammu And Ors 71 Ind Cas 65;
(1922) 43 MLJ 663 (Are Ahmediyas Muslim?)
c. Local Customs
i. H.H. Mir Abdul Hussain Khan v. Mussammat Bibi Sona Dero (1918)
20 BOMLR 528
ii. Hirbae and Others v. Sonabae. (The Kojahs and Memons case)
d. Minority:
i. Musammat Atika Begum v Muhammad Ibrahim Rashid Nawab
(Allahabad), [1916] UKPC 70 (Jul. 20, 1916)

2|Page
ii. Smt. Khatiza Qubra alias Tara Bano v Iqbal Mohd , AIR2009Raj82.
iii. Mohd. Nihal v. State MANU/DE/0980/2008
IV. WEEK 6: Muslim Divorce (7th September)
i. Shamim Ara Vs State Of U.P., AIR 2002 SC 3551
ii. Itwari Vs. Asghari and others, AIR 1960 All 684
iii. Marium v. Mohd. Shamsi Alam, AIR 1979 All 257
iv. Sabah Adnan Sami Khan v.. Adnan Sami Khan, AIR 2010 Bom 109
v. Yousuf Rawther v. Sowramma, AIR 1971 Ker 261
vi. Abdurahiman v. Khairunnessa, I(2010)DMC707,
V. WEEK 9, 10 & 11: Matrimonial Reliefs & General Bars to Matrimonial Reliefs
(14th, 21st & 28th September)
a. Restitution of Conjugal Rights / Judicial Separation
i. T. Sareetha v. T.Venkata Subbaih AIR 1983 AP 356
ii. Harvinder Kaur v. Harmander Singh AIR 1984 Del 66
iii. Saroj Rani v. Sudershan Kumar AIR 1984 SC 1562
b. Grounds of Divorce under HMA & Irretrievable Breakdown Divorce
i. N.G. Dastane v. S. Dastane, AIR 1975 SC 1534
ii. Samar Ghosh v. Jaya Ghosh, (2007)4SCC511
iii. Bipinchandra Jaisinghbai Shah v. Prabhavati, AIR 1957 SC 176
iv. Dharmendra Kumar v. Usha Kumar, AIR 1977 SC 2213
v. Hirachand Srinivas Managaonkar v. Sunanda, AIR 2001 SC 1285
vi. Sureshta Devi v. Om Prakash, AIR1992SC1904
vii. Naveen Kohli v. Neelu Kohli (2006) 3 SCC 491
viii. V.Bhagat v. D. Bhagat AIR 1994 SC 710
ix. Savitri Pandey v..Prem Chandra Pandey AIR 2002 SC 591
VI. Week 7: Christian Marriage & Divorce
a. Vinisha Tolani v. Jitesh Tolani, AIR2010SC1915
b. Lakshmi Sanyal vs Sachit Kumar Dhar, AIR 1972 SC 2667
c. Sujatha v. Jose Augustine, (1994) II DMC 442
d. Leelamma v Dilip Kumar, AIR 1993 Ker 57
e. P.J. Moore v. Valsa, AIR 1992 Ker 176
f. Reynold Rajamani v. Union of India, AIR 1982 SC 1261
g. Pramilla Khosla v. Rajnish Kumar Khosla, AIR 1979 Delhi 78
h. 8. Molly Joseph v. George Sebastian, AIR 1997 SC 109
VII. Week 8: Special Marriages Act and the Foreign Marriages Act
a. Tamali Bhattacharjee v. Samik Baidya, CHN 2004 (1) 639
b. Deepak Krishna v. District Registrar And Ors., AIR 2007 Ker 257
c. Abdur Rahim Undre Vs Padma Adbur Rahim Undre, AIR 1982 Bom 341
d. Vincent Joseph Konath vs. Jacinitha Angela Vincent Konath, AIR 1994
Bombay 120

3|Page
e. Minoti Anand & Anr vs Subhash Anand, AIR 2011 Bom 61
VIII. Week 12: Maintenance of Family Members
a. Maintenance under Muslim Law
i. Zohara Khatoon V. Mohd. Ibrahim, AIR 1981 SC 1243
ii. Mohd. Ahmed Khan V Shah Bano Begum, AIR 1985 SC 945
iii. Noor Saba Khatoon v. Mohd. Quasim, AIR 1997 SC 3280
iv. Danial Latifi v. Union of India (2001) 7 SCC 740
v. Iqbal Bano Vs State of UP (2007) 6 SCC 785
b. Spousal Maintenance of non-Muslims/Domestic Violence Act/CrPC
i. Chand Dhawan v Jawaharlal Dhawan 1993 SCC (3) 406
ii. Bhagwan Dutt v. Kamla Devi, AIR 1975 SC 83
iii. Chaturbhuj v. Sita Bai, (2008) 2 SCC 316
iv. Vinny Parmar v. Paramvir Parmar (2011) 13 SCC 112.
v. Geeta Satish Gokarna v. Satish Shankarrao Gokarna, AIR 2004 Bom
345
vi. Savitaben v. State of Gujarat AIR 2005 SC 1809
vii. Hari Lal v. Balvantia AIR 1998 All 211
viii. Indra Sarma v. V.K.V.Sarma 2013 (14) SCALE 448
c. Maintenance of Parents and Children
i. Selva Saroja v. Sasinathan (1989) Cr LJ 2032
ii. Mahendra Kumar Gaikwad v. Gulabbhai 2001 Cr LJ 2111 (Bom)
iii. Vijaya Manohar Arbat v. Kashi Rao Rajaram AIR 1987 SC 1100,
IX. Week 13: Adoption, Custody and Guardianship
a. Githa Hariharan v. Reserve Bank of India (1999) 2 SCC 228
b. Vinay Pathak And His Wife v. Unknown, 2010(1) BomCR 434
c. Lakshmi Kant Pandey v.Union of India AIR 1984 SC 469
d. Shabnam Hashmi v. Union of India 2014 (2) SCALE 529
e. Baby Manji Yamada v. Union Of India AIR 2009 SC 84
f. Vijayalaxmamma v. B.T. Shankar, (2000) 4 SCC 538
g. Khazan Singh v. Union of India AIR 1980 Delhi 60
X. Week 15: Live In Relationships
a. Chanmuniya v. Virendra Kumar (2010) 10 SCALE 602
b. Bharatha Matha v. R. Vijaya Ranganathan AIR 2010 SC 2685
c. Velusamy v. Patchaiammal AIR 2011 SC 479
d. Aysha v. Ozir Hassan, Madras High Court, 17 June, 2013
e. Revanasiddappa & Anr v. Mallikarjun ( 2011) 11 SCC

4|Page
CASE SUMMARIES/BRIEFS

I. WEEK 1: Introduction to the Course (3rd August)


a. No Cases
II. WEEK 2 & 3: Hindu Marriage Act: Valid Marriages (10th August & 17th
August)
a. Domicile
i. Sondur Gopal v. Sondur Rajini AIR2013SC2678

Summary 1:

The facts in the Sondur Gopal vs Sondur Rajini case are , That the marriage between the
husband (appellant) and the wife (respondent) took place on 25th of June, 1989 according to the
Hindu rites at Bangalore and was registered under The Hindu Marriage Act. In the first week of
July 1989 right after the marriage the husband left for Sweden and his wife joined him in the
month of November of the same year. They also had their first child in Sweden, Natasha, in the
year 1993 and the same year they bought a house in Stockholm. The couple then applied for
Swedish citizenship, which was granted to them in the year 1997. In 1997 the family moved to
Mumbai as, according to the wife, the husbands employer wanted to set up business in India.
They stayed there till mid 1999 after which the husband was offered a job in Sydney. They
moved to Australia on a sponsorship visa which permitted them to stay there for a period of four
years. But he was fired from his job in the year 2001 and since they no longer had any
sponsorship they had to leave, and they decided to move back to Stockholm. However during
their stay in Australia the husband had disposed of the house there so they had to lease a new
one. They also had a son in the year 2001 during their stay in Sydney. In October 2002 he found
another job in Sydney and left for it in December. His wife and kids joined him there in January
2003 after a short trip to Mumbai. She and the kids decided to visit Mumbai again in December
2003 on a tourist visa. According to the husband, he was informed in January 2004 that his wife
did not want to return back to Sydney at all. According to him he also went there to try and
convince her but all attempts failed and she filed for divorce before the Family Court, Bandra.

The issue for courts to decide here is whether the Hindu Marriage Act. is applicable to Hindus
not living in India and not domiciled in India and also when is somebody of Indian domicile.

The laws applied in this case are Section 1, Section 2 and Section 10 of the Hindu Marriage Act.

For any extra-territorial law to be applied it must have some relation or nexus with India,
otherwise the Indian Parliament has no right to make or apply such a law. Therefore the court
held that it is essential for the person to be a domicile of India for the Hindu Marriage Act to be
applied to him. The court also held that to prove domicile of country one needs to show that they
wish to stay in that country indefinitely and prove it sufficiently.

5|Page
Therefore the Hon. Supreme Court held that because the husband could satisfy the court with his
argument that he was a domicile of Australia the Hindu Marriage Act would apply to them.

I agree with decision taken by the Supreme Court in this case and with the reasoning provided by
them. It is essential for a law to have some sort of nexus with India otherwise the parliament
can’t enforce it on anyone. If the requirement of being a domicile of India is removed it would
imply that any Hindu residing in any part of the world can approach any Indian court with regard
to matters covered in this act. Any extra-territorial act which has no nexus with India is in itself
invalid and if the act is made to extend to people who are not domiciled in India it would cut off
all nexus with India rendering the act invalid. Also to show yourself as a domicile of a country it
is imperative to show desire to live in the country for the remainder of somebodies life. A work
document for a specific time period is not sufficient evidence to prove this as held in this case.
To establish desire one needs to show steps and actions taken towards establishing permanent
residency in that country and when somebody gives up their domicile of choice , their domicile
of origin is automatically invoked.

b. Religion
i. Gullipilli Sowria Raj v. Bandaru Pavani AIR 2009 SC 1085

Summary 1:

An Analysis of Gullipilli Sowria Raj v Bandaru Pavani alias Gullipilli Pavani

The case of Gullipilli Sowria Raj emerged in the late 1990s, and its judgment by the Supreme
Court was declared in 2008. It essentially deals with Sections 2, 5, 7, 8, 11, 12(1)(c) as well as
the Preamble of the Hindu Marriage Act, 1955.
The facts of the case begin with the marriage of the appellant to the respondent wife in a temple,
by exchange of thali, on the 24th of October 1996. Subsequently, this marriage was registered on
the 2nd of November under Section 8 of the Hindu Marriage Act. However, a few months later,
the respondent wife filed a petition in the Family Court of Vishakapatnam under Section 12(1)(c)
of Hindu Marriage Act, for a decree of nullity of the marriage. This was essentially on the
grounds of misrepresentation by the appellant regarding his social status and the fact that he was
a Hindu in religion – he was in fact a Roman Catholic Christian.
When the Family Court dismissed the petition, the respondent wife appealed to the High Court,
which allowed the appeal. However, it held that the marriage was invalid not under Section
12(1)(c) but under Section 11. In other words, a marriage between a Hindu and a Christian was
held to be void ab initio.
A few months later, the respondent married another man, after which the appellant filed a
Special Leave Petition in the Supreme Court. Thus, the main issue to be answered by the Court
was whether a marriage between a Hindu and a Christian is valid under the provisions of the
Hindu Marriage Act, 1955.

6|Page
The appellant claimed that the Hindi Marriage Act did not prohibit a marriage between a Hindu
and a non-Hindu. He also argued that because Section 5 (dealing with the conditions for a
marriage between any two Hindus) made use of the word “may”, the conditions stated were not
mandatory and must be held to be optional.
The respondent submitted that the Preamble to the Act itself indicated that the Act and its
provisions applied to Hindus and Hindus only. It is as follows: “An act to amend and codify the
law relating to marriage among Hindus”. Further, Section 2(1)(c) specifically excludes
Christians from the ambit of this law. However, the marriage did not fall under Section 11 of the
Act, which covers void marriages, and the High Court was erroneous in claiming the contrary. In
actuality, it was under Section 12(1)(c) dealing with voidable marriages that the marriage could
be nullified on the basis of fraud.
The Supreme Court dismissed the appeal and concurred with the reasoning of the respondent’s
argument. It held that the Preamble, Section 2 and Section 5 made it clear that the Act applies
only to Hindus. Usage of the word “may” in the opening line of Section 5 does not make its
provisions optional. It only indicates that a marriage can be solemnized between two Hindus if
the conditions are fulfilled. It is not directory but mandatory and depends only on the conditions
being fulfilled. Further, while the marriage of a Hindu to a non-Hindu was permitted, this can
only be carried out as per the provisions of the Special Marriage Act. Thus, the appeal was
dismissed and the marriage held to be nullified.
I believe that the Supreme Court’s decision was extremely accurate. The provisions of the Hindu
Marriage Act leave no room for doubt as to the fact that it applies to Hindus and specifically
excludes Christians. The apex court also rectified the Section rendering the marriage invalid. It
was Section 12(1)(c) and not Section 11, because the marriage did not contravene Section 11 for
it to be declared void ab initio. It was however, subject to nullity as per Section 12(1)(c).

c. Bigamy/Ceremony
i. Ram Prasad v. State of UP AIR 1961 All 334

Summary 1:

FACTS:
The main crux, the case resolves around the validity of a second marriage and whether or not
modern laws should be allowed to interfere with customary laws, the law in dispute here being
Section 17 of the Hindu Marriage Act which prohibits a person from marrying another during
the lifetime of their first spouse; bigamy.
The facts of the case are fairly simple, the appellant was an engineer employed with the Public
Works Department, UP who got married in 1934 and a few years later a girl child was born to
them. However after this the appellant’s wife began to miscarry and a medical test revealed that
she was incapable of bearing any more children. The appellant’s family being devout Hindu’s

7|Page
believed that according to the Dharma Shastra’s, salvation was impossible without a son and in
the absence of a male child, a number of religious obligations would remain unfulfilled.
This is when the appellant decided to marry a second time in order to have a son from the
second wife, a decision to which initially the wife consented and then later on changed her mind
and appealed against this decision of her husband to the State Government. The appellant also
went to seek permission from the State Government for a second marriage. To this, the State
Government, on behalf of the wife quoted Section 27 of the Government Servant Conduct
Rules, which stated that you cannot marry another without the consent of your first wife. Around
the same time, the Hindu Marriage Act 1955 came into being Section 17 of which abolished a
second marriage during the lifetime of the first wife.

Hence the permission sought by the appellant was refused, a consequence of which he filed a
writ petition, where he sought to challenge the validity Rule 27 of the Government Servants
Conduct Rules and Section 17 of the Hindu Marriage Act. He challenged these two provisions
on the ground that these infringed upon Article 25 of the Constitution, the Freedom Of
Religion, which allowed him to profess and propagate his religion in the way he deems fit and
prayed that his application be decided in accordance with the personal law laid down by the
Dharma Shastra’s.

FINAL ARGUMENTS:
* The appellant stated that Article 25(b) not only guarantees the freedom of beliefs but
also protects all religious practices in pursuance of such beliefs. He stated that attaining salvation
through the birth of a son was one such belief and the marrying another wife was in pursuance of
one such belief. The bigamy laws put an unjustified restriction on this very right.
* Clause 2 of Article 25 acts as an exception to Clause 1 and the argument was that the
two laws should be in accordance with each other and one clause cannot take away the right
given by the first clause and in case of a conflict between the two clauses, the first one should
prevail.
*The appellant’s final argument was that the operation of Clause 2(b) was to be confined
to matters of social welfare and reform of Hindu Religious Institutions and any law affecting
religious practises of the people was beyond their purview.

ISSUES:
1. Whether or not Section 17 and Rule 27 are in infringement of Article 25 of the
Constitution?
2. Is the bigamy law in contention to the Right to Freedom of Religion promised by the
Constitution?

8|Page
RULE:
Rule 27 will be in force as long as the individual is under the employment roll of the State
Government and shall have to obey the rules, for this there is no remedy. Similarly Article 25
provides citizens with the freedom of religion but at the same time also puts down the restrictions
to such a freedom. Also Section 17 of the Hindu Marriage Act does not allow for bigamy
keeping in mind public welfare and reform with respect to all religious institutions.

ANALYSIS:
The final judgment by the Allahabad High Court held against the appellant, with the court
negating the earlier submissions by the appellant. The court primarily negated these submissions
on three counts.
 With respect of the submission that the bigamy laws were contrary to the Freedom of
Religion, the court held that, it must be made known that the Constitution does not
giver absolute or unrestricted guarantee with respect to the Freedom of Religion.
Plus one section of the Constitutions cannot be read at the expense of the other. That
means that Article 25, Clause 1 cannot be read in isolation, you must read it along
with Clause 2 in order to understand the extent of the Religious Freedom provided to
us. They must be read in conjunction with one another.
 Secondly, the court stated that there was no conflict between the two clauses
contained in Article 25. To understand that they must be read in harmony with one
another. So while Clause 1 provides citizens with the right to profess and propagate
their religion, Clause 2 sets certain restrictions on their right to freedom. Hence the
question of which clause would prevail in case of a conflict does not arise.
 The learned judge stated that the bigamy section of the Hindu Marriage Act was
enacted as a measure of social welfare and reform. He further stated that- as to what
counts as social welfare and reform, the legislature of the country would be the best
and final authority. Moreover nowhere had the appellant contested that the legislation
was not in the capacity of social reform or welfare.
 Finally the Court held that Clause 25(b) was not intended to be limited to any one
religion and reads the same for all religions. If confined to any one religion, it would
go against the secular aspirations enshrined in the constitution.

HELD:
The petition of the appellant is rightly dismissed and the appeal fails. It is accordingly dismissed
with costs.

CRITICAL ANALYSIS:
Critiquing the judgment of the Allahabad High Court, we find that the decision was taken overall
was a well thought out and detailed argumentative holding. Worthy of mention was also Justice

9|Page
Mehrotra’s holding where one of his arguments was that even if the Dharma Shastra’s insisted
on a male child in the family as the only way to attain salvation, the same religious text allowed
adoption and in that way an adopted son would be for all intent and purposes as good as a natural
one. This can be seen as an attempt to bridge the gap that has long been in between modernity
and customary law.
The ruling of the case on the subject of whether one section of the constitution can be an
infringement of another has been dealt with successfully here. The judgment gives clarity on the
a very important fact- that sections of the Constitution cannot be read in isolation, in truth they
must be read in harmony with one another. The idea behind this is that all sections were
constructed in pursuance of law, order and welfare of society and there exists demarcations to the
powers given by the different sections.
Another important aspect that this judgment gave to the law of the country was that it
substantiated the extent of what constitutes as social welfare and reform. The final holding
divided ‘social welfare and reform,’ as mentioned in Article 25 into three divisions, namely:

1. Laws relating to social welfare


2. Laws relating to social reform
3. Laws made for throwing open of Hindu religious institutions to all classes and sections
of Hindu’s.
What this did is that it provided clarity to what had otherwise been vague wordings to that
section.

ii. Priya Bala Ghosh v. Suresh Chandra Ghosh AIR 1971 SC 1153

NOTE: Refer to attached slides.

iii. Bhaurao Lokhande v. State of Maharashtra AIR 1965 SC 1564

NOTE: Refer to attached slides.

iv. S. Nagalingam v. Sivagami AIR 2001 SC 3576

Summary 1

The facts of the case are as follows. The appellant, S. Nagalingam married respondent-
complainant Sivagami on 6.9.1970. They had three children together during the course of this
marriage. The respondent alleged that the appellant started ill-treating her and on many
occasions, she was physically tortured. As a result of ill-treatment and severe torture inflicted by
the appellant as well as his mother, she left her marital home and started living with her parents.
While so, the respondent came to know that the appellant had entered into a marriage with
another woman on 18.6.1984, by name Kasturi, and that the marriage was performed in a
Marriage Hall at Thiruthani. The respondent then filed a criminal complaint before the
Metropolitan Magistrate against the appellant and six others.

10 | P a g e
The case started at a trial court in Madras where the Metropolitan Magistrate gave an order of
acquittal towards the accused. Upon appeal the case reached the High Court where the Single
Judge found the accused guilty under section 494 of the Indian Penal Code. The Single Judge, by
his judgment upheld the acquittal of accused 2-7 but as regards the acquittal of the appellant, the
matter was remitted to the trial court permitting the complainant to adduce evidence regarding
the manner in which the marriage was solemnized. Upon remand the Priest, who is alleged to
have performed the marriage of the appellant with the second accused, Kasturi, on 18.6.1984,
was further examined and the appellant was allowed further cross-examination

The case then reached the Supreme Court upon appeal by the accused.

The issue in this case is if there was a valid marriage between the accused and Kasturi. A
subordinate issue deriving from this is whether the ceremony of Satapadi is essential to constitute
the marriage to be valid.

Certain statutory rules are applicable in this case. The rule for Bigamy is laid down under section
494 of the IPC which states that: Whoever, having a husband or wife living, marries in any case
in which such marriage is void by reason of its taking place during the life of such husband or
wife, shall be punished with imprisonment of either description for a term which may extend to
seven years, and shall also be liable to fine.

Firstly, in order for there to be a case of bigamy, the validity of the second marriage needs to be
proved. This is also what became the issue of this case. The counsel for the appellant contended
that as per the evidence of the Priest who performed the marriage, it is dear that "Saptapadi", an
important ritual which forms part of the marriage ceremony, was not performed and therefore,
there was no valid marriage in accordance with Hindu rites. It is undoubtedly true that the second
marriage should be proved to be a valid marriage according to the personal law of the parties,
though such second marriage is void under Section 17 of the Hindu Marriage Act having been
performed when the earlier marriage is subsisting.

Cases referred to by the bench were In Kanwal Ram and Ors. v. H.P. Administration, Smt. Priya
Bala Ghosh v. Suresh Chandra Ghosh, Lingari Obulamma v. L. Venkata Reddy and Ors, Santi
Deb Berma v. Kanchan Prava Devi. From all these cases the judges arrived to the conclusion that
Satapadi is an important ceremony in any Hindu marriage but it is not an essential ceremony to
make a marriage valid. This reasoning was taking from the amendment in section 7A.

Therefore, the court affirmed the judgment of the Single Judge of the High Court and found the
accused guilty of bigamy under section 494 of the IPC.

Critical analysis

The judgment by the bench of the Supreme Court answered the main issue of the case which
asked the question as to if the ceremony of Satapadi was essential to hold a marriage valid or not.

11 | P a g e
The judges firstly focused on the fact that the validity of the second marriage needs to be proved
in order for the accused to be held liable for bigamy.

After establishing that the personal laws govern the marriage and they will be applicable, the
judges referred to the amendment made by the state of Tamil Nadu to the Hindu marriage laws
which said that Satapadi was not essential as long as the both the parties are in agreement and
can understand a common language. Simple ceremonies like exchanging rings or putting a
garland around the others neck is sufficient enough to constitute a valid marriage. The court also
pointed out that the presence of a priest is not essential, the ceremonies can be performed in the
presence of family, friends or relatives.

All these holdings held valve as precedence to future cases and clearly laid down the
essentialness of certain ceremonies like Satapadi.

Summary 2:

This case presents one of the landmark judgment give by the Supreme Court in the arena of
Family Law primarily because of two reasons. Firstly, in a society where bigamy was widely
practiced, the judiciary in the course of various judgments did recognize bigamy as a cognizable
offence under section 494 of the Indian Penal Code. Secondly, in a constant attempt to codify
and unify the laws, through the course of this judgment, the need to recognize regional customs
and traditions was highly emphasized upon and hence marriages that were performed according
to traditional norms (not necessarily following saptapadi) were also considered valid marriages.

Facts:

 The appellant, S. Nagalingam married respondent – complaintant Sivagami on 6.9.1970.


 Due to the ill treatment and torture inflicted upon the respondent, the respondent left her
marital home and started living with her parents.
 During the time when the respondent was away from the appellant, the appellant entered
into another marriage with another woman named Kasturi on 18.6.1984.

Case History:

 The respondent filed a criminal complaint before the Metropolitan Magistrate but the
accuse was acquitted by the trial court stating that an important ceremony, namely,
"Saptapadi" had not been performed and therefore, the second marriage was not valid
marriage and no offence was committed by the appellant
 In the criminal appeal filled by the respondent at the High Court of Madras, the acquittal
was upheld and the matter was remitted to the trial court. The learned Single Judge at the
High Court held that the parties were governed by section 7 of the HMA and hence the
second marriage was also a valid marriage.

12 | P a g e
 The High Court of Madras held that the appellant had committed the offence – ‘Bigamy’
punishable under Section 494 IPC.

Issue:

 Whether Saptapadi is essential for validating a marriage?


 Is the second marriage a valid marriage according to the section 7A of the Hindu
Marriage Act?
 Is the appellant liable for the offence of bigamy under section 494 of the IPC?

Analysis:

 In the aforesaid decisions rendered by this Court, it has been held that if the parties to the
second marriage perform traditional Hindu form of marriage, "Saptapadi" and "Datta
Homa" are essential ceremonies and without there being these two ceremonies, there
would not be a valid marriage.
 "Saptapadi" was held to be an essential ceremony for a valid marriage only in cases
where it was admitted by the parties that as per the form of marriage applicable to them
that was an essential ceremony. However, according to the personal law applicable to the
parties in the present case, "Saptapadi" was not an essential ceremony for a valid
marriage. But as per the personal law of both the parties, the provisions contained in
Section 7 are applicable to the parties. Hence the traditional rituals performed for
marriage contest that there was a valid marriage on 18.6.1984 between the appellant and
the second accused, Kasturi.
 Therefore, it was proved that the appellant had committed the offence of bigamy as it was
done during the subsistence of his earlier marriage held on 6.9.1970.
 “Saptapadi” is an essential ceremony for a valid marriage only in cases where it is
admitted by the parties that as per the form of marriage applicable to them, ‘Saptapadi’ is
an essential ceremony. But even in cases where marriage is performed according to the
customary laws, with regard to section 7A of HMA, the second marriage is also a valid
marriage even if Saptapadi is not performed.
 An important aspect of the case that was not taken into consideration during the court
proceeding was the question of desertion. The appellant's first marriage had taken place
in 1970 and the second marriage had taken place in the year 1984. For a gap of 14 years,
the respondent had deserted her husband and was living with her parents. Does the
desertion period of 14 years contest a natural end of the marriage? The question finds no
mention in the judgment

Flavia Agnes Perspective:

13 | P a g e
 Until 1955, Hindu marriages were polygamous and hence Hindu men did not attract the
penal provision of section 494 of the IPC.
 There was no necessary requirement to register the contract of marriage, and all
customary forms as well as ‘shastric’ rituals were acknowledged by the act as valid forms
of ‘solemnizing’ a marriage. Due to this, uniformity amongst Hindus was a legal fiction
created by the act.
 Conversion to Islam was the only avenue left for Hindu men to commit bigamy. But an
analysis of the cases reported in law journals established that bigamy of the Hindu male
persists even with the active benevolence of the Supreme Court.
 It was in this atmosphere where the case of S. Nagalingam v. Sivagami, stood out. In a
surrounding where bigamy was highly practiced, the ruling of the case was given in favor
of the respondent and the appellant was charged with the offence of bigamy u/s 494 of
the IPC. Hence, this was a very different verdict given by the Supreme Court where
bigamy was truly recognized as an offence. Therefore, the approach presented by Flavia
Agnes doesn’t hold true in light of this case.

Conclusion:

 Even though Saptapadi was not performed, various other ceremonies such as exchange of
garlands, exchange of rings and tying of thali had taken place in front of several friends
and relatives. Hence according to the personal law of both the parties, the ceremonies
performed by them were sufficient to solemnize the marriage and hence the second
marriage was also valid under section 7A of the HMA.
 Hence, section 7 of the HMA also recognizes the marriage according to the customary
laws to be a valid marriage.
 Also, since both the marriages are valid marriages according to the definition of marriage
in the HMA, the appellant has committed the offence of bigamy and is liable for the same
under section 494 of the IPC.

v. Balusami Reddiar v. Balakrishna Reddiar, AIR 1957 Mad 97

Summary 1:

FACTS:

The case revolves around a dispute with regard to division of a property left behind by the late
Ramaswami Reddiar in his registered will or settlement deed.
The first defendant is the son of the deceased third wife.
The second wife (second defendant) is alive and childless.

14 | P a g e
The first wife pre-deceased him, leaving behind one daughter and her daughter’s daughter (grand
daughter). The grand daughter is the fourth wife and fourth plaintiff.
The marriage between the late Reddiar and the fourth plaintiff resulted in three sons (plaintiffs 1-
3) and three daughters (defendants 3-5) .
Plaintiff claims that the partition of properties was decided secretly without the knowledge of the
plaintiffs in an attempt to defraud plaintiffs 1-3. Property should be divided such that each son
gets one share and the remaining is divided between the two co-widows.

The defendant argued that:


• Marriage between Ramaswami Reddiar and the fourth plaintiff was incestuous
and invalid in law as it is prohibited under the Hindu Law. Sons born of this so –
called marriage: illegitimate.
• At the time of the so-called marriage, Ramaswami was aged 70 years and the
fourth plaintiff aged 12 year. Further, they did not live together as husband and
wife: that the fourth plaintiff was having illegal intimacy with other people-
plaintiffs 1-3 are children of adulterous relation- no claim to the properties of
Ramaswami Reddiar.
• First defendant is the only legitimate son of Ramaswami Reddiar with the second
defendant being the only legitimate widow.

The plaintiff argued that:


 Prima facie the fourth defendant married her own grandfather as per the
prevailing custom of their community. Being a prevailing custom, such
marriage is not invalid.
 Even if plaintiffs 1-3 were begotten illegitimately, decency and morality
dictated that they be given a share of the property as well.

Issues:
3. Whether the marriage of a man with his grand daughter is valid?

a. Which customs should be legally allowed in the modern, progressive India?

RULE:
A custom, which is abhorrent to decency or morality, however long practiced and recognized by
a particular community, can find no kind of enforcement by a Court of law. The marriage
between a man and his daughter's daughter comes within the prohibited degrees of relationship
as laid down in the Mitakshara which is to the effect that a man cannot marry a girl if their
common ancestor being traced through his or father is not beyond the seventh degree in the line

15 | P a g e
of ascent from or her or if their common ancestor being traced through the mother is not beyond
the fifth degree in the line of ascent from him or her.

ANALYSIS:
It is to be noted that the settlement deed, which is a registered, will recognizes the fourth
defendant as the fourth wife of the deceased.

Further, it is also to be noted that incestuous marriage is common practice in neighboring


villages. However, the marriage of the deceased to the fourth defendant is the only such case in
their village.

The court believes that the Subordinate Judge was incorrect in recognizing and legitimizing this
custom.

This court believes that the main characteristics that a custom must have to be legally recognized
are that it should not be opposed to public policy, abhorrent to decency and morality or
inconsistent with practices of practices of good men. This case of incestuous marriage does not
possess any of the above mentioned characteristics.

No civilized society can accept such a custom wherein marriage between a man and his
daughter’s daughter is permitted. Further, this relationship is also comes within the prohibited
degrees of relationship (mitakshara) seventh degree from the fathers’ side and fifth from the
mother.

The Subordinate Judge was of the opinion that such a marriage should be recognized as it was a
recognized custom for such marriages to take place. This was also decided on the basis of the
observations of the court in the case Muthusami Mudaliar v. Masilamani, ILR 33 Mad 342,
which validates incestuous marriages if the caste and community recognize the same.

Hence, a custom, which goes against the morality of society not just today, but also by ancient
texts, can find no enforcement by a court of law. Hence, this relationship is void.

HELD:
Since the marriage of the fourth plaintiff with Ramaswami Reddiar is illegal the plaintiffs are not
entitled to succeed.

Appeal allowed and suit dismissed.

Summary 2:

Balusami Reddiar (minor by guardian) vs. Balakrishna Reddiar and ors.

16 | P a g e
0 This case is one of Hindu law first taken to the Court of the Subordinate Judge of
Tuticorin after which an appeal was filed against the judgment at the Madras High Court.

History and Facts of the CASE-


• T. Ramaswami belonging to a village of Virusampatti expired leaving behind his
property to his huge family. He had 4 wives and his fourth wife is his granddaughter who
has 5 children (3 sons and 2 daughters).
• The 3 children who are the plaintiffs have claimed that the partition was bought about
secretly and to defraud them and not give them their legitimate share in the property.
• They claimed that the property of Ramaswami Reddiar be divided into 5 equal shares
where each of the sons should get a share, the fourth be given to the 4th Plaintiff and the
last being given to the 4 widows.
• The learned judge of Tuticorin passed a decree, which stated that the 4th wife and her
children were not illegitimate and that they had a right to the property.
• The other children and the widows were aggrieved by this and appealed to High Court of
Madras.
Issue-

0 The issue is whether or not the fourth wife and her children are legitimate and have a
right to the share in Ramaswami Reddiar’s property?
0 Whether (as stated by) the Subordinate Judge the custom of marrying one’s daughter’s
daughter among the Reddiars of South India valid or not?
Rule-

0 A custom, which is abhorrent to decency or morality, however long practiced and be


recognized by a particular community, can find no enforcement in a court of law. A
marriage between a man and his daughter’s daughter (grand-daughter) comes within the
prohibited degrees of relationships as laid down by the Mitakshara.
Analysis-

0 The case is one of appeal where the appellants challenge the legitimacy of the fourth wife
and her children and her right to a share in the property of her so-called husband.
0 The fourth wife is the granddaughter of Ramaswami who claims that she was married to
her grandfather according to customs prevalent in their society.
0 The court of law after investigating on such customs was of opinion that relying on oral
evidences of various families in the village stated that the marriage was legal and such
custom can be recognized valid.
0 The judge was inclined to accept the custom because the entire caste of that village had
accepted the marriage as a valid one and treated the parties as members of the caste and
declared the marriage legal.

17 | P a g e
0 Upon taking the matter to appeal and after cross- examination it was found that there
were no other Reddiars who followed such a custom and as per the Manu and
Yajnayalkya, custom is the usage of virtuous men or Sadachara. According to these, a
valid custom is that which should be ancient, certain and reasonable and should not be
opposed to decency and morality. No custom opposed to public policy can be recognized
valid and no matter how much ever practiced can be accepted in a court of law.
0 The alleged custom is revolting to all principles of morality, decency and public policy.
The marriage between a grandfather and granddaughter comes against the prohibited
degrees of relationship as laid down in the Mitakshara. (No common genes or relations
seven degrees from the father’s side and five degrees from the mother’s side. Now it has
been changed to five degrees from the father’s side and three degrees from the mother’s
side). Therefore the marriage is prohibited.

Conclusion-

0 Upon bringing the case to the court of law, the court attempted not to give judicial
recognition to such an obscene practice under guise and pretext of usage or custom.
0 Since the High Court has refused to accept the custom as legal and valid the marriage of
the Ramaswami and his granddaughter is invalid and their children are declared
illegitimate. They are therefore not allowed to succeed. The appeal is allowed.

d. Religious Conversion
i. Lily Thomas v.Union of India AIR 2000 SC 1650

Summary 1:
Issue

 Will the second marriage entered by the spouse after conversion to Islam during the
subsistence of the first marriage would be a valid marriage and whether the apostate
husband would be guilty of the offence of Section 494 of the Indian Penal Code, 1860?
 Is there a breach of fundamental right to life and liberty by forbidding bigamy after
conversion to Islam?
 Whether there is a violation of Article 22 of the Indian Constitution which prevents arrest
and detention in certain cases, if there is a prosecution under Section 494 of the Indian
Penal Code, 1860 in respect of a second marriage under Islamic law?
 Whether the Sarla Mudgal judgement which criminalizes bigamy after conversion to
Islam violates Article 25 of the Indian Constitution which guarantees right to freedom of
religion?

18 | P a g e
 When can a judgement be reviewed by the Supreme Court and whether Jamiat Ulema
Hind and others did the right thing by filing a review petition against the Sarla Mudgal
judgement?
 Is there a necessity for a Uniform Civil Code in India and if the Supreme Court had
issued directions for the codification of various personal laws into a Uniform Civil Code
in the Sarla Mudgal judgement?

Rule
Hindu Marriage Act, 1955:

Section 5: Condition for a Hindu Marriage: A marriage may be solemnized between any two
Hindus, if the following conditions are fulfilled, namely:

(i) neither party has a spouse living at the time of the marriage
Section 11: Nullity of marriage and divorce- Void marriages: Any marriage solemnized after
the commencement of this Act shall be null and void and may, on a petition presented by either
party thereto, against the other party be so declared by a decree of nullity if it contravenes any
one of the conditions specified in clauses (i), (iv) and (v), Section 5

Section 17: Punishment of Bigamy: Any marriage between two Hindus solemnized after the
commencement of this Act is void if at the date of such marriage either party had a husband or
wife living; and the provisions of Sections 494 and 495 of the Indian Penal Code (45 of 1860)
shall apply accordingly.

Indian Penal Code, 1860:

Section 494: Marrying again during lifetime of husband or wife: Whoever, having a husband
or wife living, marries in any case in which such marriage is void by reason of its taking place
during the life of such husband or wife, shall be punished with imprisonment of either
description for a term which may extend to seven years, and shall also be liable to fine.

Constitution of India:
Article 21: Protection of life and personal liberty.
Article 22: Protection against arrest and detention in certain cases.
Article 25-28: Right to freedom of religion.
Article 32: Remedies for enforcement of rights.
Article 44: Uniform civil code for the citizens.

19 | P a g e
Article 137: Review of judgements or orders by the Supreme Court.
Analysis

Mere conversion does not bring to an end, the marital ties unless a decree for divorce on that
ground by the non-convert spouse is obtained from the Court. Till a decree is passed, the
marriage subsists. Therefore, according to Section 11 of the Hindu Marriage Act, 1955, the
marriage entered into by one of the spouses after conversion would be void as Section 11 states a
marriage would be void if it flouts Section 5(i) which states that neither party should have a
spouse living at the time of the marriage. Any other marriage, during the subsistence of first
marriage would constitute an offence of Section 494 read with Section 17 of the Hindu Marriage
Act, 1955, and the person, in spite of his conversion to some other religion, would be liable to be
prosecuted for the offence of bigamy. The Supreme Court also followed that if the first marriage
was solemnised under the Hindu Marriage Act, the ‘husband’ or the ‘wife’, by mere conversion
to another religion, cannot bring to an end, the marital ties already established on account of a
valid marriage having been performed between them. So long as that marriage subsists, another
marriage cannot be performed, not even under any other personal law and on such marriage
being performed, the person would be liable to be prosecuted for the offence of Section 494 of
the Indian Penal Code.

The Supreme Court claimed that Article 21 of the Indian Constitution pledges that no person
shall be deprived of his personal life and liberty except according to the procedure established by
law. The Supreme Court argued that in the present case, they were only interpreting the
prevailing law established by the Sarla Mudgal judgement and were not laying down any new-
fangled law and therefore rebuffing claims of Jamiat Ulema Hind and others that there was
breach of fundamental right to life and liberty due to outlawing of bigamy after conversion to
Islam.

The Supreme Court felt that although Islamic law allows bigamy, Article 22 which prevents
arrest and detention in certain cases, cannot be taken as a defence for prosecution under Section
494 of the Indian Penal Code, 1860. The Supreme Court argued that the defence of Article 22 for
prosecution under Section 494 of the Indian Penal Code, 1860 could have only be taken in
respect of a second marriage under Islamic law if the first marriage was also under the Islamic
law and not if the first marriage was under any other personal law, where there was a prohibition
on contracting a second marriage in the lifetime of the spouse. In the present case, the first
marriage was under the Hindu Marriage Act, 1955 which prohibits bigamy and the second
marriage was under Islamic law and as the first marriage and the second marriage was under
different personal laws therefore Article 22 will not be a defence for prosecution under Section
494 of the Indian Penal Code, 1860 and therefore there was no violation of Article 22 of the
Indian Constitution.
The Supreme Court believed that apart from guaranteeing a right to freedom of religion, Article
25 also requires the beliefs to be exhibited in such a manner which does not infringe the religious

20 | P a g e
rights of others. They argued that as the Hindu Marriage Act, 1955 has established monogamy as
law among Hindus, the Islamic personal laws should not infringe those Hindu laws and therefore
rejected the claim that there was violation of Article 25 of the Indian Constitution. Moreover the
Supreme Court ruled that even under the Islamic law, plurality of marriages is not
unconditionally conferred upon the husband. The condition is that equal treatment should be
meted to all the wives. It would, therefore, be doing injustice to Islamic Law to urge that the
convert is entitled to practice bigamy notwithstanding the continuance of his marriage under the
law to which he belonged before conversion. They felt that it was not possible to treat all the
wives equally after conversion and as the main aim of the laws are to provide equality and justice
to everyone, the Supreme Court believed that the primary aim of the laws were being defeated in
allowing bigamy by conversion to Islam. This was the another reason given by the Supreme
Court for criminalizing bigamy by conversion and still ruling that there is no violation of Article
25 of the Indian Constitution.

The Supreme Court can exercise the power to review a particular judgement to make corrections
in the judgement and not to substitute a view in them. Such powers can be exercised only within
the limits of the statute dealing with the exercise of power. The mere possibility of two views on
the subject is not a ground for review. If there is an infringement of the fundamental rights, then
the Supreme Court can take a different view on the subject. The Supreme Court believed that for
a judgement to be reviewed, the error must be apparent on the face of the judgement and not
such which has to be fished out and searched and as they found no apparent error on the face of
the Sarla Mudgal judgement, they ruled that Jamiat Ulema Hind and others were wrong in filing
a review petition. Moreover just because the religious group had a different take on the issue of
bigamy by conversion, this was no ground for changing the established law. There was also no
infringement of the fundamental rights due to the Sarla Mudgal judgement and hence more
reason for not changing the recognised law.

The Supreme Court believed that in spite of the fact that the attractive value of a Uniform Civil
Code can scarcely be questioned, there ought to be a legitimate social atmosphere constructed by
the elites of the general public and the pioneers who as opposed to increasing personal and
political mileage can transcend and stir the mass about the change and along these lines they felt
that the present time is not a good fit for this issue. They felt that in spite of the fact that a
Uniform Civil Code is exceedingly alluring, if instituted now can turn out to be counter-
profitable to solidarity and trustworthiness of the country. Accordingly it would be inexpedient
and off base to be considering making every one of the laws uniform. Besides if there is any
imperfection in law, it can be cured by the procedure of law at stages. The Supreme Court also
ruled that there is no question of them issuing directions for the codification of various personal
laws into a Uniform Civil Code in the Sarla Mudgal judgement as this matter is beyond their
scope of power. Moreover, it is the duty of the legislature to take effective steps regarding this
issue and they cannot legislate. They also pointed out to the counter affidavit filed on the behalf
of the government of India in the Sarla Mudgal case where it was clearly stated that it would be

21 | P a g e
the government who would take steps regarding this matter and that too only on the desire of the
different communities. Therefore they clarified that there had been no direction for the enactment
of a Common Civil Code in the Sarla Mudgal judgement.

Conclusion

The Supreme Court bench of Sagir Ahmed, J. and Sethi, J. upheld the decision of Sarla Mudgal
case and enforced the same. They held that marriage resulting from conversion to Islam from any
other faith during the existence of previous marriage before conversion is deemed void and the
apostate husband would be liable to be prosecuted for the offence of bigamy and would be
punishable under Section 494 of the Indian Penal Code, 1860 along with Section 17 of the Hindu
Marriage Act, 1955. The Supreme Court dismissed the review petition and the writ petition filed
by the Jamiat Ulema Hind and others as they found no substance in their claim. They also
assured the Jamiat Ulema Hind and the Muslim Personal Law Board that the judiciary or the
Union have no intention of codifying various personal laws into a Uniform Civil Code in the
near future.

Henceforth, the law on this issue reads: Any marriage instituted after conversion to Islam
while a marriage already remains from before conversion, will be void.

Critical Analysis

Every judgement which is on a very prevent matter or custom or practice of the society is always
subjected to many debates in the public forum. Similarly, this judgement too was/is subjected to
many contentions. It is also understandable that it is not possible to keep each and every one
content on an issue when they have got different takes on that issue. In this case, the issue was
that of bigamy by conversion. Some people felt that there is nothing wrong with this practice
whereas many believed including the Supreme Court that this practice should be stopped. As far
as I am concerned, I agree with the ruling of Supreme Court on each and every issue that was
raised during the case in the Supreme Court by different parties.

The first issue was whether mere conversion to different religion brings about an end to the first
marriage which was under different personal law. I believe that a marriage celebrated under one
personal law cannot be dissolved by the application of another personal law to which only one of
the spouses convert. Where a marriage takes place under one personal law, the parties acquire
certain rights and status by that marriage itself. If, therefore, one of the parties is allowed to
dissolve the marriage by adopting and enforcing a new personal law, it would be tantamount to

22 | P a g e
destroying the existing rights of the non-convert spouse. Therefore, the practice of bigamy by
conversion should not be allowed. Even the 227th Report of The 18th Law Commission of India
stated that there should be strict constraints on the issue of bigamy by conversion and
recommended amendments in the Hindu Marriage Act, 1955 to prevent such further practices.

Coming to the issue of Article 21 of the Indian Constitution, even though Article 21 of the Indian
Constitution guarantees protection of personal life and liberty, it is subjected to certain
restrictions and one of those restrictions is the judicial precedents. In this case, the Supreme
Court was just following the judicial precedent which was established in the Sarla Mudgal case
and was not laying down any law of their own. So therefore, there was no violation of Article 21
of any of those persons who were denied bigamy by conversion. As far as infringement of the
fundamental right to freedom of religion is concerned due to the Sarla Mudgal judgement, I
believe that although a person is free to profess any faith or convert to any religion but while
doing so, there should not be an infringement of someone else’s religious beliefs. If Hindus
believe in monogamy, then the Islamic law should not infringe with those laws and if in the
process of protecting the practice of monogamy among Hindus, the bigamy by conversion is
criminalized, then there is no breach of anyone’s right to freedom of religion.

Another issue was regarding the implementation of a Uniform Civil Code. According to me, the
issue of a Uniform Civil Code is a very sensitive and grave issue. Some are in favour of the
implementation of a Uniform Civil Code whereas some opposes it’s implementation. I believe
that at present or in near future, there should not be any tampering with this issue and should be
left as it is i.e. not being implemented. The first reason I believe is that each and every
community has their own sets of personal laws, basically the religious laws. They do not want
any type of interference in those types of laws and if there is an interference, it would prove to be
counter-productive to the security, unity and integrity of the country. I believe that before it is
implemented, everyone should be made aware of the pro and cons of this issue as half
knowledge can be misguiding and dangerous. Moreover, each and every community should be
should be in favour of the same as dissent of one community can be harmful to the unity of the
country. Secondly, Dr B. R. Ambedkar was also of the opinion that there is no obligation upon
any community to do away with their personal laws. If one of the framers of the Indian
Constitution believes so, then there definitely has to be certain substance in that claim. Thirdly,
even if there is a need for a Uniform Civil Code, the request of such should come from all the
communities first and then the government should act. As of now, no such request has come
from any of the community. There are the few reasons which compels me to believe that at
present, the issue of a Uniform Civil Code should be forgotten about. However, one thing that I
would like to flag is that although every community has it’s sovereignty, that sovereignty should
be limited and should not overlap with the principles of other community. I believe this case was
the one where there was an overlapping of the principles of the two community and which
propelled the debate of a need for a Uniform Civil Code. However, I believe that these types of

23 | P a g e
issues are not grave enough which should lead to the implementation of a Uniform Civil Code
and there is judiciary to combat such types of issue and do justice.

ii. Sarla Mudgal v. Union of India 1995 SCC (3) 635

Summary 1:

STATEMENT OF FACTS:

There are two main petitioners to the case. Petitioner one is a registered society ‘Kalyani’
working to help suffering women. Sarla Mudgal is the head of this organization. Another
petitioner is Meena Mathur who was married to Jitender Mathur on February 27, 1978. Three
children were born out of this marriage. In early 1988, the second petitioner found out that her
husband solemnized a second marriage with Sunita Narula aka Fathima after both of them
converted themselves to Islam for the sole purpose of marriage and also have a son out of the
marriage. Jitender Mathur however contended that been converted to Islam, he can have four
wives despite his first wife still being a Hindu.

One of the interesting facts is that Sunita Narula herself is involved in a writ petition 347 of
1990. She contended that Jitender Mathur, due to the influence of his first Hindu wife, gave an
undertaking in April, 1988 that now he has reverted himself to Hinduism and will maintain his
marriage with Meena along with their three children. Her grievances are that she remains to be a
Muslim, without being maintained by her husband and no protection under any personal law.

Geeta Rani is another petitioner under petition 424, 1992 wherein her husband (Pradeep Kumar)
harassed and tortured her and later eloped with another woman and got married to her after
converting to Islam.

Sushmita Ghosh filed under Civil Writ Petition 509, 1992. She was married in 1984 and in 1992
her husband asked for a mutual divorce as he didn’t want to live with her any more. Soon the
husband told her that he has converted to Islam and will soon be marrying Vinita Gupta. The
petitioner prayed for restraining her husband from a second marriage.

ISSUES :

I. Does India need a Uniform Civil Code for all Indians?


II. Whether a Hindu husband, married under Hindu law, solemnize a second marriage after
converting into Islam?
III. Whether such a marriage (as mention in part II.) without dissolving the first marriage is
valid in case the first wife remains to be a Hindu?
IV. Whether the apostate husband be liable under Section 494 of IPC?

RULES:

24 | P a g e
INDIAN PENAL CODE, 1860:

 Section 494- marrying again during the lifetime of the husband or wife. As per this
section, a second marriage is void done during the lifetime of a husband or a wife.

CONSTITUTION OF INDIA:

 Article 14, 25, 26, 27, 28 and 32


 Importantly Article 44- Uniform Civil Code for all the citizens of India.

HINDU MARRIAGE ACT, 1955:

 Section 11- VOID MARRIAGES


 Section 14- DIVORCE
 Section 15- WHEN CAN DIVORCED GET MARRIED

THE HINDU SUCCESSION ACT, 1956

HINDU MINORITY AND GUARDIANSHIP ACT, 1956

HINDU ADOPTION AND MAINTENANCE ACT, 1956

ANALYSIS:

The constitution of India provides for a Uniform Civil Code for all its citizens which is a goal to
be achieved. In the case of Sarla Mudgal v. Union of India, the court has worked towards
removing the ‘inter-personal conflicts of law’ which have arisen due to the lack of a uniform
system.

Justice Kuldip Singh in the present case finds no reason to hold back the Uniform Civil Code.
So far all the governments have failed in achieving a unified system of personal laws in the
country. The reasons are seen to be obvious. The Hindu law has been codified which covers
almost 80% of the citizens; law makers find it un-reasonable to now introduce a “uniform civil
code”.

Until the government doesn’t employ a uniform civil code, whenever a Hindu would want to do
a second marriage without dissolving the first, he will resort to converting into Islam because
Islam allows for bigamy whereas Hinduism restricts to monogamy. The doctrine of
indissolubility of marriage under the Hindu law did not recognize the consequences of one of the
spouses converting into another religion and getting married.

In the cases of Re Ram Kumari and Nandi alias Zainal V. The Crown, the spouses were
charged under Section 494 of IPC for converting themselves to Islam for the purpose of bigamy.

25 | P a g e
The court sited various cases are principles that laid down a basic understanding to which I agree
as a student was that merely conversion to a religion in order to enjoy a privilege under its
personal laws does not abstain one from the penal consequences of breaching the laws of his/her
previously governing personal law.

When a marriage takes place under the Hindu personal law, certain rights and duties are
created and also the individuals acquire a status as per the Hindu marriage. When one
spouse tries to change their religion for marrying someone else without dissolving the first,
it attacks the status and rights of the other spouse who is still a Hindu.

Also the second marriage of an apostate husband would be in violation of natural justice. It is
against the principles of justice, equity and good conscience.

Justice R.M. Sahai gave a concurring judgment where he said that the major problem was that
most of the people were converting to Islam merely for the purpose of excusing themselves from
the penal consequences of bigamy. But no religion supports deliberate distortions. He finally
concluded that a unified code is imperative both for the protection of the oppressed and
maintenance of unity and solidarity of the country.

CONCLUSION:

Marriage is the foundation of the family and in turn of the society without which no civilization
can exist. The personal laws should not be allowed to be employed and subjugated for worldly
gains. Thus, if a person wishes to engage himself into a second marriage by converting the
religion while also not dissolving the first marriage, then the marriage will be valid only if
his/her original personal laws allow for the same.

“The second marriage would be void as per the provisions of Section 494 of the IPC and the
apostate husband would be guilty for the offense of bigamy” the court added. The rule used in
Sarla Mudgal V. Union of India was upheld in the case of Lily Thomas V. Union of India.

Summary 2:

FACTS:

These are four petitions under Article 32 of the Constitution of India. Basically, all the four
petitions have almost the same facts. The Hindu husband, without dissolving the first marriage
converts into Islam for the purpose of remarrying another woman.

The first petitioner, Meena Mathur was married to Jitender. Afterwards, the petitioner was
shocked to learn that her husband had solemnised second marriage with one Sunita Narula alias
Fathima. The marriage was solemnised after they converted themselves to Islam.

26 | P a g e
The second petition was filed by Fathima herself. She states that after marrying her, Jitender
decided to revert back to Hinduism and had agreed to maintain his first wife and three children.
Her grievance is that she continues to be Muslim, not being maintained by her husband and has
no protection under either of the personal laws.

Geeta Rani, the third petitioner was married to Pradeep Kumar. Afterwards, the petitioner learnt
that Pradeep Kumar ran away with one Deepa and after conversion to Islam married her.

Sushmita Ghosh is another lady who was married to G.C. Ghosh. The husband finally told the
petitioner that he had embraced Islam and would soon marry one Vinita Gupta.

JUDGEMENT

Does India need a Uniform Civil Code for all its citizens?

Justice Kuldip has stated that “when more than 80% of the citizens have already been brought
under the codified personal law there is no justification whatsoever to keep in abeyance, any
more, the introduction of ‘uniform civil code’ for all citizens in the territory of India.” He also
justifies the introduction of a unified personal law for all Indians by stating that a Hindu
husband, wants to enter into a second marriage while the first marriage still continues, can be
allured to convert into Islam as it provides room for bigamy and a Muslim can keep four wives at
a time. But since Hindu law only permits monogamy, Hindu husband embraces Islam to
circumvent the provisions of the Hindu law and to escape from penal consequences.

Whether a Hindu husband, married under Hindu law, by embracing Islam, can solemnize
second marriage? Whether such a marriage without having the first marriage dissolved
under law, would be a valid marriage where the first wife who continues to be Hindu?

When a marriage takes place under Hindu personal law, some rights and duties are created by
way of it and the parties acquire certain status under the laws governing the Hindu Marriage. If
one spouse tries to end the marriage by converting to another religion without dissolving the
marriage then it will amount to destruction of the rights and status of the other spouse who is still
a Hindu. It is, therefore, maintained that under the Hindu Personal Law as it existed prior to its
codification in 1955, a Hindu marriage continued to subsist even after one of the spouses
converted to Islam. There was no automatic dissolution of the marriage. Also, A Hindu marriage
solemnized under the Act can only be dissolved on any of the grounds specified under the Act.

Whether the apostate husband would be guilty of the offence under Section494 of the
Indian Penal Code (IPC)

A marriage cannot be dissolved except under the provisions laid down in Section 13 of Hindu
Marriage Act. The second marriage of an apostate would, therefore, be illegal marriage as his
wife who married him under the Act and continues to be a Hindu. Between the apostate and his

27 | P a g e
Hindu wife the second marriage is in violation of the provisions of the Act and as such would be
under Section 494 of Indian Penal Code.

The primary components of the section are:

1.
Having a husband or a wife living;
2.
Marries in any case;
3.
In which such marriage is void;
4.
By reason of its taking place during the life of such husband or wife.
All the four aspects of Section 494 IPC are satisfied in the case of a Hindu husband who
marries for the second time after conversion to Islam with his first wife living. The said
marriage is void by reason of its taking place during the life of the first wife. Therefore,
the second marriage of a Hindu husband after his conversion to Islam is a void marriage
in terms of Section 494 IPC.
CRITICAL ANALYSIS

According to me, Justice Sahai has given some of the most valuable suggestions. He hasstated
that there can be two sides of the same coin and there also can be an opposing argument stated
against the Uniform Civil Code. He states that its application of such a code can cause
dissatisfaction and disintegration among the masses of the Indian society rather than serve as a
common umbrella to promote homogeneity and national solidarity.

He also states that many Hindus change their religion and convert to Islam only for
purposes of escaping the consequences of bigamy. Therefore, to curb these incidents, “the first
step should be to rationalise the personal law of the minorities to develop religious and cultural
amity. The Government would be well advised to entrust the responsibility to the Law
Commission which may in consultation with Minorities Commission examine the matter and
bring about the comprehensive legislation in keeping with modern day concept of human rights
for women.”

Flavia Agnes has stated that the current legal position can be summarised as follows: A Hindu
husband can live in a polygamous marriage with impunity so long as he does not perform either
one of the ceremonies of homa or saptapadi. Since most communities do not even consider these
formalities as essential, non-performance of these ceremonies does not pose any obstacle on the
path of a Hindu husband desirous of committing bigamy.

She has basically argued that the current legal position helps the husbands to circumvent the
provisions of S.494 of IPC since the courts have refused to consider the second marriage legal.
The courts have acquitted the husbands if they haven’t performed the specific ceremonies in their
marriage which has set a wrong precedent and has encouraged the husbands to engage in bigamy
and evade the law.

28 | P a g e
While the Hindu male is granted a clear advantage, the disadvantage is suffered by two sets of
women. The first wife stands the risk of invalidating her own marriage in a prosecution for
bigamy. If she cannot prove the valid ceremonies of her marriage, the court will validate the
second marriage and thus she will not only lose out in the bigamy proceedings but will lose her
economic rights in matrimonial proceedings.

The disadvantages suffered by the second wife have not received any attention by the advocates
of monogamy. When a woman approaches the court for maintenance a routine ploy adopted by
husbands is to question the validity of her marriage, by stating that he either has an earlier
marriage subsisting or that the woman is a mistress. Then it is left to her to prove the validity of
her marriage both in terms of ceremonies as well as its monogamous character. If the husband
can prove that he has an earlier marriage subsists, the second wife will have no legal recourse
left. In the above case, one of the petitions was filed by the second wife, whose husband induced
her to convert and marry him and subsequently due to pressure from the first wife reconverted to
Hinduism and lived with the first wife. The woman has a two-year old son out of this marriage.
While the Supreme Court has stated that the uniform code is a remedy for all ills, it has not
commented upon the status of this child.

She goes on to state that Justice Sahai has made a valuable suggestion, which is that the
government should consider the feasibility of appointing a committee to enact a Conversion of
Religion Act to check the abuse of religion by any person. The law may provide that every
citizen who changes his religion cannot marry another wife unless he divorces his first wife. The
provision should be made applicable to every person whether Hindu, Muslim, Christian, Sikh,
Jain or Buddhist. Such an act could effectively deal with the problem of conversion and bigamy.

According to me, The Uniform Civil Code will be a good medium to curb the incidents of
conversion for the purpose of remarriage. But it won't be able to successfully serve its purpose
unless and until the above mentioned loopholes are not filled either by the higher courts or by the
parliament by way of a new legislation. Certain laws need to be given a liberal interpretation to
uphold the sanctity and validity of marriage and monogamy under Hindu law. For example, the
ceremonies of homa and saptapadi do not gurantee a valid marriage in all the cultures and all of
them either should have a strict uniform law or different customs of all these cultures need to be
give sufficient legal recognition.

CONCLUSION

“Marriage is the very foundation of the civilized society. The relation once formed, the law steps
in and binds the parties to various obligations and liabilities there under. Marriage is an
institution in the maintenance of which the public at large is deeply interested. It is the
foundation of the family and in turn of the society without which no civilization can exist.”

29 | P a g e
The personal laws should not be allowed to be manipulated and exploited for worldly gains and
carnal pleasures. Thus, if a person wishes to enter second marriage, after converting to another
personal law and without dissolving his/her first marriage, such second marriage must be held to
be valid only if his/ her original personal law allows such second marriage. For instance, if a
Hindu enters into a second marriage after converting to Islam, but without dissolving his first
marriage, the he should be held liable for bigamy because his original personal law does not
allow polygamy. Similarly, if a married Muslim converts to Hindu religion, without dissolving
his first marriage, and enters into a second marriage he should not be held liable for bigamy
because his original personal law allows polygamy, though capacity to do justice between co-
wives is the condition precedent. The Supreme Court has reiterated that the second marriage of a
Hindu man after conversion to Islam without having his first marriage dissolved under the law
would be invalid. This decision of the court in Sarla Mudgal case was upheld in Lily Thomas v.
Union of India.

e. Caste:
i. Smt. Kastoori Devi v. Chiranji Lal AIR 1960 All 446

Summary 1:

Facts:

 This was an intercaste marriage between a Brahman husband and a Thakur wife.

 Smt.Kastoori Devi (plaintiff) was married to Chiranji Lal (defendant) for 20 years. At the
time of her marriage, Kastoori Devi was a widow.

 But in 1954, after twenty years, Chiranji Lal took another wife and turned the plaintiff
out of the house.

 He even refused to maintain her i.e. provide her with the basic amenities of life.

 Hence plaintiff was forced to file for maintenance against the defendant under the Code
of Criminal Procedure.

 Later, on 29th April 1954, there was a gathering of the village panchayat, resulting in a
settlement and a written agreement under which the defendant agreed to pay the plaintiff
a maintenance allowance of Rs.30 a month

 But he didn’t honour the agreement and even refused point blank to pay the plaintiff .
Hence the plaintiff was forced to file a suit to enforce the agreement and claim arrears of
maintenance due to her.

 Defendant argued that the plaintiff’s application was baseless and that he had signed the
agreement under coercion from the plaintiff and one Nathoo Singh. He alleged that the

30 | P a g e
Sub-Inspector in charge of Police Station Tanda tried to persuade him to provide
accommodation and maintenance for the plaintiff.

Issue(s):

1. Is the marriage between a higher caste husband and lower caste wife valid?
2. Can Smt.Kastoori Devi successfully claim maintenance from Chiranji Lal?

Rule(s):

1. The Hindu Marriages Validity Act, 1949: Which laid down that no marriage between
Hindus would be deemed to be invalid or 'ever to have been invalid by reason only of the
fact that the parties thereto belong to different religions, caste, sub-castes or sex.
2. The Hindu Marriage Act No. 25 of 1955. Section 29(1) of this Act provides that" "a
marriage solemnized between Hindus before commencement of this Act which is
otherwise valid, shall not be deemed to be invalid or ever to have been invalid by reason
only of the fact that the parties "thereto belong to the same Cotra or Pravara or belong to
different religion, castes or sub-divisions of the same caste,"

Analysis:

The learned Judge after hearing the evidence of the parties disbelieved the defendants' allegation
that he had signed the agreement for maintenance under coercion, but he dismissed the suit on
two grounds. First, he held that a marriage between a Brahman husband and a Thakur wife is
invalid being an inter-caste marriage. He took the view that "parties to a Hindu marriage must
belong to the same caste, a marriage between persons who do not belong to the same caste, is
invalid unless it is sanctioned by custom." Secondly, he held that the marriage ceremony alleged
by the plaintiff had not been properly solemnised. To quote his own words "There is no proof
that Saptapadi, Such a marriage cannot be called a valid marriage and the plaintiff cannot be held
to be the legally wedded wife of the defendant". Accordingly he dismissed the plaintiffs' suit for
maintenance with costs.

Mr. S.B.L. Gaur on behalf of the respondent raised a preliminary objection that the suit was not
maintainable in a court of Small Causes. He relied on the second schedule to the Provincial
Small Cause Courts Act which contains a list of suits excepted from the cognizance of a Court of
Small Causes. Entry No. 38 of this list relates to "a suit relating to maintenance''. The effect of
this appears to be that if a wife files a suit to establish her claim for maintenance against the
husband, the suit would not lie in a Court of Small Causes. But if there is a pre-existing
agreement between the husband and the wife providing for maintenance, or if the wife obtained a
decree for maintenance, she can file a suit for recovery of arrears of maintenance on the basis of
such a decree or written agreement.

31 | P a g e
Learned counsel for the plaintiff-applicant Mr. Gayanendra Kumar, who argued this case with
ability and with commendable brevity, impugned the legality of the decision of the trial court on
two grounds. First, he argued that its finding that the marriage of the plaintiff with Chiranjilal
was invalid because of being an inter-caste marriage is erroneous. Secondly, he urged that his
finding that this marriage had not been properly solemnised and was therefore invalid, is also
wrong in law.

It would not be out of place to point out that even before the passing of this Act; Hindu Law
recognised the institution known as “Anuloma Marriage” which was sanctioned by the Smritis.
According to the Smritis it is open to a high caste male to marry a lower caste female. Such a
marriage was known as Anuloma. Several High Courts in India have upheld anuloma marriage
as valid. Later on, in another decision of the Bombay High Court, in the case of Bai Gulab v.
Jiwanlal Hanlal, Sir Norman Macleod, Kt. Chief Justice and Shah, J. held that the institution of
Anuloma marriage could not become illegal or prohibited by law simply because it had become
obsolete.

The finding that the marriage was not properly solemnized because there was no evidence of the
vital ceremony of Saptapadi is also wrong. To establish a valid marriage, there must be evidence
of parties who were present at the marriage ceremony and who should state that they saw the
marriage being performed. Thus onus was fully discharged by wife in the present case who went
into the witness box and stated that a marriage ceremony took place between herself and the
defendant Chairanjilal. She even named the priest or Purohit who performed the ceremony. The
priest himself gave evidence and stated that he had performed it. The other witnesses, who were
present at the marriage ceremony, also gave evidence.

Critical Analysis:

Referring to the article on Hindu Men, Monogamy and Uniform Civil Code by Flavia Agnes
and to the Priya Bala Ghosh v. Suresh Chandra Ghosh case, it is acknowledged that for a
Hindu marriage to be deemed as valid, it is necessary that the rituals of Homa and Saptapadi are
necessary. In the current case, the learned judge of the trial court had at first dismissed the
plaintiff’s (Smt. Kastoori Devi’s) appeal for maintenance on the grounds that there was no proof
that Saptapadi had been performed. But, in the end it was held that only the evidence of the
marriage from the witnesses present at the marriage ceremony was considered as important for
judging whether a marriage was valid or not.

Conclusion:

For these reasons the plaintiff's case was held that she was lawfully married to defendant
Chiranji Lal, that the latter turned her out after he had married another woman, that there was a

32 | P a g e
Panchayat in which defendant Chiranji Lal agreed to pay her a maintenance allowance of Rs.
30/- per mensem and that subsequently he refused to pay is fully proved.

This revision is allowed and the plaintiff's suit for maintenance is decreed. The defendant
respondent shall pay the costs of the plaintiff in this Court as well as in the Court below.

f. Minority:
i. P. Venkataramana v. State AIR 1977 AP 43

Summary 1:

Facts: The first respondent in this case, who is the wife, filed a case against her husband
(Ist petitioner) and ten others alleging that her husband has committed an offence,
punishable under section 494 of the Indian penal code and that the others have committed
an offence punishable under section 494 read with section 109 of IPC. At the time of
marriage, in the year 1959, the petitioner (husband) was 13 years old and the respondent
(wife) 9 years of age. The petitioner (husband) asserted that in view of the decision of the
Division Bench of the Court in P.A. Saramma v. G. Ganapatulu , the marriage between
him and the Ist respondent was void ab initio and no marriage in the eyes of law and
hence the action of the Ist petitioner in marrying a girl did not amount to an offence
punishable under Section 494.

Issue: Whether a Hindu Marriage governed by the provisions of the Hindu Marriage Act,
1955 where the parties to the marriage or either of them are below their respective ages as
set out in Clause (iii) of Section 5 of the Hindu Marriage Act, is void ab initio and is no
marriage in the eyes of law.

Rule: Section 5- Conditions for a Hindu Marriage : A marriage may be solemnized


between any two Hindus, if the following conditions are fulfilled, namely:—

(i) Neither party has a spouse living at the time of the marriage;

(ii) At the time of the marriage, neither party—


(a) is incapable of giving a valid consent to it in consequence of unsoundness of mind; or
(b) Though capable of giving a valid consent, has been suffering from mental disorder of
such a kind or to such an extent as to be unfit for marriage and the procreation of
children; or
(c) Has been subject to recurrent attacks of insanity

iii) The bridegroom has completed the age of [eighteen years] and the bride, the age

33 | P a g e
of [fifteen years] at the time of the marriage;

iv)The parties are not within the degrees of prohibited relationship unless the custom or
usage governing each of them permits of a marriage between the two;

(v) The parties are not sapindas of each other, unless the custom or usage governing each
of them permits of a marriage between the two.

Analysis: In P.A. Saramma v. G. Ganapatulu 1975, a Division Bench of this High


Court held that a Marriage between the bridegroom and the bride, if their ages do not
satisfy the requirements of clause (iii) of Section 5, cannot be solemnized as it is
prohibited under clause (iii) of Section 5. Regarding clause (iii) the marriage is neither
void nor voidable as minority of an individual getting married operates only as a bar to
his or her claiming contractual obligations. A Hindu marriage is a religious ceremony and
it is completed by the said rituals or Saptapadi. A person may be a minor or of unsound
mind and yet the marriage may be duly solemnized. A marriage solemnized in
contravention of clause (iii) is neither void nor voidable. In view of the scheme of the
Act, we have to examine as to what are the consequences of violation of clause (iii) since
the legislature, in terms has not provided for what is to happen in case of violation of
clause (iii) of Section 5. Neither in Section 11 nor in Section 12 is there any provision for
what is to happen if a marriage is solemnized in violation of the provisions of clause (iii)
of Section 5. This analysis of the different provisions of the Hindu Marriage Act clearly
brings out the fact that the Legislature itself has made a distinction between contravention
of one or the other clauses of Section 5 and such contravention is to be visited with
different consequences. In case of contravention of some clauses, the marriage is null and
void and in case of contravention of some other clauses, it becomes voidable and in case
of contravention of another clause, it is voidable if the consent of guardian is vitiated by
force or fraud; but the Legislature in terms has not provided except by way of punishment
in Section 18 for violation of Clause (iii) of Section 5. Therefore, it is not possible to read
the different clauses of Sec. 5 as laying down conditions precedent. It may be pointed out
that, under the Child Marriage Restraint Act, 1929 which was in force prior to the
enactment of the Hindu Marriage Act 1955, the legal position was that though the persons
connected with the solemnization of a marriage in contravention of the provisions of the
Child Marriage Restraint Act were liable for punishment, the marriage itself was not
rendered void or null and void. Since the reliance for the case was put on ‘Saramma Vs
G.Ganapatulu - It was felt that the view taken by the division bench regarding the same
was not in accordance with the provisions of the Hindu marriage Act. It was held that the
decision of the Division Bench of this High Court in P. A. Saramma v. G. Ganapatulu,
does not lay down the correct law and it must be held that any marriage solemnized in
contravention of clause (iii) of S. 5 is neither void nor voidable. The grounds for judicial

34 | P a g e
separation, nullity of marriage and divorce are given in S. 10, 11 and 13 of the Act
respectively. The contravention of S. 5(Iii) of the Act does not admittedly find any
mention in any of these three sections.The provisions of the Hindu Marriage Act were
extensively amended in 1976, by the Marriage Laws (Amendment) Act, 1976, the
provisions of clause (iii) of S. 5 have not been interfered with. S. 13 of the Hindu
Marriage Act. Which provides for different grounds on which decree for dissolution of
marriage can be granted, has been amended and under sub-section (2) of S. 13, a new
clause (iv) has been inserted so that after the amendment, a wife may present a petition
for dissolution of her marriage by a decree of divorce on the ground that her marriage
(whether consummated or not) was solemnized before she attained the age of fifteen
years and she has repudiated the marriage after attaining that age but before attaining the
age of eighteen years. It was held that the decision of the Division Bench of this High
Court in P. A. Saramma v. G. Ganapatulu, does not lay down the correct law and it must
be held that any marriage solemnized in contravention of clause (iii) of S. 5 is neither
void nor voidable. The criminal petition regarding bigamy was dismissed.

Conclusion: The marriage between the two parties was declared void. Since the marriage
was solemnized 1959 when the bridegroom was 13 years of age and the bride was 9 years
of age

Summary 2:

Decided on: 9th August 1976, in the HC of Andhra Pradesh


Hon’ble Judges: B.J. Divan, C.J., Alladi Kuppuswami and K.A. Muktadar, JJ.

FACTS:
In the above-mentioned case, the facts of the case are that the wife filed a complaint against her
husband, charging him of bigamy. The wife, who was the first respondent, alleged that her
husband, who is Petitioner no. 1, had committed an offence under Section 494 of the Indian
Penal Code. The ten other people, who were petitioner no. 2, were convicted for an offence
punishable under Section 494 read with Section 109, I.P.C.

The wife accused her husband of bigamy under Section 494 of the Indian Penal Code, claiming
that he had committed the offence by marrying another girl, during her lifetime. The ten others
were charged of abetment punishment under Section 109 of the Indian Penal Code.

The Trial Court held the marriage between the 1st respondent and the 1st petitioner to be legal and
under that ruling, convicted the husband of bigamy. Both the petitioner filed appeals and the
appellate court confirmed the convictions of both the petitioners, but modified their sentences to
that of payment of Rs. 200 and in default of payment of fine, each of the petitioners was

35 | P a g e
sentenced to undergo imprisonment for one month. Against their convictions and sentences, the
petitioners came by the way of revision to the High Court.

The High Court in the revision application in P.A. Saramma vs. G. Ganapatulu stated that a
marriage that is in contravention of clause (iii) of Section 5 of the Hindu Marriage Act is void ab
initio and no marriage in the eyes of law.

The petitioner argued that at the time of the marriage i.e. in 1959, he was 13 years of age and his
wife was of 9 years of age. He contends that in light of the decision of the court in P.A. Saramma
vs G. Ganapatulu, the marriage between him and the respondent was no marriage in the eyes of
law and hence, void ab initio. The matter was referred to the larger bench as it was felt that it
wasn’t in accordance with the provisions of HMA.

ISSUE:
Whether a Hindu marriage governed by the provisions of the Hindu Marriage Act, 1955, where
the parties to the marriage or either of them are below the prescribed age set out in clause (iii) of
Section 5, is void ab initio?

RULE:
Following are the sections that are relevant to this case:

Hindu Marriage Act, 1955


1. Section 5, clause (iii) which lays down the conditions for a Hindu marriage and it is in
these terms: “A marriage may be solemnized between any 2 Hindus, if the following
conditions are fulfilled namely,
(iii) The bridegroom has completed the age of 18 years and the bride age of 15 years at the
time of the marriage.

2. Section 13, sub-section 2, clause (iv), states that a wife may also present a petition for
the dissolution of her marriage by a decree of divorce on the ground that her marriage
was solemnized before she attained the age of 15 years and she has repudiated the
marriage after attaining that age but before attaining the age of 18 years.

3. Section 18 clause (a), states that every person who procures a marriage to be solemnized
under this act in contravention of the condition specified in clause (iii) of Sec 5, with simple
imprisonment which may extend to fifteen days, or with fine which may extend to one
thousand rupees, or both.

Indian Penal Code, 1860

36 | P a g e
1. Section 494, states that whoever, having a husband or wife living, marries in any case
in which such marriage is void by reason of its taking place during the life of such
husband or wife, shall be punished with imprisonment of either description for a term
which may extend to seven years, and shall also be liable to fine.

4. Section 109 refers to the punishment of abetment if the act abetted is committed in
consequence and is punished with the punishment provided for the offence.

ANALYSIS:
The analysis provided by the court looked into Section 11 and 12 of the Hindu Marriage Act and
marked the distinctions between contraventions of clauses of Section 5, which lays down the
condition precedent for solemnization of marriage.

It was analyzed in this case, that Section 11 only indicates when can a marriage be rendered void
ab initio i.e. null and void, whereas, Section 12 only indicates when a marriage is voidable and
liable to be annulled by a decree of nullity.

Contravention of clauses (i), (iv) and (v) of Section 5 marks a marriage null and void i.e. void ab
initio, under Section 11 of the Hindu Marriage Act. On the other hand, marriages contravening to
clauses (ii) and (vi) of Section 5, provided the marriage to be voidable an dliable to be annulled
by the decree of nullity.

However, it was nowhere stated either in Section 11 or 12 as to what is to happen if the marriage
is conducted in violation of clause (iii) of Section 5 of the Hindu Marriage Act, which is
regarding the ages of the parties to marriage. The only provision mentioned could be found in
Section 18 of the Hindu Marriage Act, which provides punishment for breaching of certain
conditions laid down for bigamy.

The detailed analysis by the court forces us to look up at the status of marriages of minor in
India, after this case. It is one of the most performed practices in India and has attracted many
laws to be made in order to avoid this practice.

The Child Marriage Restraint Act, enacted in 1940 restricted the practices of child marriages.
The Prohibition of Child Marriage Act (PCMA), enacted in 2007, focused mainly on
fix the shortcomings of the Child Marriage Restraint Act and worked on preventing the
child marriage. It made the child marriages voidable at the option of contracting parties
who were a child at the time of the marriage. The act also mandates the return of all
valuables, money, ornaments and gifts in marriage on declaration of the nullity order by
the court. All the children born out of the wedlock are considered legitimate and in case of

37 | P a g e
custody, the best interest of the children is considered. Even the punishment for indulging
into the practice of child marriage was increased.
The Protection of Children from Sexual Offences Act (POCSO) enacted in 2012 aimed at
protecting children from offences of sexual assault, sexual harassment and pornography. Even the
abetment of children sexual abuse is considered as an offense. The act has been criticized, as its
provisions seem to criminalize consensual sexual intercourse between two people below the age
of 18.

CONCLUSION:
The High Court of Andhra Pradesh reached to the conclusion that a marriage in contravention to
clause (iii) of Section 5 of the Hindu Marriage Act is not void. It was held by the HC in the case
that:
1. The rule laid down in P.A. Saramma vs. G. Ganapatulu doesn’t lay down the correct law.
2. Any marriage solemnized in contravention of clause (iii) of Section 5 of HMA is neither
void nor voidable.
3. The only consequence being that the persons concerned with such a marriage is liable for
punishment under Section 18 of the HMA.
4. If the requirements of clause (iv) of sub section (2) of Sec 13 [as inserted by Marriage
Laws (Amendment) Act, 1976] are satisfied, then at the instance of the bride, a decree of
divorce can be granted.

Also, the marriage between parties was held void, since the marriage was solemnized in 1959,
when the bridegroom was 13 years of age and the bride was 9 years of age. The husband and ten
others were acquitted of the charge and were not punished for the offence of bigamy.

i. Ravi Kumar v. The State 124 (2005) DLT 1

Summary 1:

ISSUE

 Whether because of the minority of the spouse, the marriage entered into is illegal and
void-ab-initio?
 Whether young lady getting married having come to the age of discretion but not attained
majority, can be sent in protective custody to a Remand Home against her want?
 Whether in a Habeas Corpus request, the Court ought to entertain the prayer for suppress
of the pending criminal proceedings for abduction, kidnapping etc., in exercise of
jurisdiction?
 Whether in the present case, the FIR & Criminal Proceedings compatible thereto for
kidnapping are liable to be subdued?

38 | P a g e
RULE

The relevant laws for the above case are as follows :-

 Section 5 (iii) of The Hindu Marriage Act, 1955 : Conditions for a Hindu marriage - A
marriage may be solemnized between any two Hindus, if the bridegroom has completed
the age of twenty-one years and the bride, the age of eighteen years at the time of the
marriage.
 Section 11 of The Hindu Marriage Act, 1955 : Void marriage - Any marriage
solemnized after the commencement of this Act shall be null and void and may, on a
petition presented by either party thereto [against the other party], be so declared by a
decree of nullity if it contravenes any one of the conditions specified in clauses (i), (iv),
(v) of section 5.
 Section 12 of The Hindu Marriage Act, 1955 : Voidable marriage
 Section 18 (a) of The Hindu Marriage Act, 1955 : In the case of a contravention of the
condition specified in clause (iii) of section 5, with rigorous imprisonment which may
extend to two years or with fine which may extend to one lakh rupees, or with both.
 Article 226 and Article 227 of The Constitution of India, 1949 : Talks about the
jurisdiction of various Indian High Courts.

ANALYSIS

 A marriage solemnized in contradiction of the age said in Section 5 (iii) of The Hindu
Marriage Act, 1955, with respect to the age restriction, made it only culpable under
Section 18 and the marriage solemnized would stay valid, enforceable and recognizable
in Courts of law.
 There was no procurement of law, which allowed minors to be kept in protective homes
against their want. Regardless of the possibility that the minor was about 15 years of age,
her wishes ought to be determined before putting her in the authority of any individual or
institution.
 The powers under Articles 226 and 227 are wide and comprehensive subject to self-
imposed restrictions.

CONCLUSION

 The marriage was neither void nor illegal on account of the spouse being less than 18
years of age and being over 15 years of age.
 The question of contravention under section 18 is not before the bench.
 The minor girl cannot be coordinated to be kept in a Remand House against her will.

39 | P a g e
 The case was observed to be fit for exercise of discretionary power by the court as
relegating the parties to separate proceedings would tantamount to denial of justice.
 Criminal charges were subdued .
 To waive off the expense that was imposed on Ravi Kumar.

g. Sapinda:
i. Navalkar v. Meena Arun Navalkar AIR 2006 Bom 342

Summary 1:

The Case of Arun Laxmanrao Navalkar v. Meena Arun Navalkar (hereinafter, Navalkar v.
Navalkar) was decided by the Bombay High Court on 12th April 2006 by a bench consisting of
Judge R Khandeparkar and Judge R Dalvi. The legal issue in this case was whether the marriage
between a husband and wife was a sapinda relationship and was thus void, and whether the wife
had a right to the matrimonial home which was her husband’s joint family property. The High
Court rightly held that the marriage of the parties was void and that, by virtue of this, there
would be no question of the wife exercising any right of residence in their matrimonial home.

The parties in this case were husband and wife. Their marital relationship began in January, 1981
and ended in July, 1981 when they separated. During their marriage, they lived in Girgaum at
their matrimonial home: a family bungalow. It was the husband’s joint family property in which
they lived with his 2 brothers and 2 cousins. The wife separated and sued the husband for
maintenance. The husband sued her for nullity of their marriage under Section 11 of the Hindu
Marriage Act on the ground of their relationship being sapinda relationship, in the alternative for
divorce on the ground of cruelty, desertion and schizophrenia under Section 13(1)(i) and
13(1)(iii) of the Hindu Marriage Act. The husband sued for an injunction restraining the wife
from coming into the matrimonial home which was his joint family property.

The Additional Principal Judge of the Bombay City Civil and Sessions Court declared the
marriage to be null and void and granted the husband a decree of divorce on the ground of
cruelty as well as an injunction restraining his wife from entering upon his joint family property.
The husband challenged the judgment setting aside the decree of nullity of his marriage, the
decree of divorce as also the order of injunction. The wife challenged the decree of judicial
separation which she didn’t claim. Thus, the Sessions Court incorrectly granted the husband and
wife decrees that they did not approach the Court for.

The issue and question before the High Court was whether the marriage between the husband
and wife was a sapinda relationship, and was therefore void, and whether the wife had a right to
the matrimonial home (her husband’s joint family property). Section 3 (f) (i) of the Hindu
Marriage Act states that a "sapinda relationship" with reference to any person extends as far as
40 | P a g e
the 3rd generation (inclusive) in the line of ascent through the mother, and the 5th (inclusive) in
the line of ascent through the father, the line being traced upwards in each case from the person
concerned, who is to be counted as the first generation. It further postulates that two persons are
"sapindas" of each other if one is a lineal ascendant of the other within the limits of sapinda
relationship, or if they have a common lineal ascendant who is within the limits of sapinda
relationship with reference to each of them.

Section 5(v) of the Hindu Marriage Act states that if the Hindu bride and the Hindu groom are
"sapindas" of each other, the marriage between the two cannot be solemnized by law and will be
legally void.

The parties in the case of Navalkar v. Navalkar came from a common ancestor, Moroba, who
had one son, Laxman, and one daughter, Champubai. The husband was the son of Laxman. The
wife was the daughter of Champubai's son. Upon this admitted relationship the husband claimed
that they were sapindas of one another. The wife claimed that they weren’t, and posited that their
relationship was not a sapinda relationship and there was a custom existing in their community
that allowed for such relationships. When it is found that a sapinda relationship exists, the burden
of proof is on the wife to prove that the sapinda relationship was allowed within the customs of
her community. If it is proved that such a custom does indeed exist, the sapinda marriage will not
be void.

The burden of proof and how the burden of proof of a particular fact is to be discharged is
articulated under Sections 101, 102 and 103 of the Indian Evidence Act. Section 101 provides
that whoever desires any Court to give a judgment as to any legal right or liability dependent on
the existence of facts which he asserts, must prove that those facts exist. When a person is bound
to prove the existence of any fact, it is said that the burden of proof lies on that person. Section
102 further states on whom burden of proof lies. The burden of proof in a suit or proceeding lies
on that person who would fail if no evidence at all were given on either side (thus, in the present
case, the wife). Finally, section 103 states that the burden of proof as to any particular fact lies on
that person who wishes the Court to believe in its existence, unless it is provided by any law that
the proof of that fact shall lie on any particular person.

As the wife was unable to produce the evidence that would have proved that her marriage was
allowed due to custom, the Court ruled correctly in stating that a sapinda relationship existed,
thus rendering the marriage void. The proof provided by the wife was insufficient for the
purposes of the Indian Evidence Act, and the wife did not have the right to her husband’s joint
family property.

Summary 2:

Facts

41 | P a g e
The parties to this litigation are husband and wife. 6 months into the marriage, the wife separated
and sued for maintenance. The husband, in turn, sued for the nullity of the marriage, divorce on
grounds of cruelty, desertion and schizophrenia and asked for an injunction preventing the wife’s
entry into their marital home. It was an established fact that the parties were sapindas of one
another, by a common ancestor named Moroba.

The Bombay City Civil and Sessions Court declared the marriage to be null and void; granted
him a decree of divorce on the ground of cruelty and issued an injunction restraining the wife
from entering upon the joint family property of the husband. The court also directed the husband
to pay costs of the Petition fixed at Rs. 3000/-
The wife appealed this decision. The appeal to a Single Judge saw the decision reversed. The
court set aside the decree of nullity of marriage; the decree of divorce and the order of injunction.
It instead granted a decree of judicial separation under Section 10 of the Hindu Marriage Act.
The order directed the husband to pay the costs of the Appeal fixed at Rs. 5000/- in addition to
the costs granted by the trial Court.
Both parties appealed this judgement. The present case is the verdict from this appeal.

Issues

I) Whether the marriage is valid under the Hindu Marriage Act, given that the couple
are sapindas of each other.
II) Whether the onus of proving the existence of customary acceptance of sapinda
marriages lies on the person claiming it, or denying it.

Rule

I) a) Section 5 (v) of the Hindu Marriage Act –

5. Condition for a Hindu Marriage - A marriage may be solemnized between any two
Hindus, if the following conditions are fulfilled, namely:

(v) the parties are not sapindas of each other, unless the custom or usage governing each
of them permits of a marriage between the two
b) Custom is characterized by its continuity, longevity and uninterruptedness.

42 | P a g e
II) It is for the party who claims such a custom to be existing, to show it by specific
illustrations.

Application

I) It is a given fact that the husband and wife are sapindas of each other. Therefore, such
a marriage would be void, unless there is custom governing both of them, which
allows such marriages. The presence of such a custom is disputed in this case. The
wife tries to prove the existence of this custom by giving examples of 9 couples.
However, of these 9 couples, only 2 fall under the type of sapinda relation in
question. Court holds that no sufficient time lapse is shown between the marriages to
qualify the practice as custom with continuity and longevity. Therefore, the court
holds that no such custom exists. Wife also claims that the lineage is broken by her
grandmother (husband’s paternal aunt) and hence they aren’t sapindas. Court doesn’t
buy this argument as such intervention is not contemplated in the act.

II) The court holds that such a sapinda marriage would be void unless custom proves
otherwise. Therefore, the one claiming such a custom exists, must prove it. The onus
to prove the lack of such a custom cannot be imposed on the husband.

Conclusion

The marriage is held to be null and void under section 11 of the Hindu Marriage Act, read with
section 5 (v). There is therefore, no need to address the other claims by the husband. The
wife’s claim of right of residence in the marital home is also not upheld. The marriage is void
and hence, no rights are created. The court doesn’t not pronounce any decision as to the cost of
litigation.

Critical Analysis

The present case is an important decision on the matter of custom, with regard to personal laws.
Two main questions arise regarding custom – what is proof of custom and who needs to prove
the existence of this custom.

In Navalkar vs. Navalkar, the Bombay High Court lays down the already accepted conditions to
prove custom: it should have longevity, continuity and uninterruptedness. These factors seem

43 | P a g e
necessary to be proven to ensure that the provision for custom doesn’t become a loophole for
validation of marriages which do not conform to statutory provisions.

The Bombay High Court also answered the question of onus of proof in this case. The wording
of section 5 of the HMA suggests that all marriages which do not conform to the conditions laid
down by the section, are void; unless custom allows for such marriages. Therefore, it is evident
that custom needs to be proved to prove that such a marriage is legitimate. The court held that,
therefore, the person willing for the marriage to be proved legitimate must prove the existence of
such a custom. If one claims that a custom exists, they must prove it. By default, the existence of
such a custom cannot be presumed and therefore, the opposition (husband in this case) cannot be
asked to prove the non-existence of such a custom. This argument is well put in the judgement.
A negative onus of proof cannot be imposed.

Apart from the question of custom, the case is also interesting for the fact that it took 22 years to
deliver the final judgement. Such a great delay in the judgement is unwarranted. Such a delay
practically renders futile, the distressed parties’ need for justice. The wife filed to lawsuit in
hopes of obtaining maintenance from her ex-husband. Assuming the court decided in the wife’s
favour, what good would such a judgement have done the wife 22 years after she sought
remedy?

The judgement is also interesting in that it brings attention to the practice of sapinda marriages in
India. It is not possible for the husband and wife to have not known the nature of their
relationship at the time of their marriage. Their voluntary acceptance of this, and their
subsequent consummation of the marriage show that sapinda marriage do occur in Indian
society. The question arises then, should custom be the only exception? Should courts also
consider the voluntary nature of the marriage and recognise its validity in interest of equity?
Perhaps it would be better if such marriages, where consent (even after knowledge of sapinda)
can be proved were made voidable and not void. Or maybe, after divorce, the wife should still be
entitled to maintenance. Either way, it is important to enter this debate and find better and more
equitable ways to sort out these scenarios. The wife’s condition in this case is proof that such a
move is necessary.

III. WEEK 4 & 5: Muslim Marriage & Sources of Muslim Law (24th August & 31st
August)
a. Nature of a Muslim Marriage
i. Abdul Kadir v. Salima And Anr (1886) ILR 8 All 149 (Nature of a
Muslim Marriage)

Summary 1:

44 | P a g e
This case was filed for the restitution of the conjugal rights. It laid down and cleared some very
core subjects of the Muhammadan law in India. The question which the court dealt with in this
case was one that was to affect the core of domestic family relationship of Muhammadans in
India.

In this case the wife (defendant) had stopped cohabiting with her husband (plaintiff) on the
ground of non-payment of the prompt dower. However, during the hearing of the lower court,
she relied on the grounds of cruelty and divorce, which were found to be false. Also, it was
found that she had never demanded the prompt dower before filing of the present case.

The issue before the court was, whether payment of prompt dower is a necessary precondition
for cohabitation.

Since the codification of the Muhammadan laws was not done at that time, the court relied on
various case laws and religious texts to find the true nature of dower. The only relevant
precedent that Justice Mahmood found was Sheikh Abdool Shukkoar v. Raheem-oon-mssa (1874
N.W.P. H.C. Rep. 94). It was held in this case that, a suit cannot lie by a Muhammadan husband
for the restitution of conjugal right until he has paid the prompt dower. This, however, was not
agreed upon by justice Mahmood.

Justice Mahmood argued that right of cohabitation and right of dower are effects of the
consummation of marriage and not part of it. Both these rights, therefore, come into existence
after consummation of marriage, simultaneously i.e. the right of dower does not precede the right
of cohabitation. He believed that if dower is made a necessary condition for cohabitation, it will
lead to many undesired complexities. He says that if this is done, then the marriage will come
into existence at one time for the husband and at another time for the wife (after dower payment).
This can prove to be disadvantageous to the wife for she won’t be able to move the court for
cohabitation before she has got a decree passed for the payment of dower as her right to
cohabitation will come into existence after the payment of the dower.

The court therefore upheld the decision of the court of the first instance.

The ritual of dower was made for the financial upliftment of the women. Also, so that the women
could survive if her husband abandons her. This was done at the time when prostitution was
widespread in areas like Turkey. With time it became a ritual and a symbol of social status but
the ground reality is that many women have to unwillingly agree to let their dower go or not
demand it, under pressure from the family and the society. Women are sometimes looked down
upon if they don’t do so as it is said that they are letting financial matters come in-between
familial relationships. Considering the fact that the social status of financial status is still not
equal to men in current times, I believe that dower (at least prompt dower) should be made a
necessary condition. The complications can be dealt with but, if the seriousness of dower is
reduced like this, the reason for which dower was made, i.e. for the welfare of women, will be
affected.

45 | P a g e
Alternate Notes:

Facts; Plaintiff Husband was married to respondent Salima, whose dower was fixed without any
specification whether it was prompt of deferred. After marriage, she cohabited with her husband
and then left for her father’s place.

When the husband requested her to return, she refused. Husband filed for RCR and the wife
raised the objection that her dower was not paid.

The trial court passed a decree in favor of the husband, the wife appealed and the appellate court
held the amount of dower was insufficient. Husband filed Second appeal.

Issue; Whether non-payment of dower can be set up as a defense in a suit for RCR after
consummation?

Held; Dower, under the Muhammadan law, is a sum of money or other property promised by
the husband to be paid or delivered to the wife in consideration of the marriage, and even where
no dower is expressly fixed or mentioned at the marriage ceremony, the law confers the right of
dower upon the wife as a necessary effect of marriage. To use the language of the Hedaya, "the
payment of dower is enjoined by the law merely as a token of respect for its object (the woman),
wherefore the mention of it is not absolutely essential to the validity of a marriage. It can be
fixed before the marriage, at the time of marriage and even later.

The right of dower does not precede the right of cohabitation which the contract of marriage
necessarily involves, but that the two rights come into existence simultaneously and by reason of
the same incident of law. That the latter right may modify and affect the former cannot be
doubted. The guardian can refuse to send the minor on non-payment.

Prompt dower may be demanded anytime after the marriage and it is only upon such demand that
it becomes payable in the sense of performance being rendered in fulfillment of an obligation.
Where nothing is specified as to whether the dower is prompt of deferred, it is considered to be
prompt.

The right of cohabitation given by marriage accrues in favor of both the spouses and to say that
payment of dower is a condition precedent to the vesting of the right, is to hold that a
relationship may come into existence at one time for one party and at another time for another.
This definitely is not the spirit behind the Muslim law relating to dower. After consummation of
marriage, non-payment of dower, even though eligible, cannot be pleaded in defense of an action
of RCR.

In the present case, the wife went away without demanding for dower (the obligation to pay only
arises when the wife demands for it). Secondly, after consummation the only right the wife has is
to set up an action in court for non-payment, otherwise, to allow a woman to not co-habit with

46 | P a g e
her husband in every case, even where a part of dower is unpaid, will amount as an absolute
option to refuse to live with the husband. This would dislocate domestic life.

b. Who is a Muslim?
i. Narantakath Avullah v. Parakkal Mammu And Ors 71 Ind Cas 65;
(1922) 43 MLJ 663 (Are Ahmediyas Muslim?)

Alternate Notes:

Facts; The defendant is charged with bigamy because she married the second time, presuming
that since her former husband joined the Ahmediyan sect, he had renounced Islam and her
marriage was dissolved.

Issue; Are Ahmediyans considered as Muslims?

Held;

Ahmadiyya is an Islamic religious movement founded in British India, originated with the life
and teachings of Mirza Ghulam Ahmad who claimed to bring about the final triumph of Islam as
per Islamic prophecy. He claimed that he was the divine reformer of the 14th Islamic century.
Followers believe that he divested Islam of fanatical beliefs and practices by championing what
is in their view, Islam’s true and essential teachings as practiced by the Prophet Muhammad.

Ahmed's principles are stated in his own words, beginning. "We are Muslims by the grace of
God: Mustafa, the Holy Prophet of Arabia, is our leader and guide. The wine of our spiritual
knowledge is from the cup of the Book of God, which is called the Koran. Every Prophet hood
has found its culmination in that Messenger of God, whose name is Muhammad. Ahmediyans
are merely a sect of Mohammedans, their divergences from ordinary principles not involving
abandonment of that creed or making them schismatic – only a difference in degrees of
orthodoxy. Ahmadees believe that Jesus Christ was actually crucified but he did not die on the
cross. He came out alive and went to Kashmir and died and is buried there. The second advent of
Jesus is to be fulfilled not by Jesus coming back in person but by his spirit entering another man
and they believe this has been fulfilled by the advent of Ahmad himself.

Sir Ameer Ali says in his book on Mohammedan Law (4th Edition - Vol. II) page 36 that: "Any
person who professes the religion of Islam, in other words, accepts the unity of God and the
prophetic character of Mohammed is a Muslim subject and is subject to the Mohammedan Law.

It was expressly ruled that the sect known as Ahmadees are Mohammedans notwithstanding their
pronounced dissent or several important matters of doctrine from the orthodox Mohammedan
faith. A Mohammedan does not become an apostate by merely accepting the doctrine of
Ahmadees. The Ahamadiyans are in my view only a reformed sect of Mohammedans.

Wife was under a mistake of law and not mistake of fact, hence, punished for bigamy.

47 | P a g e
A Muslim wife who marries the second time during the subsistence of the first marriage commits
an offence u/s 494 IPC.

c. Local Customs
i. H.H. Mir Abdul Hussain Khan v. Mussammat Bibi Sona Dero
(1918) 20 BOMLR 528

Summary 1:

FACTS

 Mir Hussein Ali Khan of Talpur died intestate on 30th January, 1907.

 Did not leave behind widow or child.

 Surviving relatives were the plaintiff, Abdul Hussein, son of his brother by half blood,
one sister, the first defendant, and his sister’s son the second defendant.

 Property which the deceased left behind was of great value and thus of great interest to
the relatives.

 The appeal was regarding the dispute about the inheritance of this property.

 The deceased was Mahomedan.

 The plaintiff alleges that the rights of inheritance are not to be determined according to
Mahomedan law, but are regulated by a custom well-known and distinctly ascertained, by
which women are excluded from any share in the inheritance from a paternal relative.

 Plaintiff further alleges that if this contention does not prevail, the deceased was Sunni
and not Shia, thus entitling him to half of the property.

ISSUES

1. Whether there exists a custom by which women are excluded from any share in the
inheritance from a paternal relative, and thus should the plaintiff be entitled to
inheritance?

2. Whether the deceased belonged to Shia or to Sunni sect?

RULE

 According to Mahomedan law, if the deceased belonged to Shia sect, the sister would
inherit the whole estate, but if he belonged to Sunni sect, the plaintiff would be entitled to
half of the property.

 To modify the ordinary law, a custom should be ancient and invariable.

48 | P a g e
 Where either party to a suit relies upon a "custom" as a rule of decision, it is incumbent
upon him to allege and prove the custom on which he relies.

ANALYSIS

 It was decided to dispose of the later contention first.Both the Judicial Commissioner
sitting as District Judge, before whom the case was first tried and the Court of the
Judicial Commissioner of Sind, before whom the first appeal was heard came to the same
conclusion that the deceased was of Shia persuasion. And there was found no reason to
interfere with the earlier findings.

 Where a large number of conditions have to be fulfilled to test a custom, and a number of
instances adduced in proof of that custom, it is necessary that one or more examples of
each condition should be established, but it is not necessary that all the conditions should
be proved in each instance. The hitter requirement would greatly weaken the evidence by
tradition to which in certain cases great weight is due.

 Custom binding inheritance in a particular family, although not known to English law and
foreign to its spirit has long been recognised in India; and if the custom were in fact well
established in one particular family, whether it were enjoyed or no by another family
would not affect the question, since the custom might be independent in each case, and
the evidence would not establish that the custom failed by reason of the inability to define
the exact limits within which it was to be found when once it was established that, within
certain and definite limits, it was undoubtedly existing.

 For the claim regarding the custom, plaintiff was asked to prove the existence of such
custom. Plaintiff provided several witnesses in support of his claim, which gave sixty-one
instances of this custom.

 It was noted that in all these instances the women was given dowry during her wedding,
thus was decided that evidence provided fails to satisfy the necessary standard of proof
for existence of such a custom.

 Also an evidence was found in the Revenue Records of October 1903 relating to
inheritance of property of Mir Ghulam Ali Khan, which showed a clear division on the
basis if Mahomedan law and no such alleged custom.

CONCLUSION

 The deceased was of Shia persuasion. Thus according to Mahomedan law, plaintiff was
not entitled to any share in the property.

49 | P a g e
 That such instances cannot be confidently relied upon as establishing the custom, for the
position and relationship of the different members of the family must always be
considered in determining whether claims were not mot because the rights to which they
relate did not exist) or whether they were put on one side because, in the circumstances,
there was no need that they should be asserted.

 The existence of the custom which would entitle the plaintiff to inheritance could not be
proved and hence the appeal was dismissed.

Alternate NOTES:

Facts; Mir Hussein Ali Khan died intestate, leaving neither widow nor child. His nearest
surviving relations were plaintiff, Abdul Hussein, the son of his step brother and
defendants, sister and her son. The plaintiff alleges that the rights of inheritance
are not to be determined according to Mohammedan law, but that they are
regulated by a custom well-known and distinctly ascertained, by which women are
excluded from any share in the inheritance of a paternal relation.

The plaintiff relied on S. 26 of the Bombay Regulation of 1827, which states that the law to be
observed in the trial of suits shall be Acts of Parliament and Regulations of
Government applicable to the case; in the absence of such Acts and Regulations, the
usage of the country in which the suit arose; if none such appears, the law of the
defendant; and in the absence of specific law and usage, justice, equity, and good
conscience alone.

Issue; Custom or personal Law will apply? If personal law, then whether Sunni or Shia because
they have different laws of inheritance.

Held; It lies upon the person asserting that he is ruled by custom, to prove that he is so
governed, and further to prove what the particular custom is. There is no presumption created by
the clause in favor of custom; it is only when the custom is established that it is to be the rule of
decision. The Legislature is not enamored of custom nor does it show any tendency to extend the
'principles' of custom to any matter to which a rule of custom is not clearly proved to apply.
When either party to a suit sets up 'custom' as a rule of decision, it lies upon him to prove the
custom which he seeks to apply; if he fails to do so the rule of decision must be the personal law
of the parties subject to the other provisions of the clause.

A very large number of witnesses were called in its support, and a number of instances,
amounting to sixty-one in all, were given as evidence of its operation, number of cases were
cited, the eight different families to which the said custom was said to apply, their heads were
called and they stated that nothing of the sort, as claimed by the plaintiff, existed. Property was
not exclusively for the men.

50 | P a g e
There have been instances in which the claims of daughters and sisters to a share have been
ignored, or they have been allotted maintenance. There are also cases in which married daughters
have been treated as estranged from the family. But instances of this kind will be found to occur
where there is no doubt that the family is governed by pure Mohammedan law. Indeed, in many
parts of the country it is unusual for Mohammedan ladies to insist on their unquestioned rights.
They will often prefer being maintained by their brothers to taking a separate share for
themselves, and when they are married, the marriage expenses and presents are often, by express
or implied agreement, taken as equivalent to the share which they could claim. Mohammedan
females are so much under the influence of their male relations, that the mere partition of the
property among the males without reference to them cannot count for much.

The most important piece of evidence was the revenue records which relate to the descent of the
property of Mir Ghulam Ali Khan, who died leaving three widows, one sister, two daughters,
and a brother; one of these Records, shows a clear division according to the Mohammedan Law,
and not according to any such custom as is alleged. On the whole the evidence has fallen short of
the standard to which it must attain in order to succeed in altering the devolution of property
according to Mohammedan law to a devolution determined by a family custom – is it the right or
the circumstance which has put aside the right in the present case – Appeal dismissed.

ii. Hirbae and Others v. Sonabae. (The Kojahs and Memons case)

Alternate Notes:

Two cases, Case of Khojas and case of Memons, which were so intrinsically similar that Justice
Perry pronounced the same judgment for these two cases.

Facts of Khojas; Hadjibhai, a merchant in Bombay, died and left behind a widow and two
children, Plaintiff Hirbae and her infant sister. He had property consisting of land and movables,
worth 3 lakhs of rupees. The brother of Hadjibhai took possession of all the property. The
plaintiff has filed a suit for claiming this property under Muhammedan Law, whereas the
defendant’s plea was that they belong to an exclusive sect of Muslims, Khojas, who were
originally Hindus, converted to Islam hence, as per the Hindu custom, a woman was not allowed
to get any share in the property.

Similarly, Memon Cutchi females, at the time when this judgment was pending, filed a case for
succession to their father’s property.

Held; The Khojas are a small cast in Western India, who appear to have originally come from
Sindh or Cutch and were converted from Hinduism to Islam some 400 years ago. Their language
is Cutchi, religion – Islam and dressing and appearance – Hindu. They knew nothing of the
Prophet or the religion and the divide of Shia and Sunni. No translation of Quran in their own

51 | P a g e
language. No scholar or men of learning to teach the Quran. Under the British Rule – Muslims
were governed by their customs and Hindus by their own custom because these were considered
Divine and British did not want to set up one single unit of law to hurt sentiments.

Examples of Christian – when sub sects began to inter-marry and a common law for all was
framed – but since the laws are common it generally applies to all Christians. The issue is when
an alien settles in Christian land.

He cited a Jurist and stated that whether the Christian or alien law will apply is for the court to
decide. The traders from Europe and west when entered Christian land, they would be governed
by the local laws for comity of nation. Then the same rule should apply to Muslims and Hindus.
At least in certain matters like marriage, succession, divorce, adoption and the like.

d. Minority:
i. Musammat Atika Begum v Muhammad Ibrahim Rashid Nawab
(Allahabad), [1916] UKPC 70 (Jul. 20, 1916)

Summary 1:

Musammat Atika Begum v. Muhammad Ibrahim Rashid Nawab (Allahabad), [1916] UKPC 70
(Jul. 20, 1916)

Facts:
1) The parties to this litigation are Musammat Atkia Begum who is the appellant and
Muhammad Ibrahim Rashid Nawab who is the respondent.
2) The respondent was married to the appellant (Atkia Begum) with the consent of her
parents in accordance with Mohammedan Law.
3) Atkia Begum’s parents died at Mecca before the marriage.
4) The marriage took place soon after the death of the father. However, the facts of the
solemnisation of the marriage are unclear.
5) Respondent wants a restitution of conjugal rights.
Issue:

1) Whether the appellant and respondent were legally married according to Muhammadan
law at Mecca on the 20th July, 1907.
2) Whether the appellant did in fact give her consent to this marriage

3) Whether the appellant was at the time of the marriage an adult and competent to give her
consent thereto.

Rule:
1) According to Mohamedan law a girl becomes a major on the happening of either of the
two events:

52 | P a g e
2) a) Completion of her fifteenth year
b) Attainment of a state of puberty at an earlier period.
3) Essentials of Muslim Marriage:

a) Every Muslim of sound mind, who has obtained adult age or puberty, may enter into a
contract of marriage.
b) A marriage brought about without the consent of such a Muslim is void.

c) If the parties are major, they must make the offer and acceptance themselves, but for
minors and lunatics, guardians can perform this function.

4) Capacity for Marriage:


a) Every Muslim of sound mind, who has attained puberty, may enter into a contract of
marriage.
b) In Muslim Law, age of majority is understood with reference to attaining the age of
puberty.

c) A Muslim girl becomes major if she has completed the age of 15 years or has attained
a state of puberty at an earlier period.

5) Child Marriage Act, 2006


With a view to restraining solemnization of child marriage the Child Marriage Restraint
Act, 1929 was enacted.
a) The act was subsequently amended in 1949 and 1978 in order to raise the age limit of
the male and female persons for the purpose of marriage.
b) The Act, though restrains solemnization of marriage ( making it punishable) yet it
does not declare them to be void or invalid.

Analysis:
1) Court gathered evidence from appellant and her maternal grandmother and her uncle
seeking evidence as to whether her parents had agreed to the marriage with the
respondent. Further, the court received evidence regarding the age of the appellant at the
time of the marriage.
2) Subordinate judge decided that the marriage at Mecca was false and thus the first issue is
negative as only the paternal uncle could give her away in marriage.
3) Subordinate judge proceeded to deal with the age of the appellant. He found that there
was no evidence to show that she had attained puberty, according to Mohommedan law,
on the date of the alleged marriage. Thus, he dismisses the suit.
Appeal in High Court:

53 | P a g e
4) High court only differed with the conclusion of the subordinate judge on one point which
is the date at which the appellant has attained puberty.
5) Second question is that even if she had attained that age, was she legally competent to
consent to this marriage?
6) Respondent has to prove the above contentions.
7) Court examines evidence and is not satisfied.
8) Court further examined witnesses and found that
9) They were not informed at the Nikah ceremony that the appellant’s father had died 3-4
days before the marriage.
10) On the whole, the court are of opinion that the evidence provided to show that the
appellant has attained puberty before the marriage is vague, unsatisfactory and
inconclusive.

Conclusion:

1) Lordship decides that the respondent has failed to establish that the appellant has
attained puberty before the date of marriage and second was not merely given away in
marriage by her grandmother, but had herself consented to the marriage.
2) Lordship thus decides that the appeal should be allowed and the decree appealed
should be reversed and decree of subordinate judge be restored.
3) The respondent will pay the costs of the appeal.

Critical Analysis:
Position in India:
a) The situation in India is, therefore, that the marriage of a girl (of any religious affiliation
or none) below the age of eighteen is a criminal offence ; in the case of a Hindu or
Muslim girl below the age of eighteen and married under her personal law, the marriage
is valid in spite of U\e liability for criminal sanctions that the celebration of the marriage
incurs.
b) If a Hindu or Muslim girl is given in marriage below the age of fifteen she may repudiate
the marriage between the ages of eighteen (subject, in the case of a Muslim girl, to the
proviso that the marriage has not been consummated with her consent after the age of
fifteen). Marriages of Hindu and Muslim girls between the ages of fifteen and eighteen
are "illegal" and invite criminal sanctions, but they are nonetheless valid and binding and
cannot be repudiated by the girl under either the Dissolution of Muslim Marriages Act or
the Hindu Marriage Act.

Whether Child Marriage act can override Muslim personal law?

54 | P a g e
a) The Madras High Court has held that the act prohibiting child marriages was not
against the Muslim Personal Law and would prevail over the latter as it had been
enacted for the welfare of girl children.
b) The court was dismissing a public interest litigation which sought a direction to
government officials not to interfere with marriages solemnised as per the Muslim
Personal Law by invoking provisions of Prohibition of Child Marriages Act
(POCMA) of 2006, which bars marriage of girls below 18 years.
c) According to the petitioner, the personal law allows marriage of Muslim girls
between the age of 15 and 18. “The Prohibition of Child Marriages Act is not against
the Muslim Personal Law. The Act enacted for the welfare of the girl children
prevails over the personal law. The former helps the girls to get education,
empowerment etc.,” a division bench here comprising justices S Tamilvanan and V S
Ravi said in their order yesterday.
d) The PIL filed by Social Democratic Party of India functionary Mohammed Abbas
challenged the action of District Social Welfare Officer, who had stopped the
proposed marriage of a 16-year old girl, daughter of one Syed Abuthakir or
Maharajapuram village in Virudhunagar, and handing her over to the custody of
Child Welfare Committee.

Summary 2:

This is a very old case which was decided under the Mahomedan Law in 1916. In this case an
appeal was filed before the Allahabad High Court by the plaintiff, as the suit was dismissed by
the trial court of Aligarh. Muhammad Ibrahim (Plaintiff) had filed a suit against Atkia Begum
(Defendant) in the trial court, claiming that she was his lawfully wedded wife. This suit was
dismissed as the judges found the issues of the case not in favour of the plaintiff. Therefore, the
plaintiff went on to file an appeal in the High Court. The judges found that no sufficient
opportunity had been given to plaintiff and as a result of it, the High Court remanded the case for
a retrial.

The facts of the case tell us that the defendant was the eldest daughter of her parents. Her parents
for the most of their lives had resided in Aligarh, but recently, for about three to four years, they
had been living in the house of Arusa Begum at Mecca. Arusa Begum was the grandmother of
the defendant and later on went on to become the guardian of the defendant. The mother of the
appellant died about two months before the 20th of July 1907 and her father died on the 16th or
17th of that month.

The elder brother of the appellant’s father, Abdul Jalil Khan, after the death of his brother,
obtained the guardianship of the minors from the District Judge of Aligarh. On the 14th of
August, the District Judge received an application requesting to revoke the guardianship obtained
by Abdul Jalil Khan, and it should be transferred to the grandmother of the minors. Considering
this application, the District Judge made Abdul Jalil Khan sign an undertaking that he would not

55 | P a g e
perform the Nikah of any of the minors without the permission of the District Judge. In the
month of December, as to a result of an application made to District Judge, the guardianship of
the minors was finally transferred to the grandmother (Arusa Begum) of the minors.
The main issues that were raised in this case are as follows-

1) Whether the Appellant and Respondent were legally married according to the
Mahomedan Law at Mecca on the 20th of July, 1907?
2) Whether the Appellant had attained majority (State of Puberty) and was competent to
give her consent for the marriage?
3) Whether the grandmother of the appellant or the appellant herself gave the consent for the
marriage?
The rule which was laid down in this case was regarding the conditions, which have to be
fulfilled in order to prove the majority of a girl. The condition laid down were as follows-

i. The completion of her fifteenth year.


ii. Attainment of a state of puberty.
The Court said that the burden of proof relies upon those who allege it. It means that in this case
the burden of proof relies on the respondent to prove that the appellant was major at the time of
the marriage. The Court also stated that ‘legal evidence’ is must, in order to prove the above
mentioned conditions and only relying on hearsay evidence won’t be sufficient to satisfy the
burden of proof.

The judge expressed in his opinion that there was no doubt that a marriage took place between
the appellant and the defendant at Mecca. But, the judge states that believing the contention of
the respondent that the marriage was performed legally and with the consent of the appellant, is
very doubtful. The judge, due to lack of evidence, found that the case made by the respondent as
to the marriage at Mecca was utterly false.

The Court felt that the respondent has failed to establish the claims raised by him. He has not
been able to prove that the appellant had attained the state of puberty and nor has he been able to
prove that she herself consented for the marriage. The Court found the issues of this case in
negative, as there was no evidence provided by the respondents to prove that she had attained
puberty and she herself consented for the marriage.

Considering all the above mentioned points, their Lordship are of the opinion that the appeal
should be allowed and the respondent will pay the costs of the appeal.
Evolution of Law

In the present case, which was decided in the year 1916, Mahomedan Law only lays down the
conditions to prove the majority of a minor, i.e. state of puberty and completion of fifteen years.
The law does not mention anything about the situation of repudiation of marriage, in case the
minor marries during minority. The law during that time also states that after the death of the

56 | P a g e
parents, only the paternal uncle can give consent for the marriage, and no other guardian will be
allowed to give consent. The law in a way was very restrictive.

In the case of Nizammudin v. Huseni (1960), the law states that when a marriage is contracted for
a minor by any guardian other than the father or father’s father, the minor enjoys the option to
repudiate the marriage on attaining puberty. The law during that time also made a distinction that
if a minor is consented by her father, then no repudiation is allowed, however, if consented by
the mother, then the minor can successfully repudiate the marriage. Therefore, all the minority
cases during that time revolve around one question, i.e., ‘who gave the consent for the
marriage?’
In the case of Tara Bano v. Iqbal Mohammad (2009), in case of a girl married during her
minority, she has to prove in order to avail the right of repudiation that she was given in marriage
by her father or any other guardian and the marriage took place before she had attained the age of
fifteen years. This option of puberty, was earlier not available outright when the minor was
contracted in marriage by the father or grandfather.

During the case of Nihal v. State (2008), the law is well established, as many new specific acts
have come onto force covering mostly all the areas. The Prohibition of Child Marriage Act, 2006
has replaced the Child Marriage (Restraint) Act, 1929. Section 3 of PCM Act declares every
child marriage to be voidable at the option of the contracting party who was a child at the time of
the marriage. This case lays down the order or sequence in which different people become
competent to act as the ‘Wali’ or the guardian of the minor.
All the above mentioned cases, shows us that how the law has evolved overtime.

ii. Smt. Khatiza Qubra alias Tara Bano v Iqbal Mohd ,


AIR2009Raj82.

NOTE: Refer to attached PPT

iii. Mohd. Nihal v. State MANU/DE/0980/2008

Summary 1:
This case deals with complex issues within Muslim Law. There are two main issues raised in this
case:

1. Can the marriage of a Muslim girl who hasn’t attained puberty be rendered void due to
any other Law in India?
2. Is the marriage in the case is valid?
Rules applied:

1. According to the Protection of Child Marriage Act a female can only enter into a
matrimonial relation if she is above the age of 18 years. Section 2 of the Muslim Personal

57 | P a g e
Law (Shariat) Application Act, 1937 gives pre-eminence to Muslim Personal Law
(Shariat).
2. Section 2 of the Majority Act and Section 4 of the Guardians and Wards Act, 1890.
3. Under Muslim law the marriage of a girl who has not attained puberty is nevertheless
legitimate provided it has the consent of her Guardian (Wali). The wife however has the
option to repudiate the marriage when she reaches puberty.
4. In the absence of the father, the grandfather, the great grandfather, the brother, the uncle
or granduncle and the mother, in this sequence, are competent to act as the Wali.
In the absence of the mother, the paternal or maternal grandmother, the maternal
grandfather, the sister, or uterine brother or sister, paternal aunt, maternal aunt, maternal
uncle or aunt, in that order, can perform the duties of a Wali.
Application:
Section 3 of the Protection of Child Marriage Act declares every child marriage to be voidable at
the option of the contracting party who was a child at the time of marriage. The Muslim Law
says that in cases of child marriage the wife has the option to repudiate the marriage when she
reaches puberty. Therefore, the Muslim Law is in accordance with the Protection of Child
Marriage Act and declares child marriages voidable at the option of the child in the marriage.

Section 2 of the Majority Act immediately assumes significance as it declares that it shall not
affect (a) the capacity of any person to act in the matters of marriage, dower, divorce and
adoption; and (b) the religion or religious rites and usages of any class of citizens.
Section 6 of the Guardians and Wards Act, 1890 also preserves this superiority/pre-eminence of
personal laws inasmuch as it directs that in the case of a minor, nothing contained in the Act
shall be interpreted to take away or derogate from any power to appoint a guardian of his person
or property or both, which is valid by the law to which the minor is subject.

Therefore, there is no conflict between any other law and the Muslim person law and the
marriage of a Muslim girl who hasn’t attained puberty or is younger than 18 years cannot be
rendered void by any other law.

Mohd. Nihal, sought the custody of his wife by means of this habeas corpus petition. Nihal
asserts that was married to Mst. Afsana on 31.3.2008 in consonance with Muslim rites and
ceremonies at Madarasa Alia, Masjid Fatehpuri, Delhi. The age of Mst. Afsana is the cause of
controversy, both factual and forensic. But in this case medical tests were not able determine as
to whether Mst. Afsana was 15 years of age at the time of her marriage. The Medical Tests,
however, clearly indicate that Mst. Afsana was not 18 years of age. It was concluded that the
Petitioner had submitted a false Affidavit pertaining to the age of Mst. Afsana during the
marriage. The petitioner could not to establish that his wife had obtained puberty at time of
marriage, or that she had reached age of 15 years. Mst. Afsana narrated that her marriage had
been consummated. But medical test proved that her hymen was intact and so, she was a virgin.
This showed a total lack of knowledge on the part of Mst. Afsana as to biological/physical

58 | P a g e
functions. She was unable to state whether she had started menstruating. The Medical Report
confirms that she has now started menstruation. However, it is unclear whether she had attained
puberty on the date of her marriage. If Mst. Afsana was in fact 19 years old at the time of her
marriage, there would not have been any requirement for the consent of a guardian. But it was
established that the affidavits were not correct, and Mst. Afsana was much younger than 18 years
and it was uncertain whether she was even 15 years of age or had actually reached puberty.
Therefore due to the uncertainity of her age it was necessary that a guardian be present to give
her consent and validate the marriage.

Mst. Afsana’s brother in law, according to Nihal, had acted as her guardian (Wali). As
mentioned in the rules, it is common to all schools of Muslim Law that in the absence of the
father, the grandfather, the great grandfather, the brother, the uncle or granduncle and the
mother, in this sequence, are competent to act as the Wali. In the absence of the mother, the
paternal or maternal grandmother, the maternal grandfather, the sister, or uterine brother or
sister, paternal aunt, maternal aunt, maternal uncle or aunt, in that order, can perform the duties
of a Wali. Mst. Afsana's father, mother and elder brother, amongst others, are alive and available
but were not present when the marriage took place. Since her father was alive, only he was
competent to act has her Wali for the purposes of her marriage as prima facie she was a minor at
that time. Therefore, the purported marriage was batil or void ab initio.

An F.I.R. under Section 363 IPC was been registered on 31.3.2008 by Mst. Akhatari Begum, Mst
Afsana’s mother. She opposed the handing over of her daughter to the Petitioner. On the date of
the petition Mst. Afsana had turned 18 years of age. Therefore her mother’s request for her
custody was rendered irrelevant she was an adult.

This case is important because it gave superiority to Muslim Laws and also went through
medical examinations to determine the age of a party in this case which does not happen in most
cases.
Conclusion:

Since the Petition is to the effect that a valid marriage was not performed between the Petitioner
and Mst. Afsana, he has no right to claim her custody.

Although she had attained majority by the last date of hearing, thereby rendering mother's
decision as to her custody irrelevant and the claim for conjugal custody was rejected.

This Petition was accordingly dismissed leaving Mst. Afsana free to decide her own fate and the
future.

IV. WEEK 6: Muslim Divorce (7th September)


i. Shamim Ara Vs State Of U.P., AIR 2002 SC 3551

Summary 1:

59 | P a g e
Facts:

The parties involved in this case are the appellant, Shamim Ara and the 2nd respondent, Abrar
Ahmad. The appellant and 2nd respondent were married according to Muslim Shariat Law in
1968. In the year 1979, the appellant, on behalf of her children and herself, filed an application
under Sec.125 of the Cr.P.C. complaining of desertion and cruelty on the part of the 2nd
respondent. The presiding judge at the Allahabad High Court passed an order in 1993, refusing
to grant any maintenance to the appellant on the ground that she had already been divorced by
the 2nd respondent and therefore had no right to ask for maintenance. In the year 1990, the
defendant (2nd respondent) filed his written statement denying all averments made by the
appellant while including an additional plea stating that he had divorced the appellant in the year
1987. The appellant preferred a revision before the High Court. The High Court did not agree
that the 2nd respondent had divorced the appellant in the year 1987 as there had been no
communication of the same to the appellant. Instead the High Court held that the act of filing the
written statement (1990) in which 2nd respondent in which he stated that he divorced his wife,
was the necessary communication which finalised the divorce between the appellant and the 2 nd
respondent. Therefore, the High Court decreed that the appellant was entitled to maintenance
from the year 1987 to 1990. The appellant then filed an appeal before the Supreme Court and
resulting in the present judgement.

Procedural History:

The present case is a result of an appeal by special leave in the Supreme Court. The appellant
initially filed an application in the Family Court of Allahabad. The Family Court ruled in favour
of the 2nd respondent. The appellant then preferred revision by the High Court. The appellant was
not satisfied by the High Court’s judgement and hence, filed an appeal in the Supreme Court.

Issue:

Whether the appellant can be said to have been divorced and if said divorce communicated to
the appellant so as to become effective from 05.12.1990, the date of filing the written statement
by respondent no.2?

Rule:

The two main sources of authority that the judgement has relied on are:

 Paragraph 310 of Mulla on Principles of Mahomedan Law. This para defines talaq and its
types while enumerating the conditions for the validity of each. The important part of this
para is the definition and conditions of oral and written talaq.
 Dr. Tahir Mahmood’s ‘The Muslim Law of India’, in which it is stated that under all schools
of Muslim law, the husband has the authority to unilaterally divorce his wife without the
intervention of the courts in the matter.

60 | P a g e
Application and Analysis:

The Supreme Court found the reasoning and the decision given by the High Court was flawed as
there was almost no evidence to prove that the defendant had pronounced talaq as stated by him
in his written statement filed in 1990 and the 2nd respondent could not prove the same. The
written statement only contained plea in passing stating that he had divorced the wife and there
were no other no other particulars, such as people present during the talaq. The judges further
stated that pronouncement of the talaq was mandatory. It had to be uttered formally and
communicated to the wife, without which it would not be valid. A written statement with a mere
plea stating that divorce had been given, could not amount to communication. Communication of
divorce had to be pronounced clearly with the required intent. It is important to note that this
judgement seems to stray from the religious commentaries on muslim divorce and instead gives a
beneficial construction to the term “pronounce” so as to elevate the position of the woman who is
the wife. Both the Mulla piece and the Tahir Mohamed piece argue that the husband has absolute
authority to unilaterally divorce the wife which would be valid even if she was not informed or
had no communication of the same. Yet, this judgement reinterprets the pronouncement
condition to mean that the communication of the divorce had to be made to the wife which
makes this judgement a bit revolutionary for its time.

Conclusion:

The appeal was allowed. The Supreme Court Held that the marriage was between the appellant
and 2nd respondent was still valid as it had not been dissolved in accordance with law.

ii. Itwari Vs. Asghari and others, AIR 1960 All 684

Summary 1:

Issue: Whether a Muslim husband who enters into another marriage can seek conjugal rights
against the first wife? Is the plea of second marriage available to the wife as her defense?

Rule: Restitution of conjugal rights: “Where either the husband or wife has, without lawful
ground withdrawn from the society of the other, or neglected to perform the obligations imposed
by law or by the contract of marriage, the court may decree restitution of conjugal rights, may
put either party on terms securing to the other the enjoyment of his or her rights”

Analysis: On behalf of the appellant, it was argued that the mere fact of the husband taking a
second wife was no proof of cruelty as every Muslim man has a right to take upto 4 wives , and
if the first wife is permitted to leave the husband merely on that ground, it would be a virtual
denial of his right; further, that to defeat a husband’s suit for restitution there must be proof of
cruelty of such character as to render it unsafe for the wife to return to her husband’s dominion.
Further, according to the court, in a suit for restitution by a Muslim husband against the first wife
if the court feels that the circumstances reveal that in taking a second wife, the husband has been

61 | P a g e
guilty of such conduct as to make it inequitable for the court to compel the first wife to live with
him, it will refuse relief.

Conclusion: After analyzing the case-law and entire circumstances of the case, the court held that
the wife had been deserted and neglected by the husband for so many years. In the
circumstances, it will be inequitable to compel her to join the husband. The husband’s appeal
was accordingly dismissed.

CRITICAL ANALYSIS

I feel that the judgement given by the Allahabad High Court is a good judgement as it has kept in
mind the social changes within the Indian society. But, I also feel that by passing such a
judgement which can be used in future cases as a precedent they might have defeated the
provision of polygamous marriage under Muslim marriage act. Now the first wives of a Muslim
man can simply claim cruelty if her husband marries another woman. The court in this case
emphasized the fact that even though polygamy is permissible amongst Muslims, the Muslim
law as enforced in India has considered it as an institution to be tolerated but not encouraged, and
has not conferred upon the husband any fundamental right to compel the first wife to share his
consortium with another woman in all circumstances. While the court has all throughout the
judgement disapproved of the husband’s second marriage, it seems to have kept some leeway
when it refers to “circumstances” and “weighty and convincing explanation” for the act which
means that second marriage per se is not an act of cruelty. As time has passed by few would
disagree that second marriage, without anything more, is an act of cruelty and therefore relying
on the social conditions if the court passes such a judgement the purpose of the provision on
polygamy will totaling be done away with in the Indian society. However, the court emphasized
more on the fact that the husband did not make efforts to bring back his wife for years which
made the court pass this judgement. So overall I feel that the judgement is a decent one and has
not entirely distorted the provision for polygamy. It has only pointed out the fact of the changing
social conditions within the Indian society which I feel was necessary to be done.

Summary 2:

62 | P a g e
Issue: Whether the conduct of the husband in taking a second wife is any ground for the first
wife to refuse to live with him or for dismissing his suit for restitution of conjugal rights?

Rule: In Abdul Kadir's case ILR 8 All 149 (FB), it was held that in a suit for conjugal rights, the
Courts in India shall function as mixed Courts' of equity and be guided by principles of equity
weld-established under English Jurisprudence. One of them is that the Court shall take into
consideration the conduct of the person who asks for specific performance.

Therefore the rule derived by the means of precedence is :


“In a suit for restitution of conjugal rights by a Muslim against his first wife after he has taken a
second, if the Court after a review of the evidence feels that the circumstances reveal that in
taking a second wife the husband has been guilty of such conduct as to make it inequitable for
the Court to compel the first wife to live with him, it will refuse relief.”

Analysis: According to Koran IV.3, “If ye fear that ye cannot do justice between orphans, then
marry what seems good to you of women, by twos, or threes, or fours or if ye fear that ye cannot
be equitable, then only one, or what your hand possesses.”
This extract from the holy Kuran supports polygamy to the extent of a muslim husband having
upto 4 wives. However, it supports polygamy only on the condition that the husband will provide
equitable treatment to all 4 wives. If he cannot provide equitable treatment to all the wives then
he is advised to have only one.

Section 2 clause viii(f) establishes a ground of divorce saying that “A woman married under
Muslim law shall be entitled to obtain a decree for the dissolution of her marriage if he has more
wives than one, does not treat her equitably in accordance with the injunctions of the Quran.”

63 | P a g e
The judge in this case focussed on this clause as well saying that cruelty can only be determined
by the prevailing social conditions and today it is impossible for any Indian husband with several
wives to cart them all around. Hence, under the prevailing conditions, the very act of taking a
second wife, in the absence of a weighty and convining explanation, raises a presumption of
cruelty to her on the grounds of inequal treatment. The onus would therefore be on the husband
who takes a second wife to explain his action and prove that his taking a second wife involved no
insult or cruelty to the first.

Conclusion: Reversed the finding of the trial court believing that the wife’s allegation of specific
acts of cruelty committed by husband were valid and held that she has been deserted and
neglected by the husband for so many years. In these circumstances, the court rules that it would
be inequitable to compel the first wife to live with such a husband.
Appeal dismissed.

Drawing a parallel between Section 2 of the Dissolution of Muslim Marriage Act and
Section 9 of the Hindu Marriage Act:

Section 2 of the Dissolution of Muslim Marriage Act talks about the grounds for decree of
dissolution of muslim marriage whereas Section 9 of the Hindu Marriage Act talks about the

64 | P a g e
restitution of conjugal rights stating:
“When either the husband or the wife has, without reasonable excuse, withdrawn from the
society of the other, the aggrieved party may apply, by petition to the district court, for restitution
of conjugal rights and the court, on being satisfied of the truth of the statements made in such
petition and that there is no legal ground why the application should not be granted, may decree
restitution of conjugal rights accordingly.”
This means that where the question of whether there was a reasonable excuse for withdrawal
from the society, the burden of proving reasonable excuse shall be on the person who has
withdrawn from the society.
In my opinion, this rule creates a conflict between the hindu and muslim personal laws regarding
marriage and divorce. For instance, if the case of Itwari v. Asghari came under the ambit of the
Hindu Marriage Act, the burden would have been on Asghari to prove that her withdrawing from
the society and the company of her husband was reasonable whereas according to muslim laws
now, the onus regarding the same will be on the husband.
Section 2 and 9 of the Acts are conflicting and logical in their own way at the same time. Since
section 2 of the Dissolution of the Muslim Marriage Act does not explicitly mention about the
rules regarding the restitution of conjugal rights, there are bound to be dissimilarities between the
two. On one hand, the Muslim personal law does not give the right to the wife to restitute her
conjugal rights but the maximum that she can do is obtain a divorce, the Hindu Marriage Act
gives right to both the parties to restitute their conjugal rights in case one partner has withdrawn
from the society unreasonably.

CRITICAL REVIEW

65 | P a g e
Through the medium of the readings that we have been given, namely the Lucy Carroll reading
on the right to acoid an arranged marriage contracted during minority, clearly shows that how the
Muslim personal laws on marriage, divorce, etc. are extremely biased towards men and favour
them majorly. The women are generally kept at a lower pedestal than men which is the reason
why most laws favour men than women. Somr of thr examples that prove the same are the
muslim personal laws regarding marriage and dissolution of marriage. A man is permitted to
practise bigamy and keep upto 4 wives while a women is not allowed to do the same. When it
comes to dissolution of marriage, the husband can make use of methods of triple talaq, ahsan,
hsan, etc. while the wife can only seek divorce through the meduim of Section 2 of the
Dissolution of Muslim Marriage Act, 1939.

The examples stated above are proof enough that there is a dire need for the women of the
muslim community to be elevated and treated equally as the men of the society and the case of
Itwari v. Asghari aims to take a short step in giving the women equal rights as opposed to the
men in the society. The case sets a precedent and partially makes monogamy the norm by stating
that in the prevailing societal conditions, it is impossible for a man to treat all his wives in equal
fashion and therefore imposes a restriction on practising bigamy unless it is an exceptional
situation where the second marriage was indeed performed on the suggestion of the first wife.

66 | P a g e
iii. Marium v. Mohd. Shamsi Alam, AIR 1979 All 257

Summary 1:

Facts

The plaintiff was married to the defendant on March 31, 1967. On 17th of July, 1969, certain
differences arose between the parties on account of which the defendant or the husband being fed
up, sent his wife to her parental house in June, 1969. On 22nd June, 1969, he went to his wife’s
parental house and demanded that she immediately sent back with him, to which her parents
objected. The defendant in a rage and in one breath uttered the words of talaq thrice over.
The plaintiff filed for a suit for declaration to declare that the plaintiff had been divorced by
triple talaq and a decree for recovery of Rs. 975.65p. as dower, Rs. 300 as maintenance for the
period of 'Iddat‘, Rs. 2,800 on account of the price of ornaments alleged to have been detained
by the defendant and maintenance for the daughter amounting to Rs. 600.

In the alternative, it was also pleaded that if the court found that relationship of husband and wife
still existed between the parties the marriage may be dissolved by a decree of divorce.

The defendant denied all charges and submitted that he had pronounced talaq only once, which
was a revocable talaq, in order to get a control over the situation and compel the wife to come
along with him. He had also sent several letters to the plaintiff and her father. He had also filed
for restitution of conjugal rights.

Issue

1. The question raised before the Court was whether there was an irrevocable divorce
between the husband and wife.
2. Whether the husband is liable to pay for the suit of recovery filed by the wife for the
recovery of ‘Dowar’, maintenance for Iddat, price of ornaments detained by her husband,
and maintenance for daughter.
Rule

The court looked into the following texts and upheld the rules laid down in the texts:

According to Mulla, principles of Mohamedan Law, a talaq in the Ahsan would become
irrevocable and complete on the expiration of the period of Iddat, and until a talaq becomes
irrevocable, the husband has the option to revoke it, which may be done either expressly, or
impliedly as by resuming sexual intercourse.

67 | P a g e
According to Tyabji's Treatise on Muslim Law P the revocation of a talaq may be implied or
conclusively presumed from the conduct of the husband even though he be of unsound mind and
that under Shia law, the revocation must always be express or by resumption of sexual
intercourse.

Analysis

On the question of whether the divorce was revocable, the court held that the a divorce
pronounced thrice in one breath would have no effect if it was given without the intention of an
irrevocable divorce. Here the husband had expressed his intention to take the wife back and had
also instituted a suit for restitution of conjugal rights. By doing this he had revoked the divorce,
and the marriage between the parties could not be said to have been dissolved.

The claim for maintenance for the child was negatived as the daughter was dead. The
maintenance for the wife was refused on the ground that she was living with her parents, of her
own free will, and had neither been divorced nor deserted by the husband.

Under Hanafi law, the revocation of a talaq may be implied or presumed from the conduct of the
husband.

The period of iddat under Muslim law is 90 days. In this case the suit was filed even before the
iddat period had expired. Based on the facts, the husband had revoked the talaq within a week of
giving it, when he personally went to fetch the wife, and by expressly stating before two
Muslims that he had revoked the talaq. There was no reason to disbelieve the defendant, and his
conduct, admitted by the wife herself was ample proof of the fact that he had not irrevocably
divorced his wife.

Conclusion

The appeal failed and was dismissed. It was a revocable Talaq and the husband had revoked it by
filing for restitution for conjugal rights within the period of 90 days of iddat.

iv. Sabah Adnan Sami Khan v.. Adnan Sami Khan, AIR 2010 Bom
109

Summary 1:

ISSUE – Whether a divorce between the appellant(wife) and respondent(husband) under the
Divorce Agreement of April 2004 a Talak Ahasan as claimed by the former, or Khula as claimed
by the latter, and whether the appellant was obliged to undergo Halala prior to her remarriage
with the respondent in April 2007?

RULE – The dispute regarding the type of Talak is of two types, i.e. Talak Ahasan or Khula.

68 | P a g e
The former type of divorce is a single pronouncement by the husband, made during a tuhr
(period between 2 menstrual course), which follows with an abstinence from sex for period of
iddat (the period a woman must observe after the cessation of nikah, during which she may not
marry another man, to differentiate the male parent of any offspring produced after the divorce or
death of the former husband), after which the same becomes irrevocable.

The latter type of divorce is at the instance of the wife, who makes the offer by giving a
consideration (dower/other rights) to the husband for her release from the marriage tie. The
husband must accept the offer for it to become irrevocable, before which the wife has three days
to revoke the offer.

Halala is necessitated if the talak was made through a triple pronouncement by the husband, i.e.
in the cases of Talak Hasan and Talak-i-Badai the wife cannot re-marry her former spouse
unless in the intervening period she marries someone else and her marriage had been dissolve
either through divorce or death of that person and the Iddat* of divorce or death has expired.

ANALYSIS – The parties, Sunni Muslims, conducted their first marriage in 2001, which was
followed by a divorce in 2004, signed by both among witnesses. Consequently, they remarried in
2008, after which the appellant filed a petition under Section 2 (viii) of the Dissolution of
Muslim Marriages Act, 1939 and relief under the Domestic Violence Act. The respondant in his
reply petition claimed that the remarriage was null and void, as the appellant did not perform
halala after the dissolution of their first marriage, and categorized the divorce as the Khula form.

In determining whether the divorce was Talak Ahasan or Khula, the courts examined the divorce
agreement of 2004, acoording to which, even before the execution of the agreement, the
respondent uttered by spoken words a divorce from the appellant, hence qualifying it as Talak
Ahasan. There was no offer in the form of monetary consideration given by the appellant to the
respondent for her release and hence it was not Khula.

In determining whether the appellant was required to perform Halala before her remarriage with
the respondent, the rule applied by the courts necissated that Halala was essential only when the
husband repudiated his wife in three pronouncements, in Triple Talak. The court further stated
that the irrevocable nature of Talak Ahasan had no relationship with Halala, and the 2004
divorce agreement too, removed any restrictions of Halala.

CONCLUSION –

The Bombay High Court ruled in favour of the appellant in 2010, by classifying the divorce
between the two parties as per the 2004 agreement to be Talak Ahasan, and dismissed the
respondent’s appeal of the remarriage being null and void, as the appellant had no obligations to
perform Halala. The appellant was directed to be awarded due damages as per the Domestic
Violence Act, to be heard by the Family Court.

69 | P a g e
Summary 2:

Sabah Adnan Sami Khan vs. Adnan Sami Khan


(AIR 2010 Bom 109)

Facts
Sabah Khan, hereinafter referred to as Petitioner, married Adnan Sami Khan, hereinafter referred
to as Respondent, in 2001. However took Divorce in 2004 by signing a ‘Talaqnama’ (Divorce
Agreement). They remarried again in 2004.
Petitioner again filed for divorce in 2009 under Sec. 2 of The Dissolution of Muslim Marriages
Act, 1939. The Respondent raised the claim that the re-marriage was void because the Petitioner
had not performed the custom of ‘Halala’. The family court upheld this claim. The Petitioner
files a petition against the Family Court’s order in the Bombay High Court.

Issue

Whether the divorce between both parties was by ‘Ahasan’ mode or by ‘Khula’ mode?

Rule

Divorce by ‘Ahasan’ mode is when the husband pronounces divorce (Talaq) once during two
consecutive ‘tuhrs’ (menstrual periods), which was followed by a period of 3 lunar months called
the period of ‘Iddat’ during the which the husband can revoke the divorce and upon the
expiration of which the divorce becomes irrevocable.

Divorce by ‘Khula Mode’ is when the wife initiates a proceeding for divorce. In the ‘Khula’
mode, the wife must make an offer to the husband and could renounce her claim to ‘mehr’ as
consideration for the husband to relieve her from marital right and duties. Upon acceptance of
the offer, the marriage becomes irrevocable and the wife can revoke her offer before the husband
accepts it.

Analysis

In this case, the court referred to the ‘talaqnama’ signed between the parties, which stated that
the initiation for the proceeding of divorce was started upon the wishes of the petitioner,
however, the court made special attention to one clause in the ‘talaqnama’, which stated that the
respondent had said, “My wife Sabah Khan is divorced”. The court concluded that the divorce
was by a single pronouncement if ‘talaq’ and was by ‘Ahasan’ mode. The court dismissed the
claim of the respondent saying that divorce was by ‘Khula’ mode by saying that in the
‘talaqnama’ the petitioner, wife, was not the only one who relinquished her claim to financial

70 | P a g e
rights over the husband; the respondent, husband, also relinquished his claim to financial rights
over his wife.

Conclusion

The divorce between the Petitioner and the Respondent was by ‘Ahasan’ mode and not by
‘Khula’ mode.

Issue

Whether the custom of ‘Halala’ needs to be performed for re-marriage in all cases of irrevocable
marriage?

Rule

1. The court referred to the Sections 19 & 20 in Chapter III of Part-I of a Compendium of
Islamic Laws, published by the All India Muslim Personal Law Board which stated that: -

19 -“An irrevocable Talaq, whether express or implied, (words of complication are


explained hereinafter) is of two kinds; bainunat-e-khafifah (minor separation) and
bainunat-e-ghalizah (major separation). Less than three Talaqs effect bainunat-e-khafifah,
otherwise there will be bainunat-e-ghalizah.”

20- “In bainunat-e-khafifah though the wife goes out of the marital bond but the parties
may by mutual consent remarry during or after the "Iddat". In bainunat-e-ghalizah
remarriage is possible only where after the expiry of "Iddat" the woman has married
another man who has either died or divorced her and the "Iddat" of death or divorce has
expired “

2. ‘Halala’ is the custom which needs to be performed by the woman before she can
remarry her husband. She needs to marry another person, the second marriage needs to be
consummated and this marriage must become void either by divorce or by death of
second husband. Once this is done, then only can the wife remarry her previous husband.

Analysis

The court dismissed the claims of the respondent that the marriage was by ‘Khula’ mode and that
the petitioner had not performed the necessary custom of ‘Halala’.
Having concluded that the ‘talaq’ was by ‘Ahasan’ mode, the court by referring to the
compendium came to the conclusion that in cases where there was minor seperation or only a

71 | P a g e
single pronouncement of divorce, ‘Halala’ need not be performed. It said that ‘Halala’ needs to
be only performed in the cases where the divorce was by ‘triple talaq’ method which was a more
severe form of ‘talaq’. In the present case, the respondent in the ‘talaqnama’ made a single
pronounce of ‘talaq’ by saying, “My wife Sabah Khan is divorced” and amounted a divorce by
‘Ahasan’ mode which was a revocable and less severe form of divorce. The court also concluded
in cases where ‘talaq’ is by ‘Khula’ mode the husband need not perform ‘Halala’ because
divorce or ‘talaq’ by this method was less severe as compared to those where there is a triple
pronouncement of ‘talaq’.

Conclusion

‘Halala’ need not be performed after divorce by single pronouncement of ‘Talaq’. It is needs to
be performed after triple pronouncements of ‘talaq’

This case highlights the plight of women in Muslim Marriages. All forms of divorce available for
use are biased towards the husband. He can initiate divorce by saying ‘talaq’ without any valid
reasoning or explaination. The custom of ‘halala’ is also very discriminatory because the woman
needs to consummate her second marriage, which has led to many cases of marital rape. Women
are given to financial right or even maintenance rights over the husband leaving them displaced
from their homes and their children. In the present case, the lower courts decision also worked in
the favour of the husband relieving him from all liability or responsibility towards his wife.

v. Yousuf Rawther v. Sowramma, AIR 1971 Ker 261

Summary 1:
Facts:
Sowramma (P), a hanafi girl, around 15 was married to Yousif (D)
who was twice her age.
Soon after the wedding P got shifted to the D’s house.
The very next day Yousif moved to Coimbatore where he was
running a radio equipment store.
After a month’s stay in husband’s house Sowramma moved back
to her parents house.
This separation lasted for over two years during which span the
defendant admittedly failed to maintain the wife, the ground
alleged by the D being that he was willing to keep her
with him but she wrongfully refused to return to the conjugal
home thanks to the objectionable inhibition by the father of the
girl.
After preliminary skirmishes, a litigation for dissolution of

72 | P a g e
marriage was filed by the wife.
The trail court dismissed the suit but the subordinate court
granted a decree for dissolution of marriage.
The defendant again appealed in the high court.

Issue:
Does the female who wrongfully leaves her matrimonial home
has a right to claim dissolution of marriage through court for
mere failure of the husband to maintain the erring wife for 2
years ?

Rule:
Statues referred to in the case:
Section 2 clause (ii) of the dissolution of Muslim Marriage act
1939. Section 2(ii) states that a women is entitled to obtain a
decree for dissolution of her marriage if her husband has
neglected or has failed to provide for her maintenance for a
period of two years.

The special marriage act, 1954

Child marriage restraint act, 1929.

Application:

According to the previous case laws, failure or neglect to


provide maintenance in order to give rise to claim for
dissolution, must be without any justification. For if there
is justification, there cannot be said to be neglect. Neglect or
failure implies non-performance of a duty. But if the
husband is released from the duty on account of the
conduct of the lady herself, the husband cannot be said to
have neglected or failed to provide maintenance. Hence no
decree of dissolution under section 2(b) of the dissolution
of marriage act.
But according to this court women may be tempted to
claim divorce by their own delinquency and family ties may
become tenuous and troublesome

Conclusion:

73 | P a g e
In the end it was held that a muslim woman, under Section 2 (ii) of the Act, can sue for
dissolution on the ground that she has not been maintained even if it is because of her own fault.

Analysis:

I think this is an remarkable decision as it tried to bring at par the women’s right to claim divorce
and men’s right to claim divorce at par.

This view also has great advantage over other that the Muslim woman (like any other woman)
can enjoy roughly equal rights as their spouses, when the talaq technique of instant divorce is
matched somewhat by the Khulaa (women claiming divorce) device of delayed dissolution
operated under judicial supervision. The social imbalance between the sexes will thus be
removed and equality can be realized

vi. Abdurahiman v. Khairunnessa, I(2010)DMC707,

Summary 1

Facts of the Case


The spouses in this case are parties. They got married in 1980 in accordance to custom Muslim
religious rights. Four children were begotten in their marriage. The wife was taken by the
husband abroad because he was employed there. She also found herself a well- paid job there.
Unfortunately the husband lost his job, as result the wife also had to resign her job and return to
India with him. They set up a joint residence and he told her that the property was purchased on
her name. In this belief she had given him some money and gold ornaments. Few days later she
got to know that the husband had cheated her. Upon finding this, the husband started treating her
with cruelty. He made her live alone in a one bedroom kitchen house, and failed to perform his
marital obligations. He also married another lady and started residing with her. On basis of this
the first wife filed a suit under 2 [ii, iv, viii (a), (f) ] under the Muslim dissolution act.

ISSUE
i. How is the expression “does not treat her equitably in accordance with the injunctions of
the Quran” in Sec.2 (viii)(f) of the Dissolution of Muslim Marriages Act, 1939 to be
understood in law?
ii. Whether Right to life under Article 21 of the Constitution must definitely include the
right to a healthy and harmonious matrimonial life.
RULE
SECTION 2:

74 | P a g e
Grounds for decree for dissolution of marriage.—A woman married under Muslim law shall be
entitled to obtain a decree for the dissolution of her marriage on any one or more of the following
grounds, namely:— —A woman married under Muslim law shall be entitled to obtain a decree
for the dissolution of her marriage on any one or more of the following grounds, namely

I. that the husband has neglected or has failed to provide for her maintenance for a period
of two years
II. (iv) that the husband has failed to perform, without reasonable cause, his marital
obligations for a period of three years

III. that the husband treats her with cruelty, that is to say, that the husband treats her with
cruelty, that is to say,—“

IV. Habitually assaults her or makes her life miserable by cruelty of conduct even if such
conduct does not amount to physical ill-treatment.

V. if he has more wives than one, does not treat her equitably in accordance with the
injunctions of the Quran;

ANALAYSIS:
The Court took a very different approach in this case. The court considered the main issue
whether the husband did not treat his wife equitably contrary to the injunctions of the Quran. The
court felt the necessity to allow a woman to walk out of a marriage if she was not happy and
satisfied with the marriage.
The court brought to the attention of article 21 to elucidate further. They further stated that the
right to life also included the right to a healthy matrimonial life. They agreed that it might not be
something everyone accepts, but the right to walk out of an emotionally dead marriage is
synonymous with what the mandate of article 21 is all about.

The courts further go on to talk about how polygamy in itself is not a correct practice. The courts
however feel judging the constitutionality of the practice is not the main issue, but the right of a
woman to walk out of a marriage if she feels the husband is not treating her equitably is the crux.
Taking the Ayat Sura of the Quran to explain the rights a woman has in a marriage. The Ayat
Sura clearly explains how a man can marry one, two or three wives. However if he cannot
maintain them he should marry only one. The courts understand that the whole idea of sharing
your husband with another woman, itself is a wrong to the first wife. If a wife wishes to walk out
of the marriage, she must be allowed to. If fair and equal treatment is absent, the court saw no
reason for a woman to tolerate the oppression faced in such situations. They go on to say that It
would be a blasphemy for anyone to say that this statement of the general principle – that
however ardent the husband may try he will not be able to be fair and just between his wives, is
incorrect. If he cannot manage to treat all of them equitably, he must not marry more than one
person.

75 | P a g e
The court uses the precedent of the Jameela case, to show how the Prophet himself granted
Jameela the right to divorce simply because she could not draw any happiness and content from a
marriage. The court held that marriage was a sacred practice meant only for two people. The
whole idea of a third person being a part of it, is enough a reason for a woman to feel the right to
walk out of a marriage.

The courts also brought in the necessity of having a uniform civil code that would ensure an
equal treatment for everyone under the law. They felt the necessity to incorporate some easy, non
complex, user friendly procedures from the Muslim Law.
CONCLUSION:

The courts declared the claim for divorce under section 2 viii (f) of the act, as well as the
assertion of the
Wife in a polygamous marriage that she has been treated inequitably by her husband must be
accepted by the court.

MY VIEW:
I think the court was correct in allowing the wife to claim for divorce. My opinion is driven by
a. If a man cannot treat all his wives equitably, he must not marry more than one person. In
such cases, a wife must have the right to walk out of a marriage.
b. The right to life does include the right to have healthy matrimonial life.
c. Being a socialist state, giving respect to all forms of justice and equality are necessary.
d. This case brought out the rights a woman had, and when she could judicially apply for
dissolution of the marriage.
e. The minute a party to a marriage feels the lack of satisfaction, happiness or content, she
must be allowed to dissolve the marriage.
For all these reasons stated above, I feel the court was right in awarding the wife divorce in
this case. This case went on to be an ideal case for women in the future to draw a line
between religious practices like polygamy and to what extent she had to tolerate and put up
with this.
Our class discussions were also along the same lines, we saw how a Muslim man could
easily get divorce. This case laid the foundation as well as the concept that a woman could
apply for divorce under the dissolution of Muslim marriage. The whole concept of divorce in
Muslim law itself was a complex process and very gender specific. This judgement was vital
in enforcing the rights a woman had, and to what extent a man cannot misuse the power of
polygamy. To reach this conclusion the court had specific people look into what the essence
of polygamy in the Ayat Sura of the Quran and bring in the correct interpretation of what the
Islamic law actually meant.

Summary 2:

76 | P a g e
FACTS:
• The parties are the spouses.
• Wife’s (respondent) position is that her husband contracted a second marriage and
started residing with the second wife. According to her, he did not treat her equitably in
accordance with the injunctions of the Quran.
• Thus, she made an application for divorce under Section 2 of the Dissolution of Muslim
Marriages Act, 1939 (hereinafter referred to as “the Act”).

PROCEDURAL HISTORY:
• The parties went on trial on certain allegations that the husband denied.
• The wife was able to prove the second marriage.
• The lower court found the wife entitled to a decree for divorce.
• The husband (appellant) appealed to the High Court. The High Court is primarily
concerned with the decree for divorce under Section 2(viii)(f) of the Act.

RULES:
Dissolution of Mustlim Marriages Act, 1939
Sec. (2): Grounds for decree for dissolution of marriage. A woman married under
Muslim law shall be entitled to obtain a decree for the dissolution of her marriage on any
one or more of the following grounds, namely:

(viii) that the husband treats her with cruelty, that is to say:

(f) if he has more wives than one, does not treat her equitably in accordance with
the injunctions of the Quran.
Translation of Quranic Injunctions:
• Ayat 3, Sura IV: The condition precedent for polygamy is the husband’s ability to justly
deal with all of his wives. If he is unable to do so, only one marriage is permitted for him.
• Ayat 129, Sura IV: In a polygamous marriage, the husband may try most ardently to
deal with his wives justly and equitably, but the Lord declares that this shall be
impossible.

ISSUE:
What is the applicable standard for assessing equitable treatment of the wife in accordance
with the injunctions of the Quran? (Is it the partisan standards of the polygamous husband?
Or the assessment of the helpless wife? Or the cold, objective standards of the Court, an
outsider?)
Mansi Sheth (20141126) B.A. LL.B. 2014 (Sec. A) Family Law I: Presentation

77 | P a g e
ANALYSIS:
• Proper understanding of the concept of marriage in Islam in the context of the
fundamental right to life is required to answer this question. Broadly speaking, Islam
allows spouses the option to dissolve their marriage if it does not facilitate the enjoyment
of life.
• Right to life must include the right to a healthy and harmonious matrimonial life.
Consenting adults can err in their decision. Marriage becomes meaningless if it is to be
endured and not enjoyed. The right to opt out of a dead marriage has to be accepted as an
incident of the right to life. Therefore, the constitutional perspective of the fundamental
right to life must be used to interpret and understand the true meaning of the expression
“does not treat her equitably” in Sec. 2(viii)(f) of the Act.
• Marriage is a space that can accommodate only two individuals. If a third one intrudes
and one of the people within the matrimony is unwilling to accommodate the third, the
unwilling second must have the option to walk out. Sec. 2(viii)(f) thus provides an escape
route for a married woman faced with this situation, whereby a wife can seek dissolution
of her marriage if her husband has not treated her equitably (different from ‘equally’).
This principle can be found in the injunctions of the Quran, too. Ayat 3 states that if a
man can deal with them justly, he can marry up to four wives. Ayat 129, however,
declares that it is impossible for him to treat them equitably, no matter how hard he tries.
• The two Ayats seem to be in contradiction with one another, though there may be an
explanation for this. At the time when the message of God was relayed through the
Prophet, males were heavily outnumbered by females in a war-ravaged society. Hence,
polygamy prevailed. On account of compulsions, women may be able to accommodate and
tolerate more than one wife under the umbrella of marriage satisfactorily. Only then can
such polygamous marriage be permitted. The underlying assumption, thus, is that the
wife would like to continue in a polygamous marriage only if she finds happiness and
contentment in such a marriage. Otherwise, she can approach the concerned authorities
to get her marriage dissolved.
• This and this alone can be the true interpretation of the words “treat her equitably in
accordance with the injunctions of the Quran”, i.e. it is the wife’s assessment that matters,
not the assessment of the partisan husband. The Quran itself declares that it is impossible
for a man, however ardent his desire be, to deal fairly and justly with more wives than
one. Hence, it is for the wife to decide whether she is satisfied that her husband is treating
her equitably; she is the best judge.
• The question then arises whether a mere assertion of inequitable treatment by a wife in a
polygamous marriage is sufficient to fulfill the requirement of sec. 2(viii)(f). When it
comes to the unilateral right of the husband to divorce a wife, he need not disclose
intimate, confidential details to any outsider and he can divorce her without the
intervention of the courts. A wife may feel that her husband is not treating her equitably.
She may feel that he does not care about satisfying her physically and mentally. In the

78 | P a g e
scheme of Islamic law, the wife should also not be obliged to reveal to anyone such
intimate and intricate details of unjust and inequitable behaviour towards her. Thus, her
assertion of being treated inequitably is sufficient. While sec. 2(viii)(f) does not recognize
a polygamous marriage by itself as a ground for divorce, when read reasonably, the
provision concedes to the wife the right to walk out of a marriage if she is satisfied that
she has not been treated equitably.
• Furthermore, to permit another to share the matrimonial bed with her husband, to be
perpetually in competition with another for the love and attention of her own husband, to
suffer the ignominy when her husband goes after another woman ignoring her – are all a
negation of the right to live a meaningful life with dignity, self respect and honour. Thus,
Mansi Sheth (20141126) B.A. LL.B. 2014 (Sec. A) Family Law I: Presentation
the right to divorce under sec. 2(viii)(f) should be given the widest possible meaning to
enable a wife to escape from a polygamous marriage.
• Several precedents also support this inference.
o Single Judge of Allahabad High Court in Itwari v. Asghari (1960) observed that the
very act of taking a second wife raises a presumption of cruelty to the first.
o Similarly, a Division Bench of the Kerala High Court in K. Muhamma Latheef v.
Nishath (2003) opined that if, during the subsistence of a valid marriage, the
husband had remarried another, necessarily that will be a mental cruelty towards
the first wife.
o There seems to be a conflicting judgment of the Kerala High Court in Saidali v.
Saleena (2008) where the Court held that the admittion/proved second marriage
and the mere refusal of the first wife to cohabit cannot by itself be reckoned as a
reason to order divorce under sec. 2(viii)(f). However, this judgment does not refer
to the Division Bench judgment in Muhamma Latheef. Also, the court below did
not go into evidence, so the High Court had no evidence to infer that the wife had
made an assertion about being treated inequitably. The matter was sent back to
the lower court for evidence and fresh decision. No law contrary to the one laid
down in this particular judgment was laid (i.e., assertion of wife sufficient to prove
inequitable treatment).

CONCLUSION:
• Ratio: In a claim for divorce under sec. 2(viii)(f) of the Dissolution of Muslim Marriages
Act,
1939, the mere assertion of inequitable treatment by a wife in a polygamous marriage is a
sufficient standard for assessing equitable treatment in accordance with the injunctions of
the Quran.
• Holding: In this case, since the second marriage was proved and the first wife asserted in
her petition and evidence that she was not treated equitably, there was sufficient grounds
for divorce under sec. 2(viii)(f) of the Act. The challenge against the order of divorce by

79 | P a g e
the Court below was thus rejected.

CRITIQUE:
• At the outset, I must acknowledge the efforts of the Kerala High Court in attempting to
elevate the position of the wife with respect to divorce laws and polygamy in Islam. This
is a very woman-friendly judgment and deserves applause for the law it has laid down.
• However, I feel that the reasoning given by the Court for ruling as above is flawed and it
has deviated from the statute in doing so. The statute provides that if a wife is being
treated with cruelty, for e.g. if her husband has more wives than one and he does not treat
her equitably in accordance with the Quran, then the wife can file for divorce. The issue
that the Court has found here is that of the standard of equitable treatment – but the
answer is given in the statute itself, i.e. “in accordance with the Quran”.
• The Court’s interpretation of the Quran is slightly dubious – a man can marry up to four
wives if he can treat them all equitably, but actually that is quire impossible. This
inherent contradiction in the Quran seems to be a result of misinterpretation.
o Firstly, the injunctions of the Quran that the Court has selected do not say
anything beyond what the statute does. The Court has not found those
injunctions which actually define the meaning of equitable or inequitable
treatment in a polygamous marriage.
o Secondly, the Court has tried to reconcile the contradiction it has found by
arriving at its own desired conclusion that it is up to the wife’s assessment because
Mansi Sheth (20141126) B.A. LL.B. 2014 (Sec. A) Family Law I: Presentation
anyway the Quran says it’s impossible for the husband to treat his wives
equitably. This is far fetched because that is not what the Quran elucidates at all.
o Thirdly, if the Quran really meant that a man can never treat all his wives
equitably, then why would Ayat 3 allow a man to marry more than one woman in
the first place? If that is what the Quran meant, then polygamy itself would not be
permissible in Islam and sec. 2(viii)(f) would be rendered meaningless.
• Therefore, because the Court is an outsider who is alien to the Islamic religion, it is
apparent that it’s bias against the Islamic practice of polygamy (of which there are
numerous instances in the written judgment) has influenced its interpretation of the
Quran with respect to the selection of verses, the order in which they are presented, and
the manner in which they are tweaked and abstracted. The Court has likely pulled these
injunctions out of context and given them an extremely narrow interpretation consistent
with its own beliefs in order to give sec. 2(viii)(f) the widest possible meaning.
• The Court, after making these unconvincing interpretations of the Quran, has concluded
that if polygamy has been proven and if the wife feels that she is being treated
inequitably, then she can walk out of the marriage. It does not matter if the wife has
consented to the second marriage, or if she is a second wife entering into a polygamous
marriage knowingly, or if she has lived happily with the co-wife. In effect, this means that

80 | P a g e
a polygamous marriage alone is sufficient grounds for a woman to file for a divorce just as
long as she makes a mere assertion that she is not being treated equitably. This begs the
very question of finding a reasonable standard for assessing equitable treatment
altogether.
• The Court justifies this interpretation by relying on precedents that state that polygamy
itself is cruelty to the first wife. Apart from the fact that this is a value judgment that
objective courts should not be passing, if sec. 2(viii)(f) is read with this interpretation, it
makes absolutely no sense. Sec. 2(viii)(f) states that a wife may walk out of a marriage if
she is treated cruelly while it allows for polygamy so long as she is treated equitably. But
if polygamy itself were cruelty to the wife, then sec. 2(viii)(f) would make no sense.
• Having said all of this, I think that the end outcome of this judgment is a positive one
because it provides a good balance between the rights of a husband and wife to dissolve a
marriage (i.e. without giving any reason). It elevates the position of women and attempts
to give them equal footing as men. However, although I understand that the Court is
attempting to give sec. 2(viii)(f) a wide interpretation in a bona fide manner, it is a poor
attempt in strict legal terms and it can be improved in the following ways:
o Commentaries on the Dissolution of Muslim Marriages Act, 1939 should be referred
to in order to pinpoint the relevant Quranic injunctions that sec. (viii)(f) points
out.
o The Quranic injunctions should be analysed in more depth and by looking at a
variety of sources and authorities rather than just the one that the Court has
followed.
o The standard for assessing equitable treatment must involve evidence, i.e. it
should be a triable issue under this section, so as to prevent misuse.
o In this particular case, the Court should have looked into the other facts to give a
more robust reasoning for its final decision of upholding the decree for divorce,
such as denial of maintenance, failure to perform marital obligations, physical
cruelty, denial of right to property, etc. However, the Court has brushed aside
these facts as unnecessary, instead.
• Nevertheless, I endorse the view that women should be allowed to walk out of a
polygamous marriage because polygamy itself is against the fundamental right to life.

V. WEEK 9, 10 & 11: Matrimonial Reliefs & General Bars to Matrimonial Reliefs
(14th, 21st & 28th September)
a. Restitution of Conjugal Rights / Judicial Separation
i. T. Sareetha v. T.Venkata Subbaih AIR 1983 AP 356

NOTE: Refer to Kusum (Book) Discussion on Restitution of Conjugal Rights

BRIEF NOTES:
81 | P a g e
T. Sareetha vs T. Venkata Subbaiah: The Court held Section 9, the restitution of conjugal
rights is a section which is un-constitutional as it goes against the right of freedom of life, liberty
and dignity and privacy. The Court held that the company of a person includes right to the
society of the spouse, and martial sex. By passing a decree under Section 9, the State will be
forcing an unwilling spouse to have sex with another spouse and attaching their property in case
of failure to do. This say, the right to consent to sex passes from the individual to the state. At the
same time this also takes away the right of the women to decide how and when her body will be
used as a vehicle for another living being and this would be totally unjustified and could alter her
life events drastically. The right to privacy belongs to each person individually, and it doesn’t get
dissolved on the grounds of marriage.

Though the state can put reasonable restrictions on the fundamental rights, it is only when there
is a greater state interest in doing so, and there is no such reason in restitution of conjugal rights.
Section 9 doesn’t even satisfy the traditional classification test because it has come from the
English society, and isn’t inherit to the Indian culture. It even fails the equality test because
though this remedy is available to both spouses, a woman, who is inherently unequal to a man,
rarely uses it, hence making the law partial and one-sided. Secondly it fails to pass the test of
minimum rationality required of any state law. Hence the Court held Section 9 to be
unconstitutional. Also, the Court held that ‘resided’ as mentioned in HMA refers mainly to the
matrimonial home and where both the parties have stayed together for a long period of time.
Temporary and causal places of residence is not accounted in the HMA.

ii. Harvinder Kaur v. Harmander Singh AIR 1984 Del 66

NOTE: Refer to Kusum (Book) Discussion on Restitution of Conjugal Rights

Summary 1:
Issue
• Whether there has been reasonable excuse for withdrawal from the society, the burden of
proving reasonable excuse "shall be on the person who has withdrawn from the society?

• Is there a just cause for the respondent to live apart and separate from the petitioning
spouse?"
Rule
• Section 9 of the Hindu Marriage Act, 1955 defines the Restitution of Conjugal Rights as:
When either the husband or the wife has, without reasonable excuse, withdrawn from
the society of the other, the aggrieved party may apply, by petition to the district court,
for restitution of conjugal rights and the court, on being satisfied of the truth of the
statements made in such petition and that there is no legal ground why the application
should not be granted, may decree restitution of conjugal rights accordingly.

82 | P a g e
• Section 13; Divorce. Any marriage solemnised, whether before or after the
commencement of this Act, may, on a petition presented by either the husband or the
wife, be dissolved by a decree of divorce on the ground that the other party has, after
the solemnisation of the marriage, had voluntary sexual intercourse with any person
other than his or her spouse; or has, after the solemnisation of the marriage, treated
the petitioner with cruelty; or has deserted the petitioner for a continuous period of not
less than two years immediately preceding the presentation of the petition; or has
ceased to be a Hindu by conversion to another religion;
Analysis:

• According to Chaudhary J, sexual cohabitation between unwilling, opposite sexual


partners." He called it "forced sex", "coerced sex" and "forcible marital intercourse". A
wife who is keeping away from her husband, because of permanent or temporary
arrangement, cannot be forced, without violating her right to privacy, to bear a child by
her husband" he said. A large number of English and American decisions have been cited
in support of this view.
• Under Sec 9. the court has power to make a decree of restitution of conjugal rights which
is the remedy available to enforce the return of a spouse who has withdrawn from
cohabitation. The decree, if granted, orders the respondent to return within a period of
one year to the aggrieved party. This period is specified in Sec 13 (i) (a) of the Act. "This
remedy is aimed at preserving the marriage and not at disrupting it as in the case of
divorce or judicial separation".

• Consortium means "companionship, love, affection, comfort, mutual services, sexual


intercourse. All these belong to the married state. Taken together they make up
consortium." Consortium has been defined as "a partnership or association; but in the
matrimonial sense it implies much more than these rather cold words surest. It involves a
sharing of two lives, a sharing of joys and sorrows of each party, of their successes and
disappointments. In its' fullest sense it implies a companionship between each of them,
entertainment of mutual friends, sexual intercourse all those elements which, when
combined. From the definitions of cohabitation and consortium It appears that sexual
intercourse is one of the elements that goes to make up the marriage.

• Further delving into their matrimonial life is not necessary. If there is no rupture of
marital relations and the parties are living together in the ordinary way of man and wife
there is no need to resort to section 9 of the Act. Section 9 is a means of saving the
marriage. It can be that the erring spouse comes on the right path and a broken home is
rebuilt. Section 9 in a sense is an extension of sub-sections (2) and (3) of section 23 of the
Act which encourage reconciliation by the court. The court is enjoined to make every
endeavour to bring about a reconciliation between the parties

83 | P a g e
• The matrimonial relationship has been terminated though marriage is subsisting. The
marriage is in name only and not in substance. In law and not in fact. The policy
underlying the legislation is that it is not conducive to the public interest that men and
women should remain bound together in permanence by the bonds of marriage the duties
of which have long ceased to be observed by either party and the purposes of which have
irremediably failed. Such a condition of marriage in law which is no marriage in fact
leads only to immorality and unhappiness

• The decree of restitution of conjugal rights acts as an index of connubial felicity. It is a


sort of litmus paper. It shows a change of heart if the restitution decree is obeyed. If the
decree is disobeyed it is an indicia that the parties have reached a stage of no return. The
object is to facilitate reconciliation in matrimonial cases.This it does by asking the
withdrawing spouse to return to the matrimonial home and gives one year's time to do so.
If the restitution decree is not obeyed the court dissolves the marriage on proof of non-
compliance of the restitution decree.

• The provision of Section 13 (1-A)(i) which provides a ground for divorce on the basis of
non-resumption of cohabitation for a period of one year or more after the decree for
judicial separation. That ground is available to both, the husband and the wife, because
the decree of judicial separation can be passed in favour of the husband also
under section 10 of the Act. But the provision in section 13(iii) is made only in favour of
the wife who has been' awarded maintenance 'against the husband either under section 18
of the Hindu Adoptions and Maintenance Act, 1956. or section 125 of the Code of
Criminal Procedure, 1973, or section 4'88 of the Code of Criminal Procedure, 1898.

• Another indication of the breakdown of the marriage because living apart and an award
of maintenance are strong indications that marriage has broken down beyond repair and
there is no hope of the wife ever returning to cohabitation with the husband.

• Section 13(1-A) is based on the breakdown theory. Section 9 is based on fault theory.
The truth is that the faulty theory has utterly failed and has provided no solution to
marriages which are on the rocks .The breakdown theory is a recognition of the defects
and demerits of guilt theory.
• According to the Indian legislature the marriage has not to be ended at once. The
breakdown of marriage does not take place in one day. It is a process in which the
spouses drift apart. First there is apathy, then indifference, and then positive aversion.
The consortium of husband' and wife, the kind of association which is only possible
between husband and wife, ultimately comes loan end.

• Section 9 is the only positive relief under the Act aiming at the preservation of the
marriage.

84 | P a g e
• A marriage is not a very casual commerce affair, . Marriage is a contract of the greatest
importance in civil institutions, and it is charged with a vast variety of rights and
obligations

• Sections 9, 13(1-A) and 13(iii) envisage the same pattern of living husband and wife
living together, husband behaving as husband and the wife behaving as a wife, doing
housewifely duties. Cohabitation is the common denominator of these provisions. This is
their warp and woof. Section 9 tries to bring them together. Sections 13 (1-A) and (iii)
dissolve -the union when there has been no resumption of cohabitation for the space of
one year and upwards.
Conclusion:

• Rohatgi J, “ the arguments which found favour with the learned judge in holding
that section 9 is constitutionally void are not sound. They, are a dangerous and fallacious
line of arguments, I cannot agree that section 9 is unconstitutional however the remedy
may be outmoded or out of tune with the times. The restitution decree in the scheme of
the- Act is a preparation for divorce if the parties do not come together. But the
Legislature first believes in coaxing 'and cajoling the withdrawing spouse to return to
cohabitation, a value it prizes most. ”

• On the whole case it appears to me that the wife deliberately broke up the matrimonial
home and brought to an end the matrimonial life, so far as she was able to bring those
results about. The breaker-up of the home is the wife. She has disrupted the matrimonial
home without sufficient cause. There is withdrawal not only from' intercourse but from
cohabitation without just cause.

• For these reasons the appeal is dismissed. The decree of the trial judge dated 20-4-1981 is
affirmed. The parties are left to bear their own costs.

BRIEF NOTES:
Harivinder kaur V Harmandar Singh : The Delhi High Court upheld the constitutional
validity of Section 9. They said that one cannot introduce constitutional rights into
personal laws as it would amount to “introducing a bull in china shop.” According to the
Court, conjugal rights include cohabitation and consortium, and not merely sex, and there
is nothing wrong with it. According to this court, conjugal rights is a way to save
marriage as per Section 23(2) and (3) and at the same time serves the two fold purpose of
saving the marriage and making the spouse return to the matrimonial home, or proving
that the marriage is beyond salvage, and divorce should be granted.

iii. Saroj Rani v. Sudershan Kumar AIR 1984 SC 1562

85 | P a g e
NOTE: Refer to Kusum (Book) Discussion on Restitution of Conjugal Rights

BRIEF NOTES ON THE CASES:

Saroj Rani V Sudershan Kumar: The Supreme Court gave its final view on the constitutional
validity of Section 9 as a valid Section. The Court held that the attachment of property with
section 9 acts as an inducement for the party to try to reconcile. The institution of marriage gives
the spouses the right to each other’s company, and this decree derives its authority from there, it
is not merely a statutory enforcement. The Court agrees with the Delhi High Court reasoning that
this one year is a cooling off period and is necessary in order to save marriages.

Additional Case:

Dadaji Bhikaji vs Rukhmabai: The Court will not give a decree for restitution of conjugal
rights in a situation where the women is unwilling to cohabit with the husband, and hates him,
and the marriage was performed at a time when she has too young to understand anything. If she
has never lived with the husband, a decree of restitution of conjugal rights will not be passed to
help the husband consummate the marriage, because this would be barbarous and not in
accordance to principles of justice or according to the legislative intent behind Section 9.
Furthermore, if there was no cohabitation initially, there cannot be restitution of conjugal rights
because there cannot be restoration of something which never existed.

b. Grounds of Divorce under HMA & Irretrievable Breakdown Divorce


i. N.G. Dastane v. S. Dastane, AIR 1975 SC 1534

Summary :

Facts:

The parties were highly educated and belonged to a financially well-off background. They were
married on May 13, 1956. Between May and October of 1960 there were tensions between them
and each blamed the other of cruelty and harassment. Two children were born of the marriage,
one in 1957 and the other in 1959.

The petitioner alleged that his wife engaged in acts of cruelty such as, tearing off her mangal
sutra, locking him outside the house when he was due to come home from work. Furthermore,
she used to rub chilly powder on their young child’s tongue and beat their child, who was
suffering from high fever at the time, mercilessly. Constant threats of burning the house and
herself alive and constantly nagging the petitioner were construed as mental cruelty by the Court.
The matrimonial suit was filed by the husband under:

1. For annulment u/s 12 (1) (c) of the Hindu Marriage Act on the ground that the consent to
the marriage was obtained by fraud.
2. For divorce u/s13 (1) (iii) on the ground that the respondent was of unsound mind.

86 | P a g e
3. For judicial separation u/s 10 (1) (b) on the ground that the respondent had treated him
with such cruelty as to cause a reasonable apprehension in his mind that it would be
injurious or harmful for him to live with her.
Issue:

Whether a decree of judicial separation on ground of cruelty can be granted if cruelty is


established, but the same is condoned, even though the respondent has not specifically pleaded
condonation?
Applicable Statutes:

1. HMA- 12(1): Voidable marriages:


Any marriage solemnized, whether before or after the commencement of this Act, shall
be voidable and may be annulled by the decree if nullity of any of the following grounds,
namely:-

(c) that the consent of the petitioner, or where the consent of the guardian in marriage of
the petitioner [was required under section 5, as it stood immediately before the
commencement of the Child Marriage Restraint (Amendment) Act, 1978], the consent of
such guardian was obtained by force [or by fraud as to the nature of the ceremony or as to
any material fact or circumstance concerning the respondent].

2. HMA – 13 (1)
Any marriage solemnized, whether before or after the commencement of this Act, may,
on a petition presented by either the husband or the wife, be dissolved by a decree of
divorce on the ground that the other party

(iii) has been incurably of unsound mind, or has been suffering continuously or
intermittently from mental disorder of such a kind and to such an extent that the
petitioner cannot reasonably be expected to live with the respondent.

3. HMA – 10
Either party to a marriage, whether solemnized before or after the commencement of this
Act, may present a petition praying for a decree for judicial separation on any of the
grounds specified in sub-section (1) of section 13.

Note: u/s 13(1) of the HMA adultery, cruelty, desertion, conversion to another religion,
unsoundness of mind, and diagnosis of a virulent, incurable and venereal disease in a
communicable form, renunciation of the world based on a religious order, and
presumption of death resulting from a lack of contact with those reasonably expected to
hear from the respondent, are valid grounds for divorce.

87 | P a g e
Analysis:

Cruelty is a matrimonial offense can be either mental or physical. Under the Hindu Marriage
Act the concept of cruelty as a ground for divorce can be pleaded by either spouse, but the
burden of proof lies on the spouse who is subject of cruelty. Acts of cruelty in a matrimonial
alliance include demand for dowry, attack on spouse’s repute either verbally or in a written form,
rude or abusive behavior, refusal to consummate marriage, or instances when the wife has an
abortion without the consent of her husband. Vague allegations and the normal wear and tear of
marriage do not constitute cruelty. The concept of cruelty has varied from time to time, place to
place and from individual to individual. The cruelty alleged may largely depend upon the type of
life the parties are accustomed to or their economic and social conditions, their cultural and
human values to which attaches importance. This means that no objective standard has been
developed to legally ascertain cruelty in a generalized fashion.

It is pertinent to note, that with respect to cruelty as a ground for divorce, the entire matrimonial
relationship must be considered. Cruelty need not consist of mere violent acts but of complaints,
accusations or taunts that may be injurious to one’ mental wellbeing and repute. The
consequences of the conduct deemed as cruelty is of greater significance than the nature of the
act in itself. Since the standard for cruelty is subjective and varied from a case to case basis,
precedent cases are of little value. Attributes such as the mental and physical state of the spouses,
their educational qualification and social backgrounds, and the spouses’ threshold or tolerance of
a certain behavior, must be taken into consideration whilst determining what may constitute
cruelty. Mala Fide intent is not an essential ingredient to cruelty but it is an important element
where it exits. In certain cases false allegations or accusations on either spouse may be held in
Court to amount to cruelty. Two cases to this effect are D. S. Zingade v. S. R. Zingade, in which
false charges of adultery by the husband amounted to cruelty on behalf of the wife. Also,
Bhagat v. Bhagat, where the husband filed for divorce on the ground of adultery. The wife
denied the allegations and accused her husband of unsoundness of mind. The husband then
amended his petition and alleged mental cruelty as a new ground for divorce. According to him,
the allegations made in the written statement per se constitute cruelty which entitles him
straightaway to a divorce without going into the original allegation of adultery.

Condonation involves forgiving the commission of a matrimonial offence by the offending


spouse and restoration of the offending spouse to the position as s/he occupied before the offence
was committed. There are two important elements of condonation, namely, forgiveness and
restoration. With respect to cruelty, where evidence of cruelty is as strong as the evidence of the
condonation, then one may not be able to bring a successful legal claim against the other party.
According to the Court “[c]ondonation of matrimonial offence is not to be like the full
presidential pardon under art. 72 of the constitution which, once granted, wipes out the guilt
beyond the possibility of revival. It is always subject to implied condition that the offending
spouse will not commit a fresh matrimonial offense”.

88 | P a g e
It is important to ascertain the extent to which a subjective standard for ascertaining cruelty may
be entertained in a court of law. Excessive subjectivity may make the legal recourse for filing for
divorce subject to more misuse as it may be premised on the whims and fancies of the parties in
a matrimonial alliance.
Conclusion:

The court held that there was condonation of the wife’s cruelty by the husband and after the
condonation there was no serious allegation against her which would revive her earlier cruel acts.
Thus even while cruelty was initially proved but in view of it being initially condoned by the
husband it was held that he was not entitled to the decree. His appeal was accordingly dismissed.

Brief Notes:

N.G. Dastane v. S. Dastane: This case laws down certain guidelines while understanding
cruelty. It firstly says that though cruelty is called a “matrimonial offence”, it does not refer to an
offence as described in Criminal statutes, and is rather a civil wrong. Hence the standard of proof
doesn't have to be beyond reasonable doubt and is through preponderance of evidence. Secondly,
to judge if cruelty took place, the Courts would not ask if a reasonable man would interpret an
action as cruelty. The Courts realize the diversity in humans and hence cruelty will be judged on
an individual standard. Thirdly, the Court clarifies that a marriage will face some wear and tear
and hence that should not be treated as cruelty. Cruelty will be something graver and more than
usual fiction in a relationship. Fourthly, the Court holds that that the educational and socio-
economic standard of the people have to be taken into account while deciding cruelty. Lastly, the
Court holds that having normal sexual intimacy with your spouse does not amount to
condonation of cruelty. Condonation means forgiveness of the matrimonial offence and the
restoration of offending spouse to the same position as he or she occupied before the offence was
committed. Its also subject to the offence not being repeated. However, sex would not amount to
condonation because cruelty isn’t an isolated affair to which you react and leave the household.
It is an ongoing affair, and sex being an integral part of marriage, would not amount to
condonation.

ii. Samar Ghosh v. Jaya Ghosh, (2007)4SCC511

Summary 1:

ISSUE:

The first issue that can be identified in this case is “Whether the respondent is guilty of cruelty as
alleged?”

The second issue in this case is “Whether the petitioner is entitled to a decree of divorce as
claimed?”

89 | P a g e
RULE:

Hindu Marriage Act, 1955

Section 13 (1): Any marriage solemnized, whether before or after the commencement of the
Act, may, on a petition presented by either the husband or the wife, be dissolved by a decree of
divorce on the ground that the other party-

(Ia) has, after the solemnization of the marriage, treated the petitioner with cruelty.

FACTS:

The appellant in this case is the husband Samar Ghosh and the respondent is the wife Jaya
Ghosh. Both of them are I.A.S officers. In 1984 the appellant and the respondent were married in
Calcutta under the Special Marriage Act, 1954.The respondent was a divorcee and had a female
child from her first marriage. The custody of the said child was given to her upon obtained a
decree of divorce against her first husband, Debashish Gupta, who was also an I.A.S. officer.

After the marriage respondent declared her decision unilaterally not to give birth to child for two
year and appellant should keep himself aloof from her as far as possible. Respondent refused
cohabitation and also stopped sharing bed with the Appellant. Appellant was not even permitted
to show his normal affection to daughter of respondent. According to the appellant, there was
imposition of rationing in emotions in the arena of love, affection, future planning and normal
human relations though he tried hard to reconcile himself to the situation created by the
respondent.

In 1985 the respondent was transferred to Calcutta in May 1985. Their residential flat at the
Minto Park Housing Estate stood allotted to the appellant. The respondent used to come to their
flat rarely. The appellant suffered from a prolonged illness. The respondent was in Bareilly. The
appellant could not go because of high temperature and indifferent health. She left him and went
to Bareilly even when there was no one to look after him during his illness. On her return, the
respondent remained in Calcutta for about four days, but she did not care to meet the appellant or
enquire about his health. The appellant and the respondent virtually began to live separately
from September 1985.

In 1986 the appellant was transferred to Murshidabad in but the respondent continued to stay in
Calcutta. The appellant stayed in Murshidabad up to April 1988. There he developed some health
problem and, therefore, he sought a transfer to Calcutta and came back there in September 1988.
The flat in which they were staying in Minto Park was allotted to the respondent as per the
standard convention.

In 1988 the appellant and the respondent again began living together in Calcutta. The appellant
again tried to establish his home with the respondent after forgetting the entire past.

90 | P a g e
In 1989 the respondent began to again live separately at her parent's house. She used to come to
the flat at Minto Park to cook food only for herself and leave for the office. The appellant began
to take his meals outside.

In 1990 the respondent started shouting that the appellant had no self-respect and as such was
staying in her flat without any right when a domestic servant associated to the appellant came to
the house. The appellant was literally asked to get out of that flat. He felt extremely insulted and
humiliated. Since then the appellant and the defendant are living separately.

Therefore, the appellant filed a petition of divorce on the ground of mental cruelty of respondent
in 1990.

JUDGEMENTS:

-TRIAL COURT

The trial court, subsequent to investigating the whole pleadings and confirmation on record,
arrived at the conclusion that the facts led to mental cruelty. The District Judge went to the
finding that the appellant had succeeded in demonstrating the instance of mental cruelty against
the respondent, consequently, the pronouncement was allowed by the request dated 19.12.1996
and the marriage between the parties was dissolved.

-HIGH COURT

The Division Bench of the High Court vide judgment dated 20.5.2003 reversed the

Judgment of the Additional District Judge on the ground that the appellant had not been able to
prove the allegation of mental cruelty. The High Court gave out the following reasons by again
and again stressing on the fact that the respondent belonged to high strata of the society and
certain situations, which were deemed as the basis of mental cruelty, could not have been
possibly applicable in her case

-It was certainly within the right of the Respondent-wife having such a high status in life to
decide when she would like to have a Child after marriage.

-Disbelieved the appellant on the issue of respondent's refusing to cohabit with him, because he
failed to give the date, month or the year when the respondent conveyed this decision to him.

-The appellant started living with the respondent, therefore, that amounted to condonation i.e. he
had forgotten each and every instances of cruelty.

-The appellant’s and the respondent's sleeping in separate rooms did not lead to the conclusion
that they did not cohabit.

91 | P a g e
-It was quite proper for the respondent with such high status and having one daughter by her
previous husband, not to sleep in the same bed with the appellant.

-Refusal to cook in such a context when the parties belonged to high strata of society and the
wife also has to go to office, cannot amount to mental cruelty.

-During illness of the husband, wife's not meeting the husband to know about his health did not
amount to mental cruelty.

-SUPREME COURT

The Supreme Court held that the High Court was wrong in putting aside a very much-
contemplated judgment of the trial court taking into account the right examination of the idea of
mental cruelty. Thus, the High's judgment Court was put aside and the District's judgment Judge
conceding the declaration of separation was restored. In the judgment provided by the Supreme
Court judge Justice Dalveer Bhandari 2 issues were discussed extensively which were:

1. Definition of mental cruelty:

The Supreme Court analyzed the situation by saying there cannot be any comprehensive
definition of the concept of 'mental cruelty' within which all kinds of cases of mental cruelty can
be covered as human mind is extremely complex and human behavior is equally complicated.

There can never be any straitjacket formula or fixed parameters for determining mental cruelty in
matrimonial matters. Mere trivial irritations, quarrels, normal wear and tear of the married life
happening in day-to-day life were not adequate for grant of divorce.

To constitute mental cruelty, conduct must be much more than jealousy, selfishness or
possessiveness, which causes unhappiness, dissatisfaction or emotional upset.

2. Instances which amount to mental cruelty in the case in question:

The instances of human behavior which were sought to be deemed as 'mental cruelty’ in this case
were:

-Respondent's refusal to cohabit with the appellant.

-Respondent's unilateral decision not to have children after the marriage.

-Respondent's going to the flat and cooking only for herself and the appellant was forced to
either eat out or cook his own meals.

-The respondent did not take care of the appellant during his prolonged illness in 1985 and never
enquired about his health even when he underwent the bye-pass surgery in 1993.

92 | P a g e
-Respondent's act of humiliating the appellant and virtually turning him out of the Minto Park
apartment. The appellant in fact had taken shelter with his friend and he stayed there till official
accommodation was allotted to him.

-Respondent and Appellant have been living separately since 1990 till 2007 i.e. the year of
pronouncement of Supreme Court’s judgment and thus, parties living separately for a sufficient
length of time and filing of petition for divorce carries the presumption that marriage has broken
down.

CRITICAL ANALYSIS

According to me, Justice Dalveer Bhandari has been able to produce a very fine judgment
including the reason as to why there is no definite definition of the term “Mental Cruelty “ in the
statutes and on what instances can mental cruelty be declared in this particular case.

In my opinion once accepted to be in institution of marriage, the couple has to respect the marital
bond and discharge obligations of marital life.

In England, under the Matrimonial Causes Act, 1973, the sole ground for which either party may
present a petition for divorce is that the marriage has broken down irretrievably. Long period of
separation is an indication of irretrievable breakdown. In our case the marriage started in 1984
lawfully but the parties never exhibited the fact that they used to live together at any point of
time. Firstly. It was their jobs that led them apart and then even being in the same state the
respondent preferred to live separately from the plaintiff. The plaintiff and the defendant
virtually began to live separately since 1984 and the final judgment of the Supreme Court was
pronounced in 2007, for nearly 20 years they lived separately and this is a considerable amount
of time which can be deemed as Irretrievable Breakdown Of Marriage.

There are certain emotions in a marriage which are depicted by both the parties entering into this
agreement, none of these emotions were depicted by the plaintiff as she carried a non-caring
attitude towards the plaintiff when she used to cook food only for her and not for her husband, a
lack of enquiry attitude towards the plaintiff when the husband was ill and she was in Bareilly
and even after coming back did not meet the plaintiff once, a lack of emotions attitude as she had
no feelings when the plaintiff had a bypass surgery in 1994.

Another element which needs to exist in a marriage is the combine decision making by both the
parties relating to the issues of cohabitation, child bearing etc. In our case the respondent
unilaterally took all these important decisions without even consulting the plaintiff and
announced it herself that she did not wanted to cohabit with him or give birth to any child ever.
Further, the plaintiff was devoid of giving any fatherly love to the existing daughter of the
defendant from her first husband and was asked to be as far as possible from her.

93 | P a g e
Lastly, the parties are expected to respect each other in order to run an arrangement of marriage
which was again violated by the defendant when she threw the plaintiff out of his own house
which was apparently named after her wife and was insulted by harsh and crude words.

These instances clearly depict that the plaintiff was mentally tortured and humiliated in all the
aspects and was never given the feeling of love, care or affection, which is expected in a
marriage. According to me where a marriage has ceased to exist in substance and reality there is
no reason to deny divorce and in our case there was no substance at all existing in the plaintiff’s
and defendant’s marriage as it was one sided. I believe that the preservation of the fiction of
marriage through a legal tie does not serve the purpose of the sanctity of marriage, and that
divorce should be looked upon as a solution, in such circumstances.

CONCLUSION:

Thus, divorce was granted to the plaintiff on the grounds of mental cruelty.

iii. Bipinchandra Jaisinghbai Shah v. Prabhavati, AIR 1957 SC 176

Summary 1:

Procedural History

This is an appeal by special leave against the judgment and decree of the High Court of
Judicature at Bombay dated August 22,1952, reversing those of a single Judge of that Court on
the Original Side, dated March 7,1952, by which he had granted a decree for dissolution of
marriage between the appellant and the respondent.

Facts of the case:

- The parties to the litigation are B.C. Jaisinghbai Shah who is the appellant and Prabhawati
who is the respondent.

- The respondent was married to the appellant in accordance to Hindu rites of the Jain
community.

- The appellant left for England in 1947 and returned to find that the respondent was having
amorous relations with one M and taxed her for developing intimacy with him.

- The respondent left for her father’s house on 24th June, 1947, on the pretext that she had to
attend her cousin’s wedding in scheduled in June.

- On July 15th, the appellant sent a notice to her stating that she had left without his wishes and
now he did not want to keep her and wanted her to send their minor son to him.

94 | P a g e
- The respondent's case that it was the appellant who by his treatment of her after his return
from England had made her life unbearable and compelled her to leave her marital home
against her wishes.

- However after the solicitor’s notice dated July 15, 1947, was received by the respondent,
attempts were made by her father and his relations to bring about reconciliation between the
parties but they failed owing to the attitude of the appellant.

- The appellant filed for divorce on the grounds under S 3.(1)(d) of the Bombay Hindu Divorce
Act that the respondent had been in desertion for over 4 years without reasonable cause and
without his consent.

Issues:

- Whether the respondent had been in desertion, entitling the appellant to have a decree for
divorce?

- Whether the defendant deserted the plaintiff for a continuous period of over four years prior to
the filing of the suit?

Rule:

Section 3 (1) (d) of the Bombay Hindu Divorce Act, 1947


3. (1) A husband or wife may sue for divorce on any of the following grounds, namely:..
(d) that the defendant has deserted the plaintiff for a continuous period of four years”.
Definition of desertion:
"Desertion" has been defined in section 2(b) under Bombay Hindu Divorce Act, 1947 in these
terms:'Desert' means to desert without reasonable cause and without the consent or against the
will of the spouse".
Section 13 (1) (ib) of The Hindu Marriage Act, 1955:
Section 13 (1) Any marriage solemnixed, whether before or after the commencement of this Act,
may, on a petition presented by either the husband or the wife, be dissolved by a decree of
divorce on the ground that that other party -
(ib) Has deserted the petitioner for a continuous period of not less than two years immediately
preceding the presentation of the petition.
Desertion:

95 | P a g e
Desertion in the Hindu Marriage Act, 1955 has been defined as the desertion of the petitioner by
the other party to the marriage without reasonable cause and without the consent or against the
wish of such party, and includes the wilful neglect of the petitioner by the other party to the
marriage, and its grammatical variations and cognate expressions shall be construed accordingly.

Rationale

- The elements of desertion are:


• Factum of separation i.e. living away from the deserted spouse.
• Animus deserendi i.e. an intention to bring cohabitation to an end permanently. Further, it
should be proved that on the part of the deserted spouse, there was-
A. Absence of consent to the separation
B. Absence of conduct giving reasonable cause to the spouse to leaving the matrimonial house

• As observed from the above elements the appellant had not been deserted as the wife of the
appellant had been displaying a continuous desire to return home which signifies that she did
not want to end cohabitation. The appellant’s wife had left him for that period of time
knowing that he was in a state of anger and not with an aim to desert him permanently. The
court determines that PERMANENCE is a crucial component of desertion and if a life partner
forsakes the other companion in a condition of temporary passion or heat of the moment, for
instance, outrage or disgust, without meaning to stop cohabitation permanently, it won't sum
to desertion. For this situation, the respondent left the litigant keeping in mind the end goal to
let his reasons for alarm of the marriage and questions on her devotion waiver or vanish
before returning home.

• All through the four years, the respondent attempted to make endeavours to return to her
conjugal home just to be rejected by the litigant. She kept staying with the appealing party's
mom at whatever point she came to visit her which demonstrates that she was not hoping to
end the marriage. On the off chance that the accomplice is willing to resume cohabitation with
all ramifications of conjugal life, before the statutory period is out or even after the lapse of
that period, unless procedures for separation have been initiated, desertion arrives at an end
and if the deserted life partner absurdly denies the offer, the latter may be in desertion and not
the former.

• On the off chance that, actually, there has been a desertion, the fundamental question
dependably is whether that act could be attributable to intention. The offense of desertion
begins when the fact of separation and the intention exist together. In any case, it is not
important for both to commence at the same time.

96 | P a g e
Conclusion

The Lower Court granted the divorce petition but the High Court held that the wife was not
guilty of the offence of desertion as she continued to want to resume her marital responsibilities,
rights and cohabitation. The appellant’s appeal was dismissed.

iv. Dharmendra Kumar v. Usha Kumar, AIR 1977 SC 2213

No case summary

Brief Notes:

Issue: Is a wife, who had obtained a decree of RCR, who doesn’t respond to her husband’s
invitation to come and stay with him, disentitled to a decree of divorce under section 13 (1A),
HMA in view of Section 23 of the Act?

Facts: Wife obtained decree of RCR. A little over 2 years after the decree, she filed for
dissolution of marriage under S. 13 (1A) (ii) on the ground that there has been no RCR between
the parties for a period of 2 years (now this period has been amended to 1 year after 1976).
Husband admitted the same but mentioned that he had tried to comply with the decree by
‘writing several letters to the wife’ inviting her to live with him, but the petitioner wife refused.
Husband argued that the wife is trying to take advantage of her own wrong.

Held: “In order to be wrong within the meaning of section 23 (1) (a) the conduct alleged has to
be something more than a mere disinclination to agree to an offer of union, it must be
misconduct serious enough to justify denial of the relief of which husband or wife is otherwise
entitled.” In this case, the allegations made by the husband, even if true, do not constitute or
amount to misconduct grave enough to disentitle the wife the decree of divorce, the court held.

Comments: Judgments, like these are only backdoor entry to the concept of irretrievable
breakdown of marriage. The wife, in all likelihood, had no intention to honor RCR decree. And
while the husband may not have been sincere in his offers to call back the wife, it provided an
easy sprung-board to get out of the marriage and get a divorce on a statutorily recognized
ground.

v. Hirachand Srinivas Managaonkar v. Sunanda, AIR 2001 SC 1285

Summary 1:

FACTS

In this case, the respondent(wife) Sunanda filed a petition under section 10 of The Hindu
Marriage Act (HMA) seeking judicial separation on the ground of adultery on the part of the
appellant (husband) a decree for judicial separation was passed by the High Court of Karnataka

97 | P a g e
in 1981. The husband was ordered by court to pay maintenance of Rs.100 and Rs.75 to the wife
and daughter respectively. Since the passing of the order, the same had not been complied with
by the husband. Thereafter, the husband filed a petition for dissolution of marriage by a decree of
divorce on the ground that there has been no resumption of cohabitation as between the parties to
the marriage for a period of more than one year after passing of the decree for judicial separation.

The wife contested the petition for divorce on the ground, inter alia (among other things), that the
appellant having failed to pay the maintenance as ordered by the Court the petition for divorce
filed by him is liable to be rejected as he is trying to take advantage of his own wrong for getting
the relief. The High Court by the judgment dated 1995 accepted the plea taken by the respondent
and refused to grant the husband prayer for divorce.

CRITICAL ANALYSIS

The main question raised in this case was Whether the appellant by refusing to pay maintenance
to the wife has committed a wrong within the meaning of section 23 and whether in seeking the
relief of divorce he is taking advantage of his own wrong.

The husband not only failed to make any attempt of resumption of cohabitation but also refused
to pay alimony to the wife and daughter. In the circumstances it can reasonably be said that he
not only commits the matrimonial wrong in refusing to maintain his wife and further estrange the
relation creating acrimony rendering any re-approchment impossible but also tries to take
advantage of the said wrong for getting the relief of divorce. In the facts and circumstances of the
case, such conduct cannot be brushed aside as not a matter of sufficient importance to disentitle
him to get a decree of divorce under section 13(1-A).

Also the husband continued to live in adultery which is a continuing matrimonial offence. Such
an offence does not get wiped out merely on passing a decree for judicial separation by the
husband. The separation between the spouses only suspends certain duties and obligations of the
spouses in connection with their marriage and does not snap the matrimonial tie. In that view of
the matter accepting the contention raised on behalf of the appellant would, defeat the very
purpose of passing the decree for judicial separation.

The contention that the right conferred by sub- section (1-A) of section 13 is absolute and
unqualified and that this newly conferred right is not subject to provisions of section 23 is false.
This argument appears to be based on the erroneous notion that to introduce consideration arising
under section 23(1) into the determination of a petition filed under sub-section (1-A) of section
13 is to render the amendments made by the amending act No.44 of 1964 wholly meaningless.
Which states that prior to the amendment under clauses (viii) and (ix) of section 13 (1) the right
to apply for divorce was restricted to the party which had obtained a decree for judicial
separation or for restitution of conjugal rights. Such a right was not available to the party against
whom the decree was passed. Sub-section (1-A) of section 13 which was introduced by the

98 | P a g e
amendment confers such a right on either party to the marriage so that a petition for divorce can
after the amendment be filed not only by the party which had obtained a decree for judicial
separation or for restitution of conjugal rights but also for the party against whom such a decree
was passed.

This is the limited object and effect of the amendment introduced by Act No.44 of 1964. The
amendment was not introduced in order that the provisions contained in section 23 should be
abrogated and that is also not the effect of the amendment. The object of sub-section (1-A) was
merely to enlarge the right to apply for divorce and not to make it compulsive that a petition for
divorce presented under sub-section (1-A) must be allowed on a mere proof that there was no
cohabitation or restitution for the requisite period.

The very language of section 23 shows that it governs every proceeding under the Act and a duty
is cast on the Court to decree the relief sought only if the conditions mentioned in the sub-section
are satisfied, and not otherwise. Therefore, the contention raised by the learned counsel for the
appellant that the provisions of section 23(1) are not relevant in deciding a petition filed under
sub-section (1-A) of section 13 of the Act, cannot be accepted.

The next contention that arises for consideration is whether the appellant by refusing to pay
maintenance to the wife has committed a wrong within the meaning of section 13 and whether in
seeking the relief of divorce he is taking advantage of his own wrong. In Mullas Hindu Law it is
stated: Cohabitation means living together as husband and wife. It consists of the husband acting
as a husband towards the wife and the wife acting as a wife towards the husband, the wife
rendering housewifely duties to the husband and the husband supporting his wife as a husband
should. Cohabitation does not necessarily depend on whether there is sexual intercourse between
husband and wife. If there is sexual intercourse, it is very strong evidence it may be conclusive
evidence that they are cohabiting, but it does not follow that because they do not have sexual
intercourse they are not cohabiting. Cohabitation implies something different from mere
residence. It must mean that the husband and wife have begun acting as such and have resumed
their status and position as husband and wife.

After the decree for judicial separation was passed on the petition filed by the wife it was the
duty of both the spouses to do their part for cohabitation. The husband was expected to act as a
dutiful husband towards the wife and the wife was to act as a devoted wife towards the husband.
If this concept of both the spouses making sincere contribution for the purpose of successful
cohabitation after a judicial separation is ordered then it can reasonably be said that in the facts
and circumstances of the case the husband in refusing to pay maintenance to the wife failed to
act as a husband. Thereby he committed a wrong within the meaning of Section 23 of the Act.
Therefore, the High Court was justified in declining to allow the prayer of the husband for
dissolution of the marriage by divorce under section 13(1-A) of the act

99 | P a g e
In this connection it is also necessary to clear an impression regarding the position that once a
cause of action for getting a decree of divorce under section 13(1-A) arises the right to get a
divorce crystallises and the Court has to grant the relief of divorce sought by the applicant. This
impression is based on a mis-interpretation of the provision in section 13(1-A). All that is
provided in the said section is that either party to a marriage may present a petition for
dissolution of the marriage by a decree of divorce on the ground that there has been no
resumption of cohabitation between the parties to the marriage for a period of one year or more
after the passing of a decree for judicial separation in a proceeding or that there has been no
restitution of conjugal rights as between the parties to the marriage for a period of one year or
more after the passing of a decree for restitution of conjugal rights in a proceeding to which both
the spouses were parties.

Another question that arises for consideration is the meaning and import of section 10(2) of the
Act in which it is laid down that where a decree for judicial separation has been passed it shall no
longer be obligatory for the petitioner to cohabit with the respondent, but the court may, on the
application by petition of either party and on being satisfied of the truth of the statements made
in such petition, rescind the decree if it considers it just and reasonable to do so.

The question is whether applying this statutory provision to the case in hand can it be said that
the appellant was relieved of the duty to cohabit with the respondent since the decree for judicial
separation has been passed on the application filed by the latter. On a fair reading of the sub-
section(2) it is clear that the provision applies to the petitioner on whose application the decree
for judicial separation has been passed. Even assuming that the provision extends to both
petitioner as well as the respondent it does not vest any absolute right in the petitioner or the
respondent not to make any attempt for cohabitation with the other party after the decree for
judicial separation has been passed. As the provision clearly provides the decree for judicial
separation is not final in the sense that it is irreversible; power is vested in the Court to withdraw
the decree if it considers it just and reasonable to do so on an application by either party. The
decree for judicial separation does not sever or dissolve the marriage tie which continues to
subsist. It affords an opportunity to the spouse for reconciliation and re-adjustment.

The question that remains to be considered is whether in the facts and circumstances of the case
in hand the appellant- husband can be said to have committed and to be committing a wrong
within the meaning of section 23(1)(a) by continuing to live with the mistress even after passing
of the decree for judicial separation on the ground of adultery. To pursue still into such an
adulterous life with no remorse, even thereafter, is yet another wrong which he deliberately
continued to commit, to thwart any attempt to re-unite and, in such circumstances can it be said
that the passing of a decree for judicial separation has put an end to the allegation of adultery; he
cannot be said to have committed a wrong by continuing to live with mistress. This is however
not acceptable because as stated earlier, living in adultery on the part of the husband in this case
is a continuing matrimonial offence.

100 | P a g e
Thus based on the above discussion of the interaction of the relevant sections to the case, it was
held that the husband’s petition for dissolution of marriage was dismissed by the court as it held
that by granting such a decree of divorce, the husband will be taking advantage of his own wrong
i.e. failing to pay his wife alimony and continuing to live in adultery and as a result thwarting
resumption of cohabitation between him and his wife.

vi. Sureshta Devi v. Om Prakash, AIR1992SC1904

Summary 1:
Facts:

The couple was married on the 21st of November, 1968. They had lived together for 6-7 months.
Thereafter, it is said that the wife did not stay with the husband, except from the 9th of December,
1984 to the 7th January, 1985.
On 8 January 1985, Appellant wife and respondent husband filed for a petition under Hindu
Marriage Act under mutual consent.
Statements were recorded on the 9th of January.

On 15th January, the wife filed for the dismissal of the petition, saying that her statements made
on the 9th were obtained under threat and pressure from the husband.

District courts ordered the dismissal of the petition, while upon appeal the high court reversed
the order, dismissing the marriage as they found sufficient reason to believe that the consent
given by the wife wasn’t coerced.

Rule:

Section 13-B was not there in the original Act. It was introduced by the Amending Act 68 of
1976.Section 13-B provides:

13-B(l) Subject to the provisions of the Act a petition for dissolution of marriage by a decree of
divorce may be presented to the district court by both the parties to a marriage together, whether
such marriage was solemnized before or after the commencement of the Marriage Laws
(Amendment) Act, 1976, on the ground that they have been living separately for a period of one
year or more, that they have not been able to live together and that they have mutually agreed
that the marriage should be dissolved.

(2) On the motion of both the parties made not earlier than six months after the date of the
presentation of the petition referred to in sub-section (1) and not later than eighteen months after
the said date, if the petition is not withdrawn in the meantime, the Court shall, on being satisfied,
after hearing the parties and after making such inquiry as it thinks fit, that a marriage has been

101 | P a g e
solemnized and that the averments in the petition are true, pass a decree of divorce declaring the
marriage to be dissolved with effect from the date of the decree."

It is also necessary to read Section 23(l)(bb): 23(1) In any proceeding under this Act, whether
defended or not, if the Court is satisfied that-

(bb) When a divorce is sought on the ground of mutual consent, such consent has not been
obtained by force, fraud or undue influence, and ....."

Section 13-B is in pari materia with Section 28 of the Special Marriage Act, 1954. Sub-section
(1) of Section 13-B requires that the petition for divorce by mutual consent must be presented to
the Court jointly by both the parties. Similarly, sub- section (2) providing for the motion before
the Court for hearing of the petition should also be by both the parties.

There are three other requirements in sub-section (1). There are:

(i) They have been living separately for a period of one year.

(ii) They have not been able to live together, and

(iii) They have mutually agreed that marriage should be dissolved.

Analysis:

On the issue of unilateral dismissal and petition of marriage, the courts have had their separate.

Section 13 B was not there in the original act, but with the amending act 68 of 1976 it now says
that the filing of the petition with mutual consent does not authorise the court to make a decree
for divorce. There is a period of waiting from 6 to 18 months. This interregnum was obviously
intended to give time and opportunity to the parties to reflect on their move and seek advice from
relations and friends. In this transitional period one of the parties may have a second thought and
change the mind not to proceed with the petition.

The 'living separately' for a period of one year should be immediately preceding the presentation
of the petition. It is necessary that immediately preceding the presentation of petition, the parties
must have been living separately. The expression 'living separately', connotes to our mind not
living like husband and wife. It has no reference to the place of living. The parties may live
under the same roof by force of circumstances, and yet they may not be living as husband and
wife. The parties may be living in different houses and yet they could live as husband and wife.
What seems to be necessary is that they have no desire to perform marital obligations and with
that attitude they have been living separately for a period of one year immediately preceding the
presentation of the petition. The second requirement that they 'have not been able to live
together' seems to indicate the concept of broken down marriage and it would not be possible to

102 | P a g e
reconcile. The third requirement is that they have mutually agreed that the marriage should be
dissolved.

The Section does not provide that if there is a change of mind it should not be by one party
alone, but by both.

Sub-section (2) requires the Court to hear the parties which means both the parties. If one of the
parties at that stage says that "I have withdrawn my consent", or "I am not a willing party to the
divorce", the Court cannot pass a decree of divorce by mutual consent. If the Court is held to
have the power to make a decree solely based on the initial petition, it negates the whole idea of
mutualitly and consent for divorce. Mutual consent to the divorce is a sine qua non for passing a
decree for divorce under Section 13-B. Mutual consent should continue till the divorce decree is
passed.

Conclusion:

Appeal by the wife had been allowed by the Supreme Court.

vii. Naveen Kohli v. Neelu Kohli (2006) 3 SCC 491

Summary 1:

FACTS
The appellant, Naveen Kohli and the respondent, Neelu Kohli are husband and wife. Their
marriage was solemnized on 20th November, 1975. After marriage, respondent (Neelu Kohli)
started quarrelling and misbehaving with the appellant and his parents. The appellant also alleged
that he had found the Respondent in a compromising position with another man. Ultimately the
appellant left the parental residence and started to reside in a rented premises from May 1994.

The Respondent filed several cases against the Appellant under IPC and also with the Women
Cell, Delhi. The Respondent had published a public notice stating that her husband, Naveen
Kohli was working with her Proprietorship firm as a Manager and that he had abandoned his job
since May 1996 and had not resumed his duties. He was, thus, no more in the employment of the
firm and anybody who would deal with him shall do so at his own risk. His authority to represent
the firm had been revoked and no one could deliver him orders, cash cheques or drafts payable to
the firm. In this manner, she humiliated him publicly. Further, Respondent alleged that the
appellant was immoral and was having affairs with numerous girls since marriage.

The case was filed before the Family Court, Kanpur City which decided in favour of the
Appellant. The respondent, aggrieved by the judgment of the Family Court, Kanpur City,
preferred the first appeal before the High Court, which was disposed of by a Division Bench of
the Allahabad High Court.

103 | P a g e
ISSUE
1. Whether irretrievable breakdown of marriage is a ground for
divorce?

2. Whether the respondent treated the appellant with cruelty by registering various criminal
cases, getting the news published and initiating civil proceedings?

RULE
The rule applied to this case was:
 Section 13 of the Hindu Marriage Act, 1955 –

(1) Any marriage solemnised, whether before or after the commencement of this Act,
may, on a petition presented by either the husband or the wife, be dissolved by a decree
of divorce on the ground that the other party
(ia) has, after the solemnisation of the marriage, treated the petitioner with cruelty.

ANALYSIS
Applying the rule to the facts, the Trial Court found that there is no alternative but to dissolve the
marriage between the parties. The judgement was found on the basis of the fact that there was no
cordiality left between the parties and there is no possibility of their living together. The couple
had started to live separately. Several cases were filed by the Respondent against her husband.
There was no more scope left to save the marriage. There indeed was no point. The Trial Court
also ordered that a decree of dissolution of marriage shall be effective after depositing the
payment of Rs.5,00,000/- by the appellant. The appellant did so. Yet, the Respondent was
unhappy with this judgement and preferred the first appeal before the Allahabad High Court.

The Division Bench of the High Court held that actions of the appellant amounted to misconduct
and was un-condonable for the purpose of Section 13 (1) (a) of the Hindu Marriage Act. The
High Court set aside the judgement of the Trial Court and the appeal was allowed. The suit filed
by the appellant seeking a decree of divorce was also dismissed.

The Appellant was not happy with the ruling of the High Court and wanted to divorce his wife.
He took the matter to the Supreme Court. Thus, this case was now appealed to the Supreme
Court. It was determined that the petition for divorce was filed primarily on the ground of
cruelty. An important point here is that prior to the 1976 amendment in the Hindu Marriage Act,
1955, cruelty was not a ground for claiming divorce. By the 1976 Amendment, Cruelty was
made ground for divorce which is laid down in Section 13(1)(i)(a) of the Hindu Marriage Act,
1955. Cruelty has not been defined but the meaning can be inferred through various cases. For
instance, in the case N.G. Dastane v. S. Dastane [(1975) 2 SCC 326: AIR 1975 SC 1534], the
words which have been incorporated are "as to cause a reasonable apprehension in the mind of
the petitioner that it will be harmful or injurious for the petitioner to live with the other party”.

104 | P a g e
Also, the case Sbhoba Rani v. Madhukar Reddi [(1988) 1 SCC 105] reported that Cruelty may be
mental or physical, intentional or unintentional and it should be "grave and weighty”. Cruelty is
subjective and it must be something more serious than "ordinary wear and tear of married life".

In this case, the concept of Irretrievable Breakdown of Marriage came into the picture. When a
divorce is based on Irretrievable Breakdown, any wrong doing by one spouse or the other does
not matter. It is simply a statement by both spouses that the marriage won’t work any longer.
There is no need to prove that one’s spouse was to blame for the failure of one’s marriage. It is
not a ground for divorce under the Hindu Marriage Act, 1955. Yet, the judges use this concept in
practice. There are precedents for this concept. For instance, in the case of Chandrakala Menon
v. Vipin Menon [(1993) 2 SCC 6], parties had been living separately for many years. The court
concluded that there is no scope of settlement between them because the marriage had
irretrievably broken down and there was no chance of their coming together. Thus, the Court
granted the decree of divorce.
In our case, the Supreme Court, ultimately stated that it is for the Legislature to decide whether
to include irretrievable breakdown of marriage as a ground of divorce or not. Further, the
Supreme Court opined that the Legislature must consider irretrievable breakdown of marriage as
a ground for grant of divorce under the Hindu Marriage Act, 1955.

Also, the 71st Report of the Law Commission of India briefly dealt with the concept of
Irretrievable breakdown of marriage. The question before the commission was, whether the grant
of divorce be based on the fault of the party, or should it be based on the breakdown of the
marriage? The former is known as the matrimonial offence theory or fault theory. The latter has
come to be known as the breakdown theory. The Report considered whether the marriage has
ceased to exist in substance and in reality. Also, the Divorce should be seen as a solution and an
escape route out of a difficult situation.

Despite the above arguments, there are jurists who have another perspective towards this and are
against the concept of Irretrievable Breakdown of Marriage as a ground for grant of the decree of
divorce. In their opinion, such an amendment in the Act would put human ingenuity at a
premium, throw wide open the doors to litigation and will create more problems than are sought
to be solved.

CONCLUSION
It was, thus, concluded by Supreme Court that the High Court was not justified in setting aside
the order of the Trial Court. It stated that the High Court ought to have visualized that
preservation of such a marriage is totally unworkable since it has ceased to be effective and
would be a greater source of misery for the parties.

105 | P a g e
It was also recommended by the Supreme Court to the Legislature that the Union of India should
consider bringing an amendment in the Hindu Marriage Act, 1955 and incorporate Irretrievable
Breakdown of Marriage as a ground for the grant of a decree of divorce.

CRITICAL ANALYSIS
In my opinion, the judgement laid down by the Supreme Court and thereby the setting aside of
the judgement of the High Court was absolutely correct. The facts of this case suggest that there
was no cordiality remaining between the husband and the wife. The wife had gone to the extent
of humiliating her husband publicly. Moreover, they were living separately for quite a few years
now. There was no trust or happiness remaining. It was not possible for them to get back
together and start living cordially as husband and wife. The case was that of Irretrievable
Breakdown of Marriage. Thus, there was no point in saving the marriage. The parties would
remain happier if a decree of divorce was granted on the basis of Irretrievable Breakdown of
Marriage.

The concept of Irretrievable Breakdown of Marriage is not a ground for granting divorce under
the Hindu Marriage Act, 1955. Yet, I believe that the legislature should accept the
recommendation of the Supreme Court and make amendments in the law such that Irretrievable
Breakdown of Marriage becomes a ground for grating a decree of divorce. If husband and wife
are not willing to live together and their relationship has turned so sour that it is not possible for
them to live cordially then they should be entitled to divorce.

The legislature should frame the law in such a manner that this concept is not used for wrong
purposes. As mentioned earlier, there are opinions that suggest that this concept can be used by
people who have mala fide intention and will cause harm to the other party. The judge while
granting a decree of divorce on this ground should be tactful and should scrutinize the facts and
evidences properly to determine that there is no mala fide intention.

viii. V.Bhagat v. D. Bhagat AIR 1994 SC 710

Summary 1:

I. Case Details

1. Case Name: V. Bhagat vs. D. Bhagat (1994) 1 SCC 337


2. Relevant Section: Hindu Marriage Act, 1955 - Section 13(1)
3. Acts/Rules/Orders: Hindu Marriage Laws (Amendment) Act, 1976 – Section
10; Hindu Marriage Act - Section 13(1)
4. Hon'ble Judges/Coram: Kuldip Singh and B.P. Jeevan Reddy, JJ.

II. Facts

106 | P a g e
The husband sued for divorce on the ground that the wife is guilty of adulterous course of life.
The wife not only denied the allegation she attributed the allegation to lack of mental equilibrium
of the husband. The husband then amended his petition; he alleged a new ground for divorce
viz., mental cruelty. According to him, the allegations made in the written statement per se
constitute cruelty which entitles him straightaway to a divorce without going into the original
allegation of adultery. He is also relying upon certain questions put to him in cross-examination
by the counsel for the respondent and the said counsel's explanatory statement made in that
connection.

III. Issue

Whether the allegations made by the wife in her written statement and the questions put by her
counsel to the petitioner in cross examination amount to mental cruelty within the meaning of
Article 13 (1)(i)(a) of the Hindu Marriage Act, 1955?

IV. Rule

1. Section 13 (1) (ia) of the Hindu Marriage Act, 1955: Divorce

(1) Any marriage solemnized, whether before or after the commencement of this Act may on a
petition presented by either the husband or the wife, be dissolved by a decree of divorce on the
ground that the other party -

(ia) has after the solemnization of the marriage treated the petitioner with cruelty.

V. Analysis

In the present case, the court held that the allegations made by the wife in her written statement
and the questions put by her counsel to the petitioner in cross examination amounted to mental
cruelty within the meaning of Article 13 (1) (ia) of the Hindu Marriage Act, 1955.

The Court looked at various Indian as well as English cases to conclude that:

Mental cruelty in Section 13(1) (ia) can broadly be defined as that conduct which inflicts
upon the other party such mental pain and suffering as would make it not possible for that
party to live with the other… It is not necessary to prove that the mental cruelty is such as
to cause injury to the health of the petitioner. While arriving at such conclusion, regard
must be had to the social status, educational level of the parties, the society they move in,
the possibility or otherwise of the parties ever living together in case they are already
living apart and all other relevant facts and circumstances which it is neither possible nor
desirable to set out exhaustively. What is cruelty in one case may not amount to cruelty in
another case. It is a matter to be determined in each case having regard to the facts and

107 | P a g e
circumstances of that case. If it is a case of accusations and allegations regard must also
be had to the context in which they were made.

The Court held that the statements made by the wife must be read in the context in which they
were made and that at the same time, it must be remembered that the wife was merely defending
herself against what are, according to her, totally unfounded allegations and aspersions on her
character. The court however stated that it was not necessary for her to allege that the petitioner
is a mental patient, that he is not a normal person, that he requires psychological treatment to
restore his mental health, that he is suffering from paranoid disorder and mental hallucinations –
or to go to the extent of calling him and all the members of his family a bunch of lunatics. The
court observed that these statements were not made in a fit of anger or under emotional stress but
in a formal pleading and the questions were put by her counsel, at her instructions, in the cross-
examination.

The wife’s statements were thus seen not as the mere protestations of an injured wife, but
positive assertions of mental imbalance and insanity. The Court also observed that the husband
was an advocate practicing in the Supreme Court as well as in the Delhi High Court and that the
divorce petition was being tried in the Delhi High Court itself. Making such allegations in the
pleadings and putting such questions to the husband while he is in the witness-box, was thus
concluded to have been bound to cause him intense mental pain and anguish besides affecting his
career and professional prospects. The wife’s statements and allegations were thus seen as going
far beyond the reasonable limits of her defense.

The court thus found it abundantly clear that the marriage between the parties had broken down
irretrievably and there was no chance of their coming together, or living together again. The
court thus dissolved the marriage between the parties under Section 13(1) (ia) of Hindu Marriage
Act.

VI. Conclusion

The court dissolved the marriage between the parties under Section 13(1) (ia) of Hindu Marriage
Act.

ix. Savitri Pandey v..Prem Chandra Pandey AIR 2002 SC 591

Summary 1:

The marriage of the appellant and respondent was solemnized on 6.5.1987. The appellant wife
lived with her husband till 21st June, 1987 and according to her, the marriage between the parties
was never consummated.

108 | P a g e
She claimed that Rs. 80000 had been spent on the wedding proceedings, ceremonies and
ornaments in cash and kind. She also claimed that after the marriage had taken placed, the
respondent and his family demanded a colour TV, a refrigerator, and some ornaments
in addition to cash of Rs 10000. Her father was able to make the payment of Rs 10000 but was
unable to meet the other demands. It is the claim of the appellant wife that it was at this time that
the respondent and his family started torturing her on false pretexts.

Aggrieved by the attitude of the respondent and his family members, the appellant states to have
filed a petition under Section 13 of the Act seeking dissolution of marriage by a decree of
divorce along with prayer for the return of the property and grant of permanent alimony. The
respondent also filed a petition seeking divorce and grant of other reliefs. However, on 14.5.1996
the respondent filed an application for withdrawal of his matrimonial case which was allowed on
19.5.1996. The appellant had alleged that the respondent was having illicit relations with a lady
residing in Gaya at Bihar with whom he was stated to have solemnized the marriage. The
allegations made in the petition were denied by the respondent and it was stated that in fact the
appellant-wife was taking advantage of her own wrongs.

The case was primarily decided in a family court and consequently she was awarded a
divorce, Rs. 12000 as the cost of a scooter that her family had given him during the wedding,
and Rs. 500 monthly as alimony. However, both the husband and wife wished to appeal against
this, the husband because he was unhappy with the divorce being granted, and the wife because
she was not given the right to some properties which she laid claim to.

However, both the appeals were disposed of by the impugned order holding that the appellant-
wife herself was a defaulting party and neither the allegations of cruelty nor of desertion were
proved. The order passed under Section 27 of the Hindu Marriage Act and for permanent
alimony was also set aside. The grievance of the appellant-wife is that the Supreme Court was
not justified in setting aside the findings of fact arrived at by the Family Court and that she had
proved the existence of cruelty and desertion against the respondent. It is contended that as the
appellant-wife was proved to have been living separately, it was to be presumed that the
respondent had deserted her. However, it was ruled that such grievances were unjustified and she
was merely trying to take advantage of her own wrongs.

Yes, I agree with the decision. The appellant should not be allowed to take advantage of her own
faults in order to further her case. Desertion should also be such that there is a prolonged period
of estrangement with animus deserendi, and no further reconciliations. The deserted party
however should be willing to resume cohabitation under the circumstances as is in order to claim
desertion. In the present case, however, it becomes quite clear that the appellant left the marital
home of her own free will, and made no attempt for reconciliation though the respondent
husband did the same. Also, in the matter of cruelty, cruelty cannot be decided on the basis of the

109 | P a g e
sensitivity of the petitioner, and rather a thorough examination of the facts of the case is required
to prove cruelty as opposed to the normal wear and tear of normal family life.

Desertion was well examined in the case Bipin Chander Jaisinghbhai Shah v. Prabhawati. It was
stated that desertion would require the fulfillment of certain conditions, both as per the deserting
spouse and the deserted spouse as well. As for the deserting spouse, the conditions are the factum
of separation, and the intention to bring cohabitation permanently to an end (animus deserendi).
For the deserted spouse, two elements are essential; the absence of consent, and the absence of
conduct that would give the spouse reasonable cause to leave the matrimonial home and form the
necessary intention of animus deserendi. This holds for the present case as well, where it is clear
these conditions were fulfilled.

Cruelty was also visited in the cases of Dastane v. Dastane, Samar Ghosh v. Jaya Ghosh, Praveen
Mehta v. Inderjeet Mehta amongst others. In the case Dastane v. Dastane, it was held that cruelty
constituted acts of such a nature that there would be reasonable apprehension that cohabitation
would be harmful or injurious to the petitioner. In the landmark case of Samar Ghosh v. Jaya
Ghosh, it was observed that mental cruelty, not being discernible through direct evidence, would
be proved through the facts and circumstances of the case. These judgements are also holding
with the judgement made in the present case.

The Judges of the lower courts who had previously reviewed the case had failed to identify that
the appellant was taking advantage of her own wrongs. This should have not have been allowed
in a court of law. Also, the appellant wife had got remarried during the pendency of the
husband's appeal. This shows that the period of appeal should be increased from a period of 30
days to that of 90 days, in order so that unscrupulous litigant spouses may not frustrate the filing
of appeals, and also any marriage solemnized during the aforesaid period would be deemed to be
void.
The clarifications made in this case as regarding Cruelty and Desertion were very important
ones, and made it a landmark case. The importance of the distinction of cruelty from the wear
and tear of normal family life, as well as the appellant being unable to take advantage of her own
wrongs in leaving the marital home and claiming desertion were significant in particular.

x. Additional Case: Mutual Consent

Manish Goel V Rohini Goel: The court held that in a mutual consent divorce, the 6 months
period of waiting after the filing of the petition cannot be done away with. The Court held that
though Article 142 gives the Court the power to grant divorce in certain cases of irretrievable
breakdown of marriage, which are beyond salvaging, on grounds which are not provided by the
legislature, this does not give the Court the power to do away with the statutory regulations and
provisions like the cooling off period of 6 months. The Court cannot ignore the substantive

110 | P a g e
provisions of a stature and can use it only in cases which cannot be effectively and appropriately
tackled by the existing provisions of law or when the existing provisions of law cannot bring
about complete justice between the parties.

VI. Week 7: Christian Marriage & Divorce


a. Vinisha Tolani v. Jitesh Tolani, AIR2010SC1915

Summary 1

Facts: This petition is about the transfer of case from Goa court to Delhi court. Petitioner i.e.
Vinisha is a Sikh by religion living in London. She married the respondent i.e. Jitesh who is a
partner in a construction business in Goa. Marriage was performed before the Civil Registrar of
Goa, on 15th November 2007. Several incidents occurred there after which caused her to
commute between the United Kingdom and India till finally she took up residence in a rented
accommodation in New Delhi.
During this period an ex-parte order was passed against the petitioner, for declaring her marriage
to be null. She was shocked as she was not informed about this by the husband.
Thus, as a result of which the petitioner was compelled to file the present transfer petition to
Delhi court.

ISSUE: Whether Marriages in Goa or other territories can only be governed by the Civil Code
and cannot be judged by any other court?

ANALYSIS: The petitioner’s lawyer presented the case of Sumita Singh vs. Kumar Sanjay,
where it was held that being a matrimonial proceeding instituted by the husband against the wife,
the convenience of the wife had to be considered in contesting the suit and, accordingly, the
proceedings ought to be transferred to Delhi, where the wife was residing.
The respondent’s lawyer raised that civil proceedings relating to marriage were governed by the
Civil code of 1867 which was in force in Goa and that as a result, the petition for annulment
could only be tried in the State of Goa and not in any other State.

RULE: Two Hindus, can contract a marriage according to Hindu religious rites or by way of a
civil marriage.

CONCLUSION: The Civil Code as enacted in 1910, and the provisions of the law of Marriage
as a Civil Contract in Goa, Daman & Diu which came into force on 26th May, 1911, we are
unable to agree that all marriages performed within the territory of Goa unless registered should
be void. Section 2 of Hindu marriage act applies to whole of India.
We are not convinced that the annulment proceedings cannot be heard outside the State of Goa in
view of the existing laws which made the Civil Code and the laws relating to marriage applicable

111 | P a g e
to all persons residing within the State of Goa. Even though the spouses are domiciled in Goa in
respect of a marriage performed outside Goa but in any other State of the Union, they would be
governed by their personal laws in so far as dissolution of marriage is concerned. Transfer
Petition filed by the husband was, therefore, dismissed.

Summary 2

The Petitioner was an Afghan born, Sikh British national, who was given the status of an Afghan
refugee in her young years. She also resided in Delhi for around ten years before moving to UK.
She further married the Respondent with Hindu rites and rituals in Goa, got her marriage
registered under the Goa Civil Code, and started to reside in Goa with her husband. Few months
after the marriage, she was asked to return to UK to fulfil some formalities for the proper
registration of the marriage. She had to make several trips to UK and fro, only to settle down in a
rented flat in New Delhi. She found out that the Respondent had filed for annulment of the
marriage under Section(12) of the HMA, and had stated the Goa address as her address. The
Petitioner filed for a transfer petition of the annulment petition to the Family District Court of
Delhi.

The issue at hand is whether the petition for annulment of a Hindu marriage, registered under the
Civil Code of Goa, can be heard by a Court having jurisdiction within the territories to which the
Hindu Marriage Act, 1955 applies.

Section(2) of the HMA states that:

It extends to the whole of India except the State of Jammu and Kashmir, and applies also
to Hindus domiciled in the territories to which this Act extends who are outside the said
territories.

The Respondent argued that since the marriage had taken place in Goa, it would be governed by
the Goa Civil Code. Also, the convenience of the Petitioner does not come into question, since
she was a resident of London, and any location in India would be equally inconvenient for her.

The Supreme Court reasoned that since the said marriage was performed with Hindu rites and
rituals, it was a Hindu marriage, falling under the ambit of the Hindu Marriage Act. The court
held that on 22nd January 1946, the provision for validity of a marriage unless registered was
altered, and that Hindus/Catholics could solemnize a marriage either by rites and rituals or by
way of a civil marriage; thereby making the given marriage between the Petitioner and
Respondent a Hindu Marriage.

It concluded that the validity of the Hindu Marriage Act, 1955 runs across the country, except
Jammu and Kashmir, and thereby, the given case isn’t limited to Goa Courts. The case can be
tried at any Court of jurisdiction across the country.

112 | P a g e
With reference to the second contention made by the Respondent, the court concluded that since
the Petitioner had resided in New Delhi for ten years of her life, she had the support of family,
friends, and acquaintances in that city, making it more convenient for her in Delhi. However, in
Goa, she only had her husband, who was the Respondent, himself. It allowed the transfer petition
filed by the Petitioner.

The ratio decidendi of the case is that if a marriage is conducted by ways of rites and rituals, and
is thereby governed by personal laws, it can be heard by any Court having jurisdiction within the
territories to which it applies. This is an extremely important precedent, considering the
significance of easy accessibility to justice. A popular debate among the people since a very long
time is the access to justice in the country.

Many critics argue that the hierarchal nature and complexity of the courts in India, deny justice
to several victims. The uniformity and ease provided by this judgement is definitely a step
towards making justice a step closer.

b. Lakshmi Sanyal vs Sachit Kumar Dhar, AIR 1972 SC 2667

Facts
 The Appellant and the Respondent are closely related, their mothers being real
sisters. It appears that prior to the date of marriage i.e. January 30, 1960 they had
sexual relations as a result of which the appellant became pregnant. The Appellant
and Respondent who were originally Hindu had got converted to Christianity and
professed the Roman Catholic faith. On January 30, 1960 the marriage of the
parties was solemnized by Father Antoine at the Church of St. Ignatius, Calcutta.
 On May 10, 1960 the first child, a daughter, was born to the appellant. She gave
birth to a second child, also a daughter, in October 1961.t would appear that the
appellant left the home of the respondent in the year 1965.
Claims and Allegations

 In the petition a number of allegations were made relating to the conduct of the
respondent. It was alleged, inter alia, that it was under duress, intimidation and
undue influence that the sexual relationship started between the appellant and the
respondent which ultimately resulted in the appellant conceiving a child.
 The conversion to Christianity as also the performance of the ceremony of
marriage were all attributed to fraud, coercion and undue influence practised by
the respondent.
 It was claimed that the appellant was a minor at the time the marriage was
solemnised and the consent of her father or her guardian was not taken nor did she
give her own consent freely to the marriage.
 Further the marriage was void because the parties were within the prohibited
degree of consanguinity. All these allegations were denied by the respondent.

113 | P a g e
Issue
 Whether the marriage was invalid and void because the appellant was a
minor at the time the marriage was solemnised and admittedly the consent
of her father or guardian had not been taken?
 Whether the parties were within the prohibited degree of
consanguinity?(section 19 of the Divorce Act)
Held

 The High Court held that from the standpoint of Canon Law of the Roman
Catholic Church the objection to the validity of the marriage on the ground of
lack of consent could not be sustained.
 The High Court expressed the view that the consent of the parents was not
necessary as' required under s. 19 of the Indian Christian Marriage Act since
the marriage was solemnised by a person failing under s. 5(1), nor was there
any provision in the Indian Divorce Act.1869 which rendered a marriage null
and void on the ground of minority of a party.
 Section 5(1) granted Father Antoine a statutory authority to sanction their
marriage. It appears that under Canon Law so long as a minor has reached the
age of capacity to contract which is 16 years in case of a man and 14 years in
case of a girl the marriage can be solemnized and the lack or absence of
consent of the parents or guardian will not invalidate the marriage.
 Section 19 which is used by the appellant for the purpose of being declared a
minor at the time of marriage is not applicable to those Ministers of religion
falling under 5(1); Father Antoine belonged to this section.
 On the question of the marriage being within the prohibited degree of
consanguinity it was found that since the consanguinity between the par- ties
was of the second degree it was certainly an impediment in the way of the
marriage under the Roman Catholic law;
 But the impediment could be removed by dispensation which was granted by
the competent authorities of the church;
 For this reason the marriage could not be held to be null and void once such
dispensation had been granted.

Critical Analysis
The Court rightly held that the marriage was valid and the grounds for divorce
were based on false allegations. Any person who has converted to another religion
cannot use the laws of former religion to his or her benefit in scenarios like these.
The relationship between the husband and wife was governed by the canon law
114 | P a g e
and the canon was rightly applied in this case. The claim for consanguinity was
also false as the couple had exempted themselves from the impediment by the
permission of Church through Father Antoine. It was as their request that he
solemnized their marriage and in this regard there is no dispute. She was not a
minor at the time of the marriage under the canon law and nor does the canon
necessitate the consent of the guardian for such solemnization. Hence rightly held
and marriage valid.

c. Sujatha v. Jose Augustine, (1994) II DMC 442

Summary 1

Issue:

Was there a valid marriage between the wife and the husband according to the law? Can their
marriage be declared as null and void?

Rule:

Section 18 of Indian divorce act,1869: Petition for decree of nullity. Any husband or wife may
present a petition to the District Court praying that his or her marriage may be declared null and
void.

The rational behind the judgment was:

Marriage between a baptised Christian and a not baptised non-Christian shall be considered as a
void marriage.

Analysis:

Petitioner i.e. the wife was a Hindu by birth which has been proved by the evidence she has
provided. The baptism ceremony done to convert the wife into a Christian is not valid as she
never turned Christian by faith. She never believed herself to be a Christian. The baptism
ceremony done was not according to the common law and was also done by force and without
the consent of the wife.

According to the courts, marriage between a baptised and a non-baptised non-Christian person is
null and void. Wife never converted to Christianity and therefore, the marriage between the two
is declared to be null and void.

115 | P a g e
Conclusion:

The petition to declare the marriage as null and void was allowed on the basis that the wife never
converted to Christianity by faith.

Critical Analysis:

As T.V. Ramakrishnan, J., Gave the judgment to be void was totally justifiable. The wife didn’t
want to marry the respondent. The respondent forcefully made her go through the baptism
ceremony and later forced her to marry him. The judge’s decision to declare the marriage as void
gave justice to the petitioner and no kind of injustice happened to anyone. If the wife didn’t
convert to Christianity by faith, she cannot be considered a Christian.

Summary 2:

Counsels for the Appelant/Petitioner/Plaintiff: S. Sreekumar, Adv.


Counsels for Respondents/Defendant: P.K. Jose, Johnson Abraham and Babu P. Poulose, Advs.

To ease the process of undertanding the case, the write up will entail the following sections:
1. Facts: These will involve the petitioner’s plea and the oppositions that were faced by the
same.
2. Issue
3. Rule
4. Analysis: This will further include the ratio decidendi of the case.
5. Conclusion

FACTS
The petitioner was a Hindu by birth and belonged to the Nair community in her mother’s custody
till her alleged baptism and marriage. The first respondent was a Latin Catholic and was
employed as a conductor on a bus. According to the petitioner, they met on this private bus on
which the First Respondent was employed, during the Petitioner’s bus journeys to and from her
college. The acquaintance soon became very close as a result of regular meetings.
Petitioner’s Case: According to the Petitioner, the First Respondent enticed her innocence and
procured her love fraudulently. Further, he coerced her into an agreement for marriage on
1.11.1989 and then again to accompany him to his house on the 7.11.1989. On 25.11.1989, she
was taken to the Holy Family Church and was made to undergo a ceremony which she later
understood as the baptism ceremony. On 30.11.1989, she was again taken to the Church and was
married to the First Respondent against her free will. She later left hin on the 29.8.1990 and got
herself converted into Hindu community on the 17.10.990.
Respondent’s Case: All the allegations of the Petitioner were denided by the Respondent as
totally false. However, the allegations in the petition and the first respondent met each other

116 | P a g e
during her journeys in the bus in which the first respondent was working as a Conductor and as a
result of the intimacy they fell in love with each other and got married after the petitioner had
undergone a baptism ceremony etc., were not denied. In fact that is the common case of the
parties.

Petitioner’s plea: The petition filed for a decree declaring the marriage betweeen Petitioner and
the first respondent as null and void on the grounds that it hasn’t been conducted as per her free
will and also not in accordance to the Cannon Law governing baptism and marriage.

Defendant’s Appeal: The First Respondent filed a counter affidavit on his behalf and on behalf
of the second respondent denying the allegations put forth by the Petitioner and also that the
marriage between himself and the Petitioner was conducted with free consent and in accordance
to the usual custom and practices followed by the Latin Catholics.

ISSUE:

The only question to be considered is whether the marriage between the petitioner and first
respondent alleged to have conducted after an attempted conversion of religion by undergoing
the ceremony of baptism was in accordance with the law regulating marriages among the Latin
Catholic Community.

RULE

The applicable rules in the givne case are as follows:


1. Section 18 of the Indian Divorce Act, 1869:
Petition for decree of nullity.- Any husband or wife may present a petition to the District
Court or to the High Court, praying that his or her marriage may be declared null and
void.
2. Cannon Law-The relevant sections of the same are stated where they’ve been used.

ANALYSIS

Ratio Decidendi: "Marriage between a baptised Christian and a not baptised non-Christian shall
be considered as a void marriage."

In this particular case, there is practically no reliable evidence to prove any of the allegations of
the Petitioner. However, her conduct of staying over a year post the alleged forced marriage with
the Respondent negates the possibility of her being coerced into marrying him. As such it is only
reasonable to conclude that the petitioner was in love with the first respondent and that

117 | P a g e
emotionally charged condition was doing all that was directed to be done by the first respondent
and his friends and relatives with the dominant motive of getting somehow or other married to
the first respondent without the knowledge and consent of her mother.

An analysis of the principles contained in the Canon Law governing baptism and marriage would
show that a marriage between a baptised Christian and a person not baptised is void (Canon
1086). The principle laid done in Leelamma's case would show that a person can become a
Christian only if it is established that he truly believes in and professes the Christian faith.
Baptism if duly administered and received may be an important circumstance to conclude that
one has become a Christian.

Canon 851 and 865 dealing with the requirements of admission to baptism of adult person would
clearly show that the person who is admitted to baptism must have manifested the intention to
receive baptism, must be adequately instructed in the truths of the faith and in the duties of a
Christian, and tested in the Christian life over the course of catechumen ate. If these are the
requirements to be satisfied before an adult can be considered as baptised, the evidence on record
regarding baptism is totally insufficient to hold that the petitioner was duly admitted to baptism
and received baptism with the required intention and faith in Christianity.

The evidence shows that she never had faith in Christianity and in its tenets. She has specifically
stated that she was not interested in getting converted into Christianity. The evidence of the
Church is insufficient to establish all the requirements to be satisfied before finding that the
petitioner was duly admitted to baptism since it is based on the entries made in the Church. There
is no evidence to hold that before being admitted to baptism, she was admitted to catchments and
was brought through the various stages to sacramental initiation. There is also no evidence to
hold that before being admitted to baptism she was adequately instructed in the truths of the faith
and duties of a Christian and tested in the Christian life over the course of catchments. The
evidence on record would on the other hand show that the baptism ceremony was conducted as
an empty formality preceding the marriage ceremony without having proper faith in Christianity
and the Christian, way of life.

The evidence in the case would further show that she never wanted to live as a true Christian.
Her intention in getting converted to Hinduism immediately after parting company with the first
Respondent would also support the conclusion that the Petitioner was not having the required
intention to receive Baptism and true faith in Christianity at any relevant time.

CONCLUSION

On the basis of the above grounds, the marriage was declared to be null and void and the
Petitioner was entitled to a declaration to that effect.

118 | P a g e
d. Leelamma v Dilip Kumar, AIR 1993 Ker 57

Summary 1:

Facts of the case


The petitioner (wife) met the respondent (husband) in December 1985 and consented to marry
him. However, her consent was acquired by making her believe that the respondent was a
Christian, and that his parents were Christian and also that he belonged to an ancient Christian
family. Due to such representation made by the husband, the petitioner believed all this to be true
and consented to marry him. They lived on their own in the respondent’s home for a while, after
which they decided to move into the respondent (husbands) parental home. After moving in, the
respondent realised that his parents were in fact ‘Ezhavas’ and weren’t Christian at all. In
addition, he too had been baptised into Christianity recently. All the earlier representations made
by him were untrue. The wife contended that the husband had not only practiced fraud on him,
but had also misappropriated his jewels. Plus, he had also tried to coerce her into having sexual
intercourse with his friends. She, however, does not bring these contentions to court, and
petitions the court to nullify the marriage only on the grounds of fraud practiced on her to obtain
her consent.

Issue
There were various issues in the case:

1. What is the identity of a Christian? Who is a Christian?


2. What is the position of canon law in the Travancore-Cochin area?
3. Whether fraud can be a grounds for nullity of a marriage?
Analysis
The wife explicitly states that upon asking her husband whether he belonged to a Christian
family or not, he answered in the affirmative. This occurred in December 1985, when the wife
consented to marry him, however facts show that he was only baptised in 1986. This proves that
as on December 1985, he fraudulently alleged that he was a Christian and made a false
representation. Since he was baptised until after the marriage and not before that, the Judge
found that the husband had, in fact, made a false representation and knowing it to be so had
obtained the consent of the wife to marriage.

According to the custom of the Syrian Catholic community in Kerala, converts are considered to
be socially inferior in status, and members of ancient Christian families do not usually marry
converts. Therefore, stemming from such custom the wife also contended that one who is not
born Christian, cannot be a true Christian. However, the concept of a ‘Born Christian’ is alien to
Law, and under Canon Law, one is inducted into Christianity by baptism. And so, the expression
‘Christianity’ is not defined precisely in any of the statues. Section 3 of the Indian Christian
Marriage Act, 1872 states that ‘Christian’ means, a person professing the Christian religion.

119 | P a g e
Similarly, stated in Section 3 of the Cochin Christian Civil Marriage Act. Due to such lose
definitions, a need arose to lay down a comprehensive definition of the expression ‘Christian’,
but ultimately the existing definitions were not modified.

However the expression was given a broad meaning by the Courts in India. In Maharam v.
Emperor AIR 1918 All 168, it was held that it is important to profess Christianity in order to be a
Christin. Similarly, in Pakkiam Solomon v. Chelliah Pillai AIR 1924 Madras 18, it was held that
even after ex-communication, a person would be a Christian if he continued to profess Christian
faith.

Held
In the present case, the marriage was declared null and void. Alimony was also decided at
Rs.350/- per month to the wife. The husbands counter-petition was dismissed. And it was held
that the parties will suffer their costs.

Critical Analysis
In my opinion, the court’s decision was justified. This is because, it is a clear case of
misrepresentation. From the facts, it is vividly clear that the consent was obtained solely on the
grounds of the respondents religious preferences. He misrepresented these religious preferences,
and chose to obtain her consent by lying to her. The concept of a marriage being a nullity from
the very beginning or being annulled subsequent to the marriage is a concept of English origin
from the times of the ecclesiastical courts which exercised jurisdiction over every aspect of
marriage. The ecclesiastical doctrine laid down that marriage was not regarded as consummated
if parties have not become one flesh by sexual intercourse, and consequently if one of the parties
was impotent and therefore unable to consummate the marriage, he or she lacked the capacity to
marry. Further, annulling a voidable marriage was given retrospective effect. According to
ecclesiastical law, a marriage was either valid forever or never, in cases similar to the above, the
marriage was declared void ab initio. Such uncontrolled and unrestrained power in the hands of
the religious leaders to declare marriages void and bastardize the issue was a cause of great
concern to the royal courts.
It was situations like this that lead to the question, whether laws which in spite of their
ecclesiastical authority character should force such arbitrary rules upon the common man. It was
as an answer to this question that laws were divided into (a) civil and (b) canonical. It was
further decided that a marriage in violation of the former would be void and latter would
voidable. It was also understood as a general principle that the validity could be questioned only
by the parties to a marriage and further that if one of the spouses died, such a question could
never arise. Fraud, as a grounds for nullity of marriage basically means situations and
circumstances as to show want of real consent to marriage. The main element here is deceit.
Unlike the Law of Contracts, misrepresentation either innocent or fraudulent will not terminate
the marriage. The important aspect here is respect to the fact that has been fraudulently

120 | P a g e
represented. If it a crucial element in the marital relation then it will affect the marital relation.
For example if there is a misrepresentation with respect to the ceremonies or identity of the
party.
Summary 2:

The defendant i.e. Dilip Kumar asked the plaintiff i.e. Leelamma to marry him in December
1985. Leelamma belong to an ancient Christian family, who had a custom to marry only those
people who were born Christians. People, who converted to Christianity, were considered to be
inferior, and members of ancient families did not marry such converts. It was essential for her
that the person she marries, should be a born Christian. Dilip told her that he was a born
Christian and even his parents were Christians. Leelamma gave her consent to marry him based
on this fact. Later she discovered that Dilip Kumar was not actually a born Christian and
converted to Christianity in 1986 by baptism, after he obtained Leelamma’s consent. She
discovered that Dilip was actually an Ezhava and his parents were also Ezhavas. Therefore she
filed for divorce on grounds of fraud. Dilip contested her claim by filing for restitution of
conjugal rights. She claimed that Dilip, only obtained her consent because he told her he was a
born Christian, which was not true and thus he obtained her consent by fraud. According to her
custom, as she belonged to an ancient family, she would have never married a person who was
not a born Christian. She also contested that a person who is not a born Christian, cannot be
considered a Christian.

Issue

The issues before the Court were:


1) Who is a Christian?
2) Is the marriage a null marriage? Which law is to be applied Statutory Law or Customary
Law?

Rule

1) According to Section 3 of the Christian Marriage Act, “Christians are persons professing
the religion of Christianity”. The Court stated that professing the religion of Christianity
is important. Even if a person is baptized at birth, or was ex-communicated by the
religious society, he will be considered a Christian only if he professes Christianity as a
religion. The Court looked at various judgments such as, Maharam v Emperor, which
upheld that a person has to profess Christianity in their life to be considered a Christian.
2) While deciding what law will apply in this case, the Court stated that in matrimonial
affairs, the Christian Marriage Act does not apply in certain states like Travancore,
Cochin, Manipur and Jammu & Kashmir. There are no statutory provisions for these

121 | P a g e
areas and therefore the Canon Law is applicable. Canon Law is different for different
communities and the Court looks at different Canon Laws and applies them as it deems
fit.

Analysis

For the first issue, the Court looked at the evidence presented in Court, and came to a conclusion
that the husband Dilip Kumar did not profess the Christian religion at any point in his life. He
did not profess the religion of Christianity at the time he asked to marry Leelamma or after the
marriage. Therefore, he was not a Christian. However this marriage would still be valid under the
Christian Marriage Act, which requires only one of the spouses to be of the Christian religion.
However, the Court held that Dilip Kumar was not Christian at the time he asked Leelama to
marry him. Later he was baptized by the Vicar but mere baptizing does not make one a Christian.
The Vicar claimed that he baptized Dilip Kumar by mistake. All this evidence made it clear to
the Court that the only reason Dilip Kumar converted to Christianity was to deceive the wife. He
did not profess the religion of Christianity at any point in his life and therefore he could not be
considered a Christian.

For the 2nd issue, the Court looked at the Canon law applicable in this case. The wife belonged to
the Syrian Christian Community, whose customary law was governed by the oriental church.
The matrimonial laws of this community were stated in the Code of Canon Law 1990, according
to which a marriage is invalid if the consent for marriage has been obtained by fraud, and fraud
about the “quality of life” of a person, as this would seriously disturb the marriage life. The
Court interpreted that the “quality of life” of a person, according to this Canon Law, is their
religious faith. As religious faith of a person is the priority in any religion, therefore “quality of
life” of a person is to be understood as their religious faith and believes. Applying this
interpretation and Canon law to the present case, the Court held that this marriage is null and
void on the grounds of fraud. The Court further stated that even under the Civil law, this
marriage would be void on the same grounds. False representation of a vital fact would render
this marriage null. Religious faith is considered to be material in a marriage, as we live in a
religious society and people do look at the religious faith and caste before giving their hand in
marriage. Thus the court held that there was an erroneous belief induced in Leelamma by Dilip
Kumar, about his “quality of life” i.e. his religious faith to be Christian when he actually was an
Ezhava. Based on this belief the wife had given her consent. Therefore, the Court declared that
the marriage is null as consent should be free and voluntary.

Conclusion

The Court held that the marriage was invalid on grounds of fraud after applying the Canon Law
of Syrian Catholic Community. The Court also declared that Dilip Kumar cannot be considered a

122 | P a g e
Christian as he did not profess the Christian religion at any point in his life. The Court awarded
Leelamma alimony of Rupees 350/-. She did not withhold her society wrongfully and therefore
the Court denied the husband’s request for restitution of conjugal rights. Further the marriage
was declared null so there can be no restitution of conjugal rights.

e. P.J. Moore v. Valsa, AIR 1992 Ker 176

Summary 1

The husband, also the appellant in the present case, was married to a lady from whom he had
three children. The wife, however, died after twelve years of marriage. The husband underwent
vasectomy for religious regions, and soon after decided to remarry. The husband claims that
before the second marriage, he had disclosed the fact about vasectomy to his wife and had
promised her that he would undergo recanalisation after marriage. Later, however, he realised
that recanalisation would be against his religious faith and decided not to undertake
recanalisation. The wife sought to have marriage annulled on the grounds of fraud.

The single judge bench of the High Court found some of the husband's claims to be false. The
facts found by the single judge to be true were that the fact about vasectomy was never disclosed
to the wife and the wife had found the same out after more than two years of marriage. The
single judge, based on these facts, decided that non-disclosure of vasectomy would amount to
fraud, and thus nullified the marriage. The husband appealed to the Division Bench of the High
Court.

The issue before the High Court was whether non-disclosure of vasectomy be a valid grounds for
nullifying a marriage, and more particularly, would it amount to fraud for the purpose of Section
19 of the Indian Divorce Act? The last part of Section 19 of the Indian Divorce Act says as
follows:

“Nothing in this section shall affect the [jurisdiction of the District Court] to make
decrees of nullity of marriage on the ground that the consent of either party was obtained
by force or fraud.”1

The issue arose because the word 'fraud' is not defined in the Indian Divorce Act. The wife
argued that the definition of fraud for the purposes of Section 19 of the Indian Divorce Act is
same as the definition of fraud as defined under Indian Contract Act. The husband argued before
the High Court that the definition of 'fraud' under Section 19 is narrower than the definition of
'fraud' in Contract Act, and must be interpreted very strictly. The husband further argued that

1
Indian Divorce Act, 1869

123 | P a g e
mere concealment of fact, unless it related to the identity of spouse or ceremony to be performed,
is not fraud for the purposes of matrimonial disputes.

Fraud, as per Indian Contract Act 1872, "is active concealment of fact to induce another"2. Mere
silence, according to the Contract Act, does not amount to fraud unless it is a person's duty to
speak. The Court ruled that Contract Act's definition of fraud is very broad and cannot be used
for matrimonial causes. The Court said that it is natural for the parties to exaggerate and
overstate the facts, and a definition imported from the Indian Contract Act would result in
nullification of all such marriages where petty facts were exaggerated to make the spouse look
better. The Court cited various decisions where overstatement of family status, income of the
husband etc. were not considered to be fraud. The Court said that if Indian Contract Act's
definition is used, it would nullify these marriages and contradict Supreme Court's decisions.
Hence, Indian Contract Act's definition of 'fraud' cannot be imported for the purposes of Indian
Divorce Act.

The Court, however, did admit that it must be liberal in all interpretations. The Court said that
non-disclosure of a fact which has a material impact on marriage is fraud for the purposes of
Indian Divorce Act. The question, then, was whether vasectomy is one such fact which can have
a material impact on marriage? The Court said that one of the objects of married life is to have
offsprings. The Court further said that one of the cravings of every married woman is to have
children. Hence, if one of the spouse is incapable of procreation, he must disclose that fact before
marriage. If that is not done, it would constitute 'fraud' under Indian Divorce Act.

The Court, thus, dismissed the appeal and allowed the divorce. The Court widened the meaning
of 'fraud' under Indian Divorce Act by including all facts which have a material impact on
marriage. A similar decision was given by the Bombay High Court in Banjamin Boming
Cardoza v. Mrs. Gladys Benjamin Cardoza3. In that case, the Bombay High Court said that
where a consent for marriage was obtained by wife by concealment of fact of incapability of
giving birth to a child, the marriage was solemnised by fraud. In that case as well, a decree of
nullity was passed. A similar decision was also passed by a full bench of Gauhati High Court in
Best Morning v. Mirmalendu4. The facts of that case were completely identical to the present
case. The husband was a widower who underwent vasectomy and marriage another lady without
disclosing the same to her. The Court found that husband was guilty of fraud and thus nullified
the marriage.

Summary 2:

Facts:

2
Indian Contract Act, 1872
3
Banjamin Boming Cardoza v. Mrs. Gladys Benjamin Cardoza, AIR 1977 Bom 175
4
Best Morning v. Mirmalendu, AIR 1987 Gau 63

124 | P a g e
A widower, father of three children, married again, but the wife after twelve years of married life
sought to have the marriage annulled on the ground that her consent was obtained by fraud.
Learned single Judge declared the marriage null and void. The fraud alleged is this; Before the
second marrigae, appellant (husband) had undergone vasectomy , but he did not disclose it to the
respondent (wife). As she did not conceive for thirty months, she was anxious to know the
reason. Appellant then divulged to her that he had undergone vasectomy before marrigae. But he
pacified her that he would undergo recanalisation. However, he later changed his mind and
declined to under recanalisation. His excuse for this volte-face is that such a course is repugnant
to Pentecostal faith to which he has subscribed on becoming a member of Assemblies of Church
of God.

Issue:

whether nondisclosure of the said fact would amount to fraud as understood in Section 19 of the
Indian Divorce Act, 1896.

Rule:

Under Section 17 of the Contract Act, active concealment of a fact with intent to induce another
person to enter into a contract is enough to constitute fraud.

the concept of fraud envisaged in section 19 of the Indian Divorce Act, is narrower than the word
"fraud" defined in Section 17 of the Contract Act; mere concealment of a fact, unless it relates to
the identity of a spouse or the ceremony to be undergone, does not amount to fraud;

Analysis:

Under Section 17 of the Contract Act, active concealment of a fact with intent to induce another
person to enter into a contract is enough to constitute fraud. In the Explanation to the section it is
made clear that mere silence as to facts likely to affect the willingness of a person to enter into a
contract is not fraud. This clarification is, however, subject to a rider that if circumstances are
such that it is the duty of the person keeping silence to speak, then, even silence may amount to
fraud.

mere concealment of a fact, unless it relates to the identity of a spouse or the ceremony to be
undergone, does not amount to fraud, according to section 19 of the Indian Divorce Act. Hence
nondisclosure of information regarding vasectomy is not fraud of such a nature as to vitiate the
consent for marriage.

It has to be borne in mind that at the time of negotiation or courtship, parties would try to project
themselves in the best possible form. Some amount of exaggeration in representation is often
experienced to make him or her acceptable to the other as a spouse. Mere concealment or over-
statement of facts by itself does not invalidate a marriage. Hence contours of concept of fraud as

125 | P a g e
widely envisaged in the Contract Act cannot be imported hook line and sinker to matrimonial
proceedings.

Conclusion:

The courts in India have regarded negation of female spouse's innate desire to become a mother
as a serious materimonial delinquency on the part of male spouse. Hence, the court is most likely
to dismiss the appeal of the appellant.

f. Reynold Rajamani v. Union of India, AIR 1982 SC 1261

Summary 1:

In the Hon’ble Supreme Court of India


July 30th, 1982

Facts- The appelants were a roman catholic couple married under s. 27 of the Indian Christian
Marriage Act, 1872. They wanted to dissolve their marriage on the ground that they had not been
living together for more than 2 years and that the marriage had broken down irritrievably. They
proceeded to the Honourable District Court with this under s. 28 of the Special Marriage Act for
divorece with mutual consent. However, the Court dismissed the petition saying that the
provisions of the Special Marriage Act can not be availed in this case since the couple was
maried under the Indian Christian Marriage Act.

The couple then filed a writ petition in the Hon’ble Supreme Court of India seeking that they be
allowed to file a joint petition under s. 7 of the Indian Divorce Act, 1869, read with s. 1(2)(d) of
the Matrimonial Clauses Act, 1973 of England. The Hon’ble Supreme Court granted the cople to
do so.

However, the District Court again dismissed the joint petition saying that the English statute can
not be relied upon. The Hon’ble High Court of Judicature affirmed this view.

Thus the present appeal is in the Hon’ble Supreme Court.

Issues-
1. Can the couple, being married under the Indian Christian Marriage Act, avail the Special
Marriage Act to seek divorce?
2. Can s. 7 of the Indian Divorce Act be read with the English Statute?
3. What is the correct interpretation of s. 7 of the Indian Divorce Act?
4. Can the Court give a loose construction to the statute to allow irritrevable breakdown of
mariage as a ground for divorce?

126 | P a g e
Rule-

A. Section 7 of the Indian Divorce Act-


Court to act on principles of English Divorce Court.- Subject to the provisions contained
in this Act, the High Courts and District Courts shall, in all suits and proceedings hereunder,
act and give relief on principles and rules which, in the opinion of the said Courts, are as
nearly as may be conformable to the principles and rules on which the Court for Divorce and
Matrimonial Causes in England for the time being acts and gives relief: Provided that nothing
in this section shall deprive the said Courts of jurisdiction in a case where the parties to a
marriage professed the Christian religion at the time of the occurrence of the facts on which
the claim to relief is founded.]
B. Section 27 of the Indian Christian Marriage Act, 1872-
All marriages hereafter solemnized in India between persons one or both of whom professes or
profess the Christian religion, except marriages solemnized under Part V or Part VI of this Act,
shall be registered
in manner hereinafter prescribed.
C. S. 28 of the Special Marriage Act-
Divorce by mutual consent.—
(1) Subject to the provisions of this Act and to the rules made thereunder, a petition for
divorce may be presented to the district court by both the parties together on the
ground that they have been living separately for a period of one year or more, that
they have not been able to live together and that they have mutually agreed that the
marriage should be dissolved.
(2) [On the motion of both the parties made not earlier than six months after the date of
the presentation of the petition referred to in sub-section (1) and not later than
eighteen months] after the said date, if the petition is not withdrawn in the meantime,
the district court shall, on being satisfied, after hearing the parties and after making
such inquiry as it thinks fit, that a marriage has been solemnized under this Act, and
that the averments in the petition are true, pass a decree declaring the marriage to be
dissolved with effect from the date of the decree.

Analysis-

The Court looked into the fact that whether or not the Enlgish statute, providing a relief under
irretrievable breakdown can be read with the Indian statute or not. It reasoned that pre-
independence mode of interpreting statutes and reading English laws with Indian ones can not be
in practice after 1947. India is a sovereign state and the statutes of other states do not have
binding authority over Indian statutes. It further said that it is the responsibility of the legislature
to decide whether or not irritrevable breakdown of marriage can be a ground for divorce.

127 | P a g e
It also said that the couple could not proceed under the Special Marriage Act to seek a divorce
due to the fact that the marriage was solemized under the Indian Christian Marriage Act. The
couple had a choice to marry under the Special Marriage Act, but however chose not to. Thus,
they can not avail the Special Marrage Act.

Alongside, the Court looked into the possibility of giving a liberal construction to the statutes. It
said that the statutes can be interpreted liberally to suit the needs of a transforming community of
Indians, entering the modern world. However, a liberal view should be distinguished from a
loose interpretation of the statute. It would be fatal to proceed against the intent of the statute as
envisaged by the Parliament.

Critically analysing the judgment, irritrevable breakdown of marriage should be included as a


ground for divorce. Afterall, if two people simply choose not to live together, there must not be
any compulsion for them to choose a particular reason to seek divorce. Having said that, we must
note that the courts are bound by the ltter of the law. No matter how liberally they interpret the
statutes, it can not go beyond the letter of the law. It is the legislator’s duty to ensure that the
statutes serve the interests of the people. There is a growing need for irritrevable breakdown of
marriage to be included as a ground for divorce in all the communities and several law
commission reports hint to the same. However due to political reasons, the Parliament has not
been able to incorporate the same in the statute.

Conslusion-

It was held by the Supreme Court that the statute can not be so loosely constructed so as to read
laws from another sovereign state with our own. Infact, s. 7 of the Indian Divorce Act has been
repealed by the amendment in 2001. The court vouched to give a liberal interpretation to the
statutes to suit the needs of a growing population of liberal citizens. It however said that it is
primarily the duty of the law maker to include such grounds in the statute.

Summary 2:

Issue: Whether the provision of the English Matrimonial Causes Act, 1973, can be applied to
petitions under the Indian Divorce Act, 1869, by virtue of Section 7 of the Indian Divorce Act.

Facts: Husband and wife belonging to the Roman Catholic community applied for a divorce
since they had been separated for about two years. They claimed in the court that they were
unable to tolerate each other’s presence and can’t make the marriage work anymore. Therefore
felt that they were entitled for the decree of divorce. The couple had been married under Section
27 of the Indian Christian Marriage Act. They filed for a decree of divorce by mutual consent
under section 28 of the Special Marriage Act.

128 | P a g e
Procedural History: The trial court dismissed their petition by stating that section 28 of the
Special Marriage Act can’t be availed. The District Court dismissed the petition holding that they
were not entitled to rely on section 1(2)(d) of the English Statute. In appeal the High Court
affirmed the view taken by the trial Court. The Supreme Court allowed them to amend their joint
petition by enabling them to rely on the Section 7 of the Indian Divorce Act and Section 1.2(d)
of the Matrimonial Causes Act. In the appeal to this court it was contended on behalf of the
appellants: (1) that the trial court and the High Court were wrong and that section 7 of the
Indian Divorce Act 1869 incorporated the provisions of section 1(2)(d) of the Matrimonial
Causes Act 1973 and that the appellants were entitled to the benefit of the ground for divorce
as set forth in the latter enactment, and (2) that the Letters Patent jurisdiction enjoyed by the
High Court in Matrimonial matters is sufficiently extensive to enable the High Court to make a
decree for divorce.

Reasoning: Mutual consent is not a ground for divorce under the Indian Divorce Act 1869. The
provisions of section 1(2)(d) of the Matrimonial Causes Act 1973 of England cannot be read into
section 7 of the Indian Divorce Act, 1869. Whether a provision for divorce by mutual consent
should be included in the Indian Divorce Act is a matter for legislative policy. The courts cannot
extend or enlarge legislative policy by adding a provision to the statute which was never enacted
there. It is for Parliament to consider whether the Indian Divorce Act, 1869 should be
amended so as to include a provision for divorce by mutual consent. The Letters Patent
jurisdiction enjoyed by the High Court in matrimonial matters cannot be construed to include a
ground for divorce not specifically set forth in section 10 of the Indian Divorce Act, 1869.

Analysis: Legislation whenever made by Parliament of a foreign state cannot automatically


become part of the law of another sovereign state. Whatever interpretation of section 7 of the
Indian Divorce Act, 1869 was permissible before August 15, 1947 when the British Parliament
had plenary powers of legislation over Indian Territory, no interpretation is now permissible
which would incorporate post-1947 British laws into the Indian laws. Thus, the appeal was
dismissed.

Conclusion: This case along with many other cases have caused great amount of hardship on
parties whose marriage had broken down beyond repair. Despite the parties wanting to mutually
end it, due to the absence of any provision permitting them to do so, they had to be tied down
with each other. The position has however changed under Section 10A of the Divorce Act as
amended in 2001 which made mutual consent a valid reason for divorce.

g. Pramilla Khosla v. Rajnish Kumar Khosla, AIR 1979 Delhi 78

Summary 1
In this case, there was a marriage between the plaintiff (wife) and the defendant (husband). The
wife professed Christianity, whereas the husband was a follower of Hinduism. It was averred that

129 | P a g e
the plaintiff and defendant were married “according to Arya Samaj rites” (a Hindu custom). The
plaintiff filed a petition under s. 22 and s. 23 of the Indian Divorce Act, 1869, for judicial
separation on the grounds of cruelty by the husband.

In the court, the defendant brought up three issues. These issues were addressed during the
course of the case. The first issue raised by the defendant was an application under Order 7 Rule
11 read with section 151, C.P.C inter alia on the grounds that the petition under Indian Divorce
Act was not maintainable as it was admitted that the marriage was performed according to Arya
Samaj rites, and thus the plaintiff should seek relief, if any, under the Hindu Marriage Act, 1955.

Secondly the defendant also pleaded by saying that based on the representations and conduct of
the plaintiff, the wife was estopped from saying that she is or ever was a Christian and for this
reason too the petition is not maintainable under the Indian Divorce Act.

Later on, the defendant raised the third issue on the assumption that the plaintiff was a Christian -
that the marriage itself was invalid, because a marriage according to Arya Samaj Rites could not
be said to be valid unless both the parties were Hindus.

The Judge for the purposes of addressing the issues raised, decided to restrict himself to the
petition in question, and held that one must assume that the allegations of the petition to be true.

Looking at the first issue raised by the defendant, the Judge concluded by saying that there was
nothing on the face of the petition by the plaintiff in this case, which made it not maintainable
under the Indian Divorce Act. The Judge looked at the Preamble of the Indian Divorce Act to
explain that its purpose is “to amend the law relating to the divorce of the persons professing the
Christian religion”.

He then read out s. 2 of the Indian Divorce Act, which reads, “Nothing hereinafter contained
shall authorize any court to grant any relief under this Act except where the petitioner or
respondent professes the Christian religion.”

He said that nowhere in the Act is it required that the marriage in respect of which relief is
sought, should have been solemnized in any particular form. It is sufficient that one of the parties
is a Christian when the petition is filed.

Since the plaintiff unequivocally alleged in her petition that she professed the Christian religion,
the requirement of the Act was fulfilled.

The judge then looked at s. 7 of the Indian Divorce Act, to look at the other aspect of the
argument. This section enjoins that in all suits and proceedings under the Act, the court shall act

130 | P a g e
and give relief on the principles and rules which in its opinion “are nearly as may be
conformable to the principles and rules on which the court for Divorce and Matrimonial Causes
in England for the time being acts and gives relief.”

In England in it a settled principle that the English Law will not recognize or grant any relief in
respect of a marriage which is not monogamous, as a marriage is defined as “the voluntary union
for life of one man and one woman to the exclusion of all others”.

However English Law would not recognize a Hindu Marriage nor grant any relief in respect of it
as Hindu Marriage was not monogamous. The Judge said that S. 7 of the Indian Divorce Act
requires the same principle to be applied by the Court, while exercising the jurisdiction under the
Indian Divorce Act. However a different case amended the act of 1912, which added a proviso to
S. 7. The proviso made it clear that, notwithstanding S. 7, the Court is not to be deprived of
jurisdiction “in a case where the parties to a marriage professed the Christian religion at the time
of occurrence of the facts on which the claim to relief was founded.”

Even though the proviso is not applicable in our case, it showed that in a case to which it applies,
relief can be had under the Indian Divorce Act in respect of any kind of marriage, whether
monogamous or not. However the whole position was radically altered by the Hindu Marriage
Act 1955, which applies to Arya Samaj Rites. Hindu Marriages were rendered monogamous by
the Act. Thus the bar to relief emanating from English law by virtue of S. 7 no longer operates.

And thus she could file for judicial separation under the Indian Divorce Act.

The Judge then looked at the second issue raised by the defendant, and said that he refuses to
accept the second plea, as it could not be entertained under Order 7 rule 11 of the C.P.C. This
was because; it involves the questions of fact on which evidence is necessary. It also did not
disclose a cause of action for the petition to be dismissed.

The Judge then looked at the third issue, and decided that all that the plaintiff had said in the
petition was that she professed the Christian religion, and nowhere did she say that she was a
Christian when the marriage was solemnized. Since no questions of validity of marriage arose
from the petition itself (as it does not say that the plaintiff was a Christian or a Hindu at the time
of marriage), the judge assumed that the ceremony of marriage was valid.

The Judge dismissed the petition of the defendant and held that relief could be claimed under the
Indian Divorce Act, 1869 as well as Hindu marriage Act, 1955. However the Judge did remark
upon the petitioner’s choice to file for judicial separation under the Indian Divorce Act, 1869
when there was a modern act (Hindu Marriage Act 1955), which contained many more
provisions for divorce and judicial separation.

131 | P a g e
Before the amendment of 2001, the Indian Divorce act contained only limited provisions, under
which women could seek relief. The draconian act was not gender neutral and made it difficult
for women to seek relief, unless they had a “reasonable excuse”. Thus the amendment of 2001
was a substantial amendment, as it sought to rectify the laws, so that they were in consonance
with the Constitutional guarantees of equality and dignity. 5

Summary 2

ISSUE
1- Whether or not can a wife file case for divorce under Indian Divorce Act if the Marriage is
covered under Hindu Marriage Act?
2- What should be the religious status of parties while filing case under Indian Divorce Act?
FACTS
In this case a wife has filed a petition under Section 22 and 23 of the Indian Divorce Act 1869.
She prays for a decree of judicial separation on the ground of cruelty by the husband.
In the wake of being given the claim, the first thing the spouse did, even before documenting his
composed articulation, was to move an application under Section 7 rule Ii and Section 151 of the
Code of Civil Procedure 1908. The spouse's question is that the solicitation by the wife is not
viable under the Indian Divorce Act as it yields that Arya Samaj Rites, which is a Hindu sort of
marriage and recommends that both sides were Hindus, performed the marriage. Along these
lines, as demonstrated by the mate, the wife can search for help, if by any methods, simply under
the Hindu Marriage Act 1955 and not the Indian Divorce Act, and, therefore, the solicitation
should be rejects. Moreover, the spouse has contended in the application that, by her
representations and conduct, the wife is estopped from saying that she was or is a Christian, and
hence in like manner the request is not viable under the Indian Divorce Act.

RULE
 Section 22 Indian Divorce Act: Bar to decree for divorce a mensa et toro; but judicial
separation obtainable by husband or wife. No decree shall hereafter be made for a divorce
a mensa et toro, but the husband or wife may obtain a decree of judicial separation, on the
ground of adultery, or cruelty, or desertion for two years or upwards, and such decree
shall have the effect of a divorce a mensa et toro under the existing law, and such other
legal effect as hereinafter mentioned.
 Section 23 Indian Divorce Act: Application for separation made by petition. Application
for judicial separation on any one of the grounds aforesaid, may be made by either
husband or wife by petition to the District Court and the Court, on being satisfied of the

5
http://www.manupatrafast.com/articles/PopOpenArticle.aspx?ID=59a6ae72-53e3-4cd8-a620-
38c292a1c6ee&txtsearch=Subject:%20Family%20Law

132 | P a g e
truth of the statements made in such petition, and that there is no legal ground why the
application should not be granted, may decree judicial separation accordingly.
 Hindu Marriage Act
Section 2: Application of Act. —
1-This Act applies—
(a)-to any person who is a Hindu by religion in any of its forms or developments,
including a Virashaiva, a Lingayat or a follower of the Brahmo, Prarthana or Arya Samaj,
 A Hindu marriage has been rendered monogamous by the Hindu Marriage Act, as section
5 makes it a condition that 'neither party his a spouse living at the time of the marriage'.
ANALYSIS
The Preamble to the Indian Divorce Act clarifies that its motivation is 'to change the law
identifying with the separation of persons purporting the Christian religion'. Section 2 sets out
specific conditions for the award of different sorts of alleviation under the Act. The second
passage of that area says: 'NOTHING hereinafter contained should approve any court to concede
any alleviation under this Act aside from where the solicitor or respondent declares the Christian
religion.' A correcting Act of 1912 added a stipulation to Section 7 of Indian Divorce Act which
makes it clear that, despite Section 7, the court is not to be denied of locale 'for a situation where
the gatherings to a marriage purported the Christian religion at the event's season of the truths on
which the case to help is established'. (Section 7 of Indian Divorce Act is been removed- current
status of section)
The Act does not require that the parties’ should be married been solemnized in one particular
form, what matters is the status of the professing religion at the time when filing the suit. If one
of the party is Christian by choice and follows Christianity is sufficient to file case.
CONCLUSION
Status of religion to be followed what was at the time of marriage solemnization is not required.
Parties may follow any religion and stand by that to file the case. Here in this case Wife followed
Christianity at the time of filing the case tough she was Hindu at time of marriage, is sufficient
order to apply the statute.

h. Molly Joseph v. George Sebastian, AIR 1997 SC 109

This case is with respect to the interplay of Statutory Law and Personal Law, i.e.,
Ecclesiastical Law.

Facts: The facts of the case are that the husband (George Sebastian) filed a petition under
Section 19(A) of the Indian Divorce Act, for nullity of his marriage. He did so on two grounds:
1. That his wife’s earlier marriage was subsisting when he married her.
2. On the ground of insanity.

133 | P a g e
The appellant wife’s (Molly Joseph) contention was that her previous marriage was already
annulled by an Ecclesiastical Tribunal (Church Court). She further claimed that the husband
knew about this.
The District Judge simply on basis of pleadings and admission of the first marriage, passed a
decree of nullity in favor of the husband. It was held that since there was no decree of nullity of
the previous marriage by a civil court under the Indian Divorce Act, the marriage would be
considered subsisting.

Issue: Do the Ecclesiastical courts have jurisdiction to decide on matters, which are governed by
the Indian Divorce Act?

Governing Statutes: Preamble of the Divorce Act says:


“Whereas it is expedient to amend the law relating to the divorce of person professing to
Christian religion, and to confer upon certain Courts jurisdiction in matters matrimonial;
it is hereby enacted as follows….”

Section 19 provides:
“Grounds of decree- such decree may be made on any of the following grounds:
(1) That the respondent was impotent at the time of the marriage and at the time of the
institution of the suit;
(2) That the parties are within the prohibited degrees of consanguinity or affinity;.
(3) That either party was a lunatic or idiot at the time of the marriage;
(4) That the former husband or wife of either party was living at the time of the
marriage, and the marriage with such former husband or wife was then in force.

Analysis: The Kerala High Court observed,


“Canon law (or personal law of Christians) can have theological or ecclesiastical
implications to the parties. But after the Divorce Act came into force, a dissolution or
annulment granted under such personal law cannot have any legal impact as statute has
provided a different procedure and different code for divorce or annulment.”
From a bare reading of the provisions of the Act including the preamble it is clear that the
Divorce Act purports to amend the law relating to divorce of persons professing the Christian
religion and to confer this act upon the courts.
Unless the Divorce Act recognizes the jurisdiction of Ecclesiastical Tribunal any order or
decree passed by such a tribunal cannot be binding on the courts, which have been recognised
under the provisions of the Divorce Act.
In this background, there is no scope for any other authority including Ecclesiastical
Tribunal to exercise power in connection with matrimonial matters, which are covered by the
provisions of the Divorce Act. When there is a statute governing the area, the statute has the
primacy over any personal law in that regard.

134 | P a g e
It is well settled that when legislature enacts a law even in respect of the personal law of a
group of persons following a particular religion, then such statutory provisions shall prevail and
override any personal law.

Conclusion: The Supreme Court agreed with the reasoning of the High Court and did not
interfere in the said order. The appeal was accordingly dismissed. Therefore, it is established that
Statutory law and not Ecclesiastical law is to govern the matrimonial matters.

In my opinion, the High Court aptly read the provisions of the Divorce Act. In making
the Statutory law have an overriding effect on Personal law, we are moving towards uniformity
in law. In other words, moving closer to a Uniform Civil Code. This, according to me, is
indicative of progress. It follows therefore that the Supreme Court’s judgment is reasonable and
justified.

VII. Week 8: Special Marriages Act and the Foreign Marriages Act
a. Tamali Bhattacharjee v. Samik Baidya, CHN 2004 (1) 639

Summary 1:

The events preceding the filing of this suit are unclear and the two different versions of the
story make it increasingly difficult to view the whole scenario lucidly. It is all the more arduous
to decide since both the versions seem to be bordering on the absurd and are somewhat
bollywoodesque, for want of a better phrase. The petitioner, TamalI, attended a party in June
1999 where even the respondent, Samik, along with some of their friends, was also present.
Sometime in the following month the petitioner got an invitation from the respondent to go over
to his residence since they had been legally married and he had all the right to bring her to his
house. The petitioner and her parents inquired further into the matter and discovered that a fake
marriage had been held before the Marriage Officer at the Dhakuria district. The petitioner in
light of the afore mentioned events, filed a suit in the district court, however the court rejected
the claim and ruled in favour of the respondent. The petitioner’s appeal in the High Court is the
suit in question.
The main issue placed before the court was whether a marriage registered under the
Special Marriage Act needs to be solemnized in a specific way or through a particular series of
procedures. The secondary issue is whether the lower court had dismissed the demand for
annulment after examining all the evidence presented, appropriately and thoroughly. Another
intriguing angle to be examined here is to gauge the Marriage Officer’s extent of power and
authority.
The court in its judgment, first and foremost, made use of Section 12, clause (2) which
states that the parties may choose to solemnize the marriage in any which way they deem
135 | P a g e
convenient as long as a verbal declaration is made in the presence of a state approved Marriage
Officer. The other section of the act that was referred to was Section 13, clause(2) which
stipulates that the certificate entered in the Marriage Certificate Book, with respect to a marriage,
shall act as conclusive evidence that the particular marriage has been solemnized and all the
formalities have been complied with.
The two points of contention before the court were whether the marriage existed in the first
place and if it did in fact persist then was it solemnized in the appropriate manner. The
arguments made by both parties reflect two entirely distinct chains of events. The “wife” claimed
that the marriage did not exist in the first place since her consent was absent; which is an
essential element for any marriage to stand valid in the eyes of law. Hence, the question whether
the marriage was solemnized appropriately or not should not even arise. Furthermore, her
council states that the respondent’s actions clearly portray his unwillingness to consume the
marriage and hence she should be allowed to obtain a decree of nullity.
The respondent on the other hand states that although a marriage did exist his consent was
obtained through misrepresentation and gleaned in an unjust manner. The respondent alleges that
the petitioner had convinced him that her brother and father would help the respondent get a job,
since they had several contacts in his respective profession. The council for the “husband”,
drawing the court’s attention to section 13(2), also went on to say that the issuance of the
Marriage Certificate would conclusively point to the fact that the marriage had been solemnized
under the act.
The court with regard to the arguments presented by both sides came to the conclusion that
it would be foolish to believe that an educated woman like the petitioner would sign a register
presented to her by a complete stranger. The court also maintained that the petitioner could not
shirk from her onus just by claiming that she had no recollection of how her signature came to
appear on the register as well as the notice. There were no challenges made regarding the
authenticity of the signature and neither were there any specific contentions when it came to the
declaration mentioned in Section 12(2). Apart from this, the certificate in the book was taken to
be enough proof that the requisite formalities had been complied with. Thus, the court dismissed
the claim to obtain a decree for annulment.
However, in this entire fiasco it is important to note the importance attached to the duties,
obligations and the powers of the Marriage Officer. The authority of the officer is almost as
much as that of a lower court since his capacity is one where he acts something like an extension
of the judiciary. At first glance it may seem unadvisable to grant such powers to individuals who
are in no formal way part of the judicial structure since it opens up the omnipresent probability
of corruption and abuse of said power. However, in my opinion, it has been done for the sake of
remaining practical and safeguarding the courts from unnecessary added strain. It is also done for
the sake of convenience since, if looked with respect to other marriage acts, marriages under the
act resemble civil contracts most closely as there is no mention of any sort of requirement of
customs and such to solemnize unions registered under this act. So logically there is no need for
a religious representative to be present at such marriages. However, that gives rise to the need for

136 | P a g e
some sort of authority to be present, namely the Marriage Officer. The officer even has the
power to make inquiries in cases wherein some objection has been raised to the marriage and if
the officer find the objection to be genuine and if the objection falls under some particular
categories, he can choose to refuse to solemnize the marriage. The officer may also compel the
person raising the objection to provide compensation if he finds that the objection has not been
made in good faith.
This case in particular showcases how the actions and obligations of the Marriage Officers
have been placed at such a high premium. However, as far as the decision of the court is
concerned, I feel that the court could have easily recommended for a separation even though the
suit for annulment fell short. On technical grounds it is obviously accurate to refuse annulment
for any marriages that are not void right from the beginning. However, it is clear that the
marriage is dubious since both parties claim that the consent required is either entirely missing or
obtained inappropriately. If both the parties involved in the suit claim that consent is not even
remotely free and duly obtained, then it quite naturally reflects negatively on the chances of
survival the marriage has.

Summary 2:

Facts

 The Petitioner Tamali Bhattacharjee and Respondent Samik Baidya were students of
coaching classes.
 The respondent invited the Petitioner for a party at Dhakuria. On the day of party one
lady took signatures of all the people present at the party in a khata.
 It was alleged by the Petitioner that the signature was taken in such a mechanical way
that she didn’t get a chance to enquire about it.
 On 9th July, the Respondent asked the petitioner to come to his home on the grounds that
she was his legally married wife.
 After due enquiry the Petitioner and her parents came to know that a fake marriage have
taken place on 21st June (day of party) before the magistrate officer of C.M.C District at
44, Dhakuria Station.

Procedural History

 The issue raised by Trial Court was whether the Petitioner was entitled to get a decree of
nullity of marriage. The Trial Court concluded that the marriage cannot be held null and
void and accordingly dismissed the suit.

137 | P a g e
 The Petitioner appealed in the High Court and challenged the decree of dismissal.

Issue

 Whether the Trial Court was right in dismissing the suit of nullity of marriage between
appellant and Respondent?
 Whether the marriage between both the parties can be held null and void?

Rule

Special Marriage Act, 1954

Section 12: Place and form of solemnization

(1) The marriage may be solemnized at the office of the marriage Officer, or at such other place
within a reasonable distance there from as the parties may desire, and upon such conditions and
the payment of such additional fees as may be prescribed.

(2) The marriage may be solemnized in any form, which the parties may choose to adopt:

Provided that it shall not be complete and binding on the parties, unless each party says to the
other in the presence of the Marriage Officer and the three witnesses and in any language
understood by the parties,-"I, (A), take thee (B), to be may lawful wife (or husband)".

Section 13: Certificate of marriage

(1) When the marriage has been solemnized, the Marriage Officer shall enter a certificate thereof
in the form specified in the Fourth Schedule in a book to be kept by him for that purpose and to
be called the Marriage Certificate Book and such certificate shall be signed by the parties to the
marriage and the three witnesses.

(2) On a certificate being entered in the Marriage Certificate Book by the Marriage Officer, the
Certificate shall be deemed to be conclusive evidence of the fact that a marriage under this Act

138 | P a g e
has been solemnized and that all formalities respecting the signatures of witnesses have been
complied with.

Judgment

The Trial Court refused to grant the petitioner a decree for nullity of marriage. The High Court
while deciding the case, took into account the allegations put forward by both Appellant and
Respondent. The contention raised by appellants was that firstly, since the respondent has
willfully refused to consummate the marriage, on that score even petitioner was entitled to a
decree of nullity of marriage. Whereas, the respond denied all the material allegations stated by
Appellant. He argued that the petitioner took his consent for the marriage through
misrepresentation. He agreed to marry the petitioner because, the petitioner stated that her father
and brother has several connections with good companies and that she would help him to get a
job in one of these companies.

At the time of hearing, the parties also argued that the marriage should be declared null and void
since the marriage was not solemnized under section 12(2) of the Special Marriage Act. Section
12(2) states that the marriage cannot be held binding on both the parties unless both the parties in
presence of Marriage Officer and in front of any three witnesses state that I Take Thee to be my
lawful wife/husband. However the Court observed there was no allegation or evidence from the
side of parties to show that this minimum declaration, which is required under section 12(2) was
not made.

The court while analyzing the judgment, ruled out the contention that the signature was taken in
an fraudulent manner since it is very hard to believe that an educated lady like petitioner would
put her signature on a register on being requested by an unknown lady. Next issue, which was
taken up by court was regarding the notice of the intended marriage. A notice is supposed to be
given to the Marriage Officer of the district under Section 5 of SMA. The Court found the
Appellant’s signature in the said notice. The court also observed that there were no reasons given
by the Appellant as to why she has put her signatures on the said notice, which was required to
be given four weeks before the marriage.

Lastly, the Court was of view that issuance of Marriage Certificate to Petitioner under section 13
of SMA is a strong indication that all formalities prior to such issuance have been complied with
and a marriage under the act has been solemnized.

Conclusion

139 | P a g e
 The Court held that the Trial Court was right in dismissing the suit.
 The marriage was not held null and void and Petitioner’s appeal was dismissed.

Critical Analysis of the Judgment

I don’t agree with the judgment because I feel that the Court has ignored several aspects of law
while giving the judgment. Firstly the Judge mention in the case that both the parties were not
the residents of Dhakuria. Still the Court did not take into account section 15 clause (f) according
to which any one of the party is supposed to live within the district of Marriage Officer for at
least not less than 30 days immediately preceding the date on which the application is made for
the registration of marriage. Also, under section 16 a public notice is issued and a time span of
30 days is given for the hearing of objections related to marriage. In our case these two sections
were not even attracted. There were no objections raised about the marriage within 30 days span.
However, In the case of Deepak Krishna And Anr. V. District Registrar And Ors. It was held that
section 15 is directory but section 16 is mandatory. The court held that it varies from case to case
but in our case the judges made no comment regarding the public notice. Also, there was no
verification regarding whether the party stayed in Dhakuria district or not.

Secondly, it can be seen through the judgment that both the parties, were equally eager to get a
divorce although based on different grounds. The petitioner while filing for divorce under
Section 25 of the Special Marriage Act raised the contention that the respondent had willfully
refused to consummate the marriage, the Respondent also argued that his consent was taken
through misrepresentation. I think that court has ignored both the contentions while giving the
judgment. It is not the case where the parties wanted divorce because of some problems in their
existing marriage rather in this case there was a question on the validity of the marriage. The
court failed to look after the fact that both the parties provided different grounds for not
accepting the marriage and didn’t consider binding on them. Therefore, the Court was wrong in
not holding the marriage null and void.

I think it is a case of irretrievable breakdown of marriage. In England, under the Matrimonial


Causes Act, 1973, the sole ground for which either party may present a petition for divorce is
that the marriage has broken down irretrievably. Long period of separation is an indication of
irretrievable breakdown. However in India there has been ongoing debate on whether the divorce
should be granted solely on the basis of the “fault of the party”, or whether it should be based on
the breakdown of marriage. Marriage as a sacrament, society’s stake in the continuance of

140 | P a g e
marriage, the duty of judges to effect reconciliation between the parties, and public interest are
some of the important factors that feature in this debate.

The fault theory is that a person cannot take advantage of his/her own wrongdoing. For example,
a man cannot commit adultery or inflict cruelty and then file a petition for divorce. The “hurt” or
aggrieved party who has been at the receiving end of the other party’s offensive conduct can only
seek divorce. Whereas when a divorce is based on Irretrievable Breakdown, any wrongdoing by
one spouse or the other does not matter. It is simply a statement by both spouses that the
marriage won’t work any longer. There is no need to prove that one’s spouse was to blame for
the failure of one’s marriage. It is not a ground for divorce under the Hindu Marriage Act, 1955.
Yet, the judges use this concept in practice. There are precedents for this concept. For example
the case of Naveen Kohli v. Neelu Kohli the Court observed that there was an irretrievable
breakdown of marriage since both the parties were living separately and respondent had already
filed several charges against her Husband and divorce was the ultimate resort left.

Although the Court has not recognized irretrievable breakdown of marriage as a ground for
divorce in India (held in V Bhagat v. D. Bhagat). The Law Commission in its 71st Report on the
Hindu Marriages Act 1955 has made recommended to make "Irretrievable Breakdown of
Marriage as a Ground of Divorce. Thus I feel that irretrievable breakdown of marriage should be
made a ground for divorce since it will save a lot of time of courts in deciding the cases where
there is no chance of reconciliation since a lot of time is gone in deciding cases based on fault
theory. If we see our case we find that no fault can be attributed to any party because no party
has consent to the marriage. The Court should have granted a decree to nullify the marriage
between the parties because marriage is an obligation to be fulfilled by both the parties and in
this case the parties are not even ready to live together and accept the marriage. The court
considered the marriage to be biding just because a marriage certificate was issued under Section
13 of the SMA. The court stated that it was a conclusive evidence of the fact that all the
condition prior to it have been complied with. By stating this, the Court had ignored that the
parties had tried so hard to dissolve the marriage. It was a mutual consent from both the sides.

“Moreover, the essence of marriage is a sharing of common life, a sharing of all the happiness
that life has to offer and all the misery that has to be faced in Life. Living together is a symbol of
such sharing in all its aspects. Living apart is a symbol indicating the negation of such sharing. It
is indicative of a disruption of the essence of marriage 'breakdown' and if it continues for a fairly
long period, it would indicate destruction of the essence of marriage-'irretrievable breakdown'.”

141 | P a g e
b. Deepak Krishna v. District Registrar And Ors., AIR 2007 Ker 257

FACTS:

This case was a petition filed in the Kerala High Court wherein the petitioners pleaded that
Section 15(f) of the Special Marriages Act, 1954 should be waived off. According to this section
it was compulsory for the parties to reside for minimum period of minimum one month, within
the jurisdiction of the marriage registrar, before filling the application to receive a marriage
certificate. The petitioners pleaded that this caused a lot of inconvenience to the individuals,
especially when they belonged to different city with regard to place where their marriage
solemnization took place.

They further contended the validity of Section 16 of the Special Marriages Act, 1954. They
requested the honourable court to strike this section too, because of the hardship it caused to the
parties.

They submitted that the honourable court should uphold the decision laid down in Giby v.
George wherein the judge directed the marriage officer to accept applications for marriage
registration before the expiry of the one month period. Also, the judgement held that it was not

142 | P a g e
mandatory for the marriage officer to wait for the lapse of one month tenure before giving the
marriage certificate to the parties.

ISSUE:

 Whether Section 15(f)of the Special Marriage Act, 1954 is a directory or mandatory
provision?

 Whether Section 16 of Special Marriage Act, 1954 is mandatory, directory or


discretionary provision?

 Whether Section 16 can be called off owing to the adverse circumstances that the parties
have to undergo?

RULE:

Special Marriage Act, 1954

 Section 15(f)- the parties have been residing within the district of the Marriage Officer
for a period of not less than thirty days immediately preceding the date on which the
application is made to him for registration of the marriage.

 Section 16- Procedure for registration.—Upon receipt of an application signed by both


the parties to the marriage for the registration of their marriage under this Chapter the
Marriage Officer shall give public notice thereof in such manner as may be prescribed
and after allowing a period of thirty days for objections and after hearing any objection
received within that period, shall, if satisfied that all the conditions mentioned in section
15 are fulfilled, enter a certificate of the marriage in the Marriage Certificate Book in the
form specified in the Fifth Schedule, and such certificate shall be signed by the parties to
the marriage and by three witnesses.

 Section 24- Void marriages.—(1) Any marriage solemnized under this Act shall be null
and void 1[and may, on a petition presented by either party thereto against the other
party, be so declared] by a decree of nullity if—(i) any of the conditions specified in
clauses (a), (b), (c) and (d) of section 4 has not been fulfilled; or

(ii) the respondent was impotent at the time of the marriage and at the time of the
institution of the suit.

143 | P a g e
 (2) Nothing contained in this section shall apply to any marriage deemed to be
solemnized under this Act within the meaning of section 18, but the registration of any
such marriage under Chapter III may be declared to be of no effect if the registration was
in contravention of any of the conditions specified in clauses (a) to (e) of section 15:
Provided that no such declaration shall be made in any case where an appeal has been
preferred under section 17 and the decision of the district court has become final.

ANALYSIS:

With regard to the question concerning whether Section 15(f) of the Special Marriage Act, 1954
was mandatory or not, the court took into consideration Section 24 of the Act. According to sub-
section (2) of Section 24, this section as such would not apply to any marriage deemed to be
solemnized under the Act. But the registration of any such marriage may be declared to be of no
effect if the registration was in contravention of any of the conditions specified in Clause (a) to
(e) of Section 15. As it can be clearly observed, in sub-section (2) of Section 24, Clause (f)
of Section 15 has been conspicuously omitted. This means noncompliance of Clause (f)
of Section 15 as such would not render the deemed solemnization of marriage void under the Act
nor will it render the registration granted invalid. Under such circumstances, the court was
inclined to take the view that Clause (f) of Section 15 was only a directory provision, and the
failure to follow the said clause strictly would not make the registration of the marriage under the
Act invalid.

With regard to the validity Section 16, the court believed that one month period as stipulated in
the section was of prime importance. Waiving off this prescribed tenure would be unjust to the
parties who wished to file legitimate objections to the granting of the marriage certificate. Thus,
in such a case the court held that this section could not be waived off as it was a mandatory
provision.

The court further said that pleas of inconvenience and hardship court not be entertained by the
court as it led to the formulation of bad judgement. Since these judgements served as precedents
for the later cases, it led to imparting of injustice.

CONCLUSION:

The court held that the judgement laid down in Giby v. George was correct in upholding that
Section 15(f) of the Special Marriages Act, 1954 was merely directory and not compulsory. But
with regard to Section 16 of the Special Marriages Act, 1954 the court upheld that this section
was mandatory and the individuals who wished to receive a marriage registration certificate had
to comply with the directions as given in this section. As far as pleas of ‘inconvenience’ and

144 | P a g e
‘hardship’ were concerned, it was said that such requests could not be entertained by the court as
it led to the formulation of bad judgements.

CRITICAL ANALYSIS

In the present case, the Court held that “the procedure stipulated under Section 16 is mandatory
in character and no discretion is cast on the Marriage Officer to deviate from those statutory
procedures.” The compulsory registration of marriage would check child marriages or non – age
marriages, bigamous marriages and fraudulent marriages. Registration must be made compulsory
to avoid alleged marriages and to prove status of women and the legitimacy of children born out
of that wedlock. Because of non – registration of marriage, a woman who has given herself
physically, emotionally and otherwise gains nothing but stands to lose everything if the marriage
is denied by the men. The children born out of the alleged wedlock also go through the mental
trauma because of their doubt on his paternity. This assault on children’s sensibilities can be
easily avoided if threes certificate of registration of marriage between his mother and father.

c. Abdur Rahim Undre Vs Padma Adbur Rahim Undre, AIR 1982 Bom 341

Summary 1:

FACTS

The appellant-plaintiff is the husband and the respondent- defendant is the wife. They got
married in England in 1966. Husband was a Muslim and wife was a Hindu at the time of
marriage. They both were Indian citizens and had domicile of India. There marriage took place
under the civil laws of England, therefore, it was secular and monogamous in nature. In 1969,
after they returned to India, wife was converted into Muslim and thereafter, Nikah was also
performed on the same day. In 1978, plaintiff alleged that he gave talaq to the defendant under
the unilateral talaq system in her absence. Later that night he conveyed the talaq to her orally.
The defendant was forbidden to enter the house where plaintiff lived which was actually their
matrimonial house.

Plaintiff filed a suit for injunction against the wife on the grounds of cruelty and mental stress.
There was a counter-claim by the defendant. Plaintiff alleged that the Muslim law would apply
as Nikah took place between them after the defendant’s conversion. But the Defendant alleged
that the Trial Court had no jurisdiction over the matter at all as the marriage happened under the
civil law.

145 | P a g e
The Trial Court held the marriage to be intact and that the talaq was not valid. The house where
the defendant was not allowed to enter was held to be their matrimonial house. Appeal by the
plaintiff. Single Judge declared that marriage would be governed under the Foreign Marriage
Act, 1969. Therefore, divorce could be obtained under the provisions of same act. Appeal was
dismissed and plaintiff is not entitled to injunction. Later, the SC granted partial injunction to the
plaintiff.

ISSUE

1. Whether Foreign Marriage Act, 1969, would apply for the validation of their marriage
given their marriage took place in England under the civil law and that the wife was a
Hindu and husband was a Muslim at the time of marriage?
2. Whether the plaintiff will be granted an injunction as alleged by him?
RULE

1. a) Section 18(1) of the Foreign Marriage Act, 1969:


18. Matrimonial reliefs to be under the Special Marriage Act, 1954 –
(1) Subject to the other provisions contained in this section the provisions of Chapters IV,
V, IV and VII of the Special Marriage Act, 1954 shall apply in relation to marriages
solemnized under this Act and to any other marriage solemnized in a foreign country
between parties of whom one at least is a citizen of India as they apply in relation to
marriages solemnized under the Act.
b) No secular civil marriage can be overpowered by any kind of religious marriage.

2. a) Section 18(4) of the Foreign Marraiage Act, 1969:

Nothing contained in sub-section (1) shall authorise any court to grant any relief under
this Act in relation to any marriage in a foreign country not solemnized under it, if the
grant of relief in respect of such marriage (whether on any of the grounds specified in the
Special Marriage Act, 1954, (45 of 1954) or otherwise) is provided for under any other
law for the time being in force.
b) Grant of injunction would depend on the house being matrimonial.
APPLICATION

1. In this case, both the parties were Indian citizens and their marriage was solemnized in
England. Therefore, their marriage is registered under FMA. After their return to India,
their marriage was still subsisting. The plaintiff could not provide any proof about the
defendant being converted into Muslim and thereafter performance of Nikah, which
comes under the Muslim personal law. The evidences provided were not enough to prove
it. Unilateral talaq is not a form of divorce under the FMA and therefore the divorce
stands invalid. Marriage was secular and monogamous under the civil law of England. As

146 | P a g e
said by the trial court that no religious marriage can overpower a secular marriage.
Hence, marriage under the Muslim personal law would not be accepted even if the
conversion was done.
2. It was proved that the house where the defendant was forbidden to enter was under joint
ownership and therefore it was there matrimonial house and both the parties had a share
in it. The defendant would be allowed to live in a portion of the house also for the sake of
the children. Plaintiff had no right to stop the defendant from entering the house also
because all the allegations made by him were false.
CONCLUSION

The marriage is held to be valid under the Foreign Marriage Act and therefore the marriage
stands undissolved under the method of unilateral talaq as per the provisions of Muslim personal
law. The house being matrimonial house has to be shared by both the parties. Relief to be
granted under the section 18(1) and 18(4) of the FMA. Plaintiff is liable to provide the defendant
with additional and ad hoc maintenance per month.

Summary 2:

Facts:-

The two parties married in the England on the 5th of May 1966. Both were citizens of India and
had domicile in the same. At the time of the marriage, the plaintiff was a Muslim and the
defendant was a Hindu. The two were married under the Marriage Act of 1949 which prevailed
in the United Kingdom thus making their marriage a civil union. The couple had two children
thereafter and returned to India with their offspring in 1969. The plaintiff alleged that the
respondent converted to Islam on the 29th of December 1969 and that a Nikah Fasid ceremony
was performed on the same day. The pair had another child following this before the marriage
became strained. The plaintiff claimed that the respondent became impossible to live with and
the marriage itself became untenable. As a result of this, he eventually divorced her through
Talaq on the 4th of April 1978. The declaration was not made in her presence and was only
intimated to her later on. The appellant contended that the respondent had converted to Islam and
he was thus within his rights to divorce her through Talaq. Furthermore, even if the conversion
had not taken place, personal law still applied which granted him the power to unilaterally
divorce his spouse. He also stated that the presence of two adult Mohammedans at the marriage
in England had de facto turned it into a Nikah Fasid. The respondent disputed these claims on the
grounds that the marriage had been solemnized according to English Civil Law and that her
alleged conversion to Islam was false. She maintained her religious status as a Hindu throughout
the marriage and that the union between her and the appellant had not been dissolved in the eyes
of the law.

147 | P a g e
Issue:-

1. Which law shall apply to the two parties with regards to marriage and divorce?
2. Whether or not the respondent had indeed converted to Islam and undergone the Nikah
Ceremony following her return to India in 1969?
3. Whether or not the appellant has divorced the respondent through means that are
recognized and validated by law?
Rule:-
Section 18(1) of the Foreign Marriage Act 1969:-

Subject to the other provisions contained in this section, the provisions of Chapters IV, V, VI and
VII of the Special Marriage Act, 1954 (43 of 1954) shall apply in relation to marriages
solemnized under this Act and to any other marriage solemnized in a foreign country between
parties of whom one at least is a citizen of India as they apply in relation to marriages
solemnized under that Act.
Section 18(4) of the Foreign Marriage Act 1969:-

Nothing contained in sub-section (1) shall authorise any court to grant any relief under this Act
in relation to any marriage in a foreign country not solemnized under it, if the grant of relief in
respect of such marriage (whether on any of the grounds specified in the Special Marriage Act,
1954 (43 of 1954) or otherwise is provided for under any other law for the time being in force.
Analysis:-

On the first issue the court decreed that the marriage had been undertaken in accordance with the
Marriage Act 1949 in force in the United Kingdom and thus the marriage was civil in nature. The
presence of two adult Muslims at the ceremony could not ipso facto turn the marriage into a
Nikah Fasid as was ruled in Eugene Berthinume v. Dame Anne Marie. A civil marriage shall
overrule all religious forms of marriage. The two parties at the time of entering the marriage had
every intention to enter into a civil union and thus it cannot be seen that there was a concurrent
desire on the part of both too engage in a religious union as well. The two forms of marriage are
antithetical to each other and cannot co-exist. If the appellants contentions were allowed it would
make civil marriages themselves an impossibility.

The second issue of whether or not a Nikah Fasid had been undertaken by the wife may be dealt
with by examining the requirements for such a declaration to be seen as valid under Muslim law.
There needs to be a degree of conviction in the declaration as well as a public acceptance of
Allah as the one true God with Mohammad being his prophet. There are various factums and
devices used by the court to deduce whether or not a valid conversion has taken place. This is to
guard against conversions which are undertaken simply in order to avail of some material or
other benefit. In this case the burden of proving conversion rested upon the appellant and was
left unfulfilled. Therefore, a valid conversion did not in fact take place at all. The court made

148 | P a g e
several observations such as, 1) there was no public declaration made by the respondent
regarding her alleged conversion, 2) there was a reference made to the date of the original
marriage itself, 3) the appellant could easily have done had the ceremony performed in England
but did not do so, 4) There was no pressing occasion or immediate need for the respondent to
convert, 5) the appellant’s father was an orthodox Muslim and if the appellant really wanted, he
would have had the conversion done before the birth of his children. Taking these factors into
account, the court held that there had been no conversion by the wife.

On the third issue of whether or not the marriage may be dissolved by way of Talaq the
reasoning by the court follows a complicated path which reveals deficiencies in the Special
Marriage Act 1954 with regards to matrimonial relief in the event of foreign marriages. The
court goes into a detailed analysis which criticizes and makes several recommendations for
amendment to the Act which, while revealing, do not hold much importance for the overall
judgment in this case. It was held by the Court that the Foreign Marriages Act was meant as a
comprehensive legislation for any and all in a foreign country. According to the court Section 18
brings all marriages within its jurisdiction. Section 18(1) FMA outlines matrimonial relief that
may be granted and to what kind of marriages with the exceptions to the same stated under
Section 18(4). The marriage in question does not fall under any of those exceptions and thus the
Foreign Marriage Act was eminently applicable to the union between the two parties. Thus the
Talaq form of divorce was ruled out as legal possibility and any attempt to divorce the wife
through it would be unrecognized by law
Conclusion:-

The court in this case decreed that marriage between the two parties was still subsisting and that
the respondent was indeed still a Hindu. The marriage was a civil one and could not be abrogated
by personal law. Furthermore the respondent had not changed her religion thus making an
attempt to divorce her through religious means legally futile. The appellants appeal was thus
dismissed.
Critical Analysis:-

The ruling was important as it laid out several guidelines for marriages undertaken in foreign
countries and the parameters of the Foreign Marriage Act. It became clear that civil unions once
undertaken would be superior to any religious considerations and would be governed by the law
in force in the country of its undertaking. The FMA would apply to all marriages undertaken
outside under India unless the form of marriage itself was not legally recognized under Indian
law. This prevented spouses such as the appellant in this case from taking the easy route out of a
marriage. The holding protected the wives in such situations from being unilaterally divorced
even after a civil union and affirmed that the Foreign Marriage Act could not be unduly
manipulated through religious conversions or personal law.

149 | P a g e
d. Vincent Joseph Konath vs. Jacinitha Angela Vincent Konath, AIR 1994
Bombay 120

Parties were Christians. They were domiciled in India. They married in Bahrain Island.
However, it was not registered under the FMA. It was, therefore, not solemnized in the manner
prescribed under the FMA.

2. The Court is required to interpret Ss. 18(1) and 18(4) of the Foreign Marriage Act 1969 and S.
2 of the Indian Divorce Act, 1869

(1) Whether the Court having jurisdiction under the Indian Divorce Act, 1869, can dissolve a
Christian Marriage not solemnised in India but in a foreign country between parties who are
citizens of India and domiciled in India?

(2) If the answer to (1) is in the negative, whether the matrimonial relief sought herein by the
Petitioner can be granted by the Court having jurisdiction to entertain and try suits and
proceedings under the Special Marriage Act, 1954?

6. Section 18(1) of the Foreign Marriage Act 1969 provides that the provisions of Chapter IV, V,
VI and VII of the Special Marriage Act, 1954 shall apply in relation to marriages solemnized
under the said Act and to any other marriage solemnized in a foreign country between the parties
of whom one at least is a citizen of India as they apply for marriages solemnized under the Act

If Section 18(4) of the Foreign Marriage Act 1969 is held to be applicable to the suits herein,
Section 18(1) of the Act shall not be applicable. Section 18(4) of the Act continues non-obstante
clause.

9. Section 2 of the Indian Divorce Act, 1869 provides that the Court shall have jurisdiction to
grant relief under the Act where the petitioner and the respondent professes Christian religion
and where the parties to the marriage are domiciled in India at the time when the petition is
presented. Section 2 of the Act nowhere specifies that the said Act can be invoked by the party
seeking dissolution of marriage only if the marriage is solemnized in India and cannot do so if
the marriage is performed out of India.

I hold that the Indian Divorce Act, 1869 can be invoked to dissolve the marriage herein even
though the marriage was performed out of India. In this view of the matter S. 18(4) of Foreign
Marriage Act, 1969 is applicable and not Section 18(1) thereto.

e. Minoti Anand & Anr vs Subhash Anand, AIR 2011 Bom 61

The petitioner and respondent married in Japan in 1972. This marriage was solemnized in the
Sumiyoshi Temple, Osaka, Japan. The marriage ceremony was performed by a Japanese Priest in
the temple and according to the rites followed by the Japanese in that temple. The marriage was
then registered under the Foreign Marriage Act. On 16th November, 1972 a certificate of

150 | P a g e
registration was issued by the Consulate General of India. Husband filed Petition under Section
13(1)(ia) of HMA. The petitioner challenged stating that they are governed by the FMA and
petition should be filed u/s 18 FMA.

Held;

The registration certificate of the marriage between the parties is conclusive evidence of the fact
that their marriage was solemnized under the FMA and not any other Act. Therefore, evidence
with regard to the fact that their marriage was actually solemnized under any other Act at any
other time cannot be allowed and cannot be seen. It may be that parties married for the purpose
of their own satisfaction or appeasement under different laws. In this case, the husband has
sought to claim that the parties had married initially under the HMA as per Hindu Vedic rites
and, therefore, their marriage can be dissolved only under the HMA. However, their marriage
has not been registered under the HMA. Their marriage has been registered under the FMA.
Hence, it must be taken to be proved conclusively that it was solemnized under the FMA and not
HMA.

The subtle distinction between this case and Vincent is in the registration itself. Since
registration certificate statutorily confers conclusive proof of solemnisation of marriage. The lack
of certificate cannot show such solemnisation since only upon registration the marriage is
deemed to be solemnnised under that Act. Upon non-registration of the marriage, the marriage
cannot be taken to be solemnised under the FMA, though it was solemnised in the foreign
country. Therefore, the fact of solemnisation is conclusively proved. Upon such solemnisation,
the provisions of Special Marriage Act would apply under Section 18(1) of the FMA for grant of
matrimonial reliefs. Therefore, when the marriage is not registered and it is not conclusively
proved that it is solemnised under the FMA, the SMA would not apply and the other law,
otherwise applicable to the parties as per their personal law, would apply.

VIII. Week 12: Maintenance of Family Members


a. Maintenance under Muslim Law
i. Zohara Khatoon V. Mohd. Ibrahim, AIR 1981 SC 1243

Summary 1:

Ms. Zohara Khatoon was married to Mohd. Ibrahim. This case deals with her right to
maintenance, under S.125 of Code of Criminal Procedure 1973, after she legally divorced her
husband.

S.125 is a secular provision that enables a magistrate to issue an order for maintenance, if any
person having sufficient means neglects or refuses to maintain his wife unable to maintain

151 | P a g e
herself. The 1973 amendment to the section expanded the definition of “wife” to include women
who “had been divorced by or had obtained a divorce from, their husband and not remarried”.
S.127 of the code deals with alteration of any allowance issued under S.125.

The High Court quashed the orders of maintenance issued by the trial court. The decision of the
court was based on the interpretation of clause (b) of the explanation to S.125. It ruled that for
the appellant to be classified as “wife” under the section, the divorce must proceed from the
husband and must be the act of the husband. Since, in the given fact situation the dissolution of
marriage was brought about at the instance of the wife under the Dissolution of Muslim
Marriages Act, 1939 - the divorce was not obtained “from the husband”.

The Supreme Court held that “woman who has obtained divorce from her husband” includes a
Muslim woman who has been granted a decree of dissolution of marriage by the court.
The judges observed that –
 S.125 is a general law that provides summary machinery for maintenance.
 The Dissolution of Muslim Marriages Act, 1939 provided an independent remedy to
women who had no right to get a decree of divorce against the will of their husband and
were subject to desertion and maltreatment. It is in this background that ‘has obtained
divorce from her husband to be read’.
 S.125 envisages all forms of divorce under the Mahomedan law i.e. unilateral divorce
from husband, divorce by mutual consent and by obtaining a decree from court. Further
S.127 (3)(c), specifically caters to those situations of maintenance where divorce has
been obtained by a wife through a decree of court.
 A wife who has been granted such a decree by civil court cannot be intended by the
legislature to be deprived of such a benefit.
 If the clause included only unilateral divorce from the husbands then this would defeat
the secular application of the clause – as such forms of divorce are allowed only under
Muslim Personal Laws.
The court restored the order of the Magistrate granting maintenance to the appellant.

CRITICAL ANALYSIS:

In this case, the Supreme Court held that - a Muslim woman who has obtained a decree of
divorce from a civil court, is entitled to maintenance under Section 125, Code of Criminal
Procedure 1973.

1. Difference in Approach - The High Court vs. The Supreme Court

152 | P a g e
This is a very well written judgment. In reaching the above conclusion the court relied on
statutory interpretations of S.125, and S.127. However, it is to be noted that these interpretations
were analyzed in context of the purpose of the statute as well as the social realities.
This can be seen as a departure from an entirely conservative approach of the High Court –
where a literal understanding of the text led to an absurd decision that only those women who
had been unilaterally divorced from their husband were entitled to maintenance under S.125.
The Supreme Court attacked such an understanding of text on various levels –
Firstly, it traced the section back to its original form i.e. S.488 of the 1898 code and analyzed the
very purpose of the statute. The section was included in the code of criminal procedure to confer
certain powers on the magistrate. The very fact that these powers include the ability to decide on
the ‘civil question’ of maintenance (governed by personal laws) illustrates the general
applicability of this section. S.488 served as a speedy remedy and a summary procedure before a
magistrate against starvation of a deserted wife or child – this is how it was intended to be by the
legislature. Can it then be argued that this section would not come to the remedy of a starving
wife who has legally obtained a decree of divorce? This defeats the very purpose of the statute.
Such an analysis was also adopted by the court in Badshah v. Urmila Badshah Godse, where it
was held that “a woman duped into marrying a person who was already having a subsisting
marriage would, at least for the purpose of claiming maintenance under S.125 is to be treated as
the legally wedded wife.”
Secondly, both the judges considered the social realities affecting women. Justice Faizal Ali
stated that it is in background of the social inequalities that Dissolution of Marriages Act, 1939
intended to eliminate – that the term ‘has obtained divorce from her husband to be read’. Further,
Justice Koshal argued that it is highly implausible that the legislature did not intend to extend the
benefits of S.125 to such a wife who has been granted a decree of dissolution of marriage by a
Civil Court.
The court had anyways established that even a literal interpretation of S.125 and 127 leads to a
conclusion that these section envisage all three forms of a Muslim Divorce – therefore the fact
that the above-mentioned analysis was still included in the judgment can be seen as somewhat
progressive.

2. Paragraph 29 of the Judgment

In concluding paragraph of the Judgment, Justice Faizal Ali states that the case is squarely
covered by clause (b) of explanation to S.125. Thus, the respondent’s case based on application
of S.125 is dismissed and further, since the husband has not given any application for
cancellation of the maintenance on the grounds enshrined in S.127(3)(b) – the claim for
maintenance is allowed. Now, a joint reading of S.125 and 127 clearly leads to a conclusion that
a situation where the wife has obtained divorce from her husband comes within the ambit of
S.127(3)(c) – where she can voluntarily relinquish her right to maintenance. Therefore the
mention of 2.127(3)(b) in the concluding paragraph is erroneous and leads to ambiguity.

153 | P a g e
3. Section 127(3)(b) and Section 127(3)(c) – Purpose?

S.127 deals with alteration of any allowance ordered under S.125 –


S.127(3)(b) states that if maintenance orders have been passed in favour of a woman who has
been divorced by her husband, such orders can be cancelled if a certain sum promised under
personal/customary laws has been paid.
While, S.127(3)(c) states that a woman who has obtained a divorce from her husband can
voluntarily relinquish her rights to maintenance.
The purpose of these separate clauses is ambiguous. According to this a woman who has been
divorced unilaterally from her husband is further not entitled to any maintenance if she has
received the promised amount – this in a way creates a situation of double privilege for the
husband. First, he can unilaterally divorce the wife (often without any justified grounds) and
second her right to maintenance under such situations is diluted by S.127(3)(b). If in such a case,
the wife has not remarried and is in a starving position – does the fact that a promised amount
was paid at the time of divorce make any difference? Does this not defeat the purpose of S.125?
This specially appears to be absurd when 127(3)(c) states that if a Muslim woman has obtained a
divorce from court, her right to maintenance can be quashed only when she voluntarily
relinquishes it. Shouldn’t this voluntary relinquishment of the right be applied to all forms of
divorce?

4. Shah Bano Case and The Muslim Women (Protection of Rights on Divorce) Act, 1986

S.488 of the 1898 Act was intended to be secular in its application - applicable to all irrespective
of caste, creed or religion. The same was upheld by the court in various cases including the
current case. The Supreme Court in Zohara Khatoon, before addressing the question of law in the
case, discussed the secular nature of S.125. This judgment was passed in 1981. Again in Shah
Bano case the Supreme Court held that Muslim women come under purview of S.125 CrPC.
However, the judgment faced a strong opposition from conservative Muslim sections of the
society and this resulted in a major political upheaval in the country. Shah Bano became a
landmark case - challenging issues of women rights, religious autonomy further affecting
country's political conditions in the years to come. Faced with strong opposition from its loyal
Muslim vote bank - the Congress government decided to pass The Muslim Women (Protection
of Rights on Divorce) Act,1986 which excluded Muslim Women from the ambit of S.125 and
thus over ruling Supreme Court's judgment in the Shah Bano case. Such a legislation, adversely
affecting women rights, borne out of political conveyance sparked off various debates within
several women organisations of the country. Were questions of interference with personal laws
more grave than violation of rights of maintenance of Muslim women (maintenance being
recognised as a basic right by United Nations etc.)? Is the state not responsible to ensure
maintenance or basic financial needs of its citizens?

154 | P a g e
The enactment of such a legislation along with the Ram Mandir case led to the downfall of the
Congress govt. that had earlier exploited the 'liberal' image of Rajeev Gandhi to secure votes.
This in the 90s, marked the beginning of coalition governments in India.

5. Gender neutral legislation

Though the current case specifically deals with rights of divorced Muslim women for
maintenance - I would like to mention here that maintenance being such a basic right should be
made gender neutral. S.125 explicitly deals with maintenance of wives, children and parents. But
considering the changing social realities - maintenance should be made a more universal and
gender neutral right and right to maintenance of a husband must be recognized.

Summary 2:

The case of Zohara Khatoon served as being the first case to question the rights of a Muslim
woman after divorce. The appellant, Smt. Zohara filed a petition under Sec. 125 of CrPC,
demanding maintainance allowance for herself and her son from her husband Mohd. Ibrahim.
She had obtained a decree of dissolution of her marriage under the provisions of Dissolution of
Muslim Marriages Act, 1939. Her husband contends that since the marriage had not been
dissolved by a decree of dissolution by the Civil Court, she was not entitled to any maintenance
allowance.

The Special Judicial Magistrate, Barabanki however did not find this defence as a strong
one and did not favour it. They allowed the petition under Section 125 to be held good and
judged that the Husband was liable to pay maintenance allowance to both the wife and their
child. They also held that the appellant had been neglected by her husband without any
reasonable and probable cause. This order of the Magistrate was upheld in revision by the High
Court after which the Respondent approached the Court under Section 482 Cr.P.C. It was then
held that because of dissolution of marriage Smt. Zohra was not entitled for any maintenance
allowance for herself but it was directed that the opposite party shall be paid maintenance
allowance at 40/- p.m. for her son.

Smt. Zohra then filed a Special Leave Petition before the Supreme Court, against this
judgment of the H.C. The Supreme Court held that in The Hon'ble Supreme Court held that in
view of decision of the Constitution Bench of that Court in the case of Mohd. Ahmed Khan v.
Shah Bano Begum , a Muslim wife whose marriage has been dissolved by a decree of Court, was
still entitled to maintenance under Section 125 Cr.P.C. It was directed that opposite party shall
pay maintainable allowance at Rs. 140/- per month i.e. Rs. 100/- per month for Smt. Zohra and
Rs. 40/- per month for her son w.e.f. the date of filing of petition. It was further directed that
opposite party shall pay arrears of maintenance allowance at the above rates within three months

155 | P a g e
from the date of judgment. On behalf of Smt. Zohra a prayer was also made before Hon'ble
Supreme Court that in view of the increased cost of living the amount of maintenance should be
increased. In this behalf it was directed that it was open to her to approach the concerned
Magistrate Under Section 127 Cr.P.C. for increasing the maintenance allowance.

The respondent however did not pay any amount despite the S.C’s direction. The
appellant then moved an application under Sec. 125(3) for execution of the order. The
Respondent files objection to this. One of the objections which found favour with the Magistrate
was that in view of the Muslim Women (Protection of Rights on Divorce) Act, 1986, Smt. Zohra
was not entitled to get any maintenance allowance and the order could not be executed. The case
was then sent for revision.

The only question for consideration in this revision is as to whether a Muslim wife whose
marriage had been dissolved at her instance by means of a Court decree under the provisions of
Dissolution of Muslim Marriages Act, 1939 prior to the enforcement of the Act and whose
application for maintenance allowance had been allowed Under Section 125, Cr.P.C. prior to the
enforcement of the Muslim Women (Protection of Rights on Divorce) Act, is entitled to get it
executed after the commencement of the Act, when her application for execution was pending at
that time the said Act came into force. The Act came into force w.e.f. May 19,1986. The decree
of dissolution of marriage was passed on 15.1.1973 i.e. prior to the commencement of the Act.
The petition Under Section 125, Cr.P.C. was moved on 17.9.1974 i.e. prior to the
commencement of the Act. It was allowed by learned Magistrate on 29.12.1976. The revision by
learned Sessions Judge was also dismissed prior to enforcement of the Act. This Court also
disposed of the petition of opposite party under Section 482, Cr.P.C. prior to the enforcement of
the Act. The Hon'ble Supreme Court also finally disposed of the appeal of Smt. Zohra on
November 26, 1985 awarding maintenance allowance which means that the judgment of Hon'ble
Supreme Court was also moved by Smt. Zohra on 27.2.1986 i.e. prior to commencement of the
Act. It has been rejected by the Magistrate on 7.7.1986 i.e. after commencement of the Act. It
follows from this that the execution application of Smt. Zohra Under Section 125(3) Cr.P.C. was
pending when the Act came into operation.

Section 125 Cr.P.C. gives a right to a wife who is unable to maintain herself, to get maintenance
allowance from the husband who inspite of having sufficient means neglects or refuses to
maintain her. Sub-section (3) of the said section provides for execution of order of maintenance
allowance in case it is not obeyed by the husband. The preamble of the Act lays down that it is
an act to protect the rights of Muslim women who have been divorced by, or have obtained
divorce from their husbands and to provide for matters connected therewith or incidental there to
Section 3 of the Act lays down that a divorced Muslim woman shall be entitled to maintenance
for the iddat period, which shall be payable by her husband. It also makes provision for down
payment of mehr or dower and restoration of properties given to dower and restoration of
properties given to wife before or after her marriage and payment of maintenance for the
children born of the wed-lock for a period of two years from respective dates of birth of such
156 | P a g e
children provided such children are being maintained by their mother. Section 7 relates to
transitional provisions, on which the learned Magistrate while passing the impugned order has
placed reliance. It has been clearly provided that any application by divorced Muslim woman
under Sections 125 and 127 of the Code of Criminal Procedure pending before the Magistrate on
the commencement of the Act shall. The view taken by learned Magistrate is that since Section 7
includes whole of Section 125 Cr.P.C. an execution application moved or filed under Sub-section
(3) of that section also gets covered by the provision of Section 7 of the Act. Also, it nowhere
lays down that orders for maintenance allowance passed prior to the commencement of the Act,
shall also be subject to the provisions of the said Act.

Section 127 is regarding alteration of maintenance allowance in the changed circumstances.


Therefore, when Section 7 of the Act also includes Section 127, it follows that by including
Section 125 along with Section 127, the main petition under Section 125 Cr.P.C. for awarding
maintenance alone is included and it does not include execution application under Sub-section
(3). Apart from this, execution of an order is almost a ministerial act and is a consequence of the
order or a decree passed by a Court. It does not create any substantive right. The substantive
rights are created by a Court's decree or an executable order. The Act does not lay down that any
order passed by a Court for maintenance in favour of a Muslim woman would become void and
would be revised in accordance with the provisions of the said Act. Thus, provisions of Section 3
of the Act are confined only for awarding of maintenance allowance for iddat period, payment of
mehr or dower and return of property given to the wife before, or at the time of or after her
marriage by her relatives or friends and payment of maintenance to the wife for the children born
from the wed-lock for a period of two years from the date of their respective birth. Section 3
does not go beyond these limitations. It follows that in accordance with the provisions of the Act
a Muslim woman is not entitled to after commencement of the Act for maintenance allowance
from her husband beyond iddat period.

Thus the Supreme Court held that the case would be sent back to the Magistrate for executing
order of maintenance allowance against Respondents under the provisions of Section 125(3)
Cr.P.C.

ii. Mohd. Ahmed Khan V Shah Bano Begum, AIR 1985 SC 945

Introduction:

The Shah Bano case is an infamous divorce lawsuit in India that has generated political
controversy in the country. The legislative overruling marked the start of an ugly brand of vote-
bank politics in India. This case caused the Rajiv Gandhi government to pass the Muslim
Women (Protection of Rights on Divorce) Act, 1986. It diluted the secular judgment of the
Supreme Court and, in reality, denied even utterly destitute Muslim divorcees the right to
maintenance from their former husbands.

Facts:

157 | P a g e
In 1978, a Muslim lawyer had divorced his wife, Shah Bano by pronouncing triple talaq. As
mandated by Muslim personal law, the husband paid 3000 rupees to his divorced wife during
iddat. Having been driven out of her matrimonial home, shah bano filled a petition under section
125 of the CrPc before the judicial magistrate at Indore. She sought maintenance from her former
husband, who she claimed had an annual professional income of about 60000 rupees. The
magistrate ordered the husband to pay Shah Bano Rs. 25 every month as maintenance. Shah
bano appealed to the Madhya Pradesh High Court in 1979. the maintenance amount was revised
to Rs. 179.20 every month. Then shah bano’s former husband Mohd. Ahmed khan petitioned the
supreme court in 1981 to challenge the high court’s decision.

Issue:

Would the payment of Mahr, regardless of how significant that payment was, absolve a Muslim
husband from providing for his divorced wife under section 125 of CrPc?

Rule:

Section 125 of CrPc

A first class magistrate could order a husband to provide a monthly allowance of up to 500
rupees to his wife/divorced wife if he neglected to maintain her and she was unable to maintain
herself.

Section 127 of CrPc

If a wife was paid any money on divorce under the personal law governing her, any order for
maintenance passed under section 125 was to be cancelled.

Analysis:

The Supreme Court in the present case quoted with approval the decisions taken in the case of
Bai Tahira V. Ali Hussain and Fuzlunbi V. K. Khader Vali. The above two cases held that the
divorced Muslim wife is entitled to apply for maintenance under Section 125. However, the court
found the decision held in the above cases in direct contravention with the section 127 of the
CrPc. Clarifying that, the court added that the payment of mahr does not absolve the husband’s
duty to pay maintenance to his wife under Section 127(3)(b) of the Code.

The court refused to accept the argument that the order for maintenance under section 125 could
be cancelled under section 127, simply because the husband had made a payment to the wife at
the time of divorce, under Muslim personal law.

It asserted that mahr was not a payment liable to be made to a muslim woman ‘on divorce’.
Exploring the meaning of mahr, the court concluded that it was an amount that a wife was
entitled to in consideration of marriage; it cannot be construed as a divorce payment. The mere

158 | P a g e
fact that mahr was sometimes paid at the time of dissolution of marriage did not imply that the
payment was occasioned by divorce.

By thus defining mahr as a marriage payment, rather than a divorce payment, the court
emphasized that the payment of mahr could not bar courts from also awarding maintenance.

According to the court, Muslim personal law did not address the situation envisaged in section
125. The personal law mandated that mahr should be paid during iddat but it did not
contemplate a scenario, where the divorced wife was unable to maintain herself after the iddat
ended. In such a scenario she would be entitled to seek maintenance under section 125 of CrPc.

The court alluded to the religious neutrality of section 125 of the CrPc, stating that whether the
spouses were ‘Hindus or Mulims, Christians or Parsis was wholly irrelevant. Its interpretation
was that the underlying purpose of section 125 was to protect dependents from vagrancy and
destitution – thus it saw no reason to exclude Muslims from its sweeping ambit.

To support the view that awarding maintenance would not go against the tenets of Islam, the
bench also interpreted the section of Quran to mean that the husbands were duty bound to
maintain their wives.

The operative part of the judgment was followed by deep dissatisfaction over the legislatures
failure to establish a Uniform Civil Code for all citizens, in accordance with article 44 of the
constitution.

Conclusion

The court concluded the following things (i) the payment of mehr by the husband on divorce is
not sufficient to absolve him of the duty to pay maintenance to the wife.

(ii) The liability of the husband to pay maintenance to the wife extends beyond the iddat period if
the wife does not have sufficient means to maintain herself.

(iii) Section 125 of the Code applies to all citizens irrespective of their religion

(iv) Section 125 overrides the personal law, if is any there conflict between the two.

(v) There is no conflict between the provisions of Section 125 and those of the Muslim Personal
Law on the question of the Muslim husband’s obligation to provide maintenance for a divorced
wife who is unable to maintain herself.

Keeping above factors in mind the court dismissed the appeal. It confirmed the maintenance
amount awarded by Madhya Pradesh high court and also awarded Shah Bano the legal cost.

Aftermath

159 | P a g e
The Shah Bano judgment, as claimed, became the centre of raging controversy, with the press
turning it into a major national issue. The Shah Bano judgment elicited a protest from many
sections of Muslims who also took to the streets against what they saw, and what they were led
to believe, was an attack on their religion and their right to their own religious personal laws

Initially, the Rajiv Gandhi led congress party that had won the parliamentary elections of 1984
by a sweeping majority, favored the judgment. However, after suffering losses in state election in
some Muslim dominated regions, the government changed its approach.

There were calls among Muslim conservatives asking for the parliament to pass a law nullifying
the SC judgment in the Shah Bano case. The Rajiv Gandhi government crumbled under pressure
and enacted the Muslim Woman (Protection of rights in divorce) Act, 1986.

Contrary to its name the MWA actually undermined the far-reaching protection granted to
divorces Muslim woman in Shah Bano and other similar cases. According to the MWA, mahr
and maintenance were to be paid to a divorced Muslim woman only during the three-month iddat
period. It effectively deprived Muslim woman of the right to file a maintenance petition under
section 125 of the CrPc.

My opinion:

The judgment of the Supreme Court in the shah bano case opens with the words – ‘this appeal
does not involve any question of constitutional importance.’ In the end, the judgment sparked a
major controversy in India’s constitutional history.

The aspects of the Supreme Court judgment were relatively non-controversial. It was not the
decision itself but the manner in which it was taken that raised the controversy. I agree with the
decision taken by the court in the Shah Bano judgment, however I believe that the courts were
very critical of the Muslim personal law. This is why, although two Supreme Court judgments
had largely already said what was said in the Shah Bano case, the latter was a cynosure of
widespread attention.

Summary 2:

Facts
In 1932 Shah Bano married Mahd. Ahmed Khan, an affluent and well-known lawyer. After 14
years of marriage, Khan remarried a younger woman. After the second marriage, the relationship
between Shah Bano and Khan deteriorated to the extent that he drove her out of their
matrimonial home along with the five children born out of their wedlock. At the time when she
was driven out, she was aged 62. In April 1978, when Khan stopped giving her Rs. 200 per
month he had apparently promised, claiming that she had no means to support herself and her
children, she filed a petition at a local court in Indore, against her husband under section 125 of

160 | P a g e
the Code of Criminal Procedure, asking him for a maintenance amount of Rs. 500 for herself and
her children.
Brief History
 April 1978- Respondent (Shah Bano) filed petition for seeking maintenance.
 November 1978- Appellant (Mohd. Khan) files irrevocable divorce.
 August 1979- The learned magistrate directs the appellant to pay Rs.25/month to the
respondent by way of maintenance.
 July, 1980- In a revision application filed by the respondent, the High Court of Madhya
Pradesh enhanced the amount of maintenance to Rs. 179.20 per month.
Judgment
The Honorable Supreme Court held the judgment of the High Court. The court used a dual
approach in its jurisdiction i.e. the Quran and the Statute together. It was seen that the Aiyat no.
241 of Quran itself supports Section 125 of CRPC by asking ordering its followers to make a
provision for the wives to support them in their daily life. The court even said that Dower
couldn’t be the amount given on divorce as no one would divorce their wife as a sign of respect
towards them whereas Dower is the amount given to the wife as a sign of respect. Hence, the
court ordered Mohd. Ahmed Khan for payment of maintenance.

Analysis
The judgment in my opinion was fair in itself. It was evident that in the case that the wife was
not able to maintain herself owing to the old age, with addition to that during the period in which
the case came to the court was the period where women did not participate in bread earning,
rather were only concerned with household work. The court taking a dual approach to the case
was the best thing, which the court could have done. It was very helpful in giving the verdict. It
on one side supported the statutory side on the other hand it annulled the use of private Islamic
laws in their own defense, which was right. Another element which we have to keep in mind is
the fact that the court in it’s jurisdiction did not increase the amount whereas it could have
increased it to 500 per month. This shows that justice should not be to the detriment of one side
as well.
iii. Noor Saba Khatoon v. Mohd. Quasim, AIR 1997 SC 3280

Facts: The appellant (wife) married the respondent according to Muslim rites on 27.10.1980.
During the wedlock, three children were born of the age 6 years, 3 years, 1 ½ years. On certain
disputes arising between the parties, the respondent allegedly turned the appellant out of the
matrimonial home along with the children and refused to maintain her and the children. After
which the respondent took a second wife, Shahnawaz Begum.

Procedural History:

A. Trial Court

161 | P a g e
Appellant wife filed an application under Section 125 Cr.P.C in the Judicial Magistrate claiming
that she has no means of maintaining her children and herself while the respondent had sufficient
means for maintaining them because of his agricultural land and business in electrical appliances.
In the application she claimed for a sum of Rs. 400/- per month for herself and Rs. 300/- per
month as maintenance for each of the three children.

The Trial Court found that the respondent had failed and neglected to maintain his wife and
children and that they had no source of income or means to maintain themselves and accordingly
held that they were entitled to the grant of maintenance from the respondent. The Trial Court
directed the respondent to pay maintenance to the appellant at the rate of Rs. 200/- per month for
herself and at the rate of Rs. 150/- per month for each of the three minor children, till they attain
the age of majority.

 After the order was passed and the matter was rested. The respondent husband divorced
the wife and filled an application in the Trial Court seeking modification under the
provisions of the Muslim Women (Protection of Rights on Divorce) Act, 1986.

B. Modified Order by the Trial Court:


The Trial court modified the order so far as the grant of maintenance to the Wife is concerned
while retaining the part of the order concerning the minor children. They held that in view of the
provisions of the 1986 Act the appellant-wife after her divorce was entitled to maintenance only
for a period of three months i.e. for the period of Iddat. The Trial Court further found that the
right to maintenance under Section 125 Cr. P. C. insofar as the children are concerned was not
affected by the 1986 Act in any manner.

This order was again challenged by the Respondent- Husband through a Revision Petition in the
Court of 2nd Additional Judge.

C. Revisional Court:
The Revsional Court dismissed the revision petition holding that the 1986 Act does not over-
ride the provisions of Section 125 Cr. P. C. for grant of maintenance to the minor children and
that Section 3(1)(b) of the 1986 Act also entitles a divorced woman to claim reasonable and fair
maintenance from her husband for maintaining the children born to her before or after her
divorce from her former husband for a period of two years from the respective dates of birth of
the children and that the said provision did not affect the right to maintenance of the minor
children granted by Section 125 Cr. P. C.

The respondent then filled a Criminal Misc. Petition under Section 482 Cr. P.C. in the High
Court challenging the correctness of that part of the order of the Revisional Court which upheld
the right to maintenance of the three minor children under Section 125 Cr. P. C.

162 | P a g e
D. High Court:

The Court accepted the contention of the Respondent and ruled that a divorced muslim woman is
entitled to claim maintenance from her previous husband for her minor children only for a period
of two years from the date of birth of the concerned child. Moreover, they denied the right to
maintenance to the minor children under Section 125 Cr.P.C. on accounts of the new legislature
of the 1986 Act.

The Court explained that two older children of the age 6 years and 3 years, at the time when the
petition was filled, were not entitled to the right of maintenance as they had completed the 2
years of age which is the criterion in 1986 Act for the grant of maintenance. While the youngest
child, who was 1 ½ years old when the petition was filled, is entitled to receive maintenance till
she attained the age of two years.

Now the appellant has come to the Supreme Court by a special leave petition.

Supreme Court:

The main issue before the court is whether Section 3(1)(b) of the 1986 Act in any way affect the
rights of the minor children of divorced Muslim parents to the grant of maintenance under
Section 125 Cr.P.C.?

 The Supreme Court cleared the confusing law, establishing that the two year period
which Section 3(1)(b) talks about is aimed at providing some extra amount to the mother
for her nourishment for nursing or taking care of the infant child.
 It does not curtail the rights of the child to claim maintenance under Section 125 Cr.P.C.
It would be unreasonable, unfair, inequitable and even preposterous to deny the benefits
of Section 125 Cr.P.C. to the children only on the grounds that they were born to Muslim
parents.
 They held that under Section 125 Cr.P.C. the maintenance of the children is obligatory on
the father irrespective of his religion as long as he is in a position to do so and that the
children have no independent means of their own.
 Muslim father's obligation, like that of a Hindu father, to maintain his minor children as
contained in Section 125 Cr. P. C. is absolute and is not at all affected by Section 3(1)(b)
of the 1986 Act.
 The right of the children to claim maintenance under Section 125 Cr. P. C. is separate,
distinct and independent of the right of their divorcee mother to claim maintenance for
herself for maintaining the infant children upto the age of 2 years from the date of birth of
the concerned child under Section 3(1) of the Act.

163 | P a g e
 It is the absolute obligation of the father to maintain his children till they attain majority
or are able to maintain themselves or whichever date is earlier. While in the case of a
female child, this obligation of the father extends till her marriage.
 Both the provisions of Section 125 Cr.P.C and Section 3(1)(b) of 1986 Act, cover
different situations and are in no conflict with each other.

Critical Analysis

In my opinion, the judgment of the Supreme Court of overruling the decisions of all the
subordinate courts was absolutely correct. The 1986 Act was enacted after the judgment of Moh.
Ahmed Khan v. Shah Bano Begum, with the aim to protect the rights of Muslim divorced
women. It was not meant to regulate the obligations of a Muslim father to maintain his minor
children. That was and is governed by Section 125 Cr.P.C.

Section 125 Cr.P.C clearly states that it applies to all religions. Moreover, the personal law also
obligates the Muslim father to maintain his childrens. Prof. Tahir Mahmood in his book “Statute-
Law relating to Muslims in India” agrees that the Code is fully applicable to the Muslims as the
muslim personal law promotes that Nafaqa (maintenance) as a birth right of children and
absolute liability of the father.

iv. Danial Latifi v. Union of India (2001) 7 SCC 740

Summary 1:

Facts

This is a writ petition filed in the Supreme Court of India where the petitioners are challenging
the constitutional validity of The Muslim Women (Protection of Rights on Divorce) Act, 1986,
which allegedly makes section 125 of CrPC inapplicable to divorced Muslim women on three
broad grounds. First, that section 125 CrPC was enacted as a public policy and in furtherance of
social justice and excluding Muslim women from it would imply discrimination as the moral
stance of law ought not to be entangled with religion-based personal laws. Secondly, the effect of
the Act would be of nullifying the judgement of the Shah Bano case. And finally, the Act is un-
Islamic and is violative of Articles 14 and 21 of the Indian Constitution and would undermine the
basic secular character of the Constitution.
On behalf of the Union of India, it was submitted that the need for giving effect to a personal law
is a valid ground for discrimination. The policy of section 125 CrPC is not to create a right to
maintenance beyond the purview of personal law. The Act has been enacted to overcome the

164 | P a g e
ratio of the Shah Bano case. The All India Muslim Personal Law Board submitted that the object
of the Act was to undo the effect of the Shah Bano case and to avoid vagrancy. In response the
Islamic Shariat Board presented details on why the proposal of a Muslim man liable for
maintaining his former wife beyond the iddat period, ought not to be accepted.

The Supreme Court decided to consider only the question of the constitutional of the Act and
upholding the same.

Issue

Whether the Muslim Women (Protection of Rights on Divorce) Act, 1986, stands
constitutionally valid?

Rule
The rules applied to this case were-
 The Muslim Women (Protection of Rights on Divorce) Act, 1986.
3. Mahr or other properties of Muslim woman to be given to her at the time of divorce.—

(1) Notwithstanding anything contained in any other law for the time being in
force, a divorced woman shall be entitled to—

a) a reasonable and fair provision and maintenance to be made and paid to her
within the iddat period by her former husband;
(3) Where an application has been made under sub-section (2) by a divorced
woman, the Magistrate may, if he is satisfied that—

a) her husband having sufficient means, has failed or neglected to make or pay
her within the iddat period a reasonable and fair provision and maintenance
for her and the children;
5. Option to be governed by the provisions of section 125 to 128 of Act 2 of 1974.

 Section 125 CrPC: Order for maintenance of wives, children and parents after divorce.
 Article 14 of the Indian Constitution: Equality before law.
 Article 21 of the Indian Constitution: Protection of life and personal liberty.

Analysis
The Court clarified that the Act makes a divorced woman entitled to reasonable and fair
provision for maintenance within the period of iddat wherein the word “provision” indicates that
something is provided in advance for meeting of some needs during the period of iddat.

165 | P a g e
But nowhere has the Parliament provided that reasonable and fair provision for maintenance only
for the iddat period and not beyond it as per section 3(3) of the Act. This clarification was made
specifically because of the differentiation made in the Shah Bano judgement between a provision
or “mata”, which is a one-time transaction made benevolently and maintenance which is made
continually. The Court went on to say that the Act incorporates “mata” as a right of the divorced
woman whether it was rendered a “maintenance” or a “provision”.

Further, the object and scope of section 125 CrPC is to prevent vagrancy and to compel
obligations to support the spouses unable to maintain themselves. On comparing this with the
Act, it is found that the same object is fulfilled and satisfied. This renders the argument of the
petitioners insignificant. It is further clarified that even under the Act, the provisions of section
125 CrPC can still be attracted, if the parties so choose to as per section 5 of the Act. In addition,
the Magistrate can appropriate provision for maintenance beyond the iddat period under section
3(3)(a) of the Act itself.
The judge remarked that the Act actually codifies what has been stated in the Shah Bano case. It
has the same impact as that of the ruling of the case and the mere fact that the legislature took
note of certain facts in enacting the law will not be of much materiality.

In conclusion it was said that the Act is actually in support of Article 21 as the “right to life and
personal liberty” would include “right to live with dignity”, and the Act achieves the object by
guaranteeing the divorced wife maintenance and avoids destitution or vagrancy.

Conclusion

In summation, the court held that the liability of a Muslim husband to his divorced wife in not
confined to the iddat period. A Muslim husband is liable to make reasonable and fair provision to
the divorced wife, which includes her maintenance extending beyond the iddat period which
must be made within the iddat period in terms of Section 3(1)(a). Furthermore, a divorced
woman unable to maintain herself may proceed for maintenance under section 4 of the Act. The
State Wakf Board is to pay such maintenance if the relatives are unable to make the payment.
Thus, the Act certainly does not offend Articles 14, 15 and 21 of the Constitution of India.

Critical Analysis

The judgment in this case acts as a clarification for why it was necessary to overrule the Shah
Bano judgment by the said Act and how the Act would none the less stand constitutionally valid
by virtue of the rights given to the divorced Muslim women. For this reason, it is essential to
understand the ruling of the Shah Bano judgment which is intrinsic to the decision made in this
case. In the case of Shah bano, where the wife was thrown out of her marital home after 43 years

166 | P a g e
of marriage and maintenance was paid up till the end of iddat period on her filing of petition
under section 125 CrPC, it was held that in such matters the Code of Criminal Procedure
overrides the personal law of the parties. Mahr is a sum that cannot ipso facto absolve the
husband’s liability. The court further held that “mata”, which is the provision or maintenance
under The Holy Quran, was not paid. Thus, it cannot be said that a Muslim husband is not
obligated to provide maintenance beyond the period of iddat if the wife is unable to maintain
herself.

Although the enactment purports to overcome the judgement in the Shah Bano case, it
accommodates much of the opinion expressed in it and overrules only the erroneous reasoning
that CrPC can override personal laws, which led to its conclusion. Additionally, the judgment
justifies the clause of “reasonable and fair provision” to a divorced Muslim wife beyond the
iddat period while respecting the personal laws equally.

Hence, it is not depriving Muslim divorced women of the social justice as envisaged in the
Constitution of India contrary to the petitioners’ claim.

Summary 2

FACTS
• The parties had been married for 43 years before the ill and elderly wife had been thrown
out of her husbands residence. For about two years the husband paid maintenance to his
wife at the rate of Rs.200/- per month. When these payments ceased she petitioned
under Section 125 CrPC. The husband immediately dissolved the marriage by
pronouncing a triple talaq. He paid Rs.3000/- as deferred mahr and a further sum to cover
arrears of maintenance and maintenance for the iddat period and he sought thereafter to
have the petition dismissed on the ground that she had received the amount due to her on
divorce under the Muslim law applicable to the parties. The important feature of the case
was that the wife had managed the matrimonial home for more than 40 years and had
borne and reared five children and was incapable of taking up any career or
independently supporting herself at that late stage of her life - remarriage was an
impossibility in that case. The husband, a successful Advocate with an approximate
income of Rs.5,000/- per month provided Rs.200/- per month to the divorced wife, who
had shared his life for half a century and mothered his five children and was in desperate
need of money to survive.
ISSUE
• The principle question for consideration before this Court was the interpretation
of Section 127(3)(b) CrPC that where a Muslim woman had been divorced by her
husband and paid her mahr, would it indemnify the husband from his obligation under the
provisions of Section 125 CrPC.

167 | P a g e
RULE
• Section 127(3)(b) in The Code Of Criminal Procedure, 1973
• (b) the woman has been divorced by her husband and that she has received, whether
before or after the date of the said order, the whole of the sum which, under any
customary or personal law applicable to the parties, was payable on such divorce, cancel
such order,-(i) in the case where, such sum was paid before such order, from the date on
Which such order was made,
• (ii) in any other case, from the date of expiry of the period, if any, for which maintenance
has been actually paid by the husband by the woman;
• Section 125 in The Code Of Criminal Procedure, 1973
• 125. Order for maintenance of wives, children and parents.(1) If any person having
sufficient means neglects or refuses to maintain-(a) his wife, unable to maintain herself,
or
• (b) his legitimate or illegitimate minor child, whether married or not, unable to maintain
itself…
• … Such allowance shall be payable from the date of the order, or, if so ordered, from the
date of the application for maintenance.

ANALYSIS
• Court reiterated that the Code of Criminal Procedure controls the proceedings in such
matters and overrides the personal law of the parties. If there was a conflict between the
terms of the Code and the rights and obligations of the individuals, the former would
prevail. This Court pointed out that mahr is more closely connected with marriage than
with divorce though mahr or a significant portion of it, is usually payable at the time the
marriage is dissolved, whether by death or divorce. This fact is relevant in the context
of Section 125 CrPC even if it is not relevant in the context of Section 127(3)(b) CrPC.
Therefore, this Court held that it is a sum payable on divorce within the meaning
of Section 127(3)(b) CrPC and held that mahr is such a sum which cannot ipso facto
absolve the husbands liability under the Act.
• Where a Muslim divorced woman is unable to maintain herself after the period of iddat,
the Magistrate is empowered to make an order for the payment of maintenance by her
relatives who would be entitled to inherit her property on her death according to Muslim
Law in the proportions in which they would inherit her property. If any one of such
relatives is unable to pay his or her share on the ground of his or her not having the
means to pay, the Magistrate would direct the other relatives who have sufficient means
to pay the shares of these relatives also. But where, a divorced woman has no relatives or
such relatives or any one of them has not enough means to pay the maintenance or the
other relatives who have been asked to pay the shares of the defaulting relatives also do
not have the means to pay the shares of the defaulting relatives the Magistrate would

168 | P a g e
order the State Wakf Board to pay the maintenance ordered by him or the shares of the
relatives who are unable to pay.

RELEVANT CASE LAW


Shah Bano Case
• Shah Bano, a 62-year-old Muslim mother of five from Indore, Madhya Pradesh, was
divorced by her husband in 1978. She filed a criminal suit in the Supreme court of India,
in which she won the right to alimony from her husband. However, she was subsequently
denied the alimony when the Indian Parliament reversed the judgement under pressure
from Islamic orthodoxy. The judgement in favour of the woman in this case evoked
criticisms among Muslims some of whom cited Qur'an to show that the judgement was in
conflict with Islamic law.
• Supreme Court concluded that "there is no conflict between the provisions of section 125
and those of the Muslim Personal Law on the question of the Muslim husband's
obligation to provide maintenance for a divorced wife who is unable to maintain herself."
• As held in Shah Banos case, the true position is that if the divorced wife is able to
maintain herself, the husbands liability to provide maintenance for her ceases with the
expiration of the period of iddat but if she is unable to maintain herself after the period of
iddat, she is entitled to have recourse to Section 125 CrPC. Thus it was held that there is
no conflict between the provisions of Section 125 CrPC and those of the Muslim Personal
Law on the question of the Muslim husbands obligation to provide maintenance to his
divorced wife, who is unable to maintain herself.

HELD
• 1) a Muslim husband is liable to make reasonable and fair provision for the future of the
divorced wife which obviously includes her maintenance as well. Such a reasonable and
fair provision extending beyond the iddat period must be made by the husband within the
iddat period.
• 2) Liability of Muslim husband to his divorced wife to pay maintenance is not confined
to iddat period.
• 3) A divorced Muslim woman who has not remarried and who is not able to maintain
herself after iddat period can proceed against her relatives who are liable to maintain her
in proportion to the properties which they inherit on her death according to Muslim law
from such divorced woman including her children and parents. If any of the relatives
being unable to pay maintenance, the Magistrate may direct the State Wakf Board
established under the Act to pay such maintenance.

v. Iqbal Bano Vs State of UP (2007) 6 SCC 785

Summary:

169 | P a g e
IQBAL BANO V. STATE OF UTTAR PRADESH
Facts:

The Appellant married Respondent 2 in 1959 and had a child with him in 1966. The son passed
away in 1991. R2 was living separately from the appellant, and after their son’s death stopped
visiting the Appellant’s house and also did not pay anything for her subsistence. An application
under S125 of the CrPC was filed in the February of 1992. Before this, the Appellant had sent R2
a notice demanding maintenance and R2 denied his liability of paying maintenance. In the May
of 1992, a written statement was filed stating that R2 had divorced his wife long back by way of
Triple Talaq. It was further stated that marital ties between them had been cut off since the
divorce was over by the utterance of Talaq. Also stated was the fact that R2 had paid Mehr and
the Iddat period was over, thereby making the claim unacceptable.
Issues:

1. Does a Muslim woman have a right to claim maintenance under Section 125 of the Code
of Criminal Procedure, which lays down an order for the maintenance of wives, children
and parents, or can she claim maintenance only under the Muslim Woman (Protection of
Rights on Divorce) Act, 1986?
2. Is a Muslim woman entitled for maintenance throughout her life, even after completion of
the Iddat period and the payment of Mehr? Should maintenance and a “reasonable
provision” be provided till her death/remarriage regardless of the end of the Iddat period?
Procedural History:

The Magistrate first held that there was no material to substantiate the plea of divorce, and
granted maintenance to the Appellant. This order was challenged by Respondent 2 before an
Additional Sessions Judge. The Judge accepted the appeal and held that Section 125 of the CrPC
is not applicable since she is a married Muslim woman. Order of the Magistrate was set aside
and the High Court dismissed the writ petition. In the present appeal, the Appellant questions
correctness of the order passed by a learned Single Judge of the Allahabad High Court,
dismissing her revision petition.
Applicable Statutes:

1. Section 125 of the Code of Criminal Procedure – “Order for maintenance of wives,
children and parents.”
2. Section 3(1) of the Muslim Woman (Protection of Rights in Divorce) Act, 1986 –
“Notwithstanding anything contained in any other law for the time being in force, a
divorced woman shall be entitled to a reasonable and fair provision and maintenance to
be made and paid to her within the Iddat period by her former husband.”
Analysis:

170 | P a g e
The case at hand deals with, on a basic level, when a specific law needs to be used and when a
general law needs to be applied. The dispute here is between Section 125 of the CrPC which lays
down and order for the maintenance of wives, and Section 3 of the Muslim Woman Act. The
Counsel for the Appellant claimed that a mere written statement about a divorce some 30 years
ago does not tantamount to there actually being a divorce. The High Court upheld the view of the
Court in the Shamim Ara case. Where the Judge has stated that the term “pronounce” can also
mean to utter, or to declare. This therefore clears the sub-issue of the existence of a divorce. The
Court also comes to a conclusion that the issues of the Iddat period being completed and the
Mehr being payed are of no relevance to the case at hand. For this, the Court makes use of the
case of Danial Latifi v. The Union of India. In this case, the Court held that, “a divorced woman
is entitled to a reasonable and fair provision for maintenance.” The Court held that a Muslim
husband is liable to make a reasonable and fair provision for the future of the divorced wife
which includes maintenance as well. The said provision extending beyond the Iddat period must
be made within the period by the husband. The Court came to a conclusion, therefore, that the
liability of a husband to his divorced wife arising under Section 3(1)(a) of the Muslim Woman
Act is not confined to just the Iddat period.
Critical Analysis:

I feel that the lower court had given the correct judgement. The Additional Sessions judge was
erroneous in reversing the judgement of the Magistrate. It is very clear that a mere written
statement being given to the wife about a divorce done by the utterance of Talaq thrice, thirty
years ago, is not valid. In my opinion, the Supreme Court was right in supporting the lower
court and using the Shamim Ara case.

b. Spousal Maintenance of non-Muslims/Domestic Violence Act/CrPC


i. Chand Dhawan v Jawaharlal Dhawan 1993 SCC (3) 406

Summary 1:

Statement of Facts:
The appellant wife and Respondent husband got married on September 19, 1972 in Amritsar,
State of Punjab. They subsequently had 3 children out of their wedlock (2 males and 1 female).
In 1985, they filed a petition for divorce on grounds of mutual consent under section 13B of
HMA, 1955 in the district court of Amritsar. In accordance to the requirements of the Section,
they claimed the grant for the decree of divorce as they had been living separately for over 1 year
and that there was no chance of reconciling.
The wife, however objected to the plea for divorce on the grounds that she had NOT
consented to filing of the petition and that her signatures were duped and taken on a false pretext.
171 | P a g e
She claimed that the couple had reconciled and that she was put back in her matrimonial home
after they had reached an agreement. The petitions were dismissed in 1987 with the parties left to
bear their own costs.

Meanwhile, the husband grew a business in Ghaziabad, UP just 3 months after the
dismissal of the petition. The husband now went to the District Court in Ghaziabad and filed a
regular petition for divorce under Section 13, HMA, 1955; making a claim of adultery against
the wife. Subsequently the wife refused all such charges and prayed for a grant of maintenance
pendent elite. The court fixed the amount at 1,000/month to which the husband objected. The
wife then moved to the Court of additional District Judge in Amritsar with a plea for
PERMANENT ALIMONY + LITIGATION EXPENSES under Section 24 and 25, HMA. The
court accordingly ordered the payment of 6,000 for litigation expenses and 2,000 as maintenance
pendent elite. The husband later challenged this order in the High Court of Punjab and Haryana
at Chandigarh and the wife appealed to the same court seeking a revision in enhancements of the
sums. Both petitions were disposed.

Issues before the Court:


- Whether a claim for alimony filed under Section 25 of HMA is maintainable when the
relationship between the spouses has NOT been terminated?( The Case was held to be good law
on this point)

- Whether the denial of relief under the Act, when making a decree in the sense appealable under
Section 28, could be a decree passed within the meaning of Section 25 entitling the respective
spouses to claim permanent alimony thereunder. (Question of jurisdiction)

Applicable Statutes:
Sections- 24, 25, 28, 13B of Hindu Marriage Act, hereinafter, HMA, 1955.

Rule: (Refer to these sections in the bare act)


Section 13- Divorce
Section 13B- Divorce by mutual consent

Section 24 & 25- Maintenance pendente lite and expenses of proceedings & Permanent alimony
and maintenance.
Section 28- Appeals from decrees and orders.

Analysis:

172 | P a g e
 Under Sec- 25, HMA; the husband’s suit would NOT be maintainable as the Court of
Amritsar had NOT passed any decree of the variables Restitution of Conjugal Rights
(Sec-9,HMA), Judicial Separation, Nullity of marriage or Divorce.
The orders granting Litigation expenses & Maintenance Pendente lite has also been
quashed.
Declaration by court that “supposed marriage” is null & void. Also called Annulment.

When husband or wife without reasonable excuse withdraws from the society of the other. The
aggrieved party may petition in court. If the court is satisfied, then the couple required to live
together again.
Legal separation (alimony, children)

 Language used in statute can be construed widely as long as remedy with true intention
of act brought out. In other words, the court may interpret the words of the statute in a
varied manner as long as justice is sought.
If language ambiguous, the court is permitted to fill the gaps. If wife’s claim over money
was justified over fact & law, procedure should not come in the way.
However, In this case, the logic used by the Trial court was that the Amritsar court had
jurisdiction to grant relief when asked for the 2nd time (Permanent maintenance +
litigation expenses) as once upon a time a petition for dissolution of marriage by mutual
consent was there, even though such petition was dismissed. This argument was
dismissed by the High Court as there was clearly NO dissolution of marriage as the
petition for divorce had been set aside and mere dismissal does not amount to dissolution
and thereby takes away jurisdiction from the Amritsar Court in passing such an order the
2nd time.

 When distinctive claims are covered distinctly under 2 statutes, it is difficult to sustain a
plea when the claim (if) valid by choosing one forum. This is NOT a technical
irregularity, it is a problem of jurisdiction.
Matrimonial court is NOT meant to pronounce a claim of maintenance without exercising
of passing a decree to disrupt marriage between parties. Passing of a decree here refers to
: Restitution of conjugal rights, judicial separation or nullity of the marriage (matrimonial
reliefs).
Rejection or dismissal of a petition does NOT mean disruption of marriage as the legal
character or status of either parties or the marriage has not been affected.

Conclusion:
The claim of a Hindu wife to permanent alimony or maintenance is based on the supposition that
either her marital status has been strained or affected by passing a decree for restitution of

173 | P a g e
conjugal rights or judicial separation in favor or against her. or her marriage stands dissolved by
a decree of nullity or divorce, with or without her consent.

Opinion:
• The Preamble of HMA- To amend & codify law relating to Hindu marriages, but also
lays down rules with respect to divorce, judicial separation, etc. When statute expressly
codifies laws, court cannot go outside law.

Prior to 1955, Law- uncodified Hindu law/case law, etc.- women could claim
maintenance. But after introduction of the Hindu Adoption & Maintenance Act and
Hindu Marriage Act these procedures have changed.
BOTH ARE CODIFIED FOR HINDUS, BUT SUBJECT ON MAINTENANCE
DIFFER.

 I absolutely agree with the judgment of the SC in this case. Such decisions set precedent
for future cases wherein there is a misuse of the matrimonial rights given to women in
terms of their right to claim maintenance. Also, disruption or dissolution of marriage is
necessary in order to claim maintenance. Even if Divorce has not been granted and the
proceedings are continuing in court or the court has passed a decree for the exercise of
any one of the matrimonial reliefs, this is a valid ground for the wife to claim
maintenance.
 It is important to note that statutes such as HMA and more closely the Hindu Adoption
and maintenance Act aim to restore the position of women and provide them with relief
when their marital status has been strained, however in doing so the court must check as
to whether such allegations are true or whether such provisions of the law are only being
used wrongly for their own advantage.
 In my personal opinion, the larger question that this case brings out is whether the grant
of maintenance to the wife in all cases can be misused by them? Should the court
intervene?
Although this case just specifically speaks about how maintenance can be granted once
some matrimonial relief has been ordered by the court and that just mere dismissal of a
divorce petition does not in any way disturb or change the legal character of the marriage,
the larger question is as to whether our marriage laws lean towards women and whether
the woman should be allowed to make false allegations against the man in order to claim
maintenance at any given point of time.
Looking at the burden on the husband to provide maintenance to his wife even in cases
where the wife is well educated and capable enough to earn for her living, a bench of
S.A. Morey J gave a landmark judgment in favour of husband to curb the misuse of the

174 | P a g e
provision of maintenance, and held that a wife who is well qualified and is capable to
earn cannot sit idle and claim maintenance from her husband.

 Watch these 2 debates regarding the same:


https://www.youtube.com/watch?v=pcEB9Xi_Ad8
https://www.youtube.com/watch?v=v_1zpJKSows

They are not exactly related to the case, but speak about the importance of maintenance
and the fallacies prevalent within the marriage laws in India.

Summary 2:

Chand Dhawan v. Jawaharlal Dhawan

In the Supreme Court of India

(1993) 3 SCC 406

Question Presented:

Whether the payment of alimony is admissible without the relationship between the spouses
being terminated?

Background and Procedural History:

Wife (appellant) was married to husband (respondent) in 1972 at Amritsar. Husband shifted to
Ghaziabad and established a business there. Filed regular petition for divorce in District Court,
Ghaziabad alleging adultery against wife. Wife refuted charges of adultery. Matter stayed.

Subsequently, the wife moved the court of Additional District Judge, Amritsar on 22-3-1990,
under Section 25 of the Hindu Marriage Act for the grant of permanent alimony on the plea that
she was facing starvation. Simultaneously she moved the court under Section 24 of the Hindu
Marriage Act for maintenance pendente lite and litigation expenses.Court allowed the petition
under Section 24 of the Act granting her a sum of Rs. 6000/- as litigation expenses and Rs.
2000/- per month as maintenance pendente lite, from the date of application.

The husband challenged the said order of grant in revision before the High Court of Punjab and
Haryana at Chandigarh. The wife too approached the High Court in revision seeking
enhancement of sums under both counts. Both the revision petitions were disposed of by the
common judgment under appeal sustaining the objection of the husband that an application under

175 | P a g e
Section 25 of the Act was, in the facts and circumstances, not maintainable, the Matrimonial
Court at Amritsar, in the earlier litigation, having not passed any decree of the variables known
as Restitution of Conjugal Rights, Judicial Separation, Nullity of Marriage, or Divorce, so as to
quash proceedings under Section 25 and sequally quashing the order under Section 24 of the Act
granting litigation expenses and maintenance pendente lite.

Rule:

Section 25 of Hindu Marriage Act States that:

25. PERMANENT ALIMONY AND MAINTENANCE (1) Any court exercising jurisdiction
under this Act may, at the time of passing any decree or at any time subsequent thereto….

Analysis:

Therefore, the issue presented before the Court was whether the words "at the time of passing
any decree or any time subsequent thereto" meant a decree passed in granting substantive relief
or also included situations where main petition is dismissed or withdrawn?

The Hindu Adoptions and Maintenance Act, 1956 entitles a Hindu wife to claim maintenance
from her husband during her life-time. Sub-section (2) of Section 18 grants her the right to live
separately, without forfeiting her claim to maintenance.So while sustaining her marriage and
preserving her marital status, the wife is entitled to claim maintenance from her husband.

On the other hand, under the Hindu Marriage Act, in contrast, her claim for maintenance
pendente lite is durated on the pendency of a litigation of the kind envisaged under Sections 9 to
14 of the Hindu Marriage Act, and her claim to permanent maintenance or alimony is based on
the supposition that either her marital status has been strained or affected by passing a decree for
restitution of conjugal rights or judicial separation in favour or against her, or her marriage
stands dissolved by a decree of nullity or divorce, with or without her consent. Thus when her
marital status is to be affected or disrupted the court does so by passing a decree for or against
her.

The Court is not at liberty to grant relief of maintenance simplicitor obtainable under one Act in
proceedings under the other. As is evident, both the statutes are codified as such and are clear on
their subjects and by liberality of interpretation interchangeability cannot be permitted so as to
destroy the distinction on the subject of maintenance.

The judgment of the High Court under appeal could be no other than the one that it was in the
present state of law and the facts and circumstances. It is still open to the wife to stake her claim
to maintenance in other fora.

Conclusion
176 | P a g e
The Court ruled that the wife can not claim alimony u/s 25 of the Hindu Marriage Act as the act
provides relief only when the status of the relationship had changed. Instead, the wife was free to
hold claim under the Hindu Adoptions and Maintenance Act. Had the legislature intended to
provide relief under Hindu Marriage Act, a same provision wouldn’t have been made under other
Acts. Therefore, keeping the legislature’s intent in mind, relief under Section 25 was denied.

ii. Bhagwan Dutt v. Kamla Devi, AIR 1975 SC 83

Overview:

The case revolves around maintenance of a wife when she has a means to support her or she has
an income to sustain herself. The question of sustainability standards has been an integral part to
the entire discussion. Also it encompasses the discussion on whether the new Criminal
Procedure Code (CrPC) of 1973 made fundamental change to the old Criminal Procedure
Code (CrPC) of 1898 or just provided clarifications.

Facts:

The facts of this case are simple. Respondent, Kamla Devi got married to the appellant, Bhagwan
Dutt in the year 1957 in accordance with the Hindu rites. Subsequently a daughter was born out
of the wedlock. In the year 1966, the respondent filed for a petition for judicial separation on
grounds of desertion and cruelty. While the petition was pending, she filed an application for
maintenance for herself and her daughter on grounds that the appellant had refused to maintain
them.

Important facts: The fact that is in consideration throughout is the income of the parties at the
time of the filing of the application. The respondent was earning ₹600 and the appellant was
earning ₹800. However, when the matter reached the Sessions court, their income had risen by
₹150 approximately.

Procedural History: At first instance, the Magistrate had directed ₹250 for maintenance
wherein ₹175 was given to the wife and ₹75 for the child. The Magistrate had not looked into the
income of the wife and this was then appealed to the Sessions Court. The income of the wife was
looked into and accordingly the maintenance was granted, wherein the maintenance for the wife
was quashed but the maintenance of the child was raised to ₹150. This was then appealed to the
High court where they argued that the income of the wife was not an important consideration

177 | P a g e
while deciding the maintenance amount. They ordered for maintenance for the wife and upheld
the maintenance for the child at ₹150. Therefore, the husband appealed to the Supreme Court.

Arguments:

The arguments put forth in the Supreme Court are mainly around the point that whether it is
important to take into consideration the wife’s income and for purpose was sought when Section
488 of CrPC was drafted and the need for addition of a new phrase in Section 125 of the new
CrPC.

Appellants argued that the language of the Section 488(1) if construed with Section 498(1),
makes it necessary to consider the wife’s income in determining the quantum for the
maintenance. This was contended by the respondents with the argument that if the legislators did
not attach the condition “unable to maintain herself” with the wife in the statute, that means her
income is not a criterion in determining the maintenance amount.

Issues:

- Can the income of the wife be taken into account in determining the amount of
maintenance payable to her under Section 488 of the CrPC, 1898?
- Whether Section 488, CrPC, 1898 is consistent with Section 23 of the Hindu Marriage
and Maintenance Act (HAMA), 1956?

Rule:

Section 488 of CrPC, 1898 –

“(1) if any person having sufficient means neglects or refuses to maintain his wife or his
legitimate or illegitimate child unable to maintain itself, the District Magistrate, a Presidency
Magistrate, a Sub-Divisional Magistrate or a Magistrate of the first class may, upon proof of
such neglect or refusal, order such person to make a monthly allowance for the maintenance of
his wife or such child at such monthly rate, not exceeding five hundred rupees in the whole as
such Magistrate thinks fit, and to pay the same to such person as the Magistrate from time to
time directs.”

Section 125 of CrPC, 1973 –


“1) If any person having sufficient means neglects or refuses to maintain-

a) his wife, unable to maintain herself, or

178 | P a g e
b) his legitimate or illegitimate minor child, whether married or not, unable to maintain
itself, or
c) his legitimate or illegitimate child (not being a married daughter) who has attained
majority, where such child is, by reason of any physical or mental abnormality or injury
unable to maintain itself, or
d) his father or mother, unable to maintain himself or herself….”

Section 498 of CrPC, 1898 –


“on proof of a change in the circumstances of any person receiving under s. 488 a monthly
allowance, the Magistrate, may make such alteration in the allowance as he thinks fit.”

Section 23 of HAMA, 1956 –


Relating to the fixation of allowance, provide fro the enforcement of the rights of Hindu wives
and dependents under their personal law.

Analysis:

Judge R.S.Sarkaria was in the view that the mere fact that the legislators did not contemplate
about the the means of support for wife does not conclude in not considering the income while
determining the maintenance amount. With the new CrPc coming in, Section 125 does attach a
necessary criterion for the wife’s income or her means of support in determining the
maintenance.

o The question asked mainly revolved around the notion whether the addition in the section
meant a fundamental change or just a clarification of how Section 488 was construed
earlier? The question was answered in the form of an explanation that basically said that
Section 488 did not ever conform an absolute right of the wife to maintenance but was
just a preventive measure.
o Looking at the judgments of Major Joginder Singh v. Bibi Raj Mohinder Kaur, where
the means of support of the wife were not looked into, Judge Sarkaria said that one
assumption had led to another false deduction. The Sections providing right to
maintenance to the wife were not a punitive measure but a social construct.
o Based on the ideology that the woman should be subjected to similar standard of living as
she did before her separation, this section prevents a woman or a child from going in a
situation of destitution or vagrancy.
o These sections just provide the right to file a petition for maintenance but not an absolute
right that maintenance will be granted. The word ‘may’ in Section 488 clearly indicates
the discretionary powers of the Magistrate in deciding the order of the maintenance.
o He further goes on to say that the language of Section 498(1) makes it amply clear that
the income of the wife is an important consideration while determination of maintenance.
This can be construed from the mention of the words ‘changes in circumstances’, this

179 | P a g e
would mean any change in financial circumstances would also have to be considered
along with other factors.

There was also an argument raised that there was no inconsistency between Section 488 of the
CrPC, 1898 and Section 23 of HAMA, 1956. Both the section can stand together. The former is
a section that is applicable to all the religions irrespective of the personal laws.

Held:

The court held that the Section 488 does not confer an absolute right of the women but it is just
for social purpose and to morally obligate the man to not neglect or desert the wife or the child in
situation where they are not able to maintain themselves.

The appeal was allowed and the magistrate was ordered to determine the amount of
maintenance while considering the financial independence of the wife.

Critical Analysis:

Critiquing the judgment, we find that the Supreme Court analysis was on the right understanding
of the legislation. Right, here, meaning morally and socially right. I firmly believe that the
section conferring the women the right to get maintenance from the husband is with the concept
of preventing her from being in a situation of destitution and desertion.

However, when one has an own individual income to support them enough as to lead a
life on similar standards, then ordering the husband to give maintenance will be a wrong call.
Obligating a husband with the responsibility of paying her wife even though she could maintain
herself would mean that one is taking away the standards of living of the husband in that regard.

This argument is based on the fact that if one deducts certain money from the husband’s
income, when he is the sole supporter of his sustainability, then that would mean that the wife
will be much better off after the separation.

This judgment, however overlooked an important point of observation. There was lesser
disparity in the income of both the parties and this would mean that both are independently
capable of maintaining themselves. The question then arises is that when the custody of the child
is granted to the wife, the husband should pay a minimal amount in the maintenance of that
child. This is because the wife’s income will also help in the maintenance of the child and both
will be obligated to fulfill their duties as parents. In this case, the Sessions court raised the
maintenance allowance of the child stating that it was too less to maintain her, but my argument
is that raising it would mean the court is freeing the wife from the obligations of properly
maintaining her child.

180 | P a g e
Apart from this one aspect, which comes from personal opinion, the court delivered a well-
thought judgment. It clarified the following things along with laying out proper reasons:

o Section 488 of CrPC, 1898 did not confer an absolute right to the wife to get
maintenance but was on the discretion of the Magistrate.
o Section 489 of CrPC, 1898 provided a clause for change in circumstances which
encompass financial circumstances as well.
o Section 125 of CrPC, 1973 did not bring fundamental changes but clarified the
vagueness in Section 488.
o Section 488 was not inconsistent with Section 23 of the HAMA as they both could stand
together and the jurisdiction of both sections varied.

It shed a light on the reasoning and vagueness of Right to Maintenance.

iii. Chaturbhuj v. Sita Bai, (2008) 2 SCC 316

Summary 1:

Facts: Appellant (Chatur Bhuj) was sued by his wife for maintenance under Section 125 (1) (a)
of Cr.PC. The trial court didn’t award any maintenance. She appealed in the High court which
set aside the trial court’s judgment and awarded her maintenance. The present appeal is filed
against the order of the High court by the appellant (Chatur Bhuj).

Issue: There are two issues in the present case. The first issue is whether a wife should be
absolutely poor or a destitute to sue her husband for maintenance under Section 125 (1) (a) of
Cr.PC. The second issue is whether earning of money by a wife can be a ground to stop her from
suing her husband under the said act.
Rule: Section 125 (1) (a) of Cr.PC
Order for maintenance of wives, children and parents.
(1) If any person leaving sufficient means neglects or refuses to maintain-
 (a) his wife, unable to maintain herself
Analysis: In the present case, the wife wanted to secure maintenance from her husband. The
husband contended on the ground that she was receiving rent on the land, which was sufficient
for her to maintain herself. However, the High Court and the Supreme Court weren’t impressed
by this contention. Under Section 125 (1) (a) it is not mentioned that a wife may only sue if she
is absolutely poor or destitute. So, if a wife of a man, wants to sue her husband for maintenance
the court is only going to consider whether her earnings are sufficient enough to maintain herself.

181 | P a g e
The court also held that earnings of a wife cannot be a grant to stop her from claiming
maintenance.

Conclusion: The Supreme Court held that the wife had a right for maintenance and that the mere
ground of her earning is not a reason to stop her from claiming maintenance.

Opinion: The court in the present case, didn’t give define the meaning of sufficient earning. This
makes it subjective. The reason to claim maintenance wasn’t clearly specified by the court.
Although the court cited the case of Bhagwan v. Kamla Desi, wherein a test is used to define the
sufficiency of earnings, however, this doesn’t give a clear understanding on how this has to be
treated in the present case. Also, the court had not addressed the contention raised by the
appellant, about him being married to another women and that giving maintenance in this case
would be impossible. This issue is critical to this case, as a husband doesn’t have any
responsibility to give maintenance to his ex-wife since they are already divorced.

Summary 2:
Chaturbhuj vs Sita Bai, (2008) 2 SCC 316
Facts:
The respondent (wife) had filed an application under Section 125 of Cr.P.C. claiming
maintenance from the appellant (husband). The appellant and the respondent had entered into a
marital knot about four decades ago and they were living separately since two decades. In the
application which was put forth it was claimed that she was unemployed and unable to maintain
herself. Appellant had retired from the post of Assistant Director of Agriculture and was getting
about Rs.8,000/- as pension and a similar amount as house rent. Besides this, he was lending
money to people on interest. His financial condition was stable. In couse of time, the appellant
(husband) claimed Rs.10,000/- as maintenance. The stand of the appellant was that the applicant
(wife) was living in the house constructed by the present appellant who had purchased 7 bighas
of land in Ratlam in the name of the applicant. She let out the house on rent and since 1979 was
residing with one of their sons.

The applicant sold the agricultural land on 13.3.2003. The sale proceeds were still with the
applicant. The appellant was getting pension of about Rs.5,700/- p.m. and was not getting any
house rent regularly. He was getting 2-3 thousand rupees per month. The plea that the appellant
had married another lady was denied. It was further submitted that the applicant at the relevant
point of time was staying in the house of the appellant and electricity and water dues were being
paid by him. The applicant can maintain herself from the money received from the sale of
agricultural land and rent.

Issue:
What falls under the definition of “unable to maintain herself” in section 125 Cr.P.C

182 | P a g e
Rule:
Section 125 Cr.P.C is applicable in the present case.

Analysis:
The main reason for providing maintenance is not to punish an individual for the past neglect but
in fact it is to prevent the wife from being homeless or in a state of beggary. The purpose is to
provide financial support to wives who cannot support themselves. The phrase "unable to
maintain herself" in the instant case would mean that means available to the deserted wife while
she was living with her husband and would not take within itself the efforts made by the wife
after desertion to survive somehow. Section 125 Cr.P.C. is a measure enacted to protect women
and children It is meant to achieve a social purpose. The object is to prevent vagrancy and
destitution. It provides a speedy remedy for the supply of food, clothing and shelter to the
deserted wife.

The burden of proof was on the wife, to establish that she could not maintain herself. In the
instant case there is no dispute that the husband has the requisite means. But there is an
inseparable condition which has also to be satisfied that the wife was unable to maintain herself.
These two conditions are in addition to the requirement that the husband must have neglected or
refused to maintain his wife. It has to be established that the wife was unable to maintain herself.
The appellant put forth evidence to prove that the respondent-wife was earning some income.
This is clearly not sufficient to rule out application of Section 125 Cr.P.C. It has to be
established that with the amount she earned the respondent-wife was able to maintain herself.

Whether the deserted wife was unable to maintain herself, has to be decided on the basis of the
material placed on record. In case the personal income of the wife is insufficient she can claim
maintenance under Section 125 Cr.P.C. The test for bestowing maintenance is based on the
factor whether the wife is in a position to maintain herself in the similar way as she was used to
before the desertion. The wife should be in a position to maintain the standard of living which is
neither luxurious nor penurious but what is consistent with status of a family. The expression
"unable to maintain herself" does not mean that the wife must be absolutely destitute before she
can apply for maintenance under Section 125 Cr.P.C.

Furthermore the 132nd law commission report, criticized the maintenance ceiling that was set.
According to section 125 of Cr.P.C , “a Magistrate of the first class may, upon proof of such
neglect or refusal, order such person to make a monthly allowance for the maintenance of his
wife or such child, father or mother, at such monthly rate not exceeding five hundred rupees in
the whole, as such Magistrate thinks fit, and to pay the same to such person as the Magistrate
may from time to time direct: Provided that the Magistrate may order the father of a minor
female child referred to in clause (b) to make such allowance, until she attains her majority, if the
Magistrate is satisfied that the husband of such minor female child, if married, is not possessed
of sufficient means.” The report stated that at the time the ceiling was set, i.e. in 1955, five

183 | P a g e
hundred rupees was a sufficient amount. Over the last few decades there has been an increase in
inflation due to which five hundred rupees is seen as an insufficient amount for a wife to
maintain herself. The law commission report suggested that the ceiling amount of five hundred
rupees should be waived off. I agree with the law commission report, I strongly feel that the
maintenance given to wives is less and should be more.

CONCLUSION
In the given case the trial Court, the Revisional Court and the High Court have analysed the
evidence and held that the respondent wife was unable to maintain herself. The conclusion is
essentially factual and not perverse. Hence there is no scope for interference in this appeal which
is dismissed.

iv. Vinny Parmar v. Paramvir Parmar (2011) 13 SCC 112.

Vinny Parmar V Paramvir Parmar: Wife use to work as an air hostess, left her job after marriage.
mutual consent divorce. wife asked for maintenance under 25 of HMA. court said 20000 per
month or 20,00,000 lump sum. wife appealed for increase as its not a reasonable amount. Court
tries to find out whats a reasonable amount. These creteria’s:

As per Section 25, while considering the claim for permanent alimony and maintenance of either
spouse, the respondent's own income and other property, and the income and other property of
the applicant are all relevant material in addition to the conduct of the parties and other
circumstances of the case. No fixed formula can be laid for fixing the amount of maintenance. It
has to be in the nature of things which depend on various facts and circumstances of each case.
The court has to consider the status of the parties, their respective needs, the capacity of the
husband to pay, having regard to reasonable expenses for his own maintenance and others whom
he is obliged to maintain under the law and statute. The courts also have to take note of the fact
that the amount of maintenance fixed for the wife should be such as she can live in reasonable
comfort considering her status and mode of life she was used to live when she lived with her
husband. At the same time, the amount so fixed cannot be excessive or 8 affect the living
condition of the other party. this idea applied to both 25 HMA and Cr C P …granted 40,000 rs.

v. Geeta Satish Gokarna v. Satish Shankarrao Gokarna, AIR 2004


Bom 345

Geeta Satish Gokarna V Satish Shankarrao Gokarna: parties dissolved their marriage by mutual
consent. The clause in the agreement said that she would ask for maintenance or file a suit for the
same. Its validity is questioned because the wife now wants maintenance, as earlier she use to
work. She left her job because of husband, and now isn’t getting one.

Issue: whether in a case where consent terms were filed which provided that the wife would not
claim maintenance or alimony in future whether it is still open to the wife to claim maintenance?

184 | P a g e
A bare perusal of Section 25(2) indicate that the section would only apply if there is a change of
circumstance of either party, at any time after an order has been made under Section 25(1) then
at the instance of either party the Court could vary, modify or rescind any such order under
Section 25(1) the order for maintenance. Therefore, it is clear that if only an order was passed
under Section 25(1) could the appellant apply under Section 25(2) for varying, modifying or
rescinding such an order.

A perusal of Section 25(1) will show that the power to grant alimony or maintenance is not only
at the time of passing of a decree. It is in the alternative i.e. at any time subsequent thereto. This
expression "or at any time subsequent thereto" can only mean after the decree for divorce has
been passed, provided that no maintenance was provided for in the order granting decree for
divorce. Having held that there is a power under Section 25(1) the next question is whether
considering the consent terms the appellant is entitled for maintenance

vi. Savitaben v. State of Gujarat AIR 2005 SC 1809

Facts; Appellant claims that she was married to respondent some time in 1994 according to the
customary rites and rituals of their caste. Though initially, the respondent treated her nicely,
thereafter he started ill-treating her and she was subjected to mental and physical torture. On
enquiry about the reason for such a sudden change in his behaviour, the appellant came to know
that respondent had developed illicit relationship with a lady named Veenaben. During the period
the appellant stayed with the respondent, she became pregnant and subsequently, a child was
born. As respondent neglected the appellant and the child born, an application in terms of
Section 125 of the Code was filed claiming maintenance. Respondent opposed the application by
taking the stand that the appellant was not his legally married wife and the child was not his son.
He also denied having developed illicit relationship with Veenaben. He claimed that actually she
was married to him more than 22 years back and two children were born. Their son Hament had
died in the road accident in July 1990. In the Claim Petition name of Veenaben was mentioned as
the legal heir and in the Voters List, Ration Card and Provident Fund records, Veenaben was
shown as the wife of respondent No.2. On 23.6.1998 learned JMFC allowed the Claim Petition
and granted maintenance.

Issue; Is the Appellant entitled to maintenance?

Held;

The appellant argued that strict proof about a valid marriage is not the sine qua non for getting
maintenance under Section 125 of the Code. The documents produced by respondent to
substantiate the plea of earlier marriage with Veenaben should not have been given primacy over
the clinching evidence adduced by the appellant to show that she was unaware of the alleged
marriage. Respondent is guilty of fraud and misrepresentation, the equity should not weigh in his

185 | P a g e
favour. In response, learned counsel for respondent submitted that law is fairly well settled
regarding the definition of the expression 'wife' and there is no scope for giving an extended
meaning to include a woman who is not legally married.

There may be substance in the plea of learned counsel for the appellant that law operates harshly
against the woman who unwittingly gets into relationship with a married man and Section 125 of
the Code does not give protection to such woman. This may be an inadequacy in law, which only
the legislature can undo. But as the position in law stands presently there is no escape from the
conclusion that the expression 'wife' as per Section 125 of the Code refers to only legally married
wife.

The provision is enacted for social justice and specially to protect women and children as also
old and infirm poor parents and falls within the constitutional sweep of Article 15(3) reinforced
by Article 39 of the Constitution of India, 1950 (in short the 'Constitution'). The provision gives
effect to the natural and fundamental duty of a man to maintain his wife, children and parents so
long as they are unable to maintain themselves. But the personal law of the parties is relevant for
deciding the validity of the marriage and therefore cannot be altogether excluded from
consideration.

vii. Hari Lal v. Balvantia AIR 1998 All 211


viii. Indra Sarma v. V.K.V.Sarma 2013 (14) SCALE 448

On 26th November 2013 a two-judge Bench of the Supreme Court constituting of K.S.
Radhakrishnan and Pinaki Chandra Ghose, JJ in Indra Sarma v. V.K.V. Sarma58 held that when
the woman is aware of the fact that the man with whom she is having living-in-relationship and
who already has a legally-wedded wife and two children, is not entitled to various reliefs
available to a legally wedded wife and also to those who enter into ‗a relationship in the nature
of marriage‘ as per provisions of PWDVA, 2005. But in this case, the Supreme Court felt that
denial of any protection would amount to a great injustice to victims of illegal relationship who
are poor, illiterate and also to their children who are born out of such relationship and has no
source of income of her own.59 Therefore, the S.C. remarked that there is a burning need to
expand the connotation of Sec. 2 (f) which defines ‗domestic relationship‘ in PWDVA, 2005
with a view to include there in victims of illegal relationship who are poor, illiterate along with
their children who are born out of such relationship and who do not have any source of income.
If the above suggestion made by the Apex Court, purely out of humanitarian consideration, is
converted into legislation, it would prove to be an effective remedy to a societal wrong caused by
such illegal relationships. During the course of its judgment, the Supreme Court has given the
following guidelines based on which the Parliament may pass a new legislation: “1) Duration of
relationship - Section 2(f) of the DV Act has used the expression “at any point of time”, which
means a reasonable period of time to maintain and continue a relationship which may vary from
case to case, depending upon the factual situation. (2) Shared household - The expression has
been defined under Section 2(s) of the DV Act and, hence, needs no further elaboration. (3)

186 | P a g e
Pooling of Resources and Financial Arrangements supporting each other, or any one of them,
financially, sharing bank accounts, acquiring immovable properties in joint names or in the name
of the woman, long term investments in business, shares in separate and joint names, so as to
have a long standing relationship, may be a guiding factor. (4) Domestic Arrangements -
Entrusting the responsibility, especially on the woman to run the home, do household activities
like cleaning, cooking, maintaining or up keeping the house, etc. is an indication of a relationship
in the nature of marriage. (5) Sexual Relationship - Marriage like relationship refers to sexual
relationship, not just for pleasure, but for emotional and intimate relationship, for procreation of
children, so as to give emotional support, companionship and also marital affection, caring etc.
(6)Having children is a strong indication of a relationship in the nature of marriage. Parties,
therefore, intend to have a long standing relationship. Sharing the responsibility for bringing up
and supporting them is also a strong indication. (7) Socialization in Public - Holding out to the
public and socializing with friends, relations and others, as if they are husband and wife is a
strong circumstance to hold the relationship is in the nature of marriage. (8) Intention and
conduct of the parties - Common intention of parties as to what their relationship is and to
involve and as to their respective roles and responsibilities, primarily determines the nature of
that relationship.”60 The Malimath Committee (on Reforms of Criminal Justice System, 2003)
61 made several recommendations in Part IV, Chap.16 under the Head “Offences against
Women” (Pg.197) has observed; “that the definition of the word „wife‟ in Section 125 should be
amended so as to include a woman who was living with the man as his wife for a reasonably
long period, during the subsistence of the first marriage.

In Dwarika Prasad Satpathy v. Bidyut Prava Dixit and Anr. (AIR 1999 SC 3348) it was held that
the validity of the marriage for the purpose of summary proceedings under Section 125 of the
Code is to be determined on the basis of the evidence brought on record by the parties. If the
claimant in proceedings under Section 125 succeeds in showing that she and the respondent have
lived together as husband and wife, the Court has to presume that they are legally wedded
spouses, and in such a situation one who denies the marital status can rebut the presumption.
Once it is admitted that the marriage procedure was followed then it is not necessary to further
probe as to whether the said procedure was complete as per the Hindu rites, in the proceedings
under Section 125 of the Code. It is to be noted that when the respondent does not dispute the
paternity of the child and accepts the fact that marriage ceremony was performed though not
legally perfect, it would hardly lie in his mouth to contend in proceedings under Section 125 of
the Code that there was no valid marriage as essential rites were not performed at the time of said
marriage. The provision under Section 125 cannot be utilized for defeating the rights conferred
by the legislature on the destitute women, children or parents who are victims of social
environment. The provision is a measure of social justice and as noted above specially enacted to
protect women and children and falls within the constitutional sweep of Article 15(3) reinforced
by Article 39 of the Constitution. The brooding presence of the constitutional empathy for the
weaker sections like women and children must inform interpretation if it has to have social
relevance. In Smt. Yamunabai's case (supra), it was held that expression 'wife' used in Section

187 | P a g e
125 of the Code should be interpreted to mean only a legally wedded wife. The word 'wife' is not
defined in the Code except indicating in the Explanation to Section 125 its inclusive character so
as to cover a divorcee. A woman cannot be a divorcee unless there was a marriage in the eye of
law preceding that status. The expression must therefore be given the meaning in which it is
understood in law applicable to the parties. The marriage of a woman in accordance with the
Hindu rites with a man having a living spouse is a complete nullity in the eye of law and she is
therefore not entitled to the benefit of Section 125 of the Code or the Hindu Marriage Act, 1955.
Marriage with person having living spouse is null and void and not voidable. However, the
attempt to exclude altogether the personal law applicable to the parties from consideration is
improper. Section 125 of the Code has been enacted in the interest of a wife and one who intends
to take benefit under sub- section (1)(a) has to establish the necessary condition, namely, that she
is the wife of the person concerned.

It may be noted at this juncture that the legislature considered it necessary to include within the
scope of the provision an illegitimate child but it has not done so with respect to woman not
lawfully married. However, desirable it may be, as contended by learned counsel for the
appellant to take note of the plight of the unfortunate woman, the legislative intent being clearly
reflected in Section 125 of the Code, there is no scope for enlarging its scope by introducing any
artificial definition to include woman not lawfully married in the expression 'wife'.

In the instant case the evidence on record has been found sufficient by the Courts below by
recording findings of fact that earlier marriage of respondent was established.

In that view of the matter, the application so far as claim of maintenance of the wife is concerned
stands dismissed.

Considering the peculiar facts of the case, we feel that the amount of maintenance to the child
can be enhanced to Rs.850/- with effect from today.

Learned counsel for the respondent has submitted that as a humanitarian gesture, the respondent
agrees to pay a lump-sum amount to settle the dispute. In case the respondent pays a sum of
rupees two lakhs only within a period of four months to the appellant, the same shall be in full
and final settlement of the claim of respondent for maintenance.

It is to be noted that recently views of the courts however, have widened the definition of wife
and included live-in partners and second wife.

c. Maintenance of Parents and Children


i. Selva Saroja v. Sasinathan (1989) Cr LJ 2032
ii. Mahendra Kumar Gaikwad v. Gulabbhai 2001 Cr LJ 2111 (Bom)
iii. Vijaya Manohar Arbat v. Kashi Rao Rajaram AIR 1987 SC 1100,

188 | P a g e
Summary 1:

Vijaya Manohar Arbat v. Kashi Rao Rajaram AIR 1987 SC 1100

Facts:

Vijaya Manohar Arbat was the married daughter of Kashi Rao Rajaram through his first wife
who died in 1948. The daughter failed to maintain her father and the father claimed the
maintenance

Procedural History:

A preliminary objection raised to the effect, that an application under section 125(1)(d) Criminal
Procedure Code by a father to claim maintenance from his daughter was not maintainable was
overruled by the Trial Magistrate and upheld by the High Court in revision. Hence the daughter's
Dr. (Mrs.) Vijaya Manohar Arbat v. Kashi Rao Rajaram Sawai and Anr on 18 February, 1987.

Rule:

25(1) (d) of the Code of Criminal Procedure, 1973:

Sub-section (1) of section 125 Cr.P.C. provides as under:-

"If any person having sufficient means neglects or refuses to maintain-

(d) his father or mother, unable to maintain himself or herself, a Magistrate of the first class may,
upon proof of such neglect or refusal, order such person to make a monthly allowance for the
maintenance of his wife or such child, father or mother, at such monthly rate not exceeding five
hundred rupees in the whole, as such Magistrate thinks fit, and to pay the same to such person as
the Magistrate may from time to time direct:

Provided that the Magistrate may order the father of a minor female child referred to in clause
(b) to make such allowance, until she attains her majority, if the Magistrate is satisfied that the
husband of such minor female child, if married, is not possessed of sufficient means."

Analysis:

The purpose of Section 125 of the Indian Penal Code is to provide a remedy to the perils of old
age and to serve a social purpose. The court found it obvious that it was the morl obligation of a
son or a daughter to maintain her parents. It was not desirable that evn though the son or the
daughter had sufficient means his or her parents would starve. And the court also found that the
Indian society had cast a duty on the children to maintain and look after their parents when they
are old and infirm.

The court said that though the Cr.P.C use the terms “he” and its derivatives, the court said that
was it was indicative of both the genders.

189 | P a g e
At the outset, the appellant raised a preliminary objection to the maintainability of the
application on the ground that section 125(1)(d) Cr.P.C. does not entitle a father to claim
maintenance from his daughter. Section 8 of the Indian Penal Code lays down that the pronoun
'he' and its derivatives are used for any person whether male or female.

Before ordering the maintenance in favour of a father or mother against their married daughter,
the court should be satisfies that the daughter has sufficient means to provide for her own family.

A daughter after her marriage does not cease to be a daughter of the father or mother. If it is not
so, parents having no son but only daughters and unable to maintain themselves, would go
destitute, if the daughters even though they have sufficient means refuse to maintain their
parents.

Conclusion:

The judgment of the High Court is affirmed and this appeal is dismissed.

IX. Week 13: Adoption, Custody and Guardianship


a. Githa Hariharan v. Reserve Bank of India (1999) 2 SCC 228

Facts; Githa signed with the consent of her husband in an application to RBI as the guardian of
her minor son in order to obtain some Relief Bonds in the name of her minor son. RBI sent back
the application advising her to produce the application signed by the father and in the
alternative, a certificate of guardianship from a competent authority in favor of the mother. A
petition filed by the mother on the grounds of this letter sent by RBI as unconstitutional,
violative of Ar. 14 and 15 of the Constitution.

RBI contended that father is the natural guardian of a legitimate son and only ‘after’ the father,
the mother is the natural guardian.

Issue; Interpretation of the term ‘after the father’.

Held;

The word `guardian' in the definition section means and implies both the parents, the same
meaning ought to be attributed to the word appearing in section 6(a) and in that perspective
mother's right to act as the guardian does not stand obliterated during the lifetime of the father
and to read the same on the statute otherwise would tantamount to a violent departure from the
legislative intent. S. 6(a) itself recognises that both the father and the mother ought to be treated
as natural guardians and the expression `after' therefore shall have to be read and interpreted in a
manner so as not to defeat the true intent of the legislature. Be it noted further, that gender
equality is one of the basic principles of our Constitution and in the event the word `after’ is to
be read to mean a disqualification of a mother to act as a guardian during the lifetime of the
190 | P a g e
father, the same would definitely run counter to the basic requirement of the constitutional
mandate and would lead to a differentiation between male and female.

Normal rules of interpretation shall have to bow down to the requirement of the Constitution
since the Constitution is supreme and the statute shall have to be in accordance therewith and not
de hors the same. The father by reason of a dominant personality cannot be ascribed to have a
preferential right over the mother in the matter of guardianship since both fall within the same
category and in that view of the matter the word `after' shall have to be interpreted in terms of the
constitutional safe-guard and guarantee so as to give a proper and effective meaning to the words
used. In our opinion the word `after' shall have to be given a meaning which would sub-serve the
need of the situation viz., welfare of the minor and having due regard to the factum that law
courts endeavor to retain the legislation rather than declaring it to be a void, we do feel it
expedient to record that the word `after' does not necessarily mean after the death of the father,
on the contrary, it depicts an intent so as to ascribe the meaning thereto as `in the absence of `-
be it temporary or otherwise or total apathy of the father towards the child or even inability of the
father by reason of ailment or otherwise and it is only in the event of such a meaning being
ascribed to the word `after' as used in Section 6 then and in that event the same would be in
accordance with the intent of the legislation viz. welfare of the child. In that view of the matter
question of ascribing the literal meaning to the word `after' in the context does not and cannot
arise having due regard to the object of the statute, read with the constitutional guarantee of
gender equality and to give a full play to the legislative intent, since any other interpretation
would render the statute void and which situation in our view ought to be avoided.

b. Vinay Pathak And His Wife v. Unknown, 2010(1) BomCR 434

Facts; Petitioners (husband and wife), both Hindus and married, having their own daughter
sought appointment as guardians of a female child who born in 2004 and her biological parents,
four days after her birth, surrendered her to a nursing home. The petitioners were appointed
guardians of the child by the Court. The child lived with the Petitioners for over four years. A
petition has been filed seeking a declaration that the Petitioners are the adoptive parents of the
child with consequential rights, privileges and responsibilities under the law.

Issue; Which Act shall be applied the Juvenile Justice Act of 2000 or the HAMA of 1956?

Held;

The provisions of the Juvenile Justice Act came up for consideration before a Constitution bench
of the Supreme Court in Pratap Singh v. State of Jharkhand. The SC held that the Act was not
only beneficial legislation but that it was also remedial in character. The Constitution bench held
that the statute must be construed in a manner that would make it effective and operative on the

191 | P a g e
principle of ‘utres magis valet quam pereat’ - That the thing may rather have effect than be
destroyed.

A similar approach had been adopted by a Bench of three Learned Judges of the Supreme Court
in Umesh Chandra v. State of Rajasthan. HMA and the Juvenile Justice Act, 2000 must be
harmoniously construed.

The Hindu Adoptions and Maintenance Act, 1956 deals with conditions requisite for adoption by
Hindus. The Juvenile Justice Act of 2000 is a special enactment dealing with children in conflict
with law and children in need of care and protection.

While enacting the Juvenile Justice Act 2000 the legislature has taken care to ensure that its
provisions are secular in character. The focus of the legislation is on the condition of the child
taken in adoption. If the child is orphaned, abandoned or surrendered, that condition is what
triggers the beneficial provisions for adoption. The religious identity of the child or of the parents
who adopt is not a precondition to the applicability of the law.

Sub section (6) of Section 41 clearly says that the Court may allow a child to be given in
adoption to parents to adopt a child of the same sex irrespective of the number of living
biological sons or daughters. This provision is intended to facilitate the rehabilitation of
orphaned, abandoned or surrendered children. The condition must apply to all persons
irrespective of religious affiliation who seek to adopt children of that description.

Consequently, where the child which is sought to be adopted falls within the description of an
orphaned, abandoned or surrendered child within the meaning of sub section (2) of Section 41 or
a child in need of care and protection under clause (d) of Section 2, the provisions of the Juvenile
Justice Act 2000 must prevail. In such a case the embargo that is imposed on adopting a child of
the same sex by a Hindu under clauses (i) and (ii) of Section 11 of HAMA 1956 must give way
to the salutary provisions made by the Juvenile Justice Act. Where, however, the child is not of a
description falling under the purview of Chapter IV of the Juvenile Justice Act, 2000, a Hindu
desirous of adopting a child continues to be under the embargo imposed by clauses (i) and (ii) of
Section 11 of the Act of 1956.

The Supreme Court applied this principle in the context of a conflict between the Companies Act
1956 and the Recovery of Debts Due to Banks and Financial Institutions Act, in its decision in
Allahabad Bank v. Canara Bank. It was held that “While enacting the Juvenile Justice Act 2000
the legislature has taken care to ensure that its provisions are secular in character. The focus of
the legislation is on the condition of the child taken in adoption. If the child is orphaned,
abandoned or surrendered, that condition is what triggers the beneficial provisions for adoption.
The religious identity of the child or of the parents who adopt is not a precondition to the
applicability of the law.

192 | P a g e
Sub section (6) of Section 41 clearly says that the Court may allow a child to be given in
adoption to parents to adopt a child of the same sex irrespective of the number of living
biological sons or daughters. This provision is intended to facilitate the rehabilitation of
orphaned, abandoned or surrendered children. The condition must apply to all persons
irrespective of religious affiliation who seek to adopt children of that description.

Consequently, where the child which is sought to be adopted falls within the description of an
orphaned, abandoned or surrendered child within the meaning of sub section (2) of Section 41 or
a child in need of care and protection under clause (d) of Section 2, the provisions of the Juvenile
Justice Act 2000 must prevail. In such a case the embargo that is imposed on adopting a child of
the same sex by a Hindu under clauses (i) and (ii) of Section 11 of HAMA 1956 must give way
to the salutary provisions made by the Juvenile Justice Act. Where, however, the child is not of a
description falling under the purview of Chapter IV of the Juvenile Justice Act, 2000, a Hindu
desirous of adopting a child continues to be under the embargo imposed by clauses (i) and (ii) of
Section 11 of the Act of 1956.

JJ Act "has one special mission" -- establishing rules of adoption for a limited subclass of
persons. Therefore, in these circumstances, the Juvenile Justice Act is a special act that overrides
the general provisions of HAMA.

The child in question did fit the description of a child in need of care and protection under
Section 2(d) (v) of the Juvenile Justice Act, 2000 and of a surrendered child under sub section (2)
of Section 41. The Petitioners were eligible to adopt the child under the Juvenile Justice Act,
2000. The child was a surrendered child and was legally free for adoption. The substance and
effect of the procedures prescribed under the Juvenile Justice (Care and Protection of Children)
Act, 2000 have been complied with.

There is abundant material before the Court for the Court to conclude that it is manifestly in the
interest and welfare of the child that the petition for adoption should be allowed. The child has
already been with the Petitioners for a period in excess of four years.

There shall be a declaration that the Petitioners are the adoptive parents with all the rights,
privileges, responsibilities and consequences under the law.

c. Lakshmi Kant Pandey v.Union of India AIR 1984 SC 469

Facts; the petitioner, an advocate of the Supreme Court addressed a letter in public interest to
the Court, complaining of malpractices indulged in by social organisation and voluntary agencies
engaged in the work of offering Indian Children in adoption to foreign parents, the petitioner
alleged that not only Indian Children of tender age, in the cases of pseudo-adoption being
“exposed to the long horrendous journey to distant foreign countries at great risk to their lives

193 | P a g e
but in cases where they survive and where these children are not placed in the shelter and Relief
Houses, they in course of time become beggars or prostitutes for want of proper care from their
alleged foster parents.” The petitioner therein sought relief restraining Indian based private
agencies “from carrying out further activity of routing children for adoption abroad” and
directing the Government of India and other government agencies to carry out their obligations
in the matter of adoption of Indian Children by Foreign parents.

Held; The Supreme Court in this case gave various directions the essence of these directions is
as follows:

(1)Every effort must be made first to see if the child can be rehabilitated by adoption within the
country and if that is not possible, then only adoption by foreign parents, i.e. ’inter-country
adoption’ should be acceptable.

(2) There is a great demand for adoption of children from India and consequently there is
increasing danger of ill-equipped and sometimes even undesirable organizations or individuals
activating themselves in the field of inter-county adoption with a view to trafficking in children.

(3) Following are the requirements which should be insisted upon so far as a foreigner wishing
to take a child in adoption is concerned:

Every application from a foreigner desiring to adopt a child must be sponsored by a social or
child welfare agency licensed by the government of the country in which the foreigner is
resident. No direct dealing is allowed in such cases due to following:

It will help to reduce the possibility of profiteering and trafficking in children, as a foreigner in
his anxiety to secure a child for adoption, be induced or persuaded to pay any unconscionable or
unreasonable amount which might be demanded by the agency or individual procuring the child.
It would be almost impossible for the court to satisfy itself that the foreigner would be suitable as
a parent for the child and would be able to provide a stable and secure family life to the child and
would be able to handle trans-racial, trans-cultural and trans-national problems likely to arise
from such adoption, because, where the application for adopting a child has not been sponsored
by a social or child welfare agency in the country of the foreigner, there would be no proper and
satisfactory home study report on which the court can rely.

There the application is made directly, there would be no authority or agency in the country of
the foreigner who could be made responsible for supervising the progress of the child and
ensuring that the child is adopted at the earliest in accordance with law and grows up in an
atmosphere of warmth and affection with moral and material security assured to it.

As per record, in every foreign country where children from India are taken in adoption, there are
social and child welfare agency licensed or recognised by the government and therefore there

194 | P a g e
would be no difficulty if adoption is required to be sponsored by asocial or child welfare agency
licensed or recognised by the government of the county in which the foreigner resides.

There maybe more than one such social or child welfare agencies, but every such social or child
welfare agency must be licensed or recognized by the government of the foreign country and a
foreigner could not be appointed as guardian unless satisfaction re the application has been
sponsored by such agency.

4) Safeguards for biological parents (both parents/father/mother:

Before a decision surrender of the child by the biological parents, they should be informed about
various possibilities and implications of adoption, like adoption by a foreigner, no connection
with the child, etc.

Also they should not be held in duress for such surrender of the child and must be given three
months’ time to reconsider their decision. Though, they should be properly assisted in taking the
decision of surrender/relinquishment, “by the institution or center or home for child care or
social or child welfare agency to which the child is being surrendered.”

But in order to eliminate any possibility of mischief and to make sure that the child has in fact
been surrendered by its biological parents, the Court gave various guidelines:

It is necessary that the institution or center or home for child care or social or child welfare
agency to which the child is surrendered by the biological parents, should take from the
biological parents a document of surrender duly signed by the biological parents and attested by
atleast two responsible persons and such document of surrender should not only contain the
names of the biological parents and their address but also information in regard to the brother of
the child and its background, health and development.

Where the child is an orphan, destitute or abandoned child (and their parents not known), such
institution or center or home in whose care the child has come, must try to trace the biological
parents of the child.

If after being traced, the biological parents do not want to take back the child, then the same
procedure as outlined above should as far as possible be followed.

If for any reason the biological parents cannot be traced, then there can be no question of taking
their consent or consulting them.

Further, the biological parents should not be induced or encouraged or even be permitted to take
a decision in regard to giving of a child in adoption before the birth of the child or within a
period of three months from the date of birth, so that they can take appropriate decision
regarding rearing/ surrender of the child and/or to allow child to overcome any health problem
experienced after birth.

195 | P a g e
A Central Adoption Resource Agency (CARA),to be set up by the Government of India with
regional branches at a few centers which are active in inter-country adoptions, was held as
desirable, for following purposes:

Such agency may act as a clearing house of information in regard to children available for inter-
country adoption and all applications by foreigners for taking Indian children in adoption can
then be forwarded by the foreign social or child welfare agency to such Central Adoption
Resource Agency (CARA) and the latter can in its turn forward them to one or the other of the
recognized social or child welfare agencies in the country.

Every social or child welfare agency, taking children under its care, could be required to send to
the CARA the names and particulars of children, under its care, available for adoption and the
names and particulars of such children could be entered in a register to be maintained by the
CARA for adoption.

Thus these comprehensive guidelines were followed in some way by the Government of India
and CARA was formed and which published” Guidelines for adoption”. Under these guidelines
every State has a VCA (Voluntary Coordinating Agencies) to co-ordinate and oversees inter-
state adoptions. In reality, in some States the VCA is a non-governmental organization (NGO)
and in some other States the Department of Women and Child Development is the VCA.

d. Shabnam Hashmi v. Union of India 2014 (2) SCALE 529

I. Facts

Shabnam Hashmi visited her first adoption centre in New Delhi’s suburbs; she was told that they
didn’t have any Muslim children. Shabman Hashmi who had a son wanted to adopt a daughter to
make a complete family. However she learnt that Muslims cannot adopt or be adopted and if
they want to do so it can only be done by virtue of The Guardians and Wards Act, 1890, which
doesn’t give a legal status of biological parents nor does adoptee have any rights of inheritance.
She filed a Writ Petition in the Supreme Court in 2005 to give the Muslim parents the same
status as that of a biological parent and to recognize adoption as a fundamental right under
Article 21.The judges called the decision to allow the civil rights activist Shabnam Hashmi to
adopt a child after an eight-year court battle, “a high watermark in the development of the law
relating to adoption.” Shabnam Hashmi was given only a guardianship right on the adopted child
who was not recognised equal to a biological child. There were no rights regarding the
adoption of children for other religious communities except the Hindus. Being a Muslim,
she was subject to the Muslim Shariat Law which did not recognise an adopted child to be on par
with a biological child. Her main argument in the court was that

Petitioner's Plea:

196 | P a g e
A PIL was filed under Article 32 of Indian Constitution to lay down guidelines to enable
adoption of children by person irrespective of caste, creed and religion.

 The petitioner in the view of the Juvenile Justice Act 2000 as amended in 2006 stated that the
prayer in the writ petition was very well answered in the Supreme Court that Juvenile Justice Act
2000 enables any person irrespective of religion she professes to adopt. She argues that every
child in India should have the right to family, explaining that often the child adopted is too young
at the time of adoption and does not know about biological parents, adding that its “highly
insensitive” to not even let them write the ‘adoptive’ parents’ name.

 The writ petitioner has also prayed for a declaration that the right of a child to be adopted and
that of the prospective parents to adopt be declared a fundamental right under Article 21 of the
Constitution.

Opposition to the plea by All India Muslim Personal Law Board:

All India Muslim Personal Law Board objected to this plea that under Islamic law adoption is not
recognised. Since there was no specific law regarding the adoption by Muslims, the board
claimed the personal law to prevail.

 According to the personal laws, they practice the kafala system rather than legal adoption. Under
this procedure the child is placed under the ‘kafil’ (a guardian) who provides the well being of
the child financially and legally takes care of him.

 Further, the Muslim Shariat law doesn't consider the adopted child to be legally equal to a
biological child.

 Also, it considers the adopted child to remain as the true descendants of his biological parents
and not the adoptive parent.

Issues:

 Since there were no civil laws for the non-Hindu adoption, in case of adoption wills the specific
Muslim personal law be applied or the common umbrella act of Juvenile Justice Act 2000?
 Will the right to adoption irrespective of caste, creed religion be considered a fundamental right
applicable to the society homogeneously?

III.Rule

There were three rules that came in to play namely,

 Article 44 of Indian Constitution -

197 | P a g e
Uniform civil code for the citizens The State shall endeavour to secure for the citizens a uniform
civil code throughout the territory of India.

 Article 41 of Juvenile Justice Act 2000-

(i) The primary responsibility for providing care and protection to children shall be that of his
family.

(ii) Adoption shall be resorted to for the rehabilitation of the children who are orphan, abandoned
or surrendered through such mechanism as may be prescribed.

(iii)In keeping with the provisions of the various guidelines for adoption issued from time to
time, by the State Government, or the Central Adoption Resource Agency and notified by the
Central Government, children may be given in adoption by a court after satisfying itself
regarding the investigations having been carried out as are required for giving such children in
adoption.

 Family and Personal Laws- Muslim Shariat Law for Adoption

IV.Background

A Bill on the Uniform Civil Code was introduced in the Indian parliament, but owing to the hue
and cry from the Muslim Community the Bill never became an Act. Moreover, the Central
Government introduced the Adoption of Children Bill, 1972 in the Rajya Sabha, but it was
subsequently dropped, presumably because of Muslim opposition to a uniform law of adoption
applicable to all communities including Muslims. It is claimed that Islam does not recognize
adoption and a child adopted by a Muslim will not be treated at par with a child adopted by a
Hindu.

However most of the non-Hindu communities do not recognize adoption viz. Muslims, Parsis,
Christians and Jews etc. Adoption for the non-Hindus is facilitated by The Guardians and
Wards Act, 1890. This however does not provide to the child the same status as a child born
biologically to the family. Unlike a child adopted under the Hindu Adoption and Maintenance
Act, 1956 the child cannot become their own, take their name or inherit their properly by right.
This Act confers only a guardian-ward relationship. Such a relationship exists until the child
completes 21 years of age.

Soon after attaining majority the legal guardian-ward relationship extinguishes and the adopted
Muslim child becomes a na-mehram (not in blood relation) to the adoptee. This results in
discrimination between a child who is adopted by a Hindu under Hindu Adoption and
Maintenance Act, 1956 and a child adopted by a non-Hindu under The Guardians and Wards
Act, 1890.

198 | P a g e
Many non-Hindus have always desired to adopt children so that the adopted child can inherit the
property, but are unable to do so due to the lack of legislation. It is unfair for those childless
parents who want to adopt a child and for the child who after the age of 21 does not have any
legal bonds with the parents who brought him up over the years.

This plea was brought by the civil activist Shabnam Hashmi in the Court.

V.Analysis

- Before the Court’s verdict there was no adoption law for people belonging to communities
other than Hindus, Sikhs and Jains. Parents of other faiths were accorded the status of a guardian
and their adopted children were described as wards. While disposing the 2005 petition of Hashmi
on February 19, a bench of Chief Justice P. Sathasivam, Justice Ranjan Gogoi and Justice Shiva
Kirti Singh allowed the adoption under the amended juvenile justice law. The following are the
major arguments given by the Supreme Court.

1. Before this judgement the non-Hindu communities have no legal options or civil laws on
adoption. In relation to adoption the non-Hindus were only entitled to guardianship which is a
legal relationship extending till the time the child becomes an adult, which is 18 years for a boy
and 21 years for a girl. This was viewed as insensitive and problematic.

2. The judgement of granting rights to adoption of Muslims is in no case a violation of the Muslim
personal Shariat law. The court has not has not given any separate law, but only expanded the
scope of the existing law (JJ Act) for people of all communities to adopt. It is an optional law
under which no one can force anyone to adopt. Muslims who want to follow Muslim laws are
free to do so. At the same time as the Supreme Court said that Muslim Personal Laws cannot
curtail a citizen from adopting.

3. The court through the (juvenile justice) act aimed to reach the goal enshrined by article 44 of
the constitution. Personal beliefs and faiths, though must be honoured, cannot dictate the
operation of the provisions of an enabling statute. Article 44, under the directive principles of
state policy, says that the state shall endeavour to secure a uniform civil code for all people
throughout India.

4. The bench of Chief Justice of India (CJI) P Sathasivam, Gogoi and Justice Shiva Kirta Singh did
not articulate a fundamental right to adopt, which would "have to await a dissipation of the
conflicting thought processes in this sphere of practices and belief prevailing in the country".

- VI. Held:

199 | P a g e
The petition of the petitioner was upheld and granted to right to adoption irrespective of the
caste, creed and religion under the Juvenile Justice Act 2006. While it was not considered as a
fundamental right as it will lead to conflicts within the heterogeneous society.

VII.Critical Analysis

The law pertaining to adoption is one aspect where the adverse impact of personal laws can be
observed. The Shabnam Hashmi case brought to the fore the judicial discrepancies surrounding
the adoption of non-Muslims by Muslim parents and the lack of legal rights thereof. At this
juncture, it is imperative for the best interests of the child to be identified and conscious efforts to
be made by the law-makers to reconcile the existing differences. There has always been
reluctance in taking stand on secularism and personal laws of a community. Even in the past,
when attempts were made in furtherance of promoting secularism, they got stalled by the
religious outcry.

The Court could have taken a better stand even if there was opposition from All India Muslim
Personal Law Board (AIMPLB). Is it for the AIMPLB to take a call on how a secular law should
be and to try and dominate the Apex Court of the country? If so, I digress. The Board has been
persistently trying to overpower the Supreme Court’s judgment. This is a blatant display of
hegemony over the Apex Court.

Many non-Hindus have always desired to adopt children so that the adopted child can inherit the
property, but are unable to do so due to the lack of legislation. It is unfair for those childless
parents who want to adopt a child and for the child who after the age of 21 does not have any
legal bonds with the parents who brought him up over the years.

Uniform Civil Code is not binding but it imposes an obligation on the State. The Supreme Court
could have interpreted adoption as a fundamental right under Article 21 for both the parents and
the child but they abstained from doing so and procrastinated the duty by stating, ‘We hardly
need to reiterate the well settled principles of judicial restraint, the fundamental of which
requires the Court not to deal with issues of Constitutional interpretation unless such an exercise
is but unavoidable.

ADOPTION BY FOREIGN NATIONALS:

The absence of a secular law not only affects Indians, but it also affects the foreign nationals who
come to India with the desire of adopting a child. However since there is no law in place for such
people, they are required to follow a cumbersome process by first taking the guardianship of the
child from the courts under the Guardians and Wards Act, 1890 and then giving due assurance to
the Court that they would legally adopt the child as per the laws of their country within two years

200 | P a g e
of their arrival there. All this can be avoided by bringing a law in place which will deal with all
such cases and consequentially the misuse of adoptees will also come down.

Newspapers have reported a number of cases where the child has gone to an alien land only to
be mistreated. Such children have been used as domestic servants, beggars and even for
prostitution. In relation to inter-country adoption certain guidelines have been laid down by the
Supreme Court as to how the adoption shall take place in Lakshmi Kant Pandey v. Union of
India. A regulatory body, i.e., Central Adoption Resource Agency (for short ‘CARA’) was
recommended for creation and accordingly set up by the Government of India in the year 1989,
Since then, the said body has been playing a pivotal role, laying down norms both substantive
and procedural, in the matter of inter as well as in country adoptions.

PROBLEMS FACED BY NON-HINDUS

But the problems in intra-country adoptions still persist, due to the lack of a secular law.
Nevertheless, the Supreme Court did not recognize adoption as a fundamental right under Article
21 and claimed self-restraint. The Court’s refusal to the right to adopt is an integral part of
Article 21, which guarantees the right to life and liberty is not convincing. In this era of judicial
activism it is absurd to say that the Supreme Court has no Constitutional responsibility to
discharge the obligation under Article 44 to strive towards the enactment to a Uniform Civil
Code is not satisfactory.

THE CONSTITUTIONAL CONFLICT

More often than not the endeavours by the courts and legislatures towards the creation of a
uniform code are disrupted on the grounds of violation of Article 25 & 26, freedom of religion
and conscience. However Article 25 protects only such practices which are essential and integral
to any religion. Laws and regulations of social relationship in the matter of marriage, divorce,
succession, adoption etc are undoubtedly secular in character and therefore “cannot be brought
within the guarantee enshrined under Article 25 and 26 of the constitution”. The right to freedom
of religion is not an absolute right. Courts have correctly weighed religious interests against the
governmental interest of promoting the health, safety, and general welfare of society. In
accordance with Article 25, the State can interfere in religious matters if it is for social welfare
and reform.

In my opinion, Uniform Civil Code is an idea much talked about but less acted upon. Political
motives of various parties and their affiliation and commitment to different sects is, without any
doubt, a main reason behind the non-implementation of Uniform Civil Code. Hence a uniform
law for adoption will eliminate the unequal status of a child adopted by a Hindu and a child
adopted by a non-Hindu.

201 | P a g e
There is indeed a severe need for a uniform law on adoption in India to avoid discrimination
between various religious communities. The day is not far when people of this country will
shove off their religious vengeance and will start living harmoniously with each other, if the
Government endeavours to secure the Uniform Civil Code and stops partitioning people on the
basis of caste, creed and religion. India will then truly become a SOVEREIGN SOCIALIST
SECULAR DEMOCRATIC REPUBLIC.

Summary 2;

Shabnam Hashmi was a Muslim woman who wanted to adopt a young girl she had bought home
from an adoption agency. She was told that she cannot adopt since she was a Muslim. Muslim
Shariat Law allowed prospective parents only guardianship rights and not adoptive rights. The
Juvenile Justice Act, 2000 however says anyone can adopt. So Ms. Shabnam Hashmi with the
help of the Human Rights Law Network filed a Writ petition seeking the right of the child to be
adopted and the right of a prospective parent to adopt be declared a fundamental right under
Article 21 of the constitution. And also demands that the Supreme Court send out directions to
all states, Union Territories and authorities under the JJ Act, 2000 to implement the provisions of
the Juvenile Justice Act because it’s a secular provision.

Here the issue was if an enabling statute should be given more authority over personal law.
Adoption is an optional legislation dependent on the discretion of the person using that statute.
So should such optional legislation be nullified by personal law? Should the right to adopt and be
adopted be made a fundamental right under article 21 of the Constitution? Are we as a country
ready for such drastic change in legislation? Or should we patiently wait till the dream of a
Uniform Civil Code be achieved?

The two statutes that are in contradiction to each other are the JJ Act, 2000 gives the adopting
parents all the rights of a biological parent whereas Muslim Shariat law only talks about The
Juvenile Justice Act, 2000 defines adoption as “adoption” means the process through which the
adopted child is permanently separated from his biological parents and become the legitimate
child of his adoptive parents with all the rights, privileges and responsibilities that are attached to
the relationship”. The other Statute that contradicts the JJ Act, 2000 is the Muslim Shariat
personal law. A Muslim board was allowed to intervene It is contended that Islamic Law does
not recognize an adopted child to be at par with a biological child. Islamic Law professes what is
known as the “Kafala” system under which the child is placed under a ‘Kafil’ who provides for
the well-being of the child including financial support and thus is legally allowed to take care of
the child though the child remains the true descendant of his biological parents and not that of
the “adoptive” parents.

202 | P a g e
The petitioners already argued and established that the JJ Act is a secular provision and that
personal law should not be allowed to cripple its working. “Personal beliefs and faiths, though
must be honoured, cannot dictate the operation of the provisions of an enabling statute. At the
cost of repetition we would like to say that an optional legislation that does not contain an
unavoidable imperative cannot be stultified by principles of personal law which, however, would
always continue to govern any person who chooses to so submit himself until such time that the
vision of a uniform Civil Code is achieved. While it is correct that the dimensions and
perspectives of the meaning and content of fundamental rights are in a process of constant
evolution as is bound to happen in a vibrant democracy where the mind is always free, elevation
of the right to adopt or to be adopted to the status of a Fundamental Right, in our considered
view, will have to await a dissipation of the conflicting thought processes in this sphere of
practices and belief prevailing in the country. All these impel us to take the view that the present
is not an appropriate time and stage where the right to adopt and the right to be adopted can be
raised to the status of a fundamental right and/or to understand such a right to be encompassed
by Article 21 of the Constitution.

The judgement comes as good news and ensures that we are nearing the realization of the dream
envisaged in Article 44 of the constitution. The judges are absolutely right in exercising judicial
restraint and though the dream of uniform civil code is tempting the judges explain why that is
not a possibility as of now. Though the right to adopt irrespective of religion has been recognized
by the Kerala and Bombay high courts (not as clearly as in this case but in regard to Christian
law) this comes as good news that we are going down a secular path.

e. Baby Manji Yamada v. Union Of India AIR 2009 SC 84

BABY MANJI YAMADA V. UNION OF INDIA

FACTS:

A Japanese couple, Dr. Ikufumi Yamada and his wife, wished to have a baby and entered into a
surrogacy contract with an Indian woman in Anand, a city in the state of Gujarat. The couple
went through matrimonial discord following which the intending mother refused to accept the
baby since it was only genetically related to the father and not her, although she was involved in
the beginning. The contracting father wanted the baby but certain provisions of the Indian law
posed a hindrance. (Indian law doesn’t allow men to adopt and the Japanese law doesn’t

203 | P a g e
recognize children born of surrogacy). The baby, however, was issued a birth certificate in the
name of her genetic father by the Municipality of Anand.

Emiko Yamada calming to be the grandmother of the child filed the petition for the custody of
the child as Manji’s closest blood relative in India, until custody could be transferred to her son,
who had returned to Japan upon the expiration of his visa.

ISSUES:

1) The main issue in this case was whether the grandmother and father could receive
custody of Babji Manji and obtain a visa for her to travel to Japan?

2) The second issue arose in form of a writ petition under Article 32 filed by Ms Sathya
claiming to be an NGO. The petition stated that in the absence of surrogacy laws, which
would clarify who Manji’s parents really are, Yamada (the grandmother) should not be
allowed to claim custody of Manji. The petitioner also claimed that that there is no law
governing surrogation in India and in the name of surrogation lot of irregularities are
being committed. According to it, in the name of surrogacy money making racket is
being perpetuated.

JUDGEMENT:

1. The Court held that, the surrogacy agreement, which is based on free consent and a
meeting of the minds of the parties concerned towards a particular outcome, has been
held to be valid in India and is therefore interpreted in light of the provisions of The
Indian Contract Act, 1872. At the time of the decision of Baby Manji, the fertility
clinics were subjected to regulation by the 2006 guidelines of the Indian Council of
Bio-medical Research (hereinafter ICMR Guidelines) which validated surrogacy
contracts. The court regarded the surrogacy contract entered into by the father as legal
and held him to be the genetic father of the child and was given custodial rights of the
child. The Government was instructed to issue the passport to Manaji Yamada and
she returned with her grand–mother.
2. The court relating to the second issue stated that complaints relating to the misuse of
surrogacy and it being an illegal racket in India should be made before the
Commission set up under the Protection of Child Rights Act, 2005. The Commission
has a right to inquire into complaints and even to take suo motu notice of matters
relating to, (i) deprivation and violation of child rights (ii) non-implementation of
laws providing for protection and development of children and (iii) non-compliance
of policy decisions, guidelines or instructions aimed at mitigating hardships to and
ensuring welfare of the children and to provide relief to such children, or take up the
issues arising out of such matters with appropriate authorities.

204 | P a g e
IMPACT OF THE BABY MANJI CASE:

 There was a public out-cry following the judgment of Baby Manji case for a national
comprehensive regulation of commercial surrogacy. In response to this case, the Indian
Council of Medical Research (ICMR) proposed the draft Assisted Reproductive
Technology (ART) (Regulation) Bill 2008. The bill addresses commercial surrogacy as
well as other emerging reproductive technologies. This bill was also to ensure that there
is a law to protect the interest of the surrogate mother and the child who is born through
surrogacy.
 An affidavit was presented by the solicitor general, Ranjit Kumar to the Supreme Court
in October that the government was planning to ban foreigners from using surrogate
mothers in the country. They reasoned that foreigners can find surrogates in India
relatively easily, especially among impoverished women, and that this creates great
potential for exploitation.

WHY THE COURT’S JUDGMENT IN BABY MANJI IS PROBLAMTIC?

 The Court in Baby Manji Case is pro-contract. Certain guidelines, precautions and
conditions that need to be included in regard to surrogacy agreements are not given
anywhere in The Indian Contract Act. This makes it easier for certain parties to evade
liability and accountability which will leave the innocent child unprotected and
vulnerable. In this case for example the mother refused to take responsibility even though
she entered into a contract.

 Free consent is one the pre-requisites of the surrogacy agreement there is an unequal
balance of power between the two main parties, i.e , the commissioning parents and the
surrogate due to the fact that the surrogate is usually more economically and social
oppressed than the couple who are willing to pay $10,000 or more for a child. It is also
well known that in many cases the husbands of the surrogates pressurize them to act as
surrogate mothers in order to make more money for the family .The comparative
illiteracy and powerlessness of the surrogate mother creates an inherently unequal
bargaining power between the parties whereby the surrogate mother will forgo certain
rights.

 One of the most distinguishing features of a surrogacy agreement from other agreements
is that it directly deals with the exchange of a human child for consideration. In the event
of a problem the matter is taken to the Courts, which will take a long time to give a
decisive verdict, and this will hamper and traumatize the child during its formative years
because of the uncertainty and insecurity in his life.

205 | P a g e
 Most surrogacy agreements in India are with foreign parents because it’s relatively cheap
in India as compared to other countries. In this context issues such as whether the parent
country of the commissioning couple recognizes and accepts the citizenship of the
surrogate child are of great importance since The Citizenship Act of India does not
accord citizenship to a child born out of surrogacy and such matters can leave the child in
a no-man’s land where citizenship is denied from both countries. For example both Japan
and Germany do not recognize surrogacy and therefore do not grant citizenship and the
surrogate children of couples from these countries will not be recognized as citizens.

CASE ANALYSIS:

Baby Manji was the first case relating to surrogacy brought forward in any Court in India. The
surrogacy industry by this time was a thriving business with the country earning itself the
moniker of “rent-a-womb” capital. Commercial surrogacy which has been legal in India since
2002 is famous mainly because the industry is highly unregulated and there are no laws to
regulate the same. The only rules available for surrogacy in India were a set of guidelines
prepared by Indian Council for Medical Research (ICMR) in the year 2005, to protect the right
of the surrogate mother, the new born and the parents but it had its fair share of loopholes.
However, those guidelines did not hold any legal validity and lacked enforceability. Although
the ICMR Guidelines did exist, there was no central or state body to ensure that these regulations
were followed strictly when it comes to surrogacy.

The judgment rendered in Baby Manji reflects not only the despicable state of laws concerning
surrogacy in India, but also the recklessness and apathy with which the Supreme Court dealt with
the case. In the Baby Manji Case, the couple who entered into the surrogacy arrangement had a
matrimonial discord following which the wife refused to claim the child as the child was
genetically related to the father and not her. The child lay stranded for months because neither
the intended mother, the birth mother nor the gestational carrier had any “legal responsibility”
towards the baby and the father who wanted custody of the baby was in catch 22 situations
because of some provisions of both the Indian and the Japanese legal system.

The Government seemed to be helpless in this matter as there were no laws governing the effect
of surrogacy and the Apex Court blatantly evaded responsibility by directing that the National
Commission for Protection of Child Rights was the apt body to deal with this issue. The Courts
need to realise that since the rapid growth of the industry has outpaced advances in the legal
system, the legal system needs to mould itself to face to these changes. The National
Commission of Child Rights, which the Court referred the NGO and others to seek recourse with
in case of any issues with surrogacy is not enough to deal with a booming multi-million dollar
industry. A comprehensive system of laws need to be enacted to protect the rights of children
born via surrogacy by clearly defining parental responsibilities as well as providing monetary
support to the surrogate child, should the parents back away from claiming the child due to
divorce or any other reasons. The law must also extend monetary and legal support to surrogate

206 | P a g e
mothers who are typically poor uneducated women from rural villages. Fertility clinics pay them
between $4,500 and $5,000 for carrying a pregnancy, and charge their clients - many of whom
come from outside the country - about twice that.

Summary 2:

 Facts:- Dr. Ikufumi Yamada and his wife, wished to have a baby and entered into a
surrogacy contract with an Indian woman in Anand, a city in the state of Gujarat where
this practice was pioneered. The couple went through matrimonial discord but the father
still insisted on having custody of the child. Under Indian Laws single father cannot
adopt a girl child. He sent his mother in his stead and a petition was filed before the
Supreme Court.

 Issue- Can the child’s genetic father claim custody of the child since under Indian laws
single father cannot adopt a girl child (Guardians and Wards act, 1890)?

 Is the agreement of surrogacy valid?

 Rule:-There are no laws governing surrogacy in the country.

 Commissions for protection of child rights act, 2005 - section 13

 Analysis:- The Government seemed to be helpless in this matter as there were no laws
governing the effect of surrogacy. The Apex Court directed that the National
Commission for Protection of Child Rights was the apt body to deal with this issue.
 Section 13 of the Commissions for protection of child rights act, 2005 states
the function of the commission
 inquire into complaints and take suo motu notice of matters relating to,-
(i) deprivation and violation of child rights;
(ii) non-implementation of laws providing for protection and development of
children;
(iii) non-compliance of policy decisions, guidelines or instructions aimed at
mitigating hardships to and ensuring welfare of the children and to provide relief
to such children, or take up the issues arising out of such matters with appropriate
authorities; and
(k) such other functions as it may consider necessary for the promotion of child
rights and any other matter incidental to the above functions.
 Guardians and warden,1890 prohibited the father from taking the custody of his
daughter
 The mother after divorce did not want the custody of her child.
 Manji technically had three mothers

207 | P a g e
1) ex-wife (intended mother)
2) anonymous egg donor
3) surrogate mother (pritiben Mehta)

 Dr. Yamada’s visa expired so he had to go back to Japan and in the meanwhile manji’s
grandmother emoki yamada arrived.
 It was becoming clear that Yamada and his daughter were caught between two legal
systems, neither of which was prepared to handle a case like theirs. He hired noted
attorney Indira Jaisingh, who took the position that Manji had the right to live with her
Japanese family and should receive Japanese nationality. She fi led an appeal with the
Indian government to issue documents for Manji and claimed the records made clear that
Yamada was her father. Two days later, on August 8, the Anand municipality issued a
birth certificate to Manji Yamada. The certificate was issued only with her father’s name.
 Satya, a Jaipur-based social justice and child welfare organization,28 fi led a habeas
corpus petition with the Rajasthan High Court. The petition claimed that Manji was a
victim of a “child-traffi cking racket” organized by Dr. Patel through her for-profi t
infertility clinic. Satya alleged that Akanskha’s aim was “furthering the illegal trade in
infants and selling them to foreigners by taking advantage of the lack of proper surrogacy
laws.” The petition stated that in the absence of such laws, which would clarify who
Manji’s parents really are, Yamada should not be allowed to claim custody of Manji.
 India’s Supreme Court stayed the request for Manji’s appearance in court,
dismissed Satya’s accusations that the baby had been abandoned, and granted
temporary custody to her grandmother, Emiko Yamada.
 Conclusion:- Justice Arijit Pasayat and Justice Mukundakan Sharma of the Supreme
Court held that the father was the genetic father of the child and he was given custodial
rights of the child. The Government was instructed to issue the passport to Manji Yamada
and she returned with her grand –mother. Most importantly, the Supreme Court held that
the Surrogacy Agreement was valid in India.
 Critical analysis- "Commercial surrogacy" is a form of surrogacy in which a gestational
carrier is paid to carry achild to maturity in her womb and is usually resorted to by well
off infertile couples who can affordthe cost involved or people who save and borrow in
order to complete their dream of being parents. This medical procedure is legal in several
countries including in India where due to excellent medical infrastructure, high
international demand and ready availability of poor surrogates it is reaching industry
proportions.

The case of Baby Manji Yamada is considered a landmark case because the supreme
court recognized the surrogacy contract as valid. Since the custody of the child was to be
given to a single father further complications were created due to the Guardians and

208 | P a g e
Wards act. I think that the supreme court took the right decision by granting Dr. Yamada
the custody of his daughter as it became extremely difficult for him to come to the
country after the expiration of his visa. The decision encouraged the rise of commercial
surrogacy in the country which benefits both the parents who are not able to conceive
children or for that matter homosexual couples who want to adopt and raise children and
also the gestational surrogate mothers who carry the child and in return get paid. There
are agencies these days which match the couples who want to have children with the
surrogate mother thereby making the entire process convenient. However the dispute
regarding the nationality and citizenship of the child born still remains unanswered.

f. Vijayalaxmamma v. B.T. Shankar, (2000) 4 SCC 538

Summary 1:

DATE OF JUDGMENT: 26/03/2001

BENCH:
D.P. Mohapatra & Doraiswamy Raju.

FACTS

A.T. Nanjappa Rao on his death, left two widows, namely, Sharadamma (senior widow) and
Neelamma (junior widow). On his death in the year 1968, he left behind him a suit schedule. On
22/06/1970, B. T. Shankar who is the son of one of A. T. Nanjappa Rao’s brothers was adopted
as per the Adoption Deed written on the same day according to the customs of the community
from which Sharadamma belonged. The adoption was said to have been undertaken by both the
widows. Disputes arose between B. T. Shankar and both the widows. Soon after, on the death of
Sharadamma, in collusion between Neelamma and another brother of A. T. Nanjappa Rao and
his wife, came to project a claim of adoption of their daughter by name Vijayalakshmamma in
the year 1970 when she was nine years old but reduced into writing and affirmed under a
registered deed dated 26.3.1984, and further said to be fortified by a Will dated 28.3.1984 jointly
claimed to have been executed by late Sharadamma and Neelamma.

B. T. Shankar then prayed for a decree for declaration that he is the only adopted son of late
Nanjappa Rao and for partition of his 3/4th share in the suit schedule properties by metes and
bounds and for delivery of separate possession of his share.

ISSUE

Whether decreeing the suit for partition and separate possession, as prayed for is valid?

209 | P a g e
RULE

• Section 7 reads as follows :-


• Capacity of a male Hindu to take in adoption.Any male Hindu who is of sound mind and
is not a minor has the capacity to take a son or a daughter in adoption:
• Provided that, if he has a wife living, he shall not adopt except with the consent of his
wife unless the wife has completely and finally renounced the world or has ceased to be a
Hindu or has been declared by a court of competent jurisdiction to be of unsound mind.
• Section 8 reads, thus Capacity of a female Hindu to take in adoption. Any female Hindu
• (a) who is of sound mind,
• (b) who is not a minor, and
• (c) who is not married, or if married, whose marriage has been dissolved or whose
husband is dead or has completely and finally renounced the world or has ceased to be a
Hindu or has been declared by a court of competent jurisdiction to be of unsound mind,
has the capacity to take a son or daughter in adoption.
• Section 12 reads as hereunder :-
• 12. Effects of adoption.An adopted child shall be deemed to be the child of his or her
adoptive father or mother for all purposes with effect from the date of the adoption and
from such date all the ties of the child in the family of his or her birth shall be deemed to
be severed and replaced by those created by the adoption in the adoptive family :
Provided that
• (a) the child cannot marry any person whom he or she could not have married if he or she
had continued in the family of his or her birth;
• (b) any property which vested in the adopted child before the adoption shall continue to
vest in such person subject to the obligations, if any, attaching to the ownership of such
property, including the obligation to maintain relatives in the family of his or her birth;
• (c) the adopted child shall not divest any person of any estate which vested in him or her
before the adoption.
• Section 14 reads, thus
• 14. Determination of adoptive mother in certain cases.(1) Where a Hindu who has a wife
living adopts a child, she shall be deemed to be the adoptive mother.
• (2) Where an adoption has been made with the consent of more than one wife, the senior-
most in marriage among them shall be deemed to be the adoptive mother and the others
to be step-mothers.
• (3) Where a widower or a bachelor adopts a child, any wife whom he subsequently
marries shall be deemed to be the step-mother of the adopted child.
• (4) Where a widow or an unmarried woman adopts a child, any husband whom she
marries subsequently shall be deemed to be the step-father of the adopted child.

ANALYSIS

210 | P a g e
The stand of the plaintiff also was that after the death of Sharadamma, the Appellants-defendants
herein with the help of their men were able to dispossess the plaintiff from some of the properties
necessitating the suit claim as noticed above.

The defendants filed a common written statement disputing the facts averred as well as claims
made by the plaintiff by contending that there was no adoption of the plaintiff by Sharadamma as
claimed; that the unregistered deed of adoption was a fabricated one and no rights can be claimed
on the basis of such a document. As an alternate plea, it was projected that in any event the
second defendant-junior widow of late Nanjappa Rao, having not either accorded her consent or
participated in the so-called adoption of the plaintiff by Sharadamma, the senior widow, the
adoption of the plaintiff, if at all, could be for Sharadamma only and not for or the estate of her
husband, late A.T. Nanjappa Rao, and that no adoption could have been properly or legally made
of the plaintiff without the consent of both the widows of late Nanjappa Rao.

In support of the claim of the plaintiff, certain witnesses were brought to court that were present
during the ceremony of the adoption. On a consideration of the oral and documentary evidence
on record, the Trial Judge decreed the suit as prayed for, after adverting to in great detail the
overwhelming materials and evidence on record.

The appellants pursued the matter in appeal before the High Court and, as noticed earlier, the
Division Bench affirmed the findings of the learned Trial Judge on the question of factum of
adoption of the plaintiff while equally confirming the findings that the defendants miserably
failed to prove the case projected by them of adoption of the first defendant. The registered deed
of adoption and the Will were held to have not been proved in respect of their genuineness and
due execution as well by examining either the Attestors or by taking any steps for proving the
signature of Sharadamma, the senior widow, on them.

The adoption was held to be for Sharadamma, the senior widow, alone and not on behalf of both
the widows of late Nanjappa Rao. Keeping in view the legal position that on the death of
Nanjappa Rao in the year 1968 under the provisions of the Hindu Succession Act, 1956, the
widows came to inherit the suit schedule properties with equal share, it was held the adoption of
the plaintiff by Sharadamma alone without the consent of the second wife did not affect the share
of Neelamma in the properties and the plaintiff would be entitled to inherit only the share of late
Sharadamma alone. To that extent, the judgment and decree passed by the Trial Court came to be
modified into one for an half share in favour of the plaintiff as against the 3/4th share granted by
the Trial Court. Not satisfied with the partial relief granted, the appellants have come before the
Supreme Court.

In V.T.S. Chandrasekhara Mudaliar vs Kulandaivelu Mudaliar & Ors. (AIR 1963 SC 185),
K.Subba Rao, J., as the learned Judge then was, on an elaborate consideration of the relevant
case law, held as follows:

211 | P a g e
“ the modern view is that she acts merely as a delegate or representative of her husband, that is to
say, she is only an instrument through whom the husband is supposed to act” “the widows power
to adopt is co-extensive with that of her husband. It is, therefore, clear that a Hindu widow in
making an adoption exercises a power which she alone can exercise, though her competency is
conditioned by other limitations which we shall consider at a later stage. Whether she was
authorised by her husband to take a boy in adoption or whether she obtained the assent of the
sapindas, her discretion to make an adoption, or not to make it, is absolute and uncontrolled.”

At this point it is essential to ask why the assent of the sapindas are required. The power to
adopt is that of the widow as the representative of her husband and the requirement of assent of
the sapindas is only a protection against the misuse of it.

The question as to how the adoption could or ought to be made when a Hindu male dies leaving
behind more than one widow came to be considered by this Court in Eramma and others vs
Muddappa (AIR 1966 SC 1137), with particular reference to the Mysore Hindu Law Womens
Rights Act 1933, stipulating that in the absence of an express prohibition in writing by the
husband, his widow, or where he has left more widows than one, the seniormost of them shall be
presumed to have his authority to make an adoption, and this position was also found to be in
conformity with law in the Bombay State. Court observed that if the consent of the husband or
sapinda was held to be necessary for the reason that a woman is incapable of exercising
independent judgment in the matter of deciding whether she should adopt a son to her deceased
husband, she can hardly be a competent adviser to another widow on the same matter and,
therefore, it was held that the absence of consent of a female sapinda would not invalidate the
adoption in a given case.

While for family peace and good relationship ordinarily a senior widow should do well to
consult the younger one before introducing a boy into the family, there is nothing in law which
compels her to do so.

What are the rights of the adopted child? On adoption by a widow, the adopted son is to be
deemed to be a member of the family of the deceased husband of the widow. Further still, he
loses all his rights in the family of his birth and those rights are replaced by the rights created by
the adoption in the adoptive family. The right, which the child had, to succeed to property by
virtue of being the son of his natural father, in the family of his birth, is, thus, clearly to be
replaced by similar rights in the adoptive family, and, consequently, he would certainly obtain
those rights in the capacity of a member of that family as an adopted son of the deceased
husband of the widow, or the married female, taking him in adoption.

CONCLUSION

B.T. Shankar was awarded one half of the property.

g. Khazan Singh v. Union of India AIR 1980 Delhi 60


212 | P a g e
Summary 1:

Statement of facts

• In the case at hand Khazan Singh is a Jat by birth who has been adopted by Kishan Lal
who is a Julaha by caste.

• It is alleged that a deed of adoption was executed on 5-1-1969 on a stamp paper and that
certain customary ceremonies were also performed on the same day which are evidenced
by a photograph taken on the occasion.
• in December, 1970 the petitioner applied to the Deputy Commissioner, Delhi in a
prescribed form for the grant of a scheduled caste certificate, apparently as was entering
Government service
• On 26-12-1970, a certificate was issued by the office of the Deputy Commissioner, Delhi
certifying that the petitioner. The certificate bore the seal of the Office of the Deputy
Commissioner
• In January, 1971, the petitioner made an application for recruitment to the post of Sub-
Inspector in the Delhi . In the application form the petitioner claimed to be a member of a
scheduled caste and attached an attested copy of the D. C.'s certificate to this effect.
• Terminated in 1975, notice issued to him by office of DC that he was not a Julaha by
birth, called for showcause

• Khazan Singh’s main contention was 1.it was not necessary for a person to be a SC only
by birth 2.certificate that was claimed was only on the basis of adoption by Kishan Lal

• Certificate issued was cancelled

Reasoning given for the cancellation of certificate

1. It was found that adoption was not valid under the Hindu adoptions and maintenance
act,1956 and therefore Khazan Singh is not entitled to have a scheduled caste certificate
in his favour.
2. As he was issued a show cause notice as to why his certificate should not be cancelled,
his reply has been considered and has been found untenable.

A writ petition was filed and the petitioner questioned the validity of the order and the
bonafide intention behind his termination. the respondent claimed that the Petitioner
obtained the certificate concealing material facts and that no such adoption ever took
place.

The Delhi High Court had held that on adoption of a Jat boy into Scheduled Caste family,
he became entitled to the benefit of reservation under Article 16(4).

213 | P a g e
During the course of the judgment, the Judge quoted "As Ganapathy Iyer points out in his
Hindu Law, it cannot be said that 'membership of caste is determined only by birth and
not by anything else'...."

"It appears to me that the answer to the question depends on the legal effect of an adoption and
that the fact that the transaction of adoption took place on the eve of the petitioner seeking a
government employment and perhaps with a view to obtain SC status' is immaterial.”

It was held in the Mohan Rao case that the legal proposition that "on adoption he became a
member of the caste to which his adoptive parents belong has to be accepted. I think that the
decisions regarding the effect of marriage have no relevance in the present context".

Another aspect in adoption that the Judge discussed was relating to the impact of adoption and
effect on the future generations - the children and grandchildren that may be born to the adoptee.

The question is - for how many generations those children have to wait for admission into the
community? Hence recognising the adoption would be the only choice that would remove the
caste factor to a greater extent.

The Judge in Khazan Singh case concluded that "once a Scheduled Caste, always a Scheduled
Caste" should not receive acceptance and added that if genuine adoptions both ways become
frequent they may eventually lead to the development of that social equality at which the
Constitution aims.

Conclusion

He held that the certificate granted to the petitioner was not liable to be cancelled on the ground
that the petitioner's claim to be a Scheduled Caste by adoption was unsustainable

(a) if the adoption is valid and a genuine one, the motive is of no consequence;

(b) the emphatic repetition of "all" in Section 12 of the Hindu Adoptions and Maintenance Act in
relation to "purposes" and "ties" is significant;

(c) the impact of adoption and effect on the future generations - for how many generations the
children, grandchildren and the great grandchildren have to wait for getting admission into the
community;

(d) the genuine adoption both ways would strengthen the social equality aimed in the Indian
Constitution; and

(e) the decisions regarding the effect of marriage have no relevance in the present context.

X. Week 15: Live In Relationships

214 | P a g e
a. Chanmuniya v. Virendra Kumar (2010) 10 SCALE 602

Summary 1

A very interesting set of questions of law have been referred by a two-judge Bench of the
Supreme Court of India to the Chief Justice of India, with a request to place the matter before a
larger Bench. The case, Chanmuniya v. Virendra Kumar Singh suggests a new and welcome
interpretation to the term “wife” for the purposes of Section 125, Cr.P.C. It suggests that “wife”
also include relationships in the nature of marriage, which would include “live-in” relationships.

The case involved a woman who had been married to the Respondent in this case, as per a local
customs which required a woman was to marry the younger brother of her husband, on the
latter’s death. Accordingly, the woman had married the Respondent, following local marriage
ceremonies. Within a few years, the marriage ran into trouble. The Respondent deserted the
Appellant, after which she filed a petition for maintenance under S. 125, Cr.P.C. and for
restitution of conjugal rights under S. 9 of the Hindu Marriage Act. The Trial Court decreed in
her favour, whereas the High Court ruled against her holding that the requirements of S. 7 of the
Hindu Marriage Act had not been satisfied and hence the Appellant and the Respondent were
never legally married. After a review petition failed, the woman approached the Supreme Court.
One of the issues to be decided by the Court was whether there is a presumption of marriage
when parties live together for a long time. If there was such a presumption, then the woman
could claim for maintenance under S. 125, Cr.P.C?

S. 125(1), Cr.P.C. defines “wife” to include “a woman who has been divorced by, or has
obtained a divorce from her husband and has not remarried.” The issue of whether the couple has
to be validly married for the woman to claim maintenance has been a constant issue before
courts, which the Supreme Court notes in Chanmuniya. It also points out the changing nature of
relationships in contemporary Indian society, reflected inter alia by the definition of “domestic
relationship” in the Protection of Women from Domestic Violence Act, 2005.

The Court says that Sir. James Fitzjames Stephen who drafted the Code of Criminal Procedure of
1872, described S. 125 as anti-vagrancy law. This has been the understanding of Indian courts,
even with respect to S. 125, Cr.P.C of the present Code. It is worth remembering that till 2001,
the maximum amount that could be awarded as maintenance under the section was Rs. 500. The
section continues to be a secular law and follows a summary procedure for determination of
maintenance. It was in contradistinction to provisions providing for maintenance under personal
law, where courts took a fairly long time to decide on the issue. Post the amendment to the
section in 2001, whether it continues to be an anti-vagrancy law is debatable. Further, describing
the provision and treating it to be anti-vagrancy measure belittles the dignity of the woman, a
sentiment that the Court shares when it says that the interpretation of the section ought to be in
line with principles of dignity enshrined in the Constitution.

215 | P a g e
It is also pertinent to note Legislative and law reform measures with regard to S. 125.The
National Commission for Women has also recently recommended that the term be widened.
However, no such Amendment Bill has been introduced in Parliament. On the other hand, the
Protection of Women from Domestic Violence Act of 2005, went a step further and defined
“domestic relationship” in S. 2(f) to include a “relationship in the nature of marriage.”
“Economic abuse” was also considered to be “domestic violence,” as per S. 3 of the Act.
Deprivation of economic or financial resources that a person is entitled to by law or custom was
considered “economic abuse.” However, since the Legislature has yet not widened the definition
of “wife” in S. 125, Cr.P.C, courts have been debating whether they need to widen the definition
through judicial interpretation. In this context, the Supreme Court in Chanmuniya has referred
the following questions to a larger bench:

1. Whether the living together of a man and woman as husband and wife for a considerable
period of time would raise the presumption of a valid marriage between them and whether
such a presumption would entitle the woman to maintenance under Section 125 Cr.P.C?
2. Whether strict proof of marriage is essential for a claim of maintenance under Section 125
Cr.P.C.having regard to the provisions of Domestic Violence Act, 2005?
3. Whether a marriage performed according to customary rites and ceremonies, without
strictly fulfilling the requisites of Section 7(1) of the Hindu Marriage Act, 1955, or any
other personal law would entitle the woman to maintenance under Section 125 Cr.P.C.?

The answers to the question by the larger bench will definitely be very interesting. Though the
bench which referred the matter recommended that the interpretation is widened, it is unlikely
that the Supreme Court will do so. It is also debatable whether the Supreme Court ought to widen
the interpretation of the section, when the Legislature has not taken that step, though it is aware
of the differing opinions of the Court and changing nature of relationships in Indian society. The
sentiment expressed by the Court in the last paragraph of the judgment: “We also believe that
such an interpretation would be a just application of the principles enshrined in the Preamble to
our Constitution, namely, social justice and upholding the dignity of the individual,” is however,
definitely welcome.

Summary 2

Facts:

1- The Appellant, Chanmuniya was married to Ram Saran and had 2 daughters.

2- Ram Saran died on 07.03.1992. Chanmuniya married Virendra Kumar Singh Kushwaha, the
younger brother of her deceased husband, in accordance with the local custom of Katha and
Sindur.

216 | P a g e
3- They were living as husband and wife together, Virendra Kumar Singh Kushwaha started
harassing and torturing Chanmuniya.

4- She asked for maintenance but Kushwaha refused saying that she wasn’t his legally

Wedded wife.

Issue:

Whether the living together of a man and woman as husband and wife for a considerable period
of time would raise the presumption of a valid marriage between them and whether such a
presumption would entitle the woman to maintenance under Section 125 Cr.P.C?

Rule:

- Section 125 of CrPc - The court addressed the question of what is meant to be a “wife” under
section 125(1) of CrPC.

- Protection of Women from Domestic Violence Act, 2005

''Women in live-in relationships are also entitled to all the reliefs given in the said Act.''

Relief in the form of protection orders, residence order, monetary relief, custody order for
children and compensation orders.

Woman's right to reside in a shared household.

Section 17 of the Act- 'shared household'

'Alternate accomodation' - This Act assigns a very broad and expansive definition to the term
`domestic abuse' to include within its purview even economic abuse. `Economic abuse' has been
defined very broadly in sub-explanation (iv) to explanation I of Section 3 of the said Act to
include deprivation of financial and economic resources.

Section 20 of the Act allows the Magistrate to direct the respondent to pay monetary relief to the
aggrieved person, who is the harassed woman, for expenses incurred and losses suffered by her.

Section 22 of the Act confers upon the Magistrate, the power to award compensation to the
aggrieved person, in addition to other reliefs granted under the Act.

In terms of Section 26 of the Act, these reliefs mentioned above can be sought in any legal
proceeding, before a civil court, family court or a criminal court, affecting the aggrieved person
and the respondent.

217 | P a g e
Most significantly, the Act gives a very wide interpretation to the term `domestic relationship' as
to take it outside the confines of a marital relationship, and even includes live-in relationships in
the nature of marriage within the definition of `domestic relationship' under Section 2(f) of the
Act.

Analysis:

The parties may not have performed the statutory essentials of a valid marriage, but they had
entered into this relationship with the intention to marry and to be called as husband and wife
before the eyes of the society. It was clearly stated down in the facts of the case that the appellant
and respondent had been living together as husband and wife and had discharged all marital
obligations towards each other.

The law provides for presumption of marriage. Where parties have cohabited together for a
certain period of time, then the courts can construe a presumption of marriage and hence apply
the rules of marriage to deal with issues of those parties. All parties that seek to rely on the
presumption of marriage must prove that indeed a union existed, and such a union can be
challenged only by strong and satisfactory evidence.

In this case the appellant and the respondent have been staying together as man and wife for a
period of time. The people around them must have believed, from their conduct that the parties
are husband and wife. The parties must be living or lived in one household and behaved in a way
that led others to believe that they were husband and wife.

A relationship, which may be adulterous at the beginning, may become matrimonial consent.
This may be evidenced by habit and repute. Courts also insist that such unions should be
qualitative and quantitative. The cohabitation should be long, continuous having substance and
not periodical.

Parties should be living under one roof, do things together like acquiring property, and maybe
even have children together.The Privy Council laid down the general proposition that where a
man and woman are proved to have lived together as man and wife, the law will presume, unless,
the contrary is clearly proved, that they were living together in consequence of a valid marriage.

The question as to whether claim of maintenance can be sought under Section. 125 of Cr.P.C. if
valid marriage is presumed and what 'wife' under Section 125 of Cr.P.C. means especially having
regard to explanation under Clause (b) of the Section, is answered here as follows- The
objectives of Section-125 of Cr.P.C are to achieve a social purpose and to prevent vagrancy and
destitution.

In those cases where a man, who lived with a woman for a long time and even though they may
not have undergone legal necessities of a valid marriage, should be made liable to pay the

218 | P a g e
woman maintenance if he deserts her. The man should not be allowed to benefit from the legal
loopholes by enjoying the advantages of a de facto marriage without undertaking the duties and
obligations. Any other interpretation would lead the woman to vagrancy and destitution, which
the provision of maintenance in Section 125 is meant to prevent.

Critique:

1- The judgement does not address her economic rights in terms of her contribution towards the
relationship in the form of her labour, care giving and material resources but has reinforced
patriarchal notions by linking sexual relationship with marriage.

2- The judgement opines that if there is sexual intimacy then it automatically becomes marriage.
It has in a vast sweep generalised all kinds of relationships and intimacies that could give rise to
a lot of confusion in trial courts.

3- According to the judgment, “any couple who choose to consummate their sexual cravings then
that act becomes a total commitment with adherence to all consequences that may follow except
on certain exceptional considerations.” (sic) These considerations are not listed.

4- The judgement further observes that either party may approach the family court for
declaration of marital status by supplementing documentary proof of evidence in order to prove
sexual relationship. Parties do file proceedings seeking such declaratory relief for marital status
and in such cases presumption of marriage is raised. But to seek evidence to prove sexual
relationship could be counterproductive. By this observation even dating with some physical
intimacy and pictures in social media could imply that there was a marriage and would require a
decree of divorce if the couple decide to break up. It could also lead to situations where young
women who have had a relationship but would want to exit the same can be harassed by the
filing of false cases using such documentary evidence.

5- If any couple who indulge in sexual gratification, then that would be considered as valid
marriage and they could be termed as ‘husband and wife’ as a result of their choice of freedom.”
It is a growing reality that young people are exploring their right to choice and sexual autonomy
before deciding on marriage and to declare that all of them should be treated as married can
cause them more harm.

Conclusion:
Women in Live-in relationships are entitled to all the relief and compensation under the Act of
2005, they should also be allowed in proceedings under Section 125 of Cr.P.C. given in the said
Act (The Protection of Women from Domestic Violence Act, 2005).

Summary:

219 | P a g e
Singhvi and Asok Kumar Ganguly, JJ)48 Facts of the case: the appellant woman contended that
she was remarried, as per the prevalent custom and usage, to the younger brother (Respondent)
of her deceased husband. They lived together as husband and wife for a pretty long time.
Thereafter, surprisingly and unfortunately the husband (respondent) started harassing the
appellant wife and also refused to provide her maintenance u/S.125 of Cr.P.C. In this case, the
High Court held that the appellant wife was not entitled to maintenance on the ground that only
legally married woman can claim maintenance u/S.125 of Cr.P.C. But the Supreme Court turned
down the judgment delivered by the High Court and awarded maintenance to the wife (appellant)
saying that provisions of Sec. 125 of Cr.P.C must be considered in the light of Sec. 26 of the
PWDVA, 2005.49 In brief, the S.C. held that women in ‗live-in-Relationship‘ are equally
entitled to all the reliefs which are available to legally wedded wife.5

b. Bharatha Matha v. R. Vijaya Ranganathan AIR 2010 SC 2685

Summary 1:

FACTS
Peria Mariammal instituted a suit, against the respondents and their predecessor-in- interest
claiming the share of her brother Muthu Reddiar, on the ground that he died unmarried and
intestate and that Smt. Rengammal in the suit was a legally wedded wife of Alagarsami Reddiar,
who was still alive.
Therefore, her claim that she had live-in-relationship with plaintiff's brother Muthu Reddiar and
had two children from him, had to be ignored.
The Trial Court decreed the suit recording the finding that Rengammal, in the suit was wife of
Alagarsami Reddiar who was alive at the time of filing the suit. There had been no legal
separation between them. Therefore, the question of live-in-relationship of Smt. Rengammal
with Muthu Reddiar could not arise.

ISSUE
1. Whether on the admitted long cohabitation of the First defendant and Muthu Reddiar, a
legal presumption of a lawful wedlock is established or not?
2. Weather the children from the live-in relationship can lawfully inherit the property or
not?

RULE
Section 12 of Hindu Marriage Act, 1955,
`any rights in or to the property of any person, other than the parents, in any case where,
but for the passing of this Act, such child would have been incapable of possessing or
acquiring any such rights by reason of this not being the legitimate child of his parents'.

220 | P a g e
Section 16 in The Hindu Marriage Act, 1955
Legitimacy of children of void and voidable marriages. —
(1) Notwithstanding that marriage is null and void under section 11, any child of such
marriage who would have been legitimate if the marriage had been valid, shall be
legitimate, whether such child is born before or after the commencement of the Marriage
Laws (Amendment) Act, 1976 (68 of 1976)*, and whether or not a decree of nullity is
granted in respect of that marriage under this Act and whether or not the marriage is held
to be void otherwise than on a petition under this Act.
(2) Where a decree of nullity is granted in respect of a voidable marriage under section
12, any child begotten or conceived before the decree is made, who would have been the
legitimate child of the parties to the marriage if at the date of the decree it had been
dissolved instead of being annulled, shall be deemed to be their legitimate child
notwithstanding the decree of nullity.
(3) Nothing contained in sub-section (1) or sub-section (2) shall be construed as
conferring upon any child of a marriage which is null and void or which is annulled by a
decree of nullity under section 12, any rights in or to the property of any person, other
than the parents, in any case where, but for the passing of this Act, such child would have
been incapable of possessing or acquiring any such rights by reason of his not being the
legitimate child of his parents.]
Section 114 in The Indian Evidence Act, 1872
(a)(i) There would be presumption in favour of wedlock if the partners lived together for
long spell as husband and wife; but it would be rebuttable and heavy burden lies on the
person who seeks to deprive the relationship of legal origin to prove that no marriage
took place; Tulsa v. Durghatiya, 2008 (1) SCR 709: 2008 (4) SCC 520

ANALYSIS
1. Presumption of marriage for having live-in- relationship with deceased does not arise
when husband of woman was alive - Further held, question of inheritance of coparcenery
property by the illegitimate children born out of live-in-relationship does not arise in
absence of evidence to show that joint family properties of deceased were partitioned or
were his self acquired properties.
2. Ancestral Copercenary Property - Right of illegitimate child - A child born of void or
voidable marriage is not entitled to claim inheritance in ancestral coparcenary property -
He is entitled to claim share only in self acquired properties of his father. (AIR 1929 PC
135 and AIR 1954 Supreme Court 176 relied on)

RELEVANT CASE LAWS


1. In S.P.S. Balasubramanyam Vs. Suruttayan @ Andali Padayachi & Ors. AIR 1992 SC
756, this Court held that if man and woman are living under the same roof and cohabiting

221 | P a g e
for a number of years, there will be a presumption under Section 114 of the Evidence Act
that they live as husband and wife and the children born to them will not be illegitimate.
2. 22. In S. Khushboo Vs. Kanniammal & Anr. JT 2010 (4) SC 478, this Court, placing
reliance upon its earlier decision in Lata Singh Vs. State of U.P. & Anr. AIR 2006 SC
2522, held that live-in-relationship is permissible only in unmarried major persons of
heterogeneous sex. In case, one of the said persons is married, man may be guilty of
offence of adultery and it would amount to an offence under Section 497 IPC.

HELD
1. It is evident that in such a fact-situation, a child born of void or voidable marriage is not
entitled to claim inheritance in ancestral coparcenery property but is entitled only to
claim share in self acquired properties, if any.
2. In the instant case, respondents had not pleaded at any stage that the Suit land was a self
acquired property of Muthu Reddiar. It is evident from the record that Muthu Reddiar did
not partition his joint family properties and died issueless and intestate in 1974.
Therefore, the question of inheritance of coparcenery property by the illegitimate
children, who were born out of the live-in-relationship, could not arise.
Summary 2:

On 17th May 2010 a Bench of the Supreme Court of India consisting of Hon‘ble Justice B.S.
Chauhan and Justice Swatanter Kumar (JJ) in Bharatha Matha & Anr vs R. Vijaya Renganathan
& Ors42 held that “20. Thus, it is evident that Section 16 of the (Hindu Marriage) Act intends to
bring about social reforms, conferment of social status of legitimacy on a group of children,
otherwise treated as illegitimate, as its prime object.”43 “27. Thus, it is evident that in such a
fact-situation, a child born of void or voidable marriage is not entitled to claim inheritance in
ancestral coparcener property but is entitled only to claim share in self acquired properties, if
any.”

c. Velusamy v. Patchaiammal AIR 2011 SC 479

Velusamy vs. D. Patchaiammal (Decided by: Markandey Katju and T.S. Thakur, JJ)51 the
Supreme Court examined the definition of ‗aggrieved person‘ (AP)52 and ‗domestic
relationship‘ taken together and opined that the expression ‗Relationship in the nature of
marriage‘ which is included within the definition of ‗domestic relationship‘ has not clearly been
defined in the Act. Hence the Supreme Court said an ―authoritative decision‖54 is required to be
taken to elucidate what is and what is not ‗a relationship in the nature of marriage‘. The S.C.
commented in the course of its judgment that the Indian Parliament while establishing the two
distinct categories viz. ‗relationship of marriage‘ and ‗relationship in the nature of marriage‘ 55
intended that the enactment should protect and benefit women in both these relationships.
Therefore the S.C. held that “Relationship in the nature of marriage” is akin to a Common Law
Marriage. Common Law Marriages require that although not being formally married:- (a) The
222 | P a g e
couple must hold themselves out to society as being akin to spouses, (b) They must be of legal
age to marry, (c) They must be otherwise qualified to enter into a legal marriage, including being
unmarried, (d) They must have voluntarily cohabited and held themselves out to the world as
being akin to spouses for a significant period of time.” The judgment further clarified the
essentials of a ‗Common Law Marriage‘ and stated that not all “livein relationships” will amount
to “a relationship in the nature of marriage.” The judgement notes by way of illustration that
―merely spending weekends together, “a one night stand” in a case where the man has a
―keep‖ whom he maintains financially but uses her merely for sexual purposes and/or as a
servant, would not qualify for protection under the Act within the definition of `domestic
relationship

d. Aysha v. Ozir Hassan, Madras High Court, 17 June, 2013


e. Revanasiddappa & Anr v. Mallikarjun ( 2011) 11 SCC

(2011) 5 MLJ 392 (SC) Revanasiddappa and Anr Vs Mallikarjun and Ors Hindu Marriage Act
(25 of 1955), Section 16(3) – Legitimacy of children of void and Voidable marriages – Extent of
property rights in coparcenary property – Suit for partition and separate possession – Claim for
share in respect of ancestral property by first wife and children – Marriage of first
defendant/husband to second wife during subsistence of first marriage – Children born out of
void or voidable marriages, legitimate – Section 16(3) limits property rights of such children to
property of their parents – Children born out of void or voidable marriages entitled to share in
property of self acquired or ancestral property of their parents – Matter referred to Larger Bench.
Held: The legislature has used the word “property” in Section 16(3) and is silent on whether such
property is meant to be ancestral or self-acquired. Section 16 contains an express mandate that
such children are only entitled to the property of their parents, and not of any other relation.
RATIO DECIDENDI: The issue of entitlement of children born out of void or voidable
marriages in coparcenary property is proposed to be referred to a Larger Bench for
reconsideration.

“Law takes its own time to articulate such social changes through a process of amendment. That
is why in a changing society law cannot afford to remain static. If one looks at the history of
development of Hindu Law, it will be clear that it was never static and has changed from time to
time to meet the challenges of the changing social pattern in different time.”- Hon‘ble Justice
A.K.Ganguly & G.S. Singhvi in Revanasiddappa & other vs Mallikarjun & others 2

On 31 March, 2011 a special Bench of the Supreme Court of India consisting of G.S. Singhvi,
Asok Kumar Ganguly in Revanasiddappa & Anr. vs Mallikarjun & Ors.45 remarked that
irrespective of the relationship between parents, birth of a child out of such relationship has to be
viewed independently of the relationship of the parents. It is as plain and clear as sunshine that a
child born out of such relationship is innocent and is entitled to all the rights and privileges

223 | P a g e
available to children born out of valid marriages. This is the crux of Section 16(3) of the
amended Hindu Marriage Act, 1955

224 | P a g e

You might also like